You are on page 1of 253

Mc lc

Li ni u . . . . . . . . . . . . . . . . . . . . . . . . . . . . . . . . . . . . . . . . . . . . . . . . . . . . . . . . . . . . . . . . . . . . . . . . . . 3
Trn Nam Dng
Nguyn l cc hn . . . . . . . . . . . . . . . . . . . . . . . . . . . . . . . . . . . . . . . . . . . . . . . . . . . . . . . . . . . . . . . . . . . 5
Trnh o Chin, L Tin Dng
Mt s dng tng qut ca phng trnh hm Pexider v p dng . . . . . . . . . . . . . . . . . . . . 16
L Sng
Xy dng mt lp phng trnh hm nh cc hng ng thc lng gic . . . . . . . . . . . . . 24
L Th Anh oan
Tnh n nh nghim ca mt s phng trnh hm Cauchy . . . . . . . . . . . . . . . . . . . . . . . . . . 35
Trn Vit Tng
Mt s lp phng trnh hm a n sinh bi phi ng thc . . . . . . . . . . . . . . . . . . . . . . . . . . . 47
L Sng, Nguyn inh Huy
T cng thc Euler n bi ton s phc . . . . . . . . . . . . . . . . . . . . . . . . . . . . . . . . . . . . . . . . . . . . . 57
Nguyn Th Tnh
Mt s ng dng ca phng trnh Pell . . . . . . . . . . . . . . . . . . . . . . . . . . . . . . . . . . . . . . . . . . . . . . 67
Hunh B Lc
Php th lng gic l cng c gii ton trong cc bi thi chn hc sinh gii . . . . . . . . . . 79
Nguyn Trung Hng
S dng vnh cc s nguyn gii mt s bi ton s hc . . . . . . . . . . . . . . . . . . . . . . . . . . . 89
Phm Th Thy Hng
Ni suy theo yu t hnh hc ca th . . . . . . . . . . . . . . . . . . . . . . . . . . . . . . . . . . . . . . . . . . . . . . 96
L Sng, V c Thch Sn
Bt bin nh l mt phng php chng minh v ng dng trong gii ton . . . . . . . . . . . 108
1
L Th Thanh Hng
Mt s dng ton lin quan n dy s c quy lut . . . . . . . . . . . . . . . . . . . . . . . . . . . . . . . . . . . 120
Trng Vn im
Vn dng tnh n iu trong cc bi ton tm gii hn dy s v gii phng trnh, bt
phng trnh, h phng trnh . . . . . . . . . . . . . . . . . . . . . . . . . . . . . . . . . . . . . . . . . . . . . . . . . . . . . . . 134
Hunh Tn Chu
ng dng mt s nh l c bn ca gii tch . . . . . . . . . . . . . . . . . . . . . . . . . . . . . . . . . . . . . . . . . 155
L Vn Thn
Mt s phng php gii h phng trnh . . . . . . . . . . . . . . . . . . . . . . . . . . . . . . . . . . . . . . . . . . . . 166
Hunh Kim Linh, T Hng Khanh
Mt s bi ton v a thc trong cc k thi hc sinh gii . . . . . . . . . . . . . . . . . . . . . . . . . . . . . 179
Nguyn Vn Ngc
Mt s bi ton v chia ht i vi cc a thc i xng . . . . . . . . . . . . . . . . . . . . . . . . . . . . . 187
Hunh Duy Thy
Nt p hm s tim n trong bi ton bt ng thc, bi ton tm gi tr ln nht v
gi tr nh nht . . . . . . . . . . . . . . . . . . . . . . . . . . . . . . . . . . . . . . . . . . . . . . . . . . . . . . . . . . . . . . . . . . . . . . 195
Nguyn Ti Chung
Thm mt phng php mi chng minh bt ng thc . . . . . . . . . . . . . . . . . . . . . . . . . . . 204
T Nguyn
Mt s vn v php nghch o trong mt phng v ng dng . . . . . . . . . . . . . . . . . . . . . 213
Trn Vn Trung
S dng mt s tnh cht ca nh x gii bi ton phng trnh hm s. . . . . . . . . . . . 235
Nguyn Hu Tm - Hong T Quyn
T gic lng tip . . . . . . . . . . . . . . . . . . . . . . . . . . . . . . . . . . . . . . . . . . . . . . . . . . . . . . . . . . . . . . . . . . . . 242
2
Li ni u
Ha nhp vi tui tr c nc hot ng si ni k nim ngy thnh lp on thanh nin
Cng sn H Ch Minh v thi ua lp thnh tch cho mng ngy sinh ca Bc H knh yu,
tin ti k nim 37 nm ngy gii phng Nha Trang v thc hin cc chng trnh i mi gio
dc ph thng, S Gio Dc v o to Khnh Ha phi hp vi Hi Ton hc H Ni ng
t chc Hi tho khoa hc Cc chuyn Ton hc bi dng hc sinh gii THPT khu vc
Duyn hi Nam Trung b v Ty nguyn.
y l hi tho ln th hai theo tinh thn cam kt ca cc tnh duyn hi Nam Trung b
v Ty Nguyn v vic hp tc pht trin kinh t - vn ha v x hi. S Gio dc v o
to Ph Yn tin hnh t chc Hi tho ln th nht vo ngy 18-19/4/2011 ti thnh ph
Tuy Ha v lin kt bi dng hc sinh gii v bi dng hc sinh gii mn ton trng Trung
hc ph thng Chuyn cc tnh duyn hi Nam Trung B v Ty Nguyn. Ti Hi tho ln th
nht thng nht giao cho S Gio dc v o to Khnh Ha t chc Hi tho ln th hai.
y l nt sinh hot truyn thng mi v sinh hot chuyn mn, v giao lu hp tc trong gio
dc, o to v cc sinh hot hc thut khc. V thc t, gi y, ti vng duyn hi Nam
Trung b v Ty Nguyn ny xut hin ngy cng nhiu nt thnh tch ni bt, c hc
sinh t gii ton Olympic quc t. Nm nay, nhiu i tuyn t gii cao trong k thi hc sinh
gii quc gia. Cc tnh k Lc, Ph Yn mnh dn c i tuyn tham d k thi Olympic
H Ni m rng bng ting Anh v t gii cao.
Khu vc Duyn hi Nam Trung b v Ty nguyn gi y thc s khi sc, to tin
vn ln tm cao mi, ch ng hi nhp, snh vai ngang bng vi cc khu vc khc trong
c nc.
Hi tho khoa hc ln ny c tin hnh t 14-15/4/2012 ti thnh ph Nha Trang, Khnh
Ha hn hnh c n tip nhiu nh khoa hc, nh gio lo thnh, cc nh qun l, cc chuyn
gia gio dc v cc nh ton hc bo co ti cc phin ton th v cc cn b ch o chuyn
mn t cc s Gio dc v o to, cc thy gio, c gio b mn Ton cc tnh, thnh khu
vc Duyn hi Nam Trung b v Ty nguyn ang trc tip bi dng hc sinh gii mn Ton
bo co ti cc phin chuyn ca hi tho.
3
Ban t chc nhn c trn 30 bo co ton vn gi ti hi tho. Song do khun kh
rt hn hp v thi gian, khu ch bn v thi lng ca cun k yu, chng ti ch c th a
vo k yu c 22 bi, nhng bi cn li s c ch bn gi qu i biu khi thc hin
chng trnh bo co chuyn chnh thc ca hi tho.
Ni dung ca k yu ln ny rt phong ph, bao gm hu ht cc chuyn phc v vic
bi dng hc sinh gii ton t i s, gii tch, hnh hc, s hc n cc dng ton lin quan
khc. Bn c c th tm thy y nhiu dng ton t cc k olympic trong nc v quc t,
mt s dng ton v hm s, l thuyt ni suy, cc tr, ...
Ban t chc xin chn thnh cm n s hp tc v gip ht sc qu bu ca qu thy
gio, c gio v c bit l ton th t ton ca trng THPT chuyn L Qu n Nha Trang,
Khnh Ha c c cun k yu vi ni dung thit thc v rt phong ph ny.
V thi gian chun b rt gp gp, nn cc khu hiu nh v ch bn cun k yu cha
c y , chi tit, chc chn cn cha nhiu khim khuyt. Rt mong c s cm thng
chia s ca qu i biu. Nhng kin ng gp lin quan n cun k yu ny xin gi v
a ch: Trng THPT Chuyn L Qu n, s 67 Yersin, Nha Trang, Khnh Ha. Email:
c3lqdon@khanhhoa.edu.vn.
Xin trn trng cm n.
Nha Trang ngy 25.03.2012
Nguyn Vn Mu
Ch tch Hi Ton hc H Ni
ng trng ban t chc hi tho
4
NGUYN L CC HN
Trn Nam Dng, Trng i hc KHTN Tp HCM
Bi vit ny c pht trin t bi vit Cc phng php v k thut chng minh m
chng ti trnh by ti Hi ngh Cc chuyn Olympic Ton chn lc ti Ba V, H Ni,
thng 5-2010 v ging dy cho i tuyn Olympic Vit Nam d IMO 2010. Trong bi ny, chng
ti tp trung chi tit hn vo cc ng dng ca Nguyn l cc hn trong gii ton.
Mt tp hp hu hn cc s thc lun c phn t ln nht v phn t nh nht. Mt tp con
bt k ca N lun c phn t nh nht. Nguyn l n gin ny trong nhiu trng hp rt c
ch cho vic chng minh. Hy xt trng hp bin! l khu quyt ca nguyn l ny.
1 Mt s v d m u
Ta xem xt mt s v d s dng nguyn l cc hn
V d 1. C 3 trng hc, mi trng c n hc sinh. Mi mt hc sinh quen vi t nht n + 1
hc sinh t hai trng khc. Chng minh rng ngi ta c th chn ra t mi trng mt bn
sao cho ba hc sinh c chn i mt quen nhau.
Li gii. Gi A l hc sinh c nhiu bn nht mt trng khc. Gi s bn nhiu nht ny
l k. Gi s A trng th nht v tp nhng bn quen A l M = {B
1
, B
2
, . . . , B
k
} trng
th 2. Cng theo gi thit, c t nht 1 hc sinh C trng th 3 quen vi A. V C quen
khng qu k hc sinh trng th nht nn theo gi thit C quen vi t nht n + 1 k hc
sinh ca trng th hai, t N = {D
1
, D
2
, ..., D
m
} l nhng ngi quen C trng th hai th
m n+1k. V M, N u thuc tp hp gm n hc sinh v |M| +|N| k +n+1k = n+1
nn ta c M N = . Chn B no thuc M N th ta c A, B, C i mt quen nhau.
V d 2. Chng minh rng khng tn ti s n l, n > 1 sao cho 15
n
+ 1 chia ht cho n
Li gii. Gi s tn ti mt s nguyn l n > 1 sao cho 15
n
+1 chia ht cho n. Gi p l c s
nguyn t nh nht ca n, khi p l. Gi s k l s nguyn dng nh nht sao cho 15
k
1
chia ht cho p (s k c gi l bc ca 15 theo modulo p).
V 15
2n
1 = (15
n
1)(15
n
+1) chia ht cho p. Mt khc, theo nh l nh Fermat th 15
p1
1
chia ht cho p. Theo nh ngha ca k, suy ra k l c s ca cc s p 1 v 2n. Suy ra
k|(p1, 2n). Do p l c s nguyn t nh nht ca n nn (n, p1) = 1. Suy ra (p1, 2n) = 2.
Vy k|2. T k = 1 hoc k = 2. C hai trng hp ny u dn ti p = 7. Nhng iu ny
mu thun v 15
n
+ 1 lun ng d 2mod 7
Trong hai v d trn, r rng vic xt cc trng hp bin em n cho chng ta nhng
thng tin b sung quan trng. Trong v d th nht, vic chn A l hc sinh c s ngi quen
nhiu nht mt trng khc cho ta thng tin s ngi quen ca C trong trng th hai t
nht l n + 1 k. Trong v d th hai, do p l c s nguyn t nh nht nn p 1 nguyn t
cng nhau vi n l bi s ca p.
Bi tp
5
1. Cho n im xanh v n im trn mt phng, trong khng c 3 im no thng hng.
Chng minh rng ta c th ni 2n im ny bng n on thng c u mt khc mu sao cho
chng i mt khng giao nhau.
2. Trn ng thng c 2n + 1 on thng. Mi mt on thng giao vi t nht n on thng
khc. Chng minh rng tn ti mt on thng giao vi tt c cc on thng cn li.
3. Trong mt phng cho n > 1 im. Hai ngi chi ln lt ni mt cp im cha c ni
bng mt vc-t vi mt trong hai chiu. Nu sau nc i ca ngi no tng cc vc t
v bng 0 th ngi th hai thng; nu cho n khi khng cn v c vc t no na m
tng vn cha c lc no bng 0 th ngi th nht thng. Hi ai l ngi thng cuc nu chi
ng?
4. Gi s n l s nguyn dng sao cho 2
n
+ 1 chia ht cho n.
a) Chng minh rng nu n > 1 th n chia ht cho 3;
b) Chng minh rng nu n > 3 th n chia ht cho 9;
c) Chng minh rng nu n > 9 th n chia ht cho 27 hoc 19;
d) Chng minh rng nu n chia ht cho s nguyn t p = 3 th p 19;
e)* Chng minh rng nu n chia ht cho s nguyn t p, trong p = 3 v p = 19 th p 163.
2 Phng php phn v d nh nht
Trong vic chng minh mt s tnh cht bng phng php phn chng, ta c th c thm
mt s thng tin b sung quan trng nu s dng phn v d nh nht. tng l chng
minh mt tnh cht A cho mt cu hnh P, ta xt mt c trng f(P) ca P l mt hm c
gi tr nguyn dng. By gi gi s tn ti mt cu hnh P khng c tnh cht A, khi s
tn ti mt cu hnh P
0
khng c tnh cht A vi f(P
0
) nh nht. Ta s tm cch suy ra iu
mu thun. Lc ny, ngoi vic chng ta c cu hnh P
0
khng c tnh cht A, ta cn c mi
cu hnh P vi f(P) < f(P
0
) u c tnh cht A.
V d 3. Cho ng gic li ABCDE trn mt phng to c to cc nh u nguyn.
a) Chng minh rng tn ti t nht 1 im nm trong hoc nm trn cnh ca ng gic (khc
vi A, B, C, D, E) c to nguyn.
b) Chng minh rng tn ti t nht 1 im nm trong ng gic c to nguyn.
c) Cc ng cho ca ng gic li ct nhau to ra mt ng gic li nh A
1
B
1
C
1
D
1
E
1
bn trong.
Chng minh rng tn ti t nht 1 im nm trong hoc trn bin ng gic li A
1
B
1
C
1
D
1
E
1
.
Cu a) c th gii quyt d dng nh nguyn l Dirichlet: V c 5 im nn tn ti t nht
2 im X, Y m cp to (x, y) ca chng c cng tnh chn l (ta ch c 4 trng hp (chn,
chn), (chn, l), (l, chn) v (l, l)). Trung im Z ca XY chnh l im cn tm.
Sang cu b) l lun trn y cha , v nu XY khng phi l ng cho m l cnh th Z c
th s nm trn bin. Ta x l tnh hung ny nh sau. rng nu XY l mt cnh, chng
hn l cnh AB th ZBCDE cng l mt ng gic li c cc nh c to u nguyn v ta
c th lp li l lun nu trn i vi ng gic ZBCDE, . . . Ta c th dng n bin chng
minh qu trnh ny khng th ko di mi, v n mt lc no s c 1 ng gic c im
nguyn nm trong.
Tuy nhin, ta c th trnh by li l lun ny mt cch gn gng nh sau: Gi s tn ti mt
ng gic nguyn m bn trong khng cha mt im nguyn no (phn v d). Trong tt c
6
cc ng gic nh vy, chn ng gic ABCDE c din tch nh nht (phn v d nh nht). Nu
c nhiu ng gic nh vy th ta chn mt trong s chng. Theo l lun trnh by cu
a), tn ti hai nh X, Y c cp to cng tnh chn l. Trung im Z ca XY s c to
nguyn. V bn trong ng gic ABCDE khng c im nguyn no nn XY phi l mt cnh
no . Khng mt tnh tng qut, gi s l AB. Khi ng gic ZBCDE c to cc nh
u nguyn v c din tch nh hn din tch ng gic ABCDE. Do tnh nh nht ca ABCDE
(phn v d nh nht pht huy tc dng!) nn bn trong ng gic ZBCDE c 1 im nguyn
T. iu ny mu thun v T cng nm trong ng gic ABCDE.
Phn v d nh nht cng l cch rt tt trnh by mt chng minh quy np ( y thng
l quy np mnh), trnh nhng l lun di dng v thiu cht ch.
V d 4. Chng minh rng nu a, b l cc s nguyn dng nguyn t cng nhau th tn ti
cc s nguynx, y sao cho ax +by = 1.
Li gii. Gi s khng nh bi khng ng, tc l tn ti hai s nguyn dng a, b nguyn
t cng nhau sao cho khng tn ti x, y nguyn sao cho ax + by = 1. Gi a
0
, b
0
l mt cp s
nh vy vi a
0
+ b
0
nh nht (phn v d nh nht).
V (a
0
, b
0
) = 1 v (a
0
, b
0
) = (1, 1) (do 1.0 + 1.1 = 1) nn a
0
= b
0
. Khng mt tnh tng qut,
c th gi s a
0
> b
0
. D thy (a
0
b
0
, b
0
) = (a
0
, b
0
) = 1. Do a
0
b
0
+ b
0
= a
0
< a
0
+ b
0
nn
do tnh nh nht ca phn v d, ta suy ra (a
0
b
0
, b
0
) khng l phn v d, tc l tn ti x, y
sao cho (a
0
b
0
)x +b
0
y = 1. Nhng t y th a
0
x +b
0
(y x) = 1. Mu thun i vi iu gi
s. Vy iu gi s l sai v bi ton c chng minh.
Bi tp
5. Gii phn c) ca v d 3.
6. Trn mt phng nh du mt s im. Bit rng 4 im bt k trong chng l nh ca
mt t gic li. Chng minh rng tt c cc im c nh du l nh ca mt a gic li.
3 Nguyn l cc hn v bt ng thc
Nguyn l cc hn thng c p dng mt cch hiu qu trong cc bt ng thc c tnh
t hp, dng chng minh tn ti k s t n s tha mn mt iu kin ny .
V d 5. (Moscow MO 1984) Trn vng trn ngi ta xp t nht 4 s thc khng m c tng
bng 1. Chng minh rng tng tt c cc tch cc cp s k nhau khng ln hn
1
4
.
Li gii. Ta cn chng minh rng vi mi n 4 s thc khng m a
1
, ..., a
n
, c tng bng 1,
ta c bt ng thc
a
1
a
2
+a
2
a
3
+... +a
n1
a
n
+a
n
a
1

1
4
.
Vi n chn (n = 2m) iu ny c th chng minh d dng: t a
1
+ a
3
+ ... + a
2m1
= a; khi
, r rng,
a
1
a
2
+a
2
a
3
+... +a
n1
a
n
+a
n
a
1
(a
1
+a
3
+... +a
2m1
) (a
2
+a
4
+... +a
2m
) = a(1a)
1
4
.
7
Gi s n l v a
k
l s nh nht trong cc s cho. ( thun tin, ta gi s 1 < k < n1 - iu
ny khng lm mt tnh tng qut khi n 4.) t b
i
= a
i
, vi i = 1, ..., k 1, b
k
= a
k
+ a
k+1
v b
i
= a
i+1
vi i = k +1, ..., n1. p dng bt ng thc ca chng ta cho cc s b
1
, ..., b
n1
,
ta c:
a
1
a
2
+... +a
k2
a
k1
+ (a
k1
+ a
k+2
)b
k
+a
k+2
a
k+3
+... +a
n1
a
n
+ a
n
a
1

1
4
.
Cui cng, ta s dng bt ng thc
a
k1
a
k
+a
k
a
k+1
+a
k+1
a
k+2
a
k1
a
k
+a
k1
a
k+1
+a
k+1
a
k+2
(a
k1
+a
k+2
)b
k
,
suy ra iu phi chng minh.
nh gi trn y l tt nht; du bng xy ra khi 2 trong n s bng
1
2
, cn cc s cn li bng
0.
V d 6. Cho n 4 v cc s thc phn bit a
1
, a
2
, . . . , a
n
tho mn iu kin
n

i=1
a
i
= 0,
n

i=1
a
2
i
= 1.
Chng minh rng tn ti 4 s a, b, c, d thuc{a
1
, a
2
, . . . , a
n
} sao cho
a +b +c +nabc
n

i=1
a
3
i
a +b +d + nabd.
Li gii. Nu a b c l ba s nh nht trong cc a
i
th vi mi i = 1, 2, . . . , n ta c bt
ng thc
(a
i
a)(a
i
b)(a
i
c) 0
Suy ra
a
3
i
(a + b +c)a
2
i
(ab +bc +ca)a
i
+ abc vi mi i = 1, 2, . . . , n.
Cng tt c cc bt ng thc ny, vi ch
n

i=1
a
i
= 0,
n

i=1
a
2
i
= 1 ta c
n

i=1
a
3
i
a + b + c + nabc.
By gi nu chn d l s ln nht trong cc a
i
th ta c
(a
i
a)(a
i
b)(a
i
d) 0
vi mi i = 1, 2, . . . , n. V cng thc hin tng t nh trn, ta suy ra bt ng thc v phi
ca bt ng thc kp cn chng minh.
V d 7. Tng bnh phng ca mt 100 s thc dng ln hn 10000. Tng ca chng nh
hn 300. Chng minh rng tn ti 3 s trong chng c tng ln hn 100.
8
Li gii. Gi s 100 s l C
1
C
2
... C
100
> 0. Nu nh C
1
100, th C
1
+C
2
+C
3
>
100. Do ta c th gi s rng C
1
< 100. Khi 100 C
1
> 0, 100 C
2
> 0, C
1
C
2

0, C
1
C
3
0, v vy
100(C
1
+C
2
+C
3
) 100(C
1
+ C
2
+C
3
) (100 C
1
)(C
1
C
3
) (100 C
2
)(C
2
C
3
)
= C
2
1
+C
2
2
+C
3
(300 C
1
C
2
)
> C
2
1
+C
2
2
+C
3
(C
3
+C
4
+. . . +C
1
00)
C
2
1
+ C
2
2
+C
2
3
+ . . . + C
2
100
) > 10000.
Suy ra, C
1
+C
2
+C
3
> 100.
Bi tp
7. Trong mi ca bng 2 n ta vit cc s thc dng sao cho tng cc s ca mi ct bng
1. Chng minh rng ta c th xo i mi ct mt s sao cho mi hng, tng ca cc s cn
li khng vt qu
n+1
4
.
8. 40 tn trm chia 4000 euro. Mt nhm gm 5 tn trm c gi l ngho nu tng s tin
m chng c chia khng qu 500 euro. Hi s nh nht cc nhm trm ngho trn tng s
tt c cc nhm 5 tn trm bng bao nhiu?
4 Nguyn l cc hn v phng trnh Diophant
Trong phn ny, ta trnh by chi tit ba v d p dng nguyn l cc hn trong phng
trnh Fermat, phng trnh Pell v phng trnh dng Markov.
V d 8. Chng minh rng phng trnh x
4
+y
4
= z
2
(1) khng c nghim nguyn dng.
Li gii. Gi s ngc li, phng trnh (1) c nghim nguyn dng, v (x, y, z) l nghim
ca (1) vi z nh nht.
(1) D thy x
2
, y
2
, z i mt nguyn t cng nhau
(2) T nghim ca phng trnh Pythagore, ta c tn ti p, q sao cho
x
2
= 2pq
y
2
= p
2
q
2
z = p
2
+q
2
(3) T y, ta li c mt b ba Pythagore khc, v y
2
+ q
2
= p
2
.
(4) Nh vy, tn ti a, b sao cho
q = 2ab
y = a
2
b
2
p = a
2
+b
2
a, b nguyn t cng nhau
(5) Kt hp cc phng trnh ny, ta c:
x
2
= 2pq = 2(a
2
+b
2
)(2ab) = 4(ab)(a
2
+b
2
)
9
(6) V ab v a
2
+b
2
nguyn t cng nhau, ta suy ra chng l cc s chnh phng.
(7) Nh vy a
2
+b
2
= P
2
v a = u
2
, b = v
2
. Suy ra P
2
= u
4
+v
4
.
(8) Nhng by gi ta thu c iu mu thun vi tnh nh nht ca z v:
P
2
= a
2
+b
2
= p < p
2
+q
2
= z < z
2
.
(9) Nh vy iu gi s ban u l sai, suy ra iu phi chng minh.
Phng php trnh by trn cn c gi l phng php xung thang. y c l l
phng php m Fermat ngh ti khi vit trn l cun sch ca Diophant nhng dng ch
m sau ny c gi l nh l ln Fermat v lm in u bao nhiu th h nhng nh
ton hc.
V d 9. Tm tt c cc cp a thc P(x), Q(x) tha mn phng trnh
P
2
(x) = (x
2
1)Q
2
(x) + 1(1)
Li gii. Khng mt tnh tng qut, ta ch cn tm nghim trong tp cc a thc c h s
khi u dng.
Nu (x +

x
2
1)
n
= P
n
(x)+

x
2
1Q
n
(x)(2) th (x

x
2
1)
n
= P
n
(x)

x
2
1Q
n
(x) (3)
Nhn (2) v (3) v theo v, ta c
1 = (x + x
2
1)
n
(x
_
x
2
1)
n
= (P
n
(x) +
_ _
x
2
1Q
n
(x))(P
n
(x) x
2
1Q
n
(x))
_
= P
2
n
(x) (x
2
1)Q
2
n
(x)
Suy ra cp a thc P
n
(x), Q
n
(x) xc nh bi (2) v (3) l nghim ca (1). Ta chng minh y
l tt c cc nghim ca (1). Tht vy, gi s ngc li, tn ti cp a thc P(x), Q(x) khng
c dng P
n
(x), Q
n
(x) tha mn (1). Ta xt cp a thc (P, Q) nh vy vi degQ nh nht. t
(P(x) +
_
x
2
1Q(x))(x
_
x
2
1) = P

(x) +
_
x
2
1Q

(x) (4)
Th r rng
(P(x)
_
x
2
1Q(x))(x +
_
x
2
1) = P

(x)
_
x
2
1Q

(x)
Suy ra (P

, Q

) cng l nghim ca (1).


Khai trin (4), ta thu c P

(x) = xP(x) (x
2
1)Q(x), Q

(x) = xQ(x) P(x). Ch l


t (1) ta suy ra (P(x) xQ(x))(P(x) + xQ(x)) = Q
2
(x) + 1. V P(x) v Q(x) u c h
s khi u > 0 v degP = degQ + 1 nn ta c deg(P(x) + xQ(x)) = degQ + 1. T y, do
deg(Q
2
(x) + 1) 2deg(Q) nn ta suy ra deg(Q

(x)) deg(Q) 1 < degQ.


Nh vy, theo cch chn cp (P, Q) th tn ti n sao cho (P

, Q

) = (P
n
, Q
n
).
Nhng khi t (4) suy ra
P(x) + x
2
1Q(x) = (P

(x) +
_ _
x
2
1Q

(x))(x +
_
x
2
1)
= (x +
_
x
2
1)
n
(x +
_
x
2
1)
= (x + x
2
1)
n+1
_
Suy ra (P, Q) = (P
n+1
, Qn + 1), mu thun.
Vy iu gi s l sai v ta c iu phi chng minh.
10
V d 10. Tm tt c cc gi tr k sao cho phng trnh (x+y +z)
2
= kxyz c nghim nguyn
dng.
Li gii. Gi s k l mt gi tr cn tm. Gi x
0
, y
0
, z
0
l nghim nguyn dng ca phng
trnh
(x +y +z)
2
= kxyz(1)
c x
0
+y
0
+ z
0
nh nht. Khng mt tnh tng qut, c th gi s x
0
y
0
z
0
.
Vit li (1) di dng x
2
(kyz 2y 2z)x + (y +z)
2
= 0,
ta suy ra x
0
l nghim ca phng trnh bc hai
x
2
(ky
0
z
0
2y
0
2z
0
)x + (y
0
+z
0
)
2
= 0(2)
Theo nh l Viet x
1
= ky
0
z
0
2y
0
2z
0
x
0
=
(y0+z0)
2
x0
cng l nghim ca (2). T
(x
1
, y
0
, z
0
) l nghim ca (1). Cng t cc cng thc trn, ta suy ra x
1
nguyn dng. Tc l
(x
1
, y
0
, z
0
) l nghim nguyn dng ca (1). T tnh nh nht ca x
0
+y
0
+z
0
ta x
1
x
0
. T
y ta c
ky
0
z
0
2y
0
2z
0
x
0
x
0
v
(y
0
+x
0
)
2
x
0
x
0
T bt ng thc th hai ta suy ra y
0
+z
0
x
0
. T , p dng vo bt ng thc th nht,
ta c ky
0
z
0
4x
0
.
Cui cng, chia hai v ca ng thc x
2
0
+y
2
0
+z
2
0
+2x
0
y
0
+2y
0
z
0
+2z
0
x
0
= kx
0
y
0
z
0
cho x
0
y
0
z
0
,
ta c
x
0
y
0
z
0
+
y
0
x
0
z
0
+
z
0
x
0
y
0
+
2
z
0
+
2
x
0
+
2
y
0
= k.
T suy ra
k
4
+ 1 + 1 + 2 + 2 + 2 k, tc l k
32
3
. Suy ra k 10.
Ch nu x
0
= 1 th y
0
= z
0
= 1 suy ra k = 9. Nu k = 9 th x
0
2 v nh gi trn tr
thnh
k
4
+ 1 +
1
2
+ 2 + 1 + 2 k suy ra k
26
3
, suy ra k 8
Vy gi tr k = 10 b loi.
Vi k = 1 phng trnh c nghim, chng hn (9, 9, 9)
Vi k = 2 phng trnh c nghim, chng hn (4, 4, 8)
Vi k = 3 phng trnh c nghim, chng hn (3, 3, 3)
Vi k = 4 phng trnh c nghim, chng hn (2, 2, 4)
Vi k = 5 phng trnh c nghim, chng hn (1, 4, 5)
Vi k = 6 phng trnh c nghim, chng hn (1, 2,3)
Vi k = 8 phng trnh c nghim, chng hn (1, 1, 2)
Vi k = 9 phng trnh c nghim, chng hn (1, 1, 1)
Ngoi ra, ta c th chng minh c rng trng hp k = 7 phng trnh khng c nghim
nguyn dng (xin c dnh cho bn c).
Vy cc gi tr k cn tm l k = 1, 2, 3, 4, 5, 6, 8, 9.
V d 11. (CRUX, Problem 1420) Nu a, b, c l cc s nguyn dng sao cho
0 < a
2
+b
2
abc c
Chng minh rng a
2
+b
2
abc l s chnh phng.
11
Li gii. Gi s ngc li rng tn ti cc s nguyn dng a, b, c sao cho 0 < a
2
+b
2
abc c
v k = a
2
+b
2
abc (1) khng phi l s chnh phng.
By gi ta c nh k v c v xt tp hp tt c cc cp s nguyn dng (a, b) tha mn phng
trnh (1), tc l ta xt
S(c, k) = {(a, b) (N

)
2
: a
2
+b
2
abc = k}
Gi s (a, b) l cp s thuc S(c, k) c a + b nh nht. Khng mt tnh tng qut c th gi
s a b. Ta xt phng trnh
x
2
bcx + b
2
k = 0
Ta bit rng x = a l mt nghim ca phng trnh. Gi a
1
l nghim cn li ca phng trnh
ny th a
1
= bc a =
(b
2
k)
a
.
Ta c th chng minh c rng (bn c t chng minh!) a
1
nguyn dng. Suy ra (a
1
, b)
cng thuc S(c, k).
Tip theo ta c a
1
= (b
2
k)/a < a
2
/a = a, suy ra a
1
+ b < a + b. iu ny mu thun vi
cch chn (a, b).
Bi tp
9. Chng minh rng phng trnh x
3
+ 3y
3
= 9z
3
khng c nghim nguyn dng.
10. Chng minh rng phng trnh x
2
+y
2
+ z
2
= 2xyz khng c nghim nguyn dng.
11. (IMO 88) Nu a, b, q = (a
2
+b
2
)/(ab + 1) l cc s nguyn dng th q l s chnh phng.
12. (PTNK 03). Tm tt c cc s nguyn dng k sao cho phng trnh x
2
(k
2
4)y
2
= 24
c nghim nguyn dng.
13. (Mathlinks) Cho A l tp hp hu hn cc s nguyn dng. Chng minh rng tn ti tp
hp hu hn cc s nguyn dng B sao cho A B,

xB
x =

xB
x
2
14.* (AMM 1995) Cho x, y l cc s nguyn dng sao cho xy + x v xy + y l cc s chnh
phng. Chng minh rng c ng mt trong hai s x, y l s chnh phng.
15. (IMO 2007) Cho a, b l cc s nguyn dng sao cho 4ab 1 chia ht (4a
2
1)
2
. Chng
minh rng a = b.
16. (VMO 2012) Xt cc s t nhin l a, b m a l c s ca b
2
+2 v b l c s ca a
2
+2.
Chng minh rng a v b l cc s hng ca dy s t nhin (v
n
) xc nh bi
v
1
= v
2
= 1 v v
n
= 4v
n1
v
n2
vi mi n 2.
5 Nguyn l cc hn trong t hp
Trn y chng ta xem xt cc v d p dng ca nguyn l cc hn trong mnh t
mu m nht dnh cho nguyn l cc hn. Nguyn l cc hn c th c ng dng chng
minh mt qu trnh l dng (trong cc bi ton lin quan n bin i trng thi) trong bi
ton v th, hay trong cc tnh hung t hp a dng khc. Cc i tng thng c em
ra xt cc hn thng l: on thng ngn nht, tam gic c din tch ln nht, gc ln
nht, nh c bc ln nht, chu trnh c di ngn nht . . .
Di y ta xem xt mt s v d:
V d 12. (nh l Sylvester) Cho tp hp S gm hu hn cc im trn mt phng tha mn
tnh cht sau: Mt ng thng i qua 2 im thuc S u i qua t nht mt im th ba
thuc S. Khi tt c cc im ca S nm trn mt ng thng.
12
Kt lun ca nh l nghe c v hin nhin nhng chng minh n th khng h n gin.
Chng minh di y ca Kelly c chng ti tham kho t Wikipedia
Gi s phn chng l tn ti mt tp hp S gm hu hn im khng thng hng nhng mi
ng thng qua hai im trong S u cha t nht ba im. Mt ng thng gi l ng
ni nu n i qua t nht hai im trong S. Gi s (P,l) l cp im v ng ni c khong
cch dng nh nht trong mi cp im-ng ni.
Theo gi thit, l i qua t nht ba im trong S, nn nu h ng cao t P xung l th tn
Hnh 1:
ti t nht hai im nm cng mt pha ca ng cao (mt im c th nm ngay chn
ng cao). Trong hai im ny, gi im gn chn ng cao hn l B, v im kia l C.
Xt ng thng m ni P v C. Khong cch t B ti m nh hn khong cch t P ti l, mu
thun vi gi thit v P v l. Mt cch thy iu ny l tam gic vung vi cnh huyn BC
ng dng v nm bn trong tam gic vung vi cnh huyn PC.
Do , khng th tn ti khong cch dng nh nht gia cc cp im-ng ni. Ni cch
khc, mi im u nm trn ng mt ng thng nu mi ng ni u cha t nht ba
im.
V d 13. V d 13. (Trn u ton hc Nga 2010) Mt quc gia c 210 thnh ph. Ban u
gia cc thnh ph cha c ng. Ngi ta mun xy dng mt s con ng mt chiu ni
gia cc thnh ph sao cho: Nu c ng i t A n B v t B n C th khng c ng i
t A n C.Hi c th xy dng c nhiu nht bao nhiu ng?
Li gii. Gi A l thnh ph c nhiu ng i nht (gm c ng i xut pht t A v
ng i n A). Ta chia cc thnh ph cn li thnh 3 loi. Loi I - C ng i xut pht t
A. Loi II - C ng i n A. Loi III: Khng c ng i n A hoc xut pht t A. t
m = |I|, n = |II|, p = |III|. Ta c m+n +p = 209.
D thy gia cc thnh ph loi I khng c ng i. Tng t, gia cc thnh ph loi 2 khng
c ng i.
13
S cc ng i lin quan n cc thnh ph loi 3 khng vt qu p(m + n). (Do bc ca
A = m+n l ln nht).
Tng s ng i bao gm:
+ Cc ng i lin quan n A: m+ n
+ Cc ng i lin quan n III : p(m+n)
+ Cc ng i gia I v II: mn
Suy ra tng s ng i nh hn mn + (p + 1)m+ (p + 1)n (m+n + p + 1)
2
/3 = 210
2
/3.
Du bng xy ra vi th 3 phe, mi phe c 70 thnh ph, thnh ph phe 1 c ng i
n thnh ph phe 2, thnh ph phe 2 c ng i n thnh ph phe 3, thnh ph phe 3 c
ng i n thnh ph phe 1.
V d 14. Trong quc hi M, mi mt ngh s c khng qu 3 k th. Chng minh rng c th
chia quc hi thnh 2 vin sao cho trong mi vin, mi mt ngh s c khng qu mt k th.
y l mt v d m ti rt thch. C nhiu cch gii khc nhau nhng y chng ta s
trnh by mt cch gii s dng nguyn l cc hn. tng tuy n gin nhng c rt nhiu
ng dng (trong nhiu bi ton phc tp hn).
Ta chia quc hi ra thnh 2 vin A, B mt cch bt k. Vi mi vin A, B, ta gi s(A), s(B) l
tng ca tng s cc k th ca mi thnh vin tnh trong vin . V s cch chia l hu hn
nn phi tn ti cch chia (A
0
, B
0
) sao cho s(A
0
) + s(B
0
) nh nht. Ta chng minh cch chia
ny tha mn yu cu bi ton.
Gi s rng cch chia ny vn cha tho mn yu cu, tc l vn c mt ngh s no c
nhiu hn 1 k th trong vin ca mnh. Khng mt tnh tng qut, gi s ngh s x thuc A
0
c t nht 2 k th trong A
0
. Khi ta thc hin php bin i sau: chuyn x t A
0
sang B
0
c cch chia mi l A

= A
0
{x} v B

= B
0
{x}. V x c t nht 2 k th trong A
0
v
A

khng cn cha x nn ta c
s(A

) s(A
0
) 4 (trong tng mt i t nht 2 ca s(x) v 2 ca cc k th ca x trong A
0
)
V x c khng qu 3 k th v c t nht 2 k th trong A
0
nn x c nhiu nht 1 k th trong
B
0
(hay B

), cho nn
s(B

) s(B
0
) + 2
T s(A

) +s(B

) s(A
0
) +s(B
0
) 2. Mu thun vi tnh nh nht ca s(A
0
) +s(B
0
). Vy
iu gi s l sai, tc l cch chia (A
0
, B
0
) tha mn yu cu bi ton (pcm).
Bi tp
17. Cho 2n im trn mt phng, trong khng c 3 im no thng hng. Chng minh rng
nhng im ny c th phn thnh n cp sao cho cc on thng ni chng khng ct nhau.
18. Trong mt phng cho 100 im, trong khng c ba im no thng hng. Bit rng ba
im bt k trong chng to thnh mt tam gic c din tch khng ln hn 1. Chng minh
rng ta c th ph tt c cc im cho bng mt tam gic c din tch 4.
19. Trn mt phng cho 2n + 3 im, trong khng c ba im no thng hng v khng c
4 im no nm trn mt ng trn. Chng minh rng ta c th chn ra t cc im ny 3
im, sao cho trong cc im cn li c n im nm trong ng trn v n im nm ngoi
ng trn.
20. Trong mt phng cho n im v ta nh du tt c cc im l trung im ca cc on
thng c u mt l cc im cho. Chng minh rng c t nht 2n 3 im phn bit c
nh du.
21. Ti mt quc gia c 100 thnh ph, trong c mt s cp thnh ph c ng bay. Bit
14
rng t mt thnh ph bt k c th bay n mt thnh ph bt k khc (c th ni chuyn).
Chng minh rng c th i thm tt c cc thnh ph ca quc gia ny s dng khng qu a)
198 chuyn bay b) 196 chuyn bay.
22*. Trong mt nhm 12 ngi t 9 ngi bt k lun tm c 5 ngi i mt quen nhau.
Chng minh rng tm c 6 ngi i mt quen nhau trong nhm .
Ti liu tham kho
[1] Nguyn Vn Mu, Cc chuyn Olympic Ton chn lc,Ba V , 5-2010 .
[2] on Qunh ch bin, Ti liu gio khoa chuyn ton - i s 10, NXB GD, 2010.
[3] http://fermatslasttheorem.blogspot.com/2005/05/fermats-last-theorem-n-4.html
[4] vi.wikipedia.org/wiki/nh l Sylvester-Gallai
[5] www.mathscope.org
[6] www.problems.ru
15
MT S DNG TNG QUT CA PHNG
TRNH HM PEXIDER V P DNG
Trnh o Chin, Trng Cao ng S Phm Gia Lai
L Tin Dng, Trng THPT Pleiku, Gia Lai
Phng trnh hm Pexider (PTHP) l phng trnh hm tng qut trc tip ca Phng trnh
hm Cauchy quen thuc. Bi vit ny cp n mt s dng tng qut ca PTHP v vi p
dng ca n trong chng trnh Ton ph thng.
1 Phng trnh hm Pexider
PTHP c bn gm bn dng di y (li gii c th xem trong [1] hoc [2])
Bi ton 1.1. Tm tt c cc hm s f, g, h xc nh v lin tc trn R tha mn iu kin
f (x +y) = g (x) + h(y) , x, y R. (1)
Gii. Nghim ca phng trnh (1) l
f (t) = ct + a + b, g (t) = ct + a, h(t) = ct + b; a, b, c R.
Bi ton 1.2. Tm tt c cc hm s f, g, h xc nh v lin tc trn R tha mn iu kin
f (x +y) = g (x) h(y) , x, y R. (2)
Gii. Nghim ca phng trnh (2) l
f (t) = abe
c
t, g (t) = ae
c
t, h(t) = be
c
; a, b, c R
hoc
f 0, g 0, h C
R
,
trong C
R
l tp hp cc hm s lin tc trn R,
hoc
f 0, h 0, g C
R
.
Bi ton 1.3. Tm tt c cc hm s f, g, h xc nh v lin tc trn R
+
tha mn iu kin
f (xy) = g (x) + h(y) , x, y R
+
. (3)
Gii. Nghim ca phng trnh (3) l
f (t) = m.lnt +a +b, g (t) = m.lnt +a, h(t) = m.lnt +b; m, a, b, c R.
Bi ton 1.4. Tm tt c cc hm s f, g, h xc nh v lin tc trn R
+
tha mn iu kin
f (xy) = g (x) h(y) , x, y R
+
. (4)
Gii. Nghim ca phng trnh (4) l
f (t) = abt
c
, g (t) = at
c
, h(t) = bt
c
; a, b, c R.
16
2 Mt s dng tng qut ca Phng trnh hm Pexider
Ty theo mc kin thc, PTHP c nhiu dng tng qut khc nhau. Di y l mt s
dng tng qut ca phng trnh (1) gn gi vi chng trnh ca h ph thng chuyn Ton.
Bi ton 2.1. Tm tt c cc hm s f, f
i
(i = 1, 2, ..., n) xc nh v lin tc trn R tha
mn iu kin
f
n

i=1
x
i
_
=
n

i=1
f
i
(x
i
), x, x
i
R. (5)
Gii. y l dng quy np mt cch t nhin ca Bi ton 1.1. Nghim ca phng trnh
(5) l
f (t) = at +
n

i=1
a
i
, f
i
(t) = at +a
i
; a, a
i
R.
Tng t Bi ton 2.1, ta cng c th gii c phng trnh hm dng
f
n

i=1
a
i
x
i
_
=
n

i=1
a
i
f
i
(x
i
), x, x
i
R, a
i
R.
Bi ton sau y l mt dng tng qut kh c bn, m phng php quy np khng th
p dng trong li gii. Mt s phn chng minh c s dng mt s kin thc c bn, khng
qu kh, ca i s tuyn tnh v Phng trnh vi phn, thuc chng trnh c s ca Ton
cao cp.
Bi ton 2.2. Tm tt c cc hm s f, f
i
, g
i
(i = 1, 2, ..., n) xc nh v tn ti o hm
(theo mi bin s c lp x, y) trn R tha mn iu kin
f (x +y) =
n

k=1
f
k
(x) g
k
(y), x, y R, n 2. (6)
Gii. Ta gii bi ton ny trong trng hp n = 2. Trng hp n 3 c gii tng t.
Xt phng trnh hm
f (x +y) = f
1
(x) g
1
(y) + f
2
(x) g
2
(y) , x, y R, (7)
trong cc hm f, f
1
, f
2
, g
1
, g
2
xc nh v tn ti o hm (theo mi bin s c lp x, y)
trn R.
Khng mt tnh tng qut, ta lun c th gi thit rng cc h hm {f
1
(x) , f
2
(x)} v
{g
1
(x) , g
2
(x)} l c lp tuyn tnh. Ta c
f

x
(x +y) = f

1
(x) g
1
(y) + f

2
(x) g
2
(y) ,
f

y
(x +y) = f
1
(x) g

1
(y) + f
2
(x) g

2
(y) .
V f

x
(x +y) = f

y
(x + y), nn
f

1
(x) g
1
(y) + f

2
(x) g
2
(y) = f
1
(x) g

1
(y) + f
2
(x) g

2
(y) . (8)
17
Ngoi ra, v {g
1
(x) , g
2
(x)} l c lp tuyn tnh, nn tn ti cc hng s y
1
, y
2
sao cho


g
1
(y
1
) g
1
(y
2
)
g
2
(y
1
) g
2
(y
2
)


= 0.
Thay y
1
, y
2
vo (8), ta c
f

1
(x) g
1
(y
1
) + f

2
(x) g
2
(y
1
) = f
1
(x) g

1
(y
1
) + f
2
(x) g

2
(y
1
) ,
f

1
(x) g
1
(y
2
) + f

2
(x) g
2
(y
2
) = f
1
(x) g

1
(y
2
) + f
2
(x) g

2
(y
2
) .
V nh thc nu trn khc 0, nn h phng trnh ny c nghim duy nht f

1
(x), f

2
(x).
Do , ta c th biu din f

1
(x) v f

2
(x) qua f
1
(x) v f
2
(x) di dng
f

1
(x) = a
11
f
1
(x) + a
12
f
2
(x) , f

2
(x) = a
21
f
1
(x) + a
22
f
2
(x) . (9)
Mt khc, thay y = 0 vo (7), ta c
f (x) = c
1
f
1
(x) + c
2
f
2
(x) . (10)
Suy ra
f

(x) = c
11
f
1
(x) + c
12
f
2
(x) , f

(x) = c
21
f
1
(x) + c
22
f
2
(x) . (11)
- Nu


c
11
c
12
c
21
c
22


= 0, th t (10) v (11), ta thu c phng trnh vi phn tuyn tnh
thun nht
a
1
f

(x) + a
2
f

(x) = 0,
trong a
1
v a
2
khng ng thi bng 0. Gii phng trnh vi phn ny, ta tm c f (x).
Tt c cc hm f (x) ny u tha mn (7) nn l nghim ca phng trnh.
- Nu


c
11
c
12
c
21
c
22


= 0, th t (10) v (11), ta c th biu din f
1
(x) v f
2
(x) bi mt t hp
tuyn tnh ca f

(x) v f

(x). Thay biu din ny vo (5), ta thu c phng trnh dng


f (x) + a
1
f

(x) + a
2
f

(x) = 0.
Gii phng trnh vi phn ny, ta tm c f (x) .
Tt c cc hm f (x) ny u tha mn (7) nn l nghim ca phng trnh. Bi ton
c gii quyt.
Di y l mt s trng hp c bit m phng trnh (7) tr thnh mt s phng trnh
hm c bn. Nhng phng trnh ny kh ni ting v c li gii hon ton s cp (c th
xem trong [1] hoc [2]).
- Vi f
1
(x) = f (x), g
1
(y) 1, f
2
(x) 1, g
2
(y) = f (y), phng trnh (7) tr thnh Phng
trnh hm Cauchy
f (x +y) = f (x) + f (y) , x, y R.
- Vi f
1
(x) = g (x), g
1
(y) 1, f
2
(x) 1, g
2
(y) = h(y), phng trnh (7) tr thnh Phng
trnh hm Pexider
f (x +y) = g (x) + h(y) , x, y R.
18
- Vi f
2
(x) 1, phng trnh (7) tr thnh Phng trnh hm Vincze
f (x + y) = f
1
(x) g
1
(y) + g
2
(y) , x, y R,
- Vi f
1
(x) = f (x), g
1
(y) = g (y), f
2
(x) = g (x), g
2
(y) = f (y), phng trnh (7) tr
thnh phng trnh hm dng lng gic (v mt nghim ca phng trnh ny l f (t) = sint,
g (t) = cost)
f (x +y) = f (x) g (y) + g (x) f (y) , x, y R.
3 p dng
PTHP tng qut c nhiu p dng trong vic nghin cu mt s vn lin quan ca Ton
ph thng. Sau y l mt p dng lin quan n cc php chuyn i bo ton yu t gc ca
mt tam gic.
Bi ton 3.1. Tm tt c cc hm s f, g, h xc nh v lin tc trn R tha mn iu kin
sau: Nu A, B, C R, A+B+C = , th A
1
+B
1
+C
1
= , trong A
1
= f (A), B
1
= f (B),
C
1
= f (C).
Gii. Gi s cc hm s f, g, h xc nh v lin tc trn R tha mn iu kin trn. Ta c
A
1
+ B
1
+C
1
= f (A) + f (B) + f (C) =
f ( B C) = f (B) f (C) . (12)
t F (x) = f ( x), G(x) =

2
g (x), H (x) =

2
h(x). Khi , phng trnh (12) c
dng
F (B + C) = G(B) + H (C) . (13)
Phng trnh (13) chnh l Phng trnh Pexider bit. Nghim lin tc tng qut ca phng
trnh ny l
F (x) = ax +c
1
+ c
2
, G(x) = ax + c
1
, H (x) = ax +c
2
,
trong a, c
1
, c
2
R.
Do
f (x) = a ( x) + c
1
+c
2
, g (x) =

2
ax c
1
, h(x) =

2
ax c
2
. (14)
t a = k, c
1
+c
2
+a = ,

2
c
1
= ,

2
c
2
= . Th th k + + + = 1. Khi
, bi (14), ta thu c
f (x) = kx + , g (x) = kx + , h(x) = kx +,
trong k + + + = 1.
R rng cc hm s f, g, h nu trn tha mn iu kin ca bi ton.
Bi ton 3.2. Tm tt c cc hm s f, g, h xc nh v lin tc trn R tha mn iu kin
sau: Nu A, B, C 0, A + B + C = , th A
1
, B
1
, C
1
0, A
1
+ B
1
+ C
1
= , trong
19
A
1
= f (A), B
1
= f (B), C
1
= f (C) .
Gii. Tng t cch gii trn, ta tm c
f (x) = kx + , g (x) = kx + , h(x) = kx +,
trong 0, 0, k + 0, k + 0, + 1, k + + 1.
Kt qu ca Bi ton 3.2 c nhiu p dng trong cc php chuyn i bo ton yu t gc
trong tam gic, chng hn cc H qu sau y m phn chng minh dnh cho bn c
H qu 3.1. Nu A, B, C l ba gc ca mt tam gic, th A
1
, B
1
, C
1
xc nh nh sau
A
1
=
B + C
2
, B
1
=
C + A
2
, C
1
=
A +B
2
cng l ba gc ca mt tam gic.
H qu 3.2. Nu A, B, C l ba gc ca mt tam gic tha mn max {A, B, C} <

2
, tc l
tam gic ABC nhn, th A
1
, B
1
, C
1
xc nh nh sau
A
1
= 2A, B
1
= 2B, C
1
= 2C
cng l ba gc ca mt tam gic.
H qu 3.3. Nu A, B, C l ba gc ca mt tam gic, th A
2
, B
2
, C
2
xc nh nh sau
A
2
=
A
2
, B
2
=
B
2
, C
2
=
+C
2
cng l ba gc ca mt tam gic, trong C
2
l gc t.
H qu 3.4. Nu A, B, C l ba gc ca mt tam gic, trong C l gc t, th A
2
, B
2
, C
2
xc nh nh sau
A
2
= 2A, B
2
= 2B, C
2
= 2 C
cng l ba gc ca mt tam gic.
H qu 3.5. Nu tam gic ABC c ba gc nhn (hoc vung ti C), th A
3
, B
3
, C
3
xc nh
nh sau
A
3
=

2
A, B
3
=

2
B, C
3
= C,
cng l ba gc ca mt tam gic t (hoc vung ti C
3
).
H qu 3.6. Nu tam gic ABC c gc C t (hoc vung), th A
3
, B
3
, C
3
xc nh nh sau
A
3
=

2
A, B
3
=

2
B, C
3
= C,
cng l ba gc ca mt tam gic nhn (hoc vung ti C
3
).
By gi, m rng mt cch t nhin cc bi ton trn, ta c cc bi ton sau
Bi ton 3.3. Tm tt c cc hm s f, g, h xc nh v lin tc trn R tha mn iu kin
sau: Nu A
i
R,
n

i=1
A
i
= (n 2) , th
n

i=1
A

i
= (n 2) , trong A

i
= f (A
i
).
Gii. Tng t cch gii Bi ton 3.1, trong phng trnh hm cm sinh chnh l Phng
trnh hm Perxider tng qut. Cc hm s tm c l
f
i
(x) = k
0
x +k
i
(n 2) (i = 1, ..., n, n 3) ,
20
trong
n

j=0
k
j
= 1.
Tng t, m rng Bi ton 3.2, ta thu c
Bi ton 3.4. Tm tt c cc hm s f
i
(i = 1, ..., n, n 3) xc nh v lin tc trn R tha
mn iu kin sau: Nu 0 A
i
2,
n

i=1
A
i
= (n 2) , th 0 A

i
2,
n

i=1
A

i
= (n 2) ,
trong A

i
= f (A
i
).
Gii. Tng t cch gii Bi ton 3.2, trong phng trnh hm cm sinh chnh l Phng
trnh hm Perxider tng qut. Cc hm s tm c l
f
i
(x) = k
0
x +k
i
(n 2) (i = 1, ..., n, n 3) ,
trong 0 k
i
(n 2) 2, 0 2k
0
+k
i
(n 2) 2.
Thu hp gi thit ca Bi ton 3.4, ta thu c
Bi ton 3.5. Tm tt c cc hm s f
i
(i = 1, ..., n, n 3) xc nh v lin tc trn R tha
mn iu kin sau: Nu 0 A
i
,
n

i=1
A
i
= (n 2) , th 0 A

i
,
n

i=1
A

i
= (n 2) ,
trong A

i
= f (A
i
).
Gii. Tng t cch gii Bi ton 3.4, trong phng trnh hm cm sinh chnh l Phng
trnh hm Perxider tng qut. Cc hm s tm c l
f
i
(x) = k
0
x +k
i
(n 2) (i = 1, ..., n, n 3) ,
trong 0 k
i
(n 2) 1, 0 k
0
+k
i
(n 2) 1.
T nhng kt qu trn ta thy rng, vi ba gc ca mt tam gic cho trc, c th to
ra c ba gc ca mt tam gic mi v do c th suy ra c nhiu h thc lng gic
lin quan n cc gc ca tam gic . Hn na, bng cch phi hp nhng phng php khc
nhau, ta cn c th to ra c nhiu ng thc v bt ng thc lng gic khc, v cng
phong ph. Sau y l mt vi v d.
Gi s rng, ta chng minh c cc h thc sau y v xem chng l nhng h thc
"gc" ban u
sin A + sin B + sin C
3

3
2
, (15)
cos Acos Bcos C
1
8
(16)
0 < sin Asin Bsin C
3

3
8
, (17)
sin 2A + sin 2B + sin 2C = 4 sin Asin Bsin C. (18)
p dng H qu 3.1 vo (15), ta c
sin
_
A
2
_
+ sin
_
B
2
_
+ sin
_
C
2
_

3
2
.
Nh vy, ta to c bt ng thc sau
Bt ng thc 1. cos
A
2
+ cos
B
2
+ cos
C
2

3

3
2
.
21
p dng H qu 3.1 vo (16), ta c
cos
_
A
2
_
cos
_
B
2
_
cos
_
C
2
_

1
8
.
Nh vy, ta to c bt ng thc sau
Bt ng thc 2. sin
A
2
sin
B
2
sin
C
2

1
8
.
p dng H qu 3.1 vo (18), ta c
sin 2
_
A
2
_
+ sin 2
_
B
2
_
+ sin 2
_
C
2
_
= 4 sin
A
2
sin
B
2
sin
C
2
.
hay
sin ( A) + sin ( B) + sin ( C) = 4 sin
A
2
sin
B
2
sin
C
2
.
Nh vy, ta to c ng thc sau
ng thc 1. sin A + sin B + sin C = 4cos
A
2
cos
B
2
cos
C
2
.
By gi, sng tc thm nhng h thc a dng hn, ta tip tc khai thc nhng kt qu
trn, chng hn t Bt ng thc 2 ta c
8 sin
A
2
sin
B
2
sin
C
2
1 32 sin
A
2
sin
B
2
sin
C
2
.cos
A
2
cos
B
2
cos
C
2
4cos
A
2
cos
B
2
cos
C
2
4
_
2 sin
A
2
cos
A
2
__
2 sin
B
2
cos
B
2
__
2 sin
C
2
cos
C
2
_
4cos
A
2
cos
B
2
cos
C
2
4 sin Asin Bsin C 4cos
A
2
cos
B
2
cos
C
2
. (19)
Nh vy, ta to c bt ng thc sau
Bt ng thc 3. sin Asin Bsin C cos
A
2
cos
B
2
cos
C
2
.
Bi (18) v ng thc 1, t (19), ta c bt ng thc sau
Bt ng thc 4. sin 2A + sin 2B + sin 2C sin A + sin B + sin C.
Ta tip tc khai thc Bt ng thc 4. Nhn xt rng, nu tam gic ABC l tam gic nhn
th, p dng H qu 3.2 vo Bt ng thc 4, ta c
sin 2 ( 2A) + sin 2 ( 2B) + sin 2 ( 2C)
sin ( 2A) + sin ( 2B) + sin ( 2C)
sin 4A sin 4B sin 4C sin 2A + sin 2B + sin 2C.
Nh vy, ta tip tc to c bt ng thc sau
Bt ng thc 5. sin 2A + sin 2B + sin 2C + sin 4A + sin 4B + sin 4C 0.
By gi, p dng H qu 3.3 vo Bt ng thc 4, ta c
sin
_
2.
A
2
_
+ sin
_
2.
B
2
_
+ sin
_
2.
+C
2
_
sin
A
2
+ sin
B
2
+ sin
+C
2
.
22
Ta to c bt ng thc sau
Bt ng thc 6. sin A + sin B sin C sin
A
2
+ sin
B
2
+ cos
C
2
.
By gi, gi s tam gic ABC c gc C t. p dng H qu 3.4 vo Bt ng thc 1, ta c
cos
2A
2
+ cos
2B
2
+ cos
2C
2

3

3
2
.
Ta to c bt ng thc sau
Bt ng thc 7. cos A + cos B + sin C
3

3
2
_
C >

2
_
.
Tip theo, gi s tam gic ABC nhn (hoc vung ti C). p dng H qu 3.5 vo (17), ta
c
0 < sin
_

2
A
_
sin
_

2
B
_
sin ( C)
3

3
8
Ta c bt ng thc sau
Bt ng thc 8. 0 < cos Acos Bsin C
3

3
8
_
C

2
_
.
By gi, gi s tam gic ABC c gc C t (hoc vung). p dng H qu 3.6 vo (15), ta
c
0 < sin
_

2
A
_
+ sin
_

2
B
_
+ sin ( C)
3

3
2
Ta c bt ng thc sau
Bt ng thc 9. 0 < cos A + cos B + sin C
3

3
2
_
C

2
_
.
TI LIU THAM KHO
[1] J. Aczl (1966), Lectures on Functional equations and their applications, Chapter 3, pp.
141-145, Chapter 4, pp. 197-199.
[2] Nguyn Vn Mu, Mt s lp phng trnh hm a n hm dng c bn, K yu Hi
tho khoa hc "Cc chuyn chuyn Ton bi dng hc sinh gii Trung hc ph thng", H
Ni, 2011.
[3] D.S. Mitrinovic, J.E. Pecaric and V. Volenec (1989), Recent advances in geometric inequal-
ities, Mathematics and its applications (East European series), Published by Kluwer Academic
Publishers, the Netherlands, Chapter V, pp. 64-69.
23
XY DNG MT LP PHNG TRNH HM NH
CC HNG NG THC LNG GIC
L Sng, Trng THPT Chuyn L Qu n - Khnh Ha
Trong cc k thi i hc cu hi v phng trnh, bt phng trnh thng c ch ,th trong
cc cu hi ca thi chn hc sinh gii quc gia hay quc t cc bi ton v phng trnh
hm cng chim phn trng tm. Trong bi vit ny chng ti th lin h kin thc v lng
gic hc trong chng trnh ph thng a n mt s bi ton c nghim l hm s
lng gic
1 Cc hm s lng gic trong chng trnh ton v vi
tnh cht
sin(x y) = sin x cos y sin y cos x, x, y R(1)
sin(x + y) sin(x y) = sin
2
x cos
2
y sin
2
y cos
2
x, x, y R(2)
cos(x y) = cos x cos y sin x sin y, x, y R(3)
T (2) a n cng thc ca phng trnh hm n l hm sin
g(x +y)g(x y) = g
2
(x) g
2
(y) vi mi x, y R.
T (3) ta cng t c cng thc ca hm cosin (phng trnh hm dAlembert )
f(x +y) + f(x y) = 2f(x)f(y) vi mi x, y R.
Ngoi ra t mt s cng thc lng gic m ta cng on c nghim
f (2x) = 2f
2
(x) 1, f (3x) = 4f
3
(x) 3f(x),x R.
Quy c: f
n
(x) = [f(x)]
n
.
Bn Phng trnh hm c bn : Trong cc bi ton sau phn nhiu trc khi i n
kt qu thng phi qua trung gian l cc phng trnh hm c bn sau
Cc phng trnh Cauchy
A(x +y) = A(x) + A(y) (I)
E(x +y) = E(x).E(y) (II)
L(xy) = L(x) + L(y) vi x > 0 (III)
F(xy) = F(x).F(y) vi x > 0 (IV )
Ta c ln lt cc nghim l A(x)=ax ,vi a=f(1) c gii bi A.L.cauchy 1821
E(x) = exp(ax) hay E(x) = 0
L(x) = alnx hay L(x) = 0
F(x) = xc hay F(x) = 0
24
2 Phng trnh hm dAlembert Hm cosin
Bi ton 1. Tm cc hm f(x) xc nh v lin tc trn v tha mn cc iu kin
_
f (x + y) + f (x y) = 2f (x) f (y) , x, y R
f (0) = 1, x
0
R : |f (x
0
)| < 1
Li gii. V f(0) = 1 v f(x) lin tc trn R nn > 0 sao cho f (x) > 0, x (, )
Khi theo (2) vi n
0
N ln th
f(
x
0
2
n0
) > 0 f(
x
0
2
n0
) < 1 (do phn chng )
Vy tn ti x
1
= 0, x
1
=
x0
2
n
0
sao cho
0 < f(x
1
) < 1.f(x) > 0, x (|x
1
| , |x
1
|) , f(x
1
) = cos , 0 < <

2
T (1) suy ra
f(2x
1
) = 2f
2
(x
1
) 1 = 2cos
2
1 = cos 2
Gi s f (kx
1
) = cos k, k = 1, 2, ..., n N
+
. Khi
f ((n + 1) x
1
) = f (nx
1
+x
1
)
= 2f (nx
1
) f (x
1
) f ((n 1) x
1
)
= 2 cos ncos cos (n 1)
= cos (n + 1) .
T suy ra f (mx
1
) = cos m, m N
+
v f(x) l hm chn trn R v nh vy
f(mx
1
= cos m, m Z(3)
Do tnh tr mt trong R , f(x) v cos x l cc hm lin tc trn R nn f (x) = cos ax, a R

Th li ta thy f (x) = cos ax (a = 0) tha mn cc iu kin ca bi ton .


Nhn xt 1. Thay trong gi thit |f (x
0
)| < 1 bi ton 1 bi |f (x
0
)| > 1 ta c nghim ca
bi ton l f(x) = cosh(x), y l hm cosin hyprebol m ta khng kho st chng trnh
hc ph thng
Nhn xt 2. Khi f l hm kh vi, ly o hm theo y hai ln , ta c f

(0) = 0, f

(x) =
k.f(x), k hng Nu k = 0 th f(x) = ax +b;
Nu k > 0 th f(x) = c sin bx +d cos bx, c, d hng
T f(0) = 1, f

(0) = 0 suy ra d = 1, bc = 0, b = 0 th f hng; c = 0 th f(x) = cos x


Nu k < 0 th f(x) = c sinh bx +d cosh bx vi b
2
= k, iu kin f(0) = 1, f

(0) = 0.
Vy nghim l f(x) = cos bx, b s thc.
nh l 1. (nh l nghim ca Phng trnh dAlembert)
Cho f : R R hm lin tc v tha mn iu kin
(C) f(x +y) + f(x y) = 2f(x)f(y), vi mi x, y
th f(x) = 0, f(x) = 1, f(x) = cos ax, hay f(x) = coshbx, a, b R.
25
Bi ton 2. (IMO1972). Tm f : R R lin tc v tha mn cc iu kin
f(x + y) + f(x y) = 2f(x)f(y), vi mi x, y
Chng minh rng nu f(x) = 0, |f(x)| 1, x R th |g(y)| 1, y R
Li gii. Do f b chn
f(x) = 0 |f(x)| |g(y)| = 2 |f(x +y) + f(x y)| |f(x + y)| +|f(x y)| 2M, x R
Bi ton 3. Tm tt c cc hm lin tc D R tha
f(x +y) =
f(x) + f(y)
1 f(x)f(y)
Li gii. Ta bit rng hm f(x) = tan x tha bi.
V th nu t A(x) = arctanf(x) th A(x + y) = A(x) + A(y) 2k .
Suy ra A(x) = kx mod 2 , t dn n f(x) = tan kcx.
Bi ton 4. Tm cc hm f(x) xc nh v lin tc trn R v tha mn cc iu kin
f (x) + f (y) = f
_
x + y
1 xy
_
, x, y : x +y > 0
Li gii. t x = cotgu, y = cotgv, 0 < u, v <
Th
x+y
1xy
= cotg (u + v)
Hay A(u + v) = A(u) + A(v) , 0 < u, v < trong A(u) = f (cotgu) .
f (x) = karcctgx, k , x R (i)
Th li ta thy hm f(x) xc nh theo (i) tha mn cc iu kin ca bi ton
Kt lun f (x) = karcctgx, k R, x R
Bi ton 5. Tm cc cp hm f(x) v g(x) xc nh v lin tc trn R v tha mn iu kin
f(x y) = f(x)g(y) + f(y)g(x)
Li gii. Nghim ca bi ton l
_
f(x) = c
g(x) =

1 c
2
hay
_
f(x) = cos kx
g(x) = sin kx
, k R

26
Bi ton 6. (Putnam1991)
Cho hai hm f : R R, g : R R, f(x), g(x) khc hng, kh vi v tha mn iu kin
_
f(x + y) = f(x)f(y) g(x)g(y)
g(x + y) = f(x)g(y) + g(x)f(y), x, y R
v f

(0) = 0 .Chng minh rng f


2
(x) + g
2
(x) = 1
Li gii. Ta ch cn chng minh rng H(x) = f
2
(x) + g
2
(x) l hng
Tht vy ly o hm theo y ri thay y = 0
ta c f

(x) = g

(0)g(x) v g

(x) = g

(0)f(x)
Do 2f(x)f

(x) + 2g(x)g

(x) = 0 suy ra f
2
(x) + g
2
(x) = C
Ngoi ra f
2
(x +y) + g
2
(x +y) = (f
2
(x) + g
2
(x))(f
2
(y) + g
2
(y)) ,nn C2 = C.
Nhng C = 0,nn C = 1
Nhn xt.T gi thit ca bi ton ta thy hai hm f(x) = cos x, g(x) = sin x l nghim ca
bi ton, nn ta t E(x) = f(x) + ig(x) ,t gi thit bi ton ta c E(x +y) = E(x)E(y)
V vy ta c 1 cch gii khc nh sau
Do E hm kh vi nn E

(0) = ib, E(x + y) = E(x)E(y) Ly o hm y ,ri cho y = 0 ta c


E(x) = Ce
ibx
,
T E(0 + 0) = E(0)E(0), ta rt ra c C = 1.
Cui cng f2(x) + g2(x) = |E(x)| =

e
ibx

2
= 1
Bi ton 7. a.Tm cc cp hm f : R R, g : R R lin tc v tha mn iu kin
_
f(x y) = f(x)g(y) f(y)g(x)
g(x y) = g(x)g(y) + f(x)f(y), x, y R
p s. Nghim ca bi ton l f(x) = sin bx, g(x) = cos bx, b s thc hay f(x) = g(x) = 0
b.
_
f(x +y) = f(x)g(y) + f(y)g(x)
g(x +y) = g(x)g(y) f(x)f(y), x, y R
p s . Nghim l f(x) = a
x/2
sin bx, g(x) = a
x/2
cos bx, a > 0, b s thc hay f(x) =
g(x) = 0.
Bi ton 8. Tm cc cp hm f(x) v g(x) xc nh v lin tc trn R v tha mn
_
[f(x) + g(x)]
2
= 1 +f(2x), f(0) = 0
f(x + y) + f(x y) = 2f(x)g(y), x, y R
p s . Nghim ca bi ton l f(x) = sin bx, g(x) = cos bx, bs thc hay f(x) = g(x) = 0
3 Phng trnh hm c cha hm s lng gic
3.1 Trc ht xt hm thc f xc nh vi mi x,y thuc R tha
f(x + y) + f(x y) = 2cosxcosy, (3.1)
27
Kt qu 1. f(x) l nghim ca 4.1 khi v ch khi f(x) = cos x
Kt qu 2. Phng trnh f(x + y) + f(x y) = 2 sin x sin y , khng c nghim
Kt qu 3. Phng trnh f(x +y) + g(x y) = 2 sin x sin y
c nghim f(x) = c cos x, g(x) = cos x c
Do cng thc bin i 2 sin x sin y = cos(x y) cos(x +y) suy ra f(u) + cos u = cos v g(v)
Kt qu 4. Phng trnh f(x +y) f(x y) = 2 sin x sin y c nghim
f(x) = c cos x, c l hng
Kt qu 5. Phng trnh f(x +y) + f(x y) = 2 cos x sin y khng c nghim
Kt qu 6. Phng trnh f(x +y) + g(x y) = 2 cos x sin y,
c nghim f(x) = c + sin x, g(x) = sin x c
Kt qu 7. Phng trnh f(x +y) + f(x y) = 2 sin x cos y ,c nghim f(x) = sin x
3.2 Xt hai hm thc f ,g xc nh trn R tha
f(x +y) + g(x y) = 2 sin x cos y, (3.2)
Kt qu 8. Nghim ca (3.2) l f(x) = c + sinx, g(x) = sinx c
Cho y = 0, ri thay y = y vo (3.2)
ta c f(x) + g(x) = 2 sin x v f(x +y) + g(x y) f(x y) + g(x +y) = 0
3.3 Cho hm thc f(x) tha f(x + y) + f(x y) = 2f(x) cos y (3.3) .
Khi hm g(x) = f(x) d cos x tha g(x +y) = g(x) cos y +g(y) cos x (3.4)
Kt qu 9. Nghim tng qut ca (3.4) l g(x) = b sin x, do f(x) = b sin x +d cos x
Tht vy, cho x = 0, ri hon i x, y trong (3.3) ta c f(y) + f(y) = 2d cos y v 2f(x +
y) + f(x y) + f(y x) = 2f(x) cos y + 2f(y) cos x
Tc l 2f(x +y) + 2d cos(x y) = 2f(x) cos y + 2f(y) cos x. Suy ra (3.4)
c nghim (3.4) , dng tnh kt hp ca hm g(x)
g(x +y +z) = (g(x) cos y +g(y)) cos x +g(z) cos(x +y)
= g(x) cos(y + z) + (g(y) cos z +g(z) cos y) cos x
Suy ra g(x) sin y sin z = g(z) sin y sin x, mi x, y, z
C nh y, z vi sin z = 0, ta c g(x) = b sin x
3.4. Cho hai hm f : R R, g : R R tha
f(x y) f(x +y) = 2g(x) sin y, vi mi s thc x, y(3.5)
khi v ch khi f(x) = a cos x d sin x + c, g(x) = a sin x +d cos x
Li gii. Cho x = 0, ri hon i x, y trong (3.5) ta c f(y) f(y) = 2d sin y v
f(x y) f(y x) = 2g(x) sin y 2g(y) sin x
Tc l 2d sin(x y) = 2g(x) sin y 2g(y) sin x.
Suy ra (g(x) d cos x) sin y = (g(y) d cos y) sin x, tc l g(x) = d cos x + a sin x (c nh y).
Thay g(x) vo (3.5) ta c kt qu
3.5.Lp lun tng t ta cng c vi phng trnh hm
sin (x +y) = f (x) sin y +f (y) sin x (3.6)
sin (x +y) = g (x) sin y +g (y) sin x (3.7)
f (x +y) = g (x) sin y +g (y) sin x (3.8)
28
Kt qu 10. Nghim (3.6 ) l f(x) = cos x ,nghim ca (3.7 ) l f(x) = cos x + d sin x, g(x) =
cos x b sin x , nghim ca(4.8) l f(x) = a cos x = g(x)
3.6. Dng tnh kt hp ca hm
Xt f(x +y)f(x y) = sin
2
(x) sin
2
(y), (3.9)
Kt qu 11. (4.9) c nghim l f(x) = k sin x, k
2
= 1
Bi ton 9. Nghim ca f(x +y)f(x y) = f
2
(x) sin
2
(y), (3.10)
L f(x) = a sin x hay f(x) = b cos x +d sin x, a, b, d hng tha a
2
= 1 = b
2
+ d
2
Li gii. Cho x = y trong (4.10).Xt 2 trng hp f(0) = 0 hay khc khng
Trng hp khc khng ta a v dng f(u)f(v) = bf(u + v) + sin usin v.
Sau xt bf(u + v +z) = f(u)(f(v)f(z) sin v sin z) sin u(sin v cos z + cos v sin z)
Tnh cht 1. Cho hm f (x)xc nh v lin tc trn R v tha mn iu kin
(S

)f(x +y + 2d) + f(x y + 2d) = 2f(x)f(y), vi mi x, y, d hng khc khng


th f l hm s l
Tnh cht 2.
(i) Nu f(0) = 1 v f(d) = 1 th hm f c chu k l d
(ii) Nu f(0) = 1 v f(d) = 1 th hm f c chu k l 2d
(iii) Nu f(0) = 1 th f(d) = 0 hm f c chu k l d
Bi ton 10. Tm hm f(x) tha mn (S*)
Li gii. Trng hp (i) v (ii) th f tha phng trnh dAlembert ca hm (C)
Trng hp (iii) th g(x) = f(x + 2d) l nghim ca phng trnh (C)
g(x + y) + g(x y) = 2g(x)g(y), g(x) c chu k 4d
Kt qu. Nghim ca bi ton (S*) l f(x) = cos
2nx
d
hayf(x) = cos
(2n+1)x
d
.
4 Phng trnh hm sin (S)f(x +y)f(x y) = f
2
(x) f
2
(y),
vi mi x, y
Tnh cht 3. Hm f khc khng, tha (S) l hm l
Bi ton 11. Cho f : R R lin tc tha (S)
Th f(x) = cx , f(x) = c sin bxhay f(x) = sinh bx
Li gii. Do f lin tc, kh vi. Ly o hm ln th nht theo y, ln th hai theo x
Suy ra f

(x) = kf(x) . Nh vy ta c kt qu
Bi ton 12. (Corovei) Cho hai hm f : R R, g : R R khc khng tha
f(x)g(y x) = f
_
y
2
_
g
_
y
2
_
f
_
x
y
2
_
g
_
x
y
2
_
, x, y R()
Khi g l nghim ca (S) v
g(x) = A(x), f(x) = c+dA(x)(1), g(x) = b(E(x)E(x)), f(x) = c(E(x)E(x))+dE(x)(2)
trong A, E tha phng trnh c bn, b v c l cc hng s.
29
Li gii. Thay x bi x + y, v y bi 2x vo (*) f(x + y)g(x y) = f(x)g(x) f(y)g(y)(3),
sau i ch x v y vi nhau ta c f(x +y)g(y x) = f(x +y)g(x y),
suy ra g(x) hm l ( do f khc khng)
Ly y = x ri thay y bi y trong (1) , tr (*) cho (3)
f(x y)g(x + y) f(x +y)g(x y) = g(y)(f(y) + f(y))
Xt 2 trng hp f(0), f(0) = 0, th f v g trong (1) l nghim; f(0) khc khng, f v g trong
(2) l nghim
Kt qu Trng hp hm lin tc, nghim khc khng l
f(x) = bx +c, g(x) = ax;
f(x) = c sin ax + d cos ax, g(x) = b sin ax;
f(x) = b sinh ax +d cosh ax, g(x) = b sinh ax.
5 M rng phng trnh hm dAlembert dng lng
gic ca W. H. Wilson1919
f(x +y) + f(x y) = 2f(x)g(y), (1)vi mi x, y
f(x +y) + g(x y) = h(x)k(y), (2) vi mi x, y
Nhn xt khi f(x) = 0 th g(x) l mt hm ty Nn ta xt f(x) = 0 ,nn c a sao cho f(a) = 0
Trong (1) thay x = a, y = y ta c g(x) l hm s chn. Nh phng php tch f thnh 2
hm chn f
1
v hm l f
2
Wilson thu gn c f
1
(x) = kg(x),vi k = f
1
(0) v g tha mn
hm dAlembert
nh l 2. (nh l Wilson.) Nghim tng qut ca phng trnh(1) l
f(x) = 0, g(x) ty , hay
f(x) = k cos bx +C sin bx v g(x) = cos bx, hay
f(x) = k cosh bx + C sinh bx v g(x) = cosh bx, hay
f(x) = k +Cx v g(x) = 1
Nhn xt. Trong phm vi ca bi ny chng ti ch nhm xt phng trnh (1) c nghim
dng lng gic, dng (2) c xt tng qut trong bi khc
Bi ton 13. Nu hm f tha f(x y) = f(x)f(y) +g(x)g(y), x, y R.th cng tha phng
trnh dAlembert f(x +y) + f(x y) = 2f(x) cos y, x, y R.
Li gii. Do bi ton 3.5
Bi ton 14. ( Bnh nh 2009 )Tm tt c cc hm s f xc nh trn R tha
f(x +y) + f(x y) = 2f(x) cos y, x, y R
.
30
6 M rng cc phng trnh hm c bn dng
f[G(x, y)] = F(f(x), f(y))
Trong bi vit ny, chng ti ch xt trng hp c bit G(x,y)=x+y, tc l dng phng
trnh hm f(x +y) = F(f(x), f(y)) , x, y R
Tnh cht 4. F c tnh kt hp tc l F[F[u, v], w] = F[u, F[v, w]] = f(x + y +w)
Trng hp
F(u, v) = Auv + Bu + Bv + C, F l a thc i xng, do tnh kt hp ta c AC = B
2
B
Khi A = 0 th B = 1 bi ton c dng f(x +y) = f(x) + f(y) + C
t A(x) = f(x) + C, bi ton c a v A(x +y) = A(x) + A(y) (phng trnh Cauchy I)
Do f(x) = A(x) C. Khi A=0 , bi ton c dng
f(x + y) = Af(x)f(y) + Bf(x) + Bf(y) +
B
2
B
A
=
[Af(x) + B] [Af(y) + B] B
A
t E(x) = Af(x) +B, bi ton c a v E(x +y) = E(x).E(y) (phng trnh Cauchy II)
Do f(x) =
E(x)B
A
Mt s bi ton lin quan
Cc nghim tng qut c a ra y trong lp hm lng gic c xc nh trong cc
khong hm s lin tc v n iu trong khong xc nh
6.1 f
1
(x +y) =
f
1
(x) + f
1
(y)
1 f
1
(x)f
1
(y)
,
6.2 f
2
(x +y) =
f
2
(x)f
2
(y) 1
f
2
(x) + f
2
(y)
,
6.3 f
3
(x +y) =
f
3
(x) + f
3
(y) 2f
3
(x)f
3
(y)
1 f
3
(x)f
3
(y)
,
6.4 f
4
(x +y) =
f
4
(x) + f
4
(y) 1
2f
4
(x) + 2f
4
(y) 2f
4
(x)f
4
(y) 1
,
6.5 f
5
(x +y) =
f
5
(x) + f
5
(y) 2f
5
(x)f
5
(y) cos a
1 f
5
(x)f
5
(y)
,
6.6 f
6
(x +y) = f
6
(x)f
6
(y)
_
1 f
2
(x)
_
1 f
2
(y)
C cc nghim ln lt l
f
1
(x) = tan kx, f
2
(x) = cot kx, f
3
(x)
=
1
1 + cot kx
, f
4
(x) =
1
1 + tan kx
, f
5
(x) =
sin kx
sin(kx +a)
, f
6
(x) = cos kx.
31
7 Lng gic ha bi ton phng trnh hm
Bi ton 15. (Putnam2000) Tm hm f(x) xc nh v lin tc trong [1, 1] v tha mn
f 2x
2
1
_
= 2xf (x) , x [1.1]
Li gii. Ta c x = 1, x =
1
2
tha mn 2x
2
1 = x suy ra f(1) = f(
1
2
) = 0
Nn f cos
2
3
+ 2n
__
= 0.T f(cos 2a) = 0 ta suy ra f(cos a) = 0
t x = cos a, Ta c f cos 2
k 2
3
+ 2n
__
= 0, n Z, k N
Hn na, tp cc s 2
k
(
2
3
+2n) tr mt trong R v f(cos x) lin tc suy ra f(cos r) = 0,mi
r
Bi ton 16. Tm hm f(x),chn, lin tc trong ln cn im O xc nh
f (x) 0 khi 0 x /2, f (x) 0 khi /2 x ,
v tha (*) f (2x) = 1 2f
2
2
- x
_
,x R, th f(x) = cos x, mi x.
Bi ton 17. Tm cc hm f (x) xc nh v lin tc trn [1, 1] v tha mn cc iu kin
f (x) + f (y) = f
_
x
_
1 y
2
+y
_
1 x
2
_
, x, y [1, 1] (18)
Li gii. t x = sin u, y = sin v, u, v
_

2
,

2

Th x
_
1 y
2
+y

1 x
2
= sin (u +v)
Khi c th vit (18) di dng
g (u +v) = g (u) + g (v) , u, v
_

2
,

2
_
vi g(u) = f(sin u)
Suy ra f (x) = aarc sin x, x [1, 1] , a R (i)
Th li ta thy hm f (x) xc nh theo ( i ) tha mn cc iu kin ca bi ton .
Bi ton 18. Tm cc hm s f (x) xc nh v lin tc trn [1, 1] v tha mn iu kin
f
_
xy
_
1 y
2
_
1 x
2
_
= f (x) + f (y) , x, y [1, 1]
Li gii. t x = cos u, y = cos v, u, v [0, ] .Khi sin u 0, sin v 0
xy
_
1 y
2
_
1 x
2
= cos (u +v) , u, v [0, ]
Phng trnh hm cho c th vit di dng
f (cos u) + f (cos v) = f (cos (u +v)) , u, v [0, ]
t f (cos u) = g (u) ta c
g (u +v) = g (u) + g (v) , u, v [0, ]
Do vy, g (u) = au, a = const , f (x) = aarccos x
Th li, ta thy hm s ny tha mn bi ton .
32
Bi ton 19. Tm cc hm f (x) xc nh v lin tc trn R v tha mn cc iu kin
f (x) + f (y) = f
_
x + y
1 - xy
_
, x,y R : |xy| < 1(17)
Li gii. t x = tgu, y = tgv,

2
< u, v <

2
. Do |xy| < 1 nn ta c
x+y
1xy
= tg (u +v)
Vy

2
< u +v <

2
Khi A(u +v) = A(u) + A(v) ,trong A(u) = f (tgu) ,

2
< u, v <

2
Vy f (x) = aarctgx, a R, x R (ii) Th li ta thy hm f (x) xc nh theo (ii) tha mn
cc iu kin ca bi ton . Kt lun f (x) = aarctgx, a R, x R
Bi ton 20. Tm tt c cc hm s f (x) lin tc trn R v
f (x) f (y) - f (x + y) = s in x sin y,x,y R()
Li gii. Vi x = y = 0,ta thu c: f
2
(0) f (0) = 0 Suy ra f(0) = 0 hay f(0) = 1
-Nu f (0) = 0 th vi y = 0 ta c f (x) = 0 T f (x) 0, x
Nhng nu thay x = y =

2
ta thy mu thun.
-Nu f (0) = 1 thay y = x ta c
f (x) f (x) = 1 sin
2
x = cos
2
x
Thay x =

2
ta c f

2
_
f

2
_
= 0
+Nu f

2
_
= 0, thay y =

2
, Ta c f x +

2
_
= sin x = cos x +

2
_
+Nu f

2
_
= 0 thay y =

2
,
f
_
x

2
_
= sin x = cos
_
x

2
_
Vy f(x) = cos x. Th li thy ng.
Bi ton 21. (Russia 2000): Tm tt c cc hm s f (x)xc nh trn R v
|f (x + y) + sinx + siny| < 2,x,y R
Li gii. Ln lt thay (x, y) = (

2
,

2
), (

2
,
3
2
),ri p dng bt ng thc tam gic ta c kt
qu
Bi ton 22. (Turkey 2000): Cho f : R R tha
|f (x +y) f(x)f(y)| 1, x, y R
Chng minh rng tn ti hm g(x) xc nh trn R sao cho
|f (x) g (x)| 1, x R
v g(x +y) = g(x) + g(y) mi x, y
33
p s. g(x) = lim
x
f(2
n
x)
2
n
Bi ton 23. (Indian TST2004) Tm tt c cc hm s f (x) xc nh trn R v
f(x +y) = f(x)f(y) c sin x sin y, x, y R c hng c > 1
p s. f (x) =

c 1sin x + cos x,x R


Bi ton 24. (Thi lan 2007) Tm cc hm xc inh trn R tha
f(x + cos(2007y)) = f(x) + 2007 cos(f(y)).
Li gii. t A(x) = f(x) d, d = f(0), c = A(1) = f(1) A(0), A(0) = 0
Ta c: A(x + cos 2007y) = A(x) + 2007 cos(A(y) + d).
ta c :A(cos 2007y) = 2007 cos(A(y)+d) 2007 suy ra A(y) 2007, y [1, 1]v A(x+y) =
A(x) + A(y) , vi mi y trong on [-1,1].
Bng quy np ta c A(x + ny) = A(x) + nA(y), v A(ny) = A(0) + nA(y) = A(0).
Suy ra A tha mn PTH Cauchy v A(x) = cx v f(x) = cx +d, mi x.
Ti liu tham kho
[1] J. Aczel,1966, Lectures on Functional equations and their applications, Academic press.
[2] Nguyn Vn Mu,1997, Phng trnh hm, NXB Gio Dc.
[3] Christopher G. Small,2000, Functional equations and how to solve them, Springer.
[4] Razvan Gelca and Titu Andreescu,2007,Putnam and Beyond,Springer.
[5] PI. Kannappan,2009 Functional equations and inqualities with application,Springer.
34
TNH N NH NGHIM CA MT S PHNG
TRNH HM CAUCHY
L Th Anh oan, THPT Chuyn L Qu n -Khnh Ha
Trong ni dung bi vit ny, ta cn quan tm n khi nim n nh. Chng hn ta ni phng
trnh hm (Cauchy) nhn tnh l n nh nu n tha mn tnh cht sau:
Gi s G l mt nhm, H(d) l mt nhm metric v f : G H, vi mi > 0 th tn ti
> 0 sao cho
d (f(xy), f(x)f(y)) < , x, y G
v do , tn ti mt ng cu M : G H sao cho
d (f(x), M(x)) < , x G.
1 Tnh n nh ca phng trnh hm (Cauchy) cng
tnh
Trc ht ta nhc li phng trnh hm (Cauchy) cng tnh (A)
f(x +y) = f(x) + f(y) (A)
Gi s hm f : X Y tha mn (A), vi X v Y l hai khng gian Banach. Khi f c
gi l hm cng tnh.
nh l 1. Gi s , hm f : X Y tha mn, vi mi > 0, ta c
f(x +y) f(x) f(y) , x, y X. (1)
Khi tn ti gii hn sau
A(x) = lim
n
2
n
f(2
n
x) (2)
vi mi x X v tn ti duy nht hm cng tnh A : X Y tha mn
f(x) A(x) , x X. (3)
Chng minh. Thay x = y vo (1) ta c

_
1
2
_
f(2x) f(x)
_
1
2
_
. (4)
S dng phng php quy np, ta c
2
n
f(2
n
x) f(x) (1 2
n
). (5)
35
Tht vy, trong (4) ta thay x bi 2x, ta c

1
2
f(2
2
x) f(2x)
1
2
.
Khi
[
1
2
f(2
2
x) 2f(x)] [f(2x) 2f(x)] =
1
2
f(2
2
x) f(2x)
1
2

hay

1
2
2
f(2
2
x) f(x)
1
2
f(2x) f(x)
1
2
2
,
nn

1
2
2
f(2
2
x) f(x)
_
1
2
+
1
2
2
_
,
do

1
2
n
f(2
n
x) f(x)
_
1
2
+
1
2
2
+ +
1
2
n
_
=
_
1
1
2
n
_
.
By gi ta s chng minh dy {
1
2
n
f(2
n
x)} l dy Cauchy vi mi x X. Chn m > n, khi

1
2
n
f(2
n
x)
1
2
m
f(2
m
x) =
1
2
n

1
2
mn
f(2
mn
.2
n
x) f(2
n
x)

1
2
n

_
1
1
2
mn
_
=
_
1
2
n

1
2
m
_
.
Do dy {
1
2
n
f(2
n
x)} l dy Cauchy v do Y khng gian Banach nn tn ti A : X Y sao
cho A(x) := lim
n
2
n
f(2
n
x) vi mi x X, hay
A(x)
1
2
n
f(2
n
x)
1
2
n
.
Tip theo ta cn chng minh A l hm cng tnh. Thay x, y bi 2
n
x v 2
n
y trong (1), ta c

1
2
n
f(2
n
(x +y))
1
2
n
f(2
n
x)
1
2
n
f(2
n
y)
1
2
n

vi n Z

+
, x, y X. Cho n , ta c
A(x +y) A(x) A(y) .
Vi mi x X, ta c
f(x) A(x) = [f(x)
1
2
n
f(2
n
x)] + [
1
2
n
f(2
n
x) A(x)]
f(x)
1
2
n
f(2
n
x) +
1
2
n
f(2
n
x) A(x)]

_
1
1
2
n
_
+
1
2
n
= .
36
Cui cng, ta cn chng minh A duy nht. Gi s tn ti mt hm cng tnh A
1
: X Y tha
mn (3). Khi , vi mi x X,
A(x) A
1
(x) =
1
n
[A(nx) f(nx)] + [A
1
(nx) f(nx)]

2
n
theo (3)
Vy A
1
= A.
nh l 2. Vi mi dy s thc bt k a
n
tha mn
|a
n+m
a
n
a
m
| < 1, n, m Z

+
, (6)
th tn ti gii hn hu hn
A := lim
n
a
n
n
v
a
n
nA < 1, n Z

+
.
Chng minh. p dng nh l 1 cho Y = R. C nh x X v t a
n
:= (
1

)f(nx), n Z

+
.
Khi , theo (1) dy (a
n
) tha mn (6). t
A := lim
k
_
a
k
k
_
=
1

lim
k
_
f(kx)
k
_
,
Theo nh l 1, ta c
|
1

f(nx) nA(x)| < 1,


vi mi n Z

+
v mi x X hay
|f(x) nA
_
x
n
_
| < .
V theo (1), vi mi x, y X ta c
|A(x +y) A(x) A(y)| = lim
n


f(nx +ny)
n

f(nx)
n

f(ny)
n


lim
n

n
= 0,
Do
|f(x) A(x)| < , x X.
37
2 Tnh n nh ca phng trnh hm (Cauchy) nhn
tnh
Trong phn ny nghin cu phng trnh
f(xy) = f(x)f(y) (M)
Gi s hm f : X Y tha mn (M), vi X v Y l hai khng gian Banach. Khi f c
gi l hm nhn tnh.
nh l 3. Gi s > 0, S l mt na nhm v f : S C sao cho
|f(xy) f(x)f(y)| , x, y S. (1)
Khi
|f(x)|
1 +

1 + 4
2
=: , x S. (2)
hoc f l hm nhn tnh vi mi x, y S.
Chng minh. Trong (2), ta c
1+

1+4
2
=: hay
2
= v > 1. Gi s (2) khng xy ra,
tc l tn ti a S sao cho |f(a)| > , hay |f(a)| = + , vi > 0 no . Trong (1), chn
x = y = a, ta c
|f(a
2
) f(a)
2
| (3)
Khi
|f(a
2
)| = |f(a)
2
(f(a)
2
f(a
2
))|
|f(a)
2
| |f(a)
2
f(a
2
)|
|f(a)|
2
theo (3)
= ( + )
2

= ( +) + (2 1) +
2
(do
2
= )
> + 2 (do > 1)
Bng php chng minh quy np, ta c
|f(a
2
n
)| > + (n + 1), n = 1, 2, . . . .
Vi mi x, y, z S,
|f(xyz) f(xy)f(z)| , v |f(xyz) f(x)f(yz)|
Ta c
|f(xy)f(z) f(x)f(yz)| |f(xyz) f(xy)f(z)| +|f(xyz) f(x)f(yz)|
2
38
v
|f(xy)f(z) f(x)f(y)f(z)| |f(xy)f(z) f(x)f(yz)|
+|f(x)f(yz) f(x)f(y)f(z)|
2 +|f(x)|
Suy ra
|f(xy) f(x)f(y)|.|f(z)| 2 +|f(x)|.
Chn z = a
2
n
, ta c
|f(xy) f(x)f(y)|
2 +|f(x)|
|f(a
2
n
)|
.
vi mi x, y S v mi n = 1, 2 . . . . Cho n , ta c f(xy) = f(x)f(y), x, y S. Vy f
l mt hm nhn tnh.
3 Cc v d p dng
V d 1. Nghim ca phng trnh Jensen.
Bi ton 1. Tm hm f : R R tha mn phng trnh sau
f
_
x +y
2
_
=
f(x) + f(y)
2
x, y R (1)
Thay y = 0 vo (1), ta c
f
_
x
2
_
=
f(x) + f(0)
2
x R (2)
Khi p dng (1) v (2), ta c
f(x) + f(y)
2
= f
_
x + y
2
_
=
f(x + y) + f(0)
2
hay
f(x) + f(y) = f(x +y) + f(0), x, y R.
t A(x) = f(x) f(0). Ta c A(x) + A(y) = A(x + y), x, y R. Vy A l mt hm cng
tnh trn R nn f(x) = A(x) + , trong = f(0).
Ch . Nu bi ton c thm gi thit: hm f lin tc th nghim tm c s l f(x) = ax+,
vi a, l cc hng s ty .
Tip theo ta xt tnh n nh nghim ca phng trnh (1).
Mnh 1. Gi s hm f tha mn


f
_
x +y
2
_

f(x) + f(y)
2


(3)
vi l s dng ty cho trc v vi mi x, y R. Khi tn ti duy nht mt hm cng
tnh A : R R sao cho
|f(x) A(x) f(0)| 4, x R.
39
Chng minh. Thay y = 0 vo (3), ta c


f
_
x
2
_

f(x) + f(0)
2


x R.
Do

f
_
x + y
2
_

f(x +y) + f(0)


2


x, y R.
Ta c


f(x) + f(y)
2

f(x +y) + f(0)
2



f(x) + f(y)
2

f(x +y)
2


+


f(x +y)
2

f(x +y) + f(0)
2


2
hay
|f(x +y) + f(0) f(x) f(y)| 4. (4)
t g(x) = f(x) f(0). Thay vo (4), ta c
|g(x +y) g(x) g(y)| 4
Theo tnh n nh ca hm cng tnh, tn ti duy nht hm cng tnh A sao cho
|g(x) A(x)| 4.
Ta c
|f(x) A(x) f(0)| = |g(x) A(x)| 4.
V d 2. Nghim ca phng trnh Cauchy hai n hm.
Bi ton 2. Tm cp hm f, g : R R tha mn phng trnh sau
f(x +y) = g(x) + g(y) x, y R (5)
Thay y = 0 vo (5), ta c
f(x) = g(x) + g(0) x R,
hay f(x) = g(x) + , vi = g(0). Do g(x) = f(x) vi mi x R.
Thay vo phng trnh (5), ta c
f(x +y) = f(x) + f(y) 2 (6)
t f(x) = A(x) + 2. Phng trnh (6) tr thnh
A(x + y) + 2 = A(x) + 2 +A(y) + 2 2
40
hay
A(x + y) = A(x) + A(y) x, y R.
Vy A l mt hm cng tnh trn R nn
_
f(x) = A(x) + 2
g(x) = A(x) + .
Ch . Nu bi ton c thm gi thit: hm f, g lin tc th nghim tm c s l
_
f(x) = ax + 2
g(x) = ax +
vi a, l cc hng s ty .
Tip theo ta xt tnh n nh nghim ca phng trnh (5).
Mnh 2. Gi s hm f, g : R R tha mn
|f(x + y) g(x) g(y)| (7)
vi l s dng ty cho trc v vi mi x, y R. Khi tn ti duy nht mt hm cng
tnh A : R R sao cho
_
|f(x) A(x) f(0)| 4
|g(x) A(x) g(0)| 3
vi mi x R.
Chng minh. Thay y = 0 vo (7), ta c
|f(x) g(x) g(0)| , x R, (8)
suy ra
|f(0) 2g(0)| . (9)
S dng (8), ta c
|f(x +y) g(x +y) g(0)| , x, y R. (10)
Ta c
|f(x +y) g(x + y) g(0)| = |f(x +y) g(x) g(y) g(x +y) + g(x) + g(y) g(0)|
nn kt hp (7) v (10) thu c
|g(x +y) g(x) g(y) + g(0)| |f(x +y) g(x + y) g(0)| +|f(x +y) g(x) g(y)|
2
hay
|[g(x +y) g(0)] [g(x) g(0)] [g(y) g(0)]| 2, (11)
41
vi mi x, y R. t
G(x) = g(x) g(0), (12)
vi mi x, y R. Th vo (11) ta c
|G(x + y) G(x) G(y)| 2, x, y R.
Theo nh l v tnh n nh ca hm cng tnh, tn ti duy nht mt hm cng tnh A : R R
sao cho
|G(x) A(x)| 2, x R. (13)
T (12) v (13) ta c
|g(x) A(x) g(0)| 2, x R. (14)
T (8), (9) v (14) ta c
|f(x) A(x) f(0)| = |f(x) g(x) g(0) + g(x) A(x) g(0) + 2g(0) f(0)|
|f(x) g(x) g(0)| +|g(x) A(x) g(0)| +|f(0) 2g(0)|
+ 2 + = 4.
V d 3. Nghim ca phng trnh Pexider.
Bi ton 3. Tm tt c cc hm f, g, h : R R tha mn phng trnh sau
f(x +y) = g(x) + h(y) x, y R (15)
Thay y = 0 vo (15), ta c
f(x) = g(x) + h(0) x R,
hay f(x) = g(x) + , vi = h(0). Do g(x) = f(x) vi mi x R.
Thay x = 0 vo (15), ta c f(y) = h(x) + , vi = g(0), hay h(x) = f(x) vi mi
x R.
Phng trnh (15)tr thnh
f(x +y) = f(x) + f(y) , x, y R. (16)
t f(x) = A(x) + + thay vo phng trnh (16), ta c
A(x +y) + + = A(x) + + +A(y) + +
hay
A(x +y) = A(x) + A(y) x, y R.
Vy A l mt hm cng tnh trn R nn
_
_
_
f(x) = A(x) + +
g(x) = A(x) +
h(x) = A(x) +
42
Ch . Nu bi ton c thm gi thit: hm f, g, h lin tc th nghim tm c s l
_
_
_
f(x) = ax + +
g(x) = ax +
h(x) = ax +
vi a, , l cc hng s ty .
Tip theo ta xt tnh n nh nghim ca phng trnh (15).
Mnh 3. Gi s hm f, g, h : R R tha mn
|f(x +y) g(x) h(y)| (17)
vi l s dng ty cho trc v vi mi x, y R. Khi tn ti duy nht mt hm cng
tnh A : R R sao cho
_
_
_
|f(x) A(x) f(0)| 6
|g(x) A(x) g(0)| 4
|h(x) A(x) h(0)| 6
vi mi x R.
Chng minh. Thay y = 0 vo (17), ta c
|f(x) g(x) h(0)| , x R, (18)
suy ra
|f(0) g(0) h(0)| . (19)
Thay y = 0 vo (17), ta c
|f(y) h(y) g(0)| , y R, (20)
T (18) v (20)
|h(x) g(x) h(0) + g(0)| = |f(x) g(x) h(0) + h(x) + g(0) f(x)|
|f(x) g(x) h(0)| +|f(x) h(x) h(0)|
hay
|h(x) g(x) h(0) + g(0)| 2, x R. (21)
S dng (18), ta c
|f(x + y) g(x +y) h(0)| , x, y R. (22)
Ta c
|f(x + y) g(x +y) h(0)| = |f(x +y) g(x) h(y) g(x +y) + g(x) + h(y) h(0)|
43
nn kt hp (17) v (22) thu c
|g(x + y) g(x) h(y) + h(0)| |f(x +y) g(x +y) h(0)| +|f(x +y) g(x) h(y)|
2
Mt khc
|g(x + y) g(x) h(y) + h(0)| = |g(x +y) g(x) g(y) + g(0) h(y) + g(y) g(0) + h(0)|
nn t (21)
|g(x +y) g(x) g(y) + g(0)| |g(x + y) g(x) h(y) h(0)| +|h(y) g(y) + g(0) h(0)|
4
hay
|[g(x +y) g(0)] [g(x) g(0)] [g(y) g(0)]| 4, (23)
vi mi x, y R. t
G(x) = g(x) g(0), (24)
vi mi x, y R. Th vo (23) ta c
|G(x +y) G(x) G(y)| 4, x, y R.
Theo nh l v tnh n nh ca hm cng tnh, tn ti duy nht mt hm cng tnh A : R R
sao cho
|G(x) A(x)| 4, x R. (25)
T (24) v (25) ta c
|g(x) A(x) g(0)| 4, x R. (26)
T (18), (19) v (26) ta c
|f(x) A(x) f(0)| = |[f(x) g(x) h(0)] + [g(x) A(x) g(0)] + [g(0) + h(0) f(0)]|
|f(x) g(x) h(0)| +|g(x) A(x) g(0)| +|f(0) g(0) h(0)|
+ 4 + = 6
T (21) v (26) ta c
|h(x) A(x) h(0)| = |[h(x) g(x) h(0) + g(0)] + [g(x) A(x) g(0)]|
|h(x) g(x) h(0) + g(0)| +|g(x) A(x) g(0)|
2 + 4 = 6.
V d 4. Tm tt c cc hm lin tc f : R R tha mn phng trnh sau
f(
x +y
2
) =
_
f(x)f(y), x, y R (27)
44
T phng trnh (27), ta c f(x) 0, x R. Gi s tn ti x
0
R sao cho f(x
0
) = 0.
Khi
f(
x
0
+y
2
) =
_
f(x
0
)f(y) = 0 y R,
hay f(x) = 0 vi mi x R.
Xt f(x) > 0, x R. Khi ly logarit hai v ca phng trnh (27), ta c
ln f(
x +y
2
) =
ln f(x) + ln f(y)
2
, x, y R
t g(x) = ln f(x) ta c
g(
x +y
2
) =
g(x) + g(y)
2
, x, y R
hay g l mt nghim ca phng trnh Jensen, tc l g(x) = ax+b. Suy ra nghim ca phng
trnh (27) l f(x) = e
ax+b
vi a, b R.
Tip theo ta xt tnh n nh nghim ca phng trnh (27).
Mnh 4. Gi s hm f : R R
+
tha mn
|f(
x + y
2
)
_
f(x)f(y)| (28)
vi mi x, y R v
|f(x) f(x)| (29)
vi , l cc s dng ty cho trc. Gi s tn ti f(a)
1
, khi tn ti mt hm E : R
R
+
sao cho
|E(x + y) E(x) E(y)| , x, y R (30)
v
|f(x)
1
2
(E(x) E(x))| , x R (31)
vi , l cc hng s no .
Chng minh. t m = sup
xR
_
f(x)f(a). T iu kin (29) th m l hu hn. Khi , ta c
_
f(x)f(a)
_
f(x)f(a) +|
_
f(x)f(a)
_
f(x)f(a)|
m+|f(
x a
2
)
_
f(x)f(a)| +|f(
x +a
2
)
_
f(x)f(a)|
+|f(
x a
2
) f(
x +a
2
)|
m+ 2 +
t h : R R
+
tha
h(x) =
_
f(x)f(x), x R.
45
Khi h l mt hm chn v
|h(x) f(x)| =
_
f(x)|
_
f(x)
_
f(x)| 2
m
2
f(a)
, x R, |h(x)
_
f(a)| m. (32)
t E : R R
+
tha mn
E(x) = h(x) +
_
f(a) x R
p dng (32) ta c
|E(x +y) E(x)E(y)| = |h(x +y) +
_
f(a) h(x)h(y) (h(x) + h(y))
_
f(a) f(a)|
|h(x +y)| +|h(x)h(y)| +|(h(x) + h(y))
_
f(a)| +|f(a)|
|h(x +y) f(x + y)| +|f(x +y)| +|h(x)h(y)f(a)f

1(a)|
+|h(x)
_
f(a)| +|h(y)
_
f(a)| +
_
f(a) +|f(a)|
2
m
2
f(a)
+
m
_
f(a)
+
m
2
f(a)
+ 2m+
_
f(a) +|f(a)| =
v
|f(x)
1
2
(E(x) E(x))| =
|f(x) h(x) + h(x)
1
2
(h(x) + h(x))
_
f(a)|
|f(x) h(x)| +
_
f(a) 2
m
2
f(a)
+
_
f(a) = .
Ti liu tham kho
[1] Nguyn Vn Mu, Phng trnh hm, Nh xut bn Gio dc, 1997.
[2] Pl. Kannappan, Functional Equations and Inequalities with Applications, Springer, 2009,
295-323.
46
MT S LP PHNG TRNH HM A N SINH
BI PHI NG THC
Trn Vit Tng, Trng THPT Trn Ph - Nng
Trong ton hc ph thng cc bi ton v phng trnh hm l cc loi ton thng mi v rt
kh, thng xuyn xut hin trong cc thi hc sinh gii quc gia, Olympic Ton khu vc
v Quc t, Olympic sinh vin gia cc trng i hc v cao ng. Lin quan n cc dng
ton ny l cc bi ton v cc c trng khc nhau ca hm s v cc tnh cht lin quan vi
chng.
h thng cc phng trnh hm, cn thit phi h thng cc kin thc c bn v nng cao
v cc dng phng trnh hm cng nh cc ng dng ca chng.
i vi cc bi ton v phng trnh hm vi nhiu n hm trong cc lp hm c th: lin tc,
kh vi, tun hon, li lm,... cn nm c mt s k thut v bin i hm s, kho st cc
tnh cht c bn ca hm thc v cc php bin hnh trn trc thc.
1 Phng trnh hm sinh bi phi ng thc a
2
+ b
2

g(a + b)h(a b)
Bi ton 1. Tm cc hm s f, g, h xc nh v lin tc trn R tha mn iu kin
f(x
2
+y
2
) = g(x +y).h(x y), x, y R. (1)
Gii. Xt trng hp g(0) = 0.
Cho y = x , phng trnh cho tr thnh
f(2x
2
) = 0, x R
Suy ra
f(x) = 0, x 0.
Thay f(x) vo phng trnh cho ta c
g(x +y).h(x y) = 0, x, y R
g(u).h(v) = 0, u, v R.
Do
_
g(x) = 0
h(x) lin tc ty trn R
hoc
_
h(x) = 0
g(x) lin tc ty trn R
47
Vy nghim trong trng hp ny l :
_

_

_
_
f(x) = 0 vi x 0
f(x) = q(x) vi q(x) lin tc ty trong (; 0] v q(0) = 0
g(x) 0
h(x) l hm lin tc ty
hoc
_

_

_
_
f(x) = 0 vi x 0
f(x) = q(x) vi q(x) lin tc ty trong (; 0] v q(0) = 0
g(x) l hm s lin tc ty v g(0) = 0
h(x) 0
Xt trng hp h(0) = 0
Cho y = x, phng trnh cho tr thnh
f(2x
2
) = 0, x R
Suy ra
f(x) = 0, x 0.
Th f(x) vo cho ta c
g(x +y).h(x y) = 0, x, y R
g(u).h(v) = 0, u, v R.
Do
_
g(x) = 0
h(x) lin tc ty trn R; h(0) = 0
hoc
_
h(x) = 0
g(x) lin tc ty trn R.
Vy nghim ca phng trnh trong trng hp ny l :
_

_

_
_
f(x) = 0 vi x 0
f(x) = q(x) vi q(x) lin tc ty trong (; 0] v q(0) = 0
g(x) l hm lin tc ty
h(x) 0
hoc
_

_

_
_
f(x) = 0 vi x 0
f(x) = q(x) vi q(x) lin tc ty trong (; 0] v q(0) = 0
g(x) 0
h(x) l hm lin tc ty vi h(0) = 0
48
Xt trng hp g(0) = 0 v h(0) = 0 .
Ta c f(0) = 0.
Cho x = y, phng trnh cho tr thnh
f(2x
2
) = g(2x).h(0), g(2x) =
f(2x
2
)
h(0)
, x R, g(x) =
f(
x
2
2
)
h(0)
, x R.
Cho x = y, phng trnh cho tr thnh
f(2x
2
) = g(0).h(2x), h(2x) =
f(2x
2
)
g(0)
, x R, h(x) =
f(
x
2
2
)
g(0)
, x R.
Thay g(x) v h(x) vo phng trnh ta c
f(x
2
+y
2
) = f[
(x + y)
2
2
].f[
(x y)
2
2
].
1
g(0)h(0)
, x, y R.
t
_

_

_
u =
(x + y)
2
2
v =
(x y)
2
2
.Khi ta c
f(u + v) = f(u).f(v).
1
g(0)h(0)
, u, v 0. (2)
t f(u) = g(0)h(0)F(u), u 0.
Phng trnh (3.2) tr thnh
g(0)h(0)F(u + v) = g(0)h(0)F(u).g(0)h(0)F(v).
1
g(0)h(0)
, u, v 0
F(u + v) = F(u)F(v), u, v 0.
Ta c
F(u) = a
u
u 0; a > 0
f(u) = b.a
u
vi b = g(0)h(0); a > 0
f(x) = b.a
x
, x 0; a > 0, b = 0.
Suy ra g(x) =
f(
x
2
2
)
h(0)
, x R = g(0).a
x
2
2
x R = m.a
x
2
2
vi m = g(0).
h(x) =
f(
x
2
2
)
g(0)
, x R = h(0).a
x
2
2
, x R = n.a
x
2
2
vi n = h(0).
49
Vy nghim ca phng trnh trong trng hp ny l :
_

_

_
_
f(x) = b.a
x
vi x 0; a > 0; b = 0
f(x) = q(x) vi q(x) lin tc ty trong (; 0] v q(0) = 0
g(x) = m.a
x
2
2
, x R
h(x) = n.a
x
2
2
, x R
Tm li nghim ca bi ton l
_

_

_
_
f(x) = 0 vi x 0
f(x) = q(x) vi q(x) lin tc ty trong (; 0] v q(0) = 0
g(x) 0
h(x) l hm lin tc ty
hoc
_

_

_
_
f(x) = 0 vi x 0
f(x) = q(x) vi q(x) lin tc ty trong (; 0] v q(0) = 0
g(x) l hm s lin tc ty v g(0) = 0
h(x) 0
hoc
_

_

_
_
f(x) = 0 vi x 0
f(x) = q(x) vi q(x) lin tc ty trong (; 0] v q(0) = 0
g(x) l hm lin tc ty
h(x) 0
hoc
_

_

_
_
f(x) = 0 vi x 0
f(x) = q(x) vi q(x) lin tc ty trong (; 0] v q(0) = 0
g(x) 0
h(x) l hm lin tc ty vi h(0) = 0
hoc
_


_


_
_
f(x) = b.a
x
vi x 0; a > 0; b = 0
f(x) = q(x) vi q(x) lin tc ty trong (; 0] v q(0) = 0
g(x) = m.a
x
2
2
, x R
h(x) = n.a
x
2
2
, x R
50
Bi ton 2. Tm cc hm s f, g, h xc nh v lin tc trn R tha mn iu kin
f(x
2
+ y
2
) = g(x +y) + h(x y), x, y R. (3)
Gii. Nghim ca bi ton l
_


_


_
_
f(x) = ax +b +c vi x 0
f(x) = q(x) vi q(x) lin tc ty trong (; 0] v q(0) = 0
g(x) = a
x
2
2
+b, x R
h(x) = a
x
2
2
+c, x R
vi b = g(0); c = h(0)
Bi ton 3. Tm cc hm s f, g, h xc nh v lin tc trn R tha mn iu kin
f(x
2
+y
2
) = g(x
2
) h(y
2
), x, y R. (4)
Gii. Nghim ca bi ton l
_

_

_
_
f(x) = ax +b vi x 0
f(x) = q(x) vi q(x) lin tc ty trong (; 0] v q(0) = 0
g(x) = ax +c, x R
h(x) = ax d, x R
2 Phng trnh hm sinh bi ng thc a
2
b
2
= (a +
b)(a b)
Bi ton 4. Tm cc hm s f, g, h lin tc v xc nh trn R tha mn iu kin
f(x
2
y
2
) = (x +y)g(x y), x, y R. (5)
Gii. Cho y = 0, phng trnh cho tr thnh
f(x
2
) = x.g(x), x R.
Nu x = 0 th
f(x) = 0. (6)
Nu x = 0 , ta c
g(x) =
f(x
2
)
x
.
51
Do , phng trnh cho tr thnh
f(x
2
y
2
) =
(x +y)f[(x y)
2
]
x y
, x = y
f(x
2
y
2
)
x + y
=
f[(x y)
2
]
x y
, x = y
f(x
2
y
2
)
x
2
y
2
=
f[(x y)
2
]
(x y)
2
, x = y.
t h(x) =
f(x)
x
vi x = 0. Khi , ta c
h(x
2
y
2
) = h[(x y)
2
].
Cho x = y + 1 , ta c
h(2y + 1) = h(1), y R
h(x) = a, x = 0.
Do
f(x) = ax vi x = 0. (7)
Kt hp (10) v (7) ta c
f(x) = ax, x R.
Suy ra
g(x) =
ax
2
x
= ax, x = 0.
Vy nghim ca bi ton l
_
f(x) = ax
g(x) = ax
Bi ton 5. Tm tt c cc hm s f, g, h xc nh v lin tc trn R tha mn iu kin
f(x
2
y
2
) = g(x y) + h(x + y), x, y R. (8)
Gii. Nghim ca bi ton l
_

_
f(x) = a, x R
g(x) = b, x R
h(x) = c, x R
trong a, b, c R; a = b + c.
Bi ton 6. Tm tt c cc hm s f, g, h xc nh v lin tc trn R tha mn iu kin
f(x
2
y
2
) = g
2
(x) h
2
(y), x, y R. (9)
52
Gii. Nghim ca bi ton l
_

_
f(x) = mx +a b, x R; a, b, m 0
g(x) =

mx
2
+a
h(x) =

mx
2
+ b
hoc
_

_
f(x) = mx + a b, x R; a, b, m 0
g(x) =

mx
2
+a
h(x) =

mx
2
+b
hoc
_

_
f(x) = mx + a b, x R; a, b, m 0
g(x) =

mx
2
+ a
h(x) =

mx
2
+ b
hoc
_

_
f(x) = mx + a b, x R; a, b, m 0
g(x) =

mx
2
+ a
h(x) =

mx
2
+b
3 Mt s bi ton phng trnh a n hm khc
Bi ton 7. Tm cc hm s f, g, h lin tc v xc nh trn R tha mn iu kin
f(x) g(y) = xh(y) yh(x), x, y R. (10)
Gii. Cho x = y, phng trnh cho tr thnh
f(x) g(x) = 0 f(x) = g(x), x R.
Cho y = 0, phng trnh cho tr thnh
f(x) g(0) = x.h(0), x R
f(x) = x.h(0) + g(0), x R
f(x) = ax +b vi a = h(0); b = g(0).
Thay f, g vo phng trnh cho ta c
(ax +b) (ay +b) = xh(y) yh(x), x, y R
ax ay = xh(y) yh(x), x, y R
a
y

a
x
=
h(y)
y

h(x)
x
, x, y R
a
x

h(x)
a
=
a
y

h(y)
y
, x, y R.
53
Suy ra
a
x

h(x)
x
= C vi C l hng s
h(x) = Cx +a
h(x) = cx +a vi c = C.
Th li phng trnh ta thy f, g, h tha mn.
Vy nghim ca phng trnh l
_
f(x) = g(x) = ax +b
h(x) = cx +a
Bi ton 8. Tm tt c cc hm s f, g xc nh v lin tc trn R tha mn iu kin
f(x) f(y) = (x +y)g(x y), x, y R. (11)
Gii. Nghim ca bi ton l
_
f(x) = ax
2
+b
g(x) = ax
vi mi a, b R.
Bi ton 9. Tm tt c cc hm s f, g xc nh v lin tc trn R tha mn iu kin
f(x) + f(y) + 2xy = (x +y)g(x +y), x, y R. (12)
Gii. Nghim ca bi ton l f(x) = x
2
+ax v g(x) =
_
x +a vi x = 0
c vi x = 0
.
Bi ton 10. Tm tt c cc hm s f, g xc nh v lin tc trn R tha mn iu kin
f(x).g(y) = x
2
y
2
, x, y R. (13)
Gii. Nu tn ti x
0
sao cho f(x
0
) = 0. Khi ta c
0 = f(x
0
).g(y) = x
2
0
y
2
, y R (v l).
Suy ra
f(x) 0, x R.
Tng t ta cng c
g(x) 0, x R.
Cho x = y, phng trnh (15) tr thnh
f(x).g(x) = 0 f(x) = 0 hoc g(x) = 0 (loi do f(x) 0; g(x) 0).
Vy khng tn ti cc hm f, g tha mn bi ton.
54
Bi ton 11. Tm tt c cc hm s f, g, h xc nh v lin tc trn R tha mn iu kin
f(x +y) + g(x y) = h(xy), x, y R. (14)
Gii. Nghim ca bi ton l
_

_

_
f(x) =
mx
2
4
+b, x R
g(x) =
mx
2
4
+ a, x R
h(x) = mx + a +b, x R.
Bi ton 12. Tm tt c cc hm s dng f, g, h xc nh v lin tc trn R tha mn iu
kin
f(x +y).g(x y) = h(xy), x, y R. (15)
Gii. Do f, g, h l cc hm s dng nn phng trnh (??) tng ng
ln f(x +y) + ln f(x y) = ln h(xy), x, y R.
t
_

_
ln f(x) = F(x)
ln g(x) = G(x)
ln h(x) = H(x)
. Khi ta c F(x +y) + G(x y) = H(xy), x, y R.
Ta c
_

_

_
F(x) =
mx
2
4
+b, x R
G(x) =
mx
2
4
+a, x R
H(x) = mx +a +b, x R.
. Suy ra
_

_

_
f(x) = e
mx
2
4
+b
, x R
g(x) = e

mx
2
4
+a
, x R
h(x) = e
mx+a+b
, x R.
Th li ta thy cc hm f, g, h tha mn iu kin bi ton.
Vy nghim ca bi ton l
_

_

_
f(x) = e
mx
2
4
+b
, x R
g(x) = e

mx
2
4
+a
, x R
h(x) = e
mx+a+b
, x R.
Bi ton 13. Tm tt c cc hm s f, g xc nh v lin tc trn R tha mn iu kin
f(x) + g(x) + f(y) g(y) = sin x cos y, x, y R. (16)
Gii. Nghim ca bi ton l
_

_

_
f(x) =
1
2
(sin x cos x), x R
g(x) =
1
2
(sin x + cos x) + a, x R.
55
Ti liu tham kho
[1] Nguyn Vn Mu, 1997, Phng trnh hm, NXB Gio Dc
[2] Nguyn Vn Mu, 2006, Cc bi ton ni suy v p dng, NXB Gio Dc.
[3] Nguyn Vn Mu, 2009, Phng trnh hm vi nhm hu hn cc bin i phn tuyn tnh,
K yu HNKH "Cc phng php v chuyn ton s cp" ti Bc Giang, 27-29/11/2009.
[4] Christopher G. Small, 2000, Functinal equations and how to solve them, Springer.
56
T CNG THC EULER N CC BI TON S
PHC
L Sng, Nguyn inh Huy, Trng THPT chuyn L Qu n - Khnh Ha
p dng s phc trong hnh hc phng c nhiu ti liu ca tc gi on Qunh,Nguyn
Hu in nh l sch gio khoa dng tham kho.Trong bi vit ny chng ti cp n mt
phng php rt hiu qu trong gii h phng trnh, chng minh ng thc, hay dng tnh
tng c gi l phng php s phc. Trc xt thm khai trin chui Taylor xy dng
cng thc Moivre m trong sch gio khoa c c t cng thc nhn s phc dng lng gic
v chng minh quy np I Khai trin Taylor v cng thc Moivre
1 Khai trin Taylor v cng thc Moivre
Cho z l s phc
e
z
= 1 +
z
1!
+
z
2
2!
+
z
3
3!
+.... +
z
n
n!
+ ...
Trng hp c bit, vi mt gc x, ta c
e
ix
= 1 +i
x
1!

x
2
2!
i
x
3
3!
+
x
4
4!
+....
Phn thc v phn o ca e
ix
ln lt l
Re(e
ix
) = 1
x
2
2!
+
x
4
4!

x
6
6
+ , Im(e
ix
) =
x
1!

x
3
3!
+
x
5
5!

x
7
7!
+
Hai chui trn l khai trin Taylor ca cos x v sin x.
Ta c c cng thc Euler nh sau e
ix
= cos x+i sin x v ta c mt cng thc tuyt p: e
i
=
1 Ngoi ra do e
inx
= (e
(ix)
n
) nn ta suy ra cng thc Moivre: cos nx+i sin nx = (cos x+i sin x)
n
V d m u
Tm khai trin Taylor ca hm f(x) = e
xcos
cos(x sin ) ti im 0, vi l tham s.
Gii. t g(x) = e
xcos
sin(x sin ) v vit :
f (x) + ig (x) = e
xcos
(cos (x sin ) + i sin (x sin ))
= e
xcos
.e
ixsin
= e
x(cos +i sin )
Dng cng thc Moivre v khai trin Taylor:
1 +
x
1!
(cos + sin ) +
x
2
1!
(cos 2 +i sin 2) + +
x
n
n!
(cos n +i sin n) +
57
Khai trin v rt gn ta thu c
f (x) = 1 +
cos
1!
x +
cos 2
2!
x
2
+ +
cos n
n!
x
n
+
Sau y l mt s bi ton p dng phng php s phc
2 Chng minh ng thc
Bi ton 1. Chng minh
1
cos 6
0
+
1
sin 24
0
+
1
sin 48
0
=
1
sin 12
0
Li gii. t z = cos 6
0
+i sin 6
0
. Ta c z
1
5 = i
M cos 6
0
=
z
2
+1
2z
, sin 12
0
=
z
4
1
2iz
2
, sin 24
0
=
z
8
1
2iz
4
, sin 48
0
=
z
16
11
2iz
8
ng thc cn chng minh tr thnh chng minh
2z
z
2
+1

2iz
2
z
4
1
+
2iz
4
z
8
1
+
2iz
8
z
16
1
= 0. Quy ng mu
s, thu gn Ta c z
16
1 iz (z
14
+ 1) = 0z
16
1 iz (z
14
+ 1) = 0 tc l iz 1 i
2
iz = 0
iu ny hin nhin ng.
Bi ton 2. Cho a, b, c l cc s thc tha cos a + cos b + cos c = sin a + sin b + sin c = 0
Chng minh rng
a) cos 2a + cos 2b + cos 2c = sin 2a + sin 2b + sin 2c = 0
b) 3(cos(a +b +c) = cos 3a + cos 3b + cos 3c v 3(sin(a +b +c) = sin 3a + sin 3b + sin 3c
Li gii. t x = cos a +i sin a, y = cos b + i sin b, z = cos c +i sin c
T gi thit ta c x + y +z = x
1
+y
1
+z
1
= 0 suy ra xy + yz +zx = 0
a) x
2
+ y
2
+z
2
= (x +y + z)
2
2(xy + yz +zx) = 0. Suy ra kt qu
b) x
3
+ y
3
+ z
3
= 3xyz. Suy ra kt qu
Bi ton 3. Chng minh ng thc:
_
1 + i tan t
1 i tan t
_
n
=
1 + i tan nt
1 i tan nt
, n 1
Li gii. Nu ta nhn t v mu v bn tri vi cos t, v v bn phi bi cos nt, th ta c uc
ng thc:
_
e
it
e
it
_
n
=
e
int
e
int
Bi ton 4. Chng minh ng thc
_
n
0
_
+
_
n
k
_
+
_
n
2k
_
+... =
2
n
k
k

i=1
cos
n
j
k
cos
nj
k
Li gii. Let C
1
, C
2
, . . . , C
k
l k nghim ca cn n v, tc l, C
j
= cos
2j
k
+ i sin
2j
k
, j =
1, 2, .., k
Nh vy
k

j=1
(1 + C
j
)
n
=
n

s=0
_
n
s
_
_
k

j=1
C
j
_
= k
n
k

j=0
_
n
jk
_
58
V 1 + C
j
= 2cos
j
k
cos
j
k
+i sin
j
k
_
Nn p dng cng thc Moivre
k

j=1
(1 + C
j
)
n
=
k

j=1
2
n
cos
n
j
k
_
cos
nj
k
+ i sin
nj
k
_
So snh phn thc 2 v ta c kt qu
Bi ton 5. Chng minh ng thc
1
_
n
2
_
+
_
n
4
_

_
n
6
_
+... = 2
n
2
_
cos
n
4
+ i sin
n
4
_
, n 1
Li gii. Dng cng thc Moivre, ta c:
(1 + i)
n
=
_

2
_
cos

4
+i sin

4
__
n
= 2
n
2
_
cos
n
4
+i sin
n
4
_
Khai trin (1 + i)
n
v cho hai v bng nhau bi phn thc,ta c c iu phi chng minh
ban u.
3 Tnh tch v tng
Bi ton 6. Chng minh tng cos
2
7
+ cos
4
7
+ cos
6
7
=
1
2
Li gii. Nu z = cos

7
+ i sin

7
th z
7
= 1 iu cn chng minh tng ng vi
1
2
_
z +
1
z
_
+
1
2
_
z
2
+
1
z
2
_
+
1
2
_
z
3
+
1
z
3
_
+
1
2
= 0
Nhn cho 2z
3
, sp xp cc s hng z
6
+z
5
+z
4
+ z
3
+ z
2
+ z + 1 =
z
7
1
z1
= 0
Bi ton 7. Tnh tng T =
3
_
cos
2
9
+
3
_
cos
4
9
+
3
_
cos
8
9
Li gii. Xt phng trnh z
9
1 = 0 c 9 nghim z
k
, k = 0, 1, . . . , 8 c tng l 0
Phng trnh t
4
+t
3
3t
2
2t + 1 = (t + 1) (t
3
3t + 1) = 0 c 4 nghim:
2 cos
2
9
, 2 cos
4
9
, 2 cos
6
9
, 2 cos
8
9
t t = z +
1
z
, phng trnh (t
3
3t + 1) = 0 c 3 nghim 2 cos
2
9
, 2 cos
4
9
, 2 cos
8
9
Dng cc biu thc i xng nghim phng trnh bc 3, ta c tng l
3
_
3
3

96
2
Bi ton 8. Tnh tch P = cos cos40
0
cos80
0
59
Li gii. . t z = cos 20
0
+i sin 20
0
Ta c 2 cos 20
0
= z +
1
z
_
, 2 cos 40
0
= z
2
+
1
z
2
_
, 2 cos 80
0
= z
4
+
1
z
4
_
, z
9
= 1
Suy ra (z z
8
) (z
2
z
7
) (z
3
z
5
) = (z z
2
+z
3
z
4
+z
5
z
6
+z
7
z
8
) = 1
Bi ton 9. Tnh tch sin

2n
sin
2
2n
sin
3
2n
sin
(n1)
2n
Li gii. Xt a thc P(X) = X
2n
1
C cc nghim x
k
= cos
k
n
+ i sin
k
n
, k = 0, 1, , 2n 1, x
0
= 1, x
n
= 1, x
k
= x
2nk
khi 1
k n 1
Khi P(X) = (X
2
1)
n1

k=1
(X x
k
)(X x
k
) = (X
2
1)
n1

k=1
(X
2
2 cos
k
n
+ 1), do x
k
+x
k
=
2 cos
k
n
, x
k
x
k
= 1
Chia 2 v cho x
2
1 , ta c
x
2n2
+x
2n4
+ x
4
+ x
2
+ 1 =
n1

k=1
_
x
2
2x cos
k
n
+ 1
_
.
Ly x = 1, ta c n = 2
n1
n1

k=1
1 cos
k
n
_
=2
n1
n1

k=1
2sin
2 k
2n
, suy ra
n1

k=1
sin
k
2n
=

n
2
n1
Bi ton 10. Tnh tng:
_
n
1
_
cos x +
_
n
2
_
cos 2x +... +
_
n
n
_
cos nx
Li gii. Gi tng cn tm l S
1
v cho
S
2
=
_
n
1
_
sin x +
_
n
2
_
sin 2x +... +
_
n
n
_
sin nx.
Dng cng thc Euler, ta c th vit
1 + S
1
+iS
2
=
_
n
0
_
+
_
n
1
_
e
ix
+ ... +
_
n
2
_
e
i2x
Nh tnh nhn ca ly tha, ta c:
n

k=0
_
n
k
_
_
e
ix
_
k
= 1 + e
ix
_
n
=
_
2cos
x
2
_
n
_
e
ix
2
_
n
Tng trong cu hi l mt phn thc ca khai trin lun b hn 1, iu ny dn n kt qu
tng l
2
n
cos
n
x
2
cos
nx
2
1
60
Bi ton 11. (USAMO 1999) Tnh (cos )(cos 2)(cos 3) . . . (cos 999)
vi =
2
1999
Li gii. Xt bi ton tng qut, vi n l s nguyn l, hy tnh:
S = (cos )(cos 2)(cos 3) . . . (cos n) vi =
2
2n + 1
Chng ta c th cho = e
i...?
v S = 2
n
n

k=1

k
+
k
_
.
Khi
k
+
k
=
2n+1k
, k = 1, 2, .., n, chng cha:
S
2
= 2
2x
2n

k=1

k
+
k
_
= 2
2n
.
2n

k=1

k
.
2n

k=1
1 +
2k
_
M
(1+2+...+2n)
= 1. Bi v (1 + 2 +. . . + 2n) = n(2n + 1) l bi ca 2n + 1.
Xt
2n

k=1
1 +
2k
_
, ch rng ta c th vit l
2n

k=1
1 +
k
_
, t khi s
2k
c hn (2n+1) nghim.
Trong khai trin
z
n+1
1 = (z 1)
2n

k=1
z
k
_
Cho z = 1 v chia c 2 v bi -2 v th
2n

k=1
1 +
k
_
= 1. Nn S
2
= 2
2n
, v v th S = 2
n
Khi 1 k n, cos k khi

2
< k < . Gi tr ca k chn , l, ty theo n s d khi chia cho 4
S =
_
+2
n
khi n 1 hay 2 (mod4)
2
n
khi n 0 hay 3 (mod4)
Khi n = 999 3 (mod4)
Ta c kt qu l 2
999
.
4 Gii phng trnh -h phng trnh
Bi ton 12. Gii phng trnh cos x + cos 2x + cos 3x = sin x + sin 2x + sin 3x
Li gii. t z = cos x +i sin x
Phng trnh thnh
z
2
1
2iz
+
z
4
1
2iz
2
+
z
6
1
2iz
3
=
z
2
+1
2z
+
z
4
+1
2z
2
+
z
6
+1
2z
3
Thu gn ta c (z
4
i) (z
2
+ z + 1) = 0
z
2
+ z + 1 = 0 suy ra z =
1 i

3
2
= cos
2
3
+isin
2
3
, x =
2
3
+ 2k, k Z
z
4
= i suy ra cos 4x +i sin 4x = cos

2
+i sin

2
, x =

8
+k

2
, k Z
61
Bi ton 13. (VMO 1996 ) Gii h phng trnh
_

_

_

3x(1 +
1
x +y
) = 2
_
7y(1
1
x +y
) = 4

2
Li gii. Bi ny c nhiu cc gii, y vn dng phng php s phc
Th

x = u,

y = v, ta c h
_

_

_
u(1 +
1
u
2
+ v
2
) =
2

3
v(1
1
u
2
+v
2
) =
4

7
t z = u + iv, do u
2
+ v
2
l bnh phng ca modun s phc z, nn nhn phng trnh sau
cho i, ri cng v ta c 1 phng trnh phc z
2
(
2

3
+ i
4

7
)z + 1 = 0 c nghim ln lt l
1

3

2

21
+ i(
2

2), du cng, tr tng ng


Do nghim x, y ln lt l (
1

3

2

21
)
2
(
2

2)
2
Bi ton 14. (Chn i tuyn chuyn Vnh Phc 2010-2011 )
Gii h phng trnh :
_
x +
3xy
x
2
+y
2
= 3 (1)
y
3x+y
x
2
+y
2
= 0 (2)
Li gii. Nhn phng trnh (2) vi i v cng theo v vi (1) ta c :
x + yi + 3
x yi
x
2
+ y
2

i (x yi)
x
2
+y
2
= 3 (3)
n y ta t z = x +yi khi
xyi
x
2
+y
2
=
1
2
v do (3) tr thnh : z +
3i
z
= 3.
Gii phng trnh s phc ny ta c 2 nghim :z = 1 i v z = 2 +i.
T ta suy ra h phng trnh ban u c 2 cp nghim l : x = 1, y = 1 v x = 2, y = 1.
Nh vy da vo vic sp xp cc n s thch hp a v mt phng trnh s phc ta c
th d dng tm c nghim ca h ban u thng qua nghim phc ny.
Bi ton tng t
Gii h phng trnh :
_
x +
x+2y
x
2
+y
2
= 2 (1)
x +
2x3y
x
2
+y
2
= 0 (2)
H ni 2007 :
_
x
3x+10y
x
2
+y
2
= 1 (1)
x +
10x3y
x
2
+y
2
= 2 (2)
62
Bi ton 15. Gii h phng trnh :
_
_
_
2x
_
1 +
1
x
2
+y
2
_
= 3 (1)
2y
_
1 +
1
x
2
+y
2
_
= 1 (2)
Li gii. Tng t nh bi ton 1 ta cng nhn (2) vi v cng vi (1) ta c :
2(x +yi) +
x yi
x
2
+y
2
= 3 +i (3).
t z = x +yi khi phng trnh (3) tr thnh: 2z +
2
z
= 3 +i.
Phng trnh ny c 2 nghim: z = 1 +i v z =
1
2

1
2
i.
Do h phng trnh ban u c 2 cp nghim: x = y = 1 v x =
1
2
, y =
1
2
.
Bi ton 16. ( Moldova TST 2011)
Gii h phng trnh :
_
x +y + 4 =
12x+11y
x
2
+y
2
(1)
y x + 3 =
11x12y
x
2
+y
2
(2)
Li gii. Nhn (2) vi v cng vi (1) ta c :
x + yi + y xi + 4 + 3i = 12
x yi
x
2
+y
2
+ 11
y +xi
x
2
+y
2
(3)
t z = x +yi khi phng trnh (3) tr thnh: z(z iz + 4 + 3i) = 12 + 11i.
Phng trnh ny c 2 nghim : 2 + i v
5
2

9
2
i.
Do h phng trnh ban u c 2 cp nghim: x = 2, y = 1 v x =
5
2
, y =
9
2
.
T cc v d trn ta nhn thy nu cc h phng trnh c i lng x
2
+ y
2
nm mu s
c hai phng trnh ca h th chng ta nn ngh n phng php s dng s phc tn
dng tnh cht z. z = |z|
2
ca s phc lin hp v modul s phc gii n. Vic cn li l gii
phng trnh s phc sau khi bin i t h phng trnh ban u.
Bi ton 17. Gii h phng trnh :
_
x
2
+x y
2
+
x
x
2
+y
2
=
3
2
(1)
2xy + y
y
x
2
+y
2
=
5
2
(2)
Li gii. Vn nh cc v d trc ta thu c :
x
2
+ 2xyi y
2
+ x + yi +
x yi
x
2
+ y
2
=
3
2
+
5
2
i (3)
t z = x +yi khi phng trnh (3) tr thnh:
z
2
+z +
1
z
=
3
2
+
5
2
i.
Gii phng trnh ny ta c 3 nghim t ta kt lun c nghim ca h ban u.
63
Bi ton 18. Gii h phng trnh :
_
x
2
y
2
+
x
2
y
2
(x
2
+y
2
)
2
+ x +
x
x
2
+y
2
=
3
2
2xy
2xy
(x
2
+y
2
)
2
+y
y
x
2
+y
2
= 1
Li gii. Hng dn : t z = x +yi h phng trnh dn n phng trnh s phc :
z
2
+
1
z
2
+z +
1
z
=
3
2
i.
Bi ton 19. Gii h phng trnh :
_
cos 2x + cos 2y +
4 cos x
2+cos 2xcos 2y
=
1+

3
2
cos x. sin y
sin y
2+cos 2xcos 2y
=

31
2
Li gii. Hng dn : t z = cos x + i sin y, h phng trnh s dn n phng trnh s
phc :
z
2
+
1
z
=
1 +

3
2
+

3 1
2
5 Cc dng khc
Bi ton 20. (Indian TST 2005) Cho a, b, c, d cc s thc khng ng thi bng 0. Cho f :
R R.
f(x) = a +b cos 2x +c sin 5x + d cos 8x.
Gi s rng c s thc t sao cho f(t) = 4a. Chng minh rng tn ti s thc s sao cho f(s) < 0.
Li gii. Cho g(x) = be
2ix
ice
5ix
+ de
8ix
. Khi f(x) = a + Reg(x).
rng
g(x) + g
_
x +
2
3
_
+ g
_
x +
4
3
_
= g(x)
_
1 + e
2i
s
+e
4i
s
_
= 0
Do f(x) + f x +
2
3
_
+ f x +
4
3
_
= 3a
Nu a < 0, th ly s = t, nu a = 0 th trong 3 s hng trn c 1 s m, nu a > 0, thay x = t
trong ng thc trn v f(t) = 4a, ta c f t +
2
3
_
+f t +
4
3
_
= a < 0.
Vy f t +
2
3
_
hay f t +
4
3
_
. Bi ton chng minh xong.
Bi ton 21. Cho z
1
, z
2
, z
3
l cc s phc phn bit c modun bng nhau, khng phi l s thc
. Chng minh rng, nu: z
1
+z
2
z
3
, z
2
+z
2
+z
3
z
1
, z
3
+z
1
z
2
l nhng s thc, th z
1
z
2
z
3
= 1.
64
Li gii. Cho z
j
= r(cos t
j
+ i sin t
j
), vi r = 0 vi r
j
(0, ), j = 1, 2, 3.
Vi gi thit
sin t
1
+r sin(t
2
+t
3
) = 0
sin t
2
+r sin(t
3
+t
1
) = 0
sin t
3
+r sin(t
1
+t
2
) = 0
t t = t
1
+t
2
+t
3
, th sin t
j
= r sin(t t
j
) = r sin t cos t
j
r cos t sin t
j
, vi j = 1, 2, 3.
C ngha l: cos t
j
sin t =
1
r
cos t
j
, j = 1, 2, 3.
Nu sin t = 0 th cos t
1
= cos t
2
= cos t
3
. Ch c 2 gi tr c th xy ra t
1
, t
2
, t
3
c th nhn gi
tr gia 0 v 2. iu ny dn ti sin t = 0. mt khc ta li c r cos t = 1 nn ta c cos t = 1
hoc 1. M iu ny ch xy ra nu cos t = 1 v r = 1 nn ta c pcm.
Bi ton 22. (Putnam 2006) Cho n l mt s nguyn dng l v l mt s thc sao cho

n
l s v t.
t a
k
= tan +
k
n
_
, k = 1, 2, ..., n.
Chng minh rng :
a
1
+a
2
+...+an
a
1
.a
2
...an
l mt s nguyn v tm gi tr nguyn .
Li gii. Xt s phc c dng = cos + i sin . Ta phi chng minh
_
1 + ix
1 ix
_
n
=
2n
C nghim l a
k
= tan +
k
n
_
, k = 1, 2, ..., n.
Vit li a thc bc n, ta c
0 = (1 + ix)
2

2n
(1 ix)
n
= (1
2x
) + ni (1 +
2x
) x +... +i
n1
(1
2x
) x +i
n
(1 +
2x
) x
n
Tng cc nghim theo khi trin trn l
ni
n1
(= a1
2x
)
i
n
(1 +
2x
)
v tch ca chng l
(1
2x
)
i
n
(1+
2x
)
.
Do :
a
1
+a
2
+...an
a
1
a
2
...an
= n.i
n1
= n(1)
(n1)
2
. pcm
Bi ton 23. Cho s nguyn dng n v hm f: F(n) = x
n
sin(nA) +y
n
sin(nB) +z
n
sin(nC) ,
x, y, z, A, B, C l cc s thc v A + B + C = k vi k nguyn . Chng minh rng, nu
F(1) = F(2) = 0, th F(n) = 0, vi mi s dng n.
Li gii. Chn cc s phc sao cho p = x.e
iA
, q = y.e
iB
, r = z.e
iC
v f(n) = p
n
+ q
n
+ z
n
.
t F(n) = Im(f(n)). Ta chng mnh bng qui np f(n) l s thc vi mi n, tc l F(n) = 0.
Chng ta cho rng f(1) v f(2) l thc v f(0) = 3 cng l mt s thc. By gi chng ta gii
65
s f(k) l thc cho tt c k n v n 3, ta s chng minh rng f(n+1) cng l s thc. Ch
rng, a = p + q + r = f(1), b = pq + qr + rp =
1
2
(f(1)
2
f(2)), v c = pqr = xyz.e
i(A+B+C)
l cc s thc. Cc s p, q, r l nghim ca tam thc P(t) = t
3
at
2
+bt c.
Dng
f(n + 1) = p
n+1
+q
n+1
+r
n+1
= a(p
n
+q
n
+r
n
) b(p
n1
+q
n1
+r
n1
) + c(p
n2
+q
n2
+r
n2
)
= af(n) bf(n 1) + cf(n 2)
Khi f(n), f(n 1) v f(n 2)l cc s thc, theo phng php qui np ta c f(n + 1) l s
thc. Ta c pcm.
Ti liu tham kho
[1] R. Gelca and T. Andresscu, 2007, Putnam and beyond, Springer.
[2] R. Gelca and T. Andresscu, 2000, Mathematical olympiad challenges, Bkhauser.
66
MT S NG DNG CA PHNG TRNH PELL
Nguyn Th Tnh, Trng THPT L Thng Kit, TX Ayun Pa, Gia Lai
Cc phng trnh Diophant ng mt vai tr quan trng trong l thuyt s hc. T thi Trung
c, cc phng trnh ny c cc nh ton hc n quan tm nghin cu v t c
nhng kt qu su sc, chng hn tm ra dng nghim tng qut ca phng trnh Diophant
tuyn tnh ax +by = c t nm 499.
Trong cc phng trnh Diophant bc hai, c dng phng trnh bc hai chnh tc rt c
bn v quan trng l x
2
dy
2
= 1 hoc x
2
dy
2
= 1, trong d l s nguyn dng khng
chnh phng.
Phng trnh trn thng c gi l phng trnh Pell, ly theo tn nh ton hc ngi
Anh John Pell (1610 - 1685) m theo nhng cu chuyn ton hc truyn li, do s nhm ln
no ca nh ton hc thin ti Euler khi i tm cng lao nhng ngi u tin khai m
phng trnh ny.
Cho n nay, Vit Nam c mt s ti liu, sch tham kho trong c cp mt
cch trn vn phng php gii phng trnh Pell v mt s vn lin quan ca GS. Nguyn
Vn Mu (ch bin), GS. H Huy Khoi, GS. ng Hng Thng,. . .
Trn c s nhng ti liu su tm c, xin gii thiu li mt s vn c bn nht, dnh
cho h ph thng chuyn ton, v phng trnh kinh in ny. Trong , khng qu i su vo
l thuyt ca phng trnh Pell, v iu ny c trong mt s ti liu nu.
1 Mt s v d dn n phng trnh Pell
T rt lu i, khi m cha c cch gii phng trnh Pell, trong ton hc c rt nhiu
nhng cu hi t nhin t ra. tr li c cc vn nhiu khi li dn n yu cu tm
tt c nhng s nguyn dng tha mn phng trnh bc 2 c hai bin. C rt nhiu nhng
bi ton m nhiu khi chng ta khng thy c s lin quan n phng trnh Pell, nhng bng
nhng php bin i i s li dn n mt yu cu chung, l i tm cc nghim nguyn
dng ca phng trnh x
2
dy
2
= n, trong d khng phi l s chnh phng, n l mt s
nguyn. Chng hn ta c kt qu rt n gin l 1+2+3+ +14 = 15+16+17+18+19+20
hoc 3 + 4 + 5 + 6 = 3.6. Nh vy mt vn t nhin t ra l liu rng cn nhng cp s
nguyn dng (m, n) no c tnh cht nh trn hay khng? Chng ti xt 2 v d sau y:
V d 1. Xc nh cc s nguyn dng m v n sao cho
m+ (m+ 1) + +n = mn (1)
67
Ta c
(1)
(m+n)(n m+ 1)
2
= mn
n
2
+n m
2
+m = 2mn
(n m)
2
2m
2
+n +m = 0
t u = n m, ta c
u
2
2m
2
+u + 2m = 0 u
2
u 2(k
2
k) = 0

_
u +
1
2
_
2

1
4
2
_
_
m
1
2
_
2

1
2
_
= 0
(2u + 1)
2
2(2m1)
2
= 1
t x = 2u + 1 = 2(n m) + 1; y = 2m1, ta c phng trnh
x
2
2y
2
= 1 (2)
Nh vy yu cu ca bi ton t ra s c gii quyt nu phng trnh (2) c gii.
V d 2. Tm tt c nhng s nguyn dng k, m sao cho k < m v
1 + 2 + +k = (k + 1) + (k + 2) + +m
Gi s k, m(k < m) l hai s nguyn dng tha mn h thc:
1 + 2 + +k = (k + 1) + (k + 2) + +m (3)
Ta c
(3) 2(1 + 2 + + k) = 1 + 2 + +k + (k + 1) + (k + 2) + + m
k(k + 1) =
m(m+ 1)
2
2k
2
+ 2k = m
2
+m
8k
2
+ 8k = 4m
2
+ 4m
2(2k + 1)
2
1 = (2m+ 1)
2
.
t x = 2m+ 1; y = 2k + 1, khi ta c phng trnh
x
2
2y
2
= 1. (4)
Nh vy vn t ra s c gii quyt nu nh phng trnh (4) c gii.
S kt hp gia S hc v Hnh hc trong mt s trng hp cng to ra nhng bi ton
kh th v. Chng hn xt v d sau y
68
V d 3. Tm cc s nguyn dng m tam gic vi 3 cnh
1
2
(m
3
+m
2
) 1;
1
2
(m
3
m
2
) + 1; m
2
hoc m
3

1
2
(m1); m
3

1
2
(m+ 1); m c din tch l s
chnh phng.
Gi 2p, S ln lt l chu vi v din tch tam gic.
1) Trng hp 3 cnh tam gic l
a =
1
2
(m
3
+ m
2
) 1; b =
1
2
(m
3
m
2
) + 1; c = m
2
, ta c
p =
1
2
(m
3
+ m
2
); p a = 1; p b = m
2
1; p c =
1
2
(m
3
m
2
)
Theo cng thc Hrng, ta c
S =
_
p(p a)(p b)(p c) =
_
1
4
(m
3
+ m
2
)(m
2
1)(m
3
m
2
)
=
1
2
m
2
(m
2
1)
2) Trng hp 3 cnh tam gic l
a = m
3

1
2
(m1); b = m
3

1
2
(m+ 1); c = m , ta c
p = m
3
; p a =
1
2
(m1); p b =
1
2
(m+ 1); p c = m
3
m
S =
_
p(p a)(p b)(p c) =
_
1
4
m
3
(m1)(m+ 1)(m
3
m)
=
1
2
m
2
(m
2
1)
cc tam gic vi s o cc cnh cho nh trn c din tch l mt s chnh phng th
1
2
(m
2
1) l mt s chnh phng, tc l
1
2
(m
2
1) = n
2
m
2
1 = 2n
2
m
2
2n
2
= 1
Chng ta thy iu kin tam gic c cnh cho trc nh trn c din tch l mt s chnh
phng ph thuc vo vic i tm cc nghim nguyn dng ca phng trnh
m
2
2n
2
= 1
Mt s vn khc, ta bit rng

2 l s v t. Mt chng minh thng gp cho khng


nh ny l phng php phn chng. Chng minh sau y, tuy di, nhng kh th v v n
dn n phng trnh Pell c bn.
V d 4. Chng minh

2 l s v t
69
Chng minh. Ta cn chng minh

2 c biu din thnh lin phn s v hn.


Xt dy s
_
_
_
r
1
= 1
r
n
= 1 +
1
1 + r
n1
, n = 2, 3, 4, . . . Ta d dng nhn thy vi mi gi tr n, r
n
c biu din di dng
p
n
q
n
, vi (p
n
, q
n
) = 1 v khi n cng tng th r
n
cng tng khi n l v
r
n
cng gim khi n chn
Ta s chng minh:
1 r
1
r
3
r
2k+1
r
2l
r
4
r
2

3
2
(5)
|r
n+1
r
n
|
1
4
|r
n
r
n1
| (6)
1. Chng minh(5)
Trc ht ta chng minh
1 r
1
r
3
r
2n+1
(7)
Ta c r
3
r
1
=
3
1
1 =
1
2
> 0
Gi s (7) ng vi mt s t nhin n = k bt k, tc l ta c r
2k+1
r
2k1
> 0. Ta cn chng
minh r
2(k+1)+1
r
2(k+1)1
> 0
r
2k+3
+r
2k+1
= 1 +
1
1 + r
2k+2
1
1
1 + r
2k
=
r
2k
r
2k+2
(1 + r
2k+2
)(1 + r
2k
)
=
1 +
1
1 + r
2k1
1
1
1 + r
2k+1
(1 + r
2k+2
)(1 + r
2k
)
=
r
2k+1
r
2k1
(1 + r
2k+2
)(1 + r
2k
)(1 + r
2k1
)(1 + r
2k+1
)
> 0
(bt ng thc lun ng v theo gi thit quy np)
Vy 1 r
1
r
3
r
2n+1
, vi n nguyn dng bt k.
Chng minh tng t bng phng php quy np nh bt ng thc trn ta c:
3
2
r
2
r
4
r
2n
v r
2n1
r
2n
, vi mi n nguyn dng.
Vy vi mi cp s nguyn dng (k; l) ta c:
1 r
1
r
3
r
2k+1
r
2l
r
4
r
2

3
2
2. Chng minh (6)
70
Ta c
|r
n+1
r
n
| =


1 +
1
1 + r
n
1
1
1 + r
n1


=
|r
n
r
n1
|
(1 + r
n
)(1 + r
n1
)
r
n
r
n1
|

|
4

r
n
r
n1

1
4
|r
n
r
n1
| .
Gi l cn trn nh nht ca tp hp cc phn t {r
1
, r
3
, , r
2k+1
, },
khi r
2m1
r
2m
Ta c
| r
2m1
| < |r
2m
r
2m1
|
1
4
2(m1)
|r
2
r
1
| =
1
2
4m3
| r
2m
| < |r
2m
r
2m1
|
1
4
2(m1)
|r
2
r
1
| =
1
2
4m3
suy ra lim
n
r
n
= lim
n
r
n1
= suy ra = 1 +
1
1 +
hay
2
= 2
=

2. Vy

2 c biu din di dng lin phn s v hn nn

2 l mt s v t.
nh ngha 1. Cc s 1, 3, 6, 10, 15, 21, 28, 36, 45, . . . ,
t
n
=
1
2
n(n + 1),. . . c gi l cc s tam gic.
V m hnh hnh hc, cc s tam gic ln lt phn b u trn cnh ca mt tam gic, th
hin trn hnh v 1.2
Nhn xt rng, tng 2 s tam gic k nhau l mt s chnh phng.
Tht vy t
n
+ t
n+1
=
1
2
n(n + 1) +
1
2
(n + 1)(n + 2) = (n + 1)
2
.
Mt cch rt t nhin ny sinh l liu c tm c tt c cc s tam gic l s chnh phng
hay khng. Chng ta xt v d sau y
V d 5. Tm cc s tam gic l s chnh phng.
T dy s lit k trn, cc s 1 v 36 l cc s chnh phng. Mt cch tng qut, ta cn
xc nh nhng gi tr n N

sao cho t
n
l s chnh phng, hay ta tm nghim nguyn dng
ca phng trnh:
1
2
n(n + 1) = m
2
.
D thy phng trnh ny tng ng vi phng trnh:
(2n + 1)
2
8m
2
= 1.
Nh vy vic tm cc s tam gic l s chnh phng quy v bi ton tm nghim nguyn
dng ca phng trnh :
x
2
8y
2
= 1
71
Cng lin quan n s tam gic, mt vn mi t ra l liu c hay khng cc b ba s
tam gic lin tip m tch ca chng hoc tng ca chng l s chnh phng. Vn ny s
c cp trong hai v d sau y
V d 6. Tm cc b ba s tam gic lin tip sao cho tch ca chng l mt s chnh phng.
Gi s 3 s tam gic lin tip c dng
1
2
(n 1)n,
1
2
n(n + 1), v
1
2
(n + 1)(n + 2) Ta c
1
2
(n 1)n
1
2
n(n + 1)
1
2
(n + 1)(n + 2) =
1
2
2

1
2
n
2
(n 1)(n + 1)
2
(n + 2).
tch ca 3 s tam gic lin tip l mt s chnh phng th
1
2
(n 1)(n + 2) phi l mt
s chnh phng, tc l
1
2
(n 1)(n + 2) = m
2
(n 1)(n + 2) = 2m
2
(2n + 1)
2
8m
2
= 9, m Z
t x = 2n + 1; y = m, ta c phng trnh
x
2
8y
2
= 9 (8)
Nh vy bi ton s c lm sng t nu nh phng trnh(8) c gii.
V d 7. Tm cc b ba s tam gic lin tip sao cho tng ca 3 s l mt s chnh phng.
nh ngha 2. Vi cc s nguyn n v k, ta nh ngha
_
n
k
_
=
n!
k!(n k)!
Nh vy lin quan n bi ton t hp, mt vn t ra y l:
V d 8. C tn ti cc s nguyn a, b khng m sao cho
_
a
b
_
=
_
a 1
b + 1
_
(9)
V d 9. Gi s rng c n vin bi trong mt ci l, trong c r vin bi mu v n r vin
bi mu xanh. Ly ra 2 vin bi mt cch ngu nhin. Bit rng, xc sut ly c 2 vin bi
cng mu l
1
2
. Xc nh cc gi tr c th c c ca n v r.
nh ngha 3. Gi s a, b, c, ) l 3 s nguyn tha mn a
2
+b
2
= c
2
. Khi b s (a, b, c) c
gi l b ba s Pitago.
72
B 1. Vi mi s nguyn k, m, n, b s sau y l b ba s Pitago:
k(m
2
n
2
), 2kmn, k(m
2
+n
2
)
_
(10)
H qu 1. Vi k = 1, m = y, n = z th b ba
(y
2
z
2
, 2yz, y
2
+z
2
) (11)
l b ba s Pitago.
V d 10. Ta bit rng 2
2
+ 4
2
+ 6
2
+ 8
2
+ 10
2
= 4.5 + 5.6 + 6.7 + 7.8 + 8.9.
Vn t ra l tm nhng v d khc, tng t, ngha l: "Tm nhng ng thc m mt v
l tng cc bnh phng ca n s chn u tin v mt v l tng ca n cp s nguyn lin k".
Qua mt s v d c trnh by trn, chng ta nhn thy c rt nhiu vn n gin,
nhng li c tng qut ho thnh nhng bi ton kh v rt th v. Vic gii quyt mt s
v d trn ph thuc vo vic gii phng trnh dng:
x
2
dy
2
= 1 (12)
hoc
x
2
dy
2
= 1 (13)
hoc
x
2
dy
2
= n, (14)
y d l s nguyn dng, khng phi l s chnh phng; n l mt s nguyn.
2 Tm nghim nguyn dng ca phng trnh.
Bi ton 1. Tm cc nghim nguyn dng ca phng trnh
x
2
6xy + y
2
= 1
Gii.
Ta c x
2
6xy +y
2
= 1 (x 3y)
2
8y
2
= 1.
t u = x 3y, ta c phng trnh:
u
2
8y
2
= 1 (15)
Nghim nguyn dng nh nht ca phng trnh (15) l (a; b) = (3; 1) nn nghim ca phng
trnh (15) cho bi dy sau:
_
u
0
= 1; u
1
= 3; u
n+2
= 6u
n+1
u
n
y
0
= 0; y
1
= 1; y
n+2
= 6y
n+1
y
n
, n = 0, 1, 2,
73
M u = x 3y nn x = u + 3y lun l s nguyn dng khi u, y nguyn dng. Do
x
n+2
= u
n+2
+ 3y
n+2
= 6u
n+1
u
n
+ 3(6y
n+1
y
n
)
= 6(u
n+1
+ 3y
n+1
) (u
n
+ 3y
n
)
= 6x
n+1
x
n
.
Theo nh l (??) ni v cng thc nghim ca phng trnh Pell loi I, ta c nghim ca phng
trnh cho c cc nghim nguyn dng tho mn h thc sau y:
_
x
0
= 1; x
1
= 6; x
n+2
= 6x
n+1
x
n
y
0
= 0; y
1
= 1; y
n+2
= 6y
n+1
y
n
, n = 0, 1, 2,
Bi ton 2. Tm cc nghim nguyn dng ca phng trnh
(x 1)
2
+ (x + 1)
2
= y
2
+ 1
Gii. Nu dng phng php xy dng nghim th chng ta c th chng minh c phng
trnh trn c v s nghim nguyn dng, nhng nu bin i phng trnh trn v phng
trnh Pell c bn th chng ta s ch ra c tt c cc nghim nguyn dng ca n v li gii
cng s gn gng hn.
Ta c (x 1)
2
+ (x + 1)
2
= y
2
+ 1 2x
2
+ 2 = y
2
+ 1
y
2
2x
2
= 1 (16)
Phng trnh (16) l phng trnh Pell loi I, c nghim nguyn dng nh nht l (a; b) =
(3; 2).Vy theo nh l (??) ta c nghim ca phng trnh (16) cng nh phng trnh cho l:
_
x
0
= 0; x
1
= 2; x
n+2
= 6x
n+1
x
n
y
0
= 1; y
1
= 3; y
n+2
= 6y
n+1
y
n
, n = 0, 1, 2,
Bi ton 3. Tm cc s nguyn dng (x, y, z, w) tha mn x
2
+ y
2
+ z
2
= w
2
sao cho x =
y; z = x 1.
3 Tnh tng ca cc s nguyn lin tip.
Bi ton 4. Tm tt c nhng s nguyn dng k, m sao cho k < m v
1 + 2 + +k = (k + 1) + (k + 2) + + m
Gii. Gi s k, m(k < m) l hai s nguyn dng tha mn h thc:
1 + 2 + +k = (k + 1) + (k + 2) + + m (17)
74
Ta c:
(17)) 2(1 + 2 + + k) = 1 + 2 + +k + (k + 1) + (k + 2) + + m
k(k + 1) =
m(m+ 1)
2
2k
2
+ 2k = m
2
+m
8k
2
+ 8k = 4m
2
+ 4m
2(2k + 1)
2
1 = (2m+ 1)
2
.
t x = 2m+ 1; y = 2k + 1, khi ta c phng trnh Pell loi II:
x
2
2y
2
= 1. (18)
Lin kt vi (18) l phng trnh Pell loi I:
x
2
2y
2
= 1. (19)
Phng trnh Pell loi I c nghim nguyn dng b nht l
(x, y) = (3, 2).
Xt h phng trnh sau:
_
u
2
+ 2v
2
= 3
2uv = 2
(20)
D thy (u, v) = (1, 1) l nghim nguyn dng b nht ca h (20). Theo l thuyt xy dng
dy th phng trnh Pell loi II c nghim l
_
x
0
= 1; x
1
= 7; x
n+2
= 6x
n+1
x
n
y
0
= 1; y
1
= 5; y
n+2
= 6y
n+1
y
n
, n = 0, 1, 2,
Ta thy x
k
1 (mod 2); y
k
1 (mod 2), vi mi k = 0, 1, .T suy ra dy nghim
m
i
=
x
i
1
2
; k
i
=
y
i
1
2
c cho bi cng thc:
m
0
= 0; m
1
= 3; m
i+2
= 6m
i+1
m
i
+ 2
k
0
= 0; k
1
= 2; k
i+2
= 6k
i+1
k
i
+ 2,
vi i = 0, 1, 2, Bn p s u tin l:
(m, k) = (3, 2); (20, 14); (119, 84); (696, 492).
Bi ton 5. Xc nh cc s nguyn dng m v n sao cho
m+ (m+ 1) + +n = mn (21)
Bi ton 6. Ta c 2
2
+ 4
2
+ 6
2
+ 8
2
+ 10
2
= 4.5 + 5.6 + 6.7 + 7.8 + 8.9.
Tm nhng ng thc khc m mt v l tng cc bnh phng ca n s chn u tin v mt
v l tng ca n cp s nguyn lin k.
75
4 S tam gic v tnh cht ca s tam gic
Bi ton 7. Tm cc s nguyn dng n sao cho s tam gic
1
2
n(n+1) l mt s chnh phng.
Gii.
Ta c
n(n + 1)
2
= y
2
n(n + 1) = 2y
2
4n
2
+ 4n + 1 = 8y
2
+ 1
(2t + 1)
2
8y
2
= 1.
t x = 2n + 1 th (x; y) l nghim ca phng trnh:
x
2
8y
2
= 1 (22)
Ngc li nu (x; y) l nghim ca phng trnh (22) th x l nn n =
x 1
2
tha mn
bi.
Nghim nh nht ca phng trnh (22) l (3; 1) nn phng trnh c nghim l dy
_
x
0
= 1; x
1
= 3; x
i+2
= 6x
i+1
x
i
y
0
= 0; y
1
= 1; y
i+2
= 6y
i+1
y
i
, i = 0, 1, 2,
Khi vi x
i
= 2n
i
+ 1 th 2n
i+2
+ 1 = 6(2n
i+1
+ 1) (2n
i
+ 1). Dy (n
i
) cn tm xc nh
bi n
0
= 0; n
1
= 1; n
i+2
= 6n
i+1
n
i
+ 2.
l cc s 1; 8; 49; 288;
Bi ton 8. Tm cc b ba s tam gic lin tip sao cho tch ca chng l mt s chnh phng.
Bi ton 9. Tm cc b ba s tam gic lin tip sao cho tng ca 3 s l mt s chnh
phng.
Bi ton 10. Xc nh s nguyn dng b sao cho s (111 1)
b
, gm k ch s 1, vi b l c
s, l s tam gic vi bt k gi tr no ca k.
5 Tm s chnh phng
Mt p dng cng rt quan trng na l trong ton hc c rt nhiu bi ton lin quan
n s chnh phng. Vic gii phng trnh Pell s gip chng ta tm c cc s chnh phng
tho mn yu cu cho trc no .
Bi ton 11. Tm tt c cc s nguyn dng t sao cho tng ca t s nguyn dng u tin
l mt s chnh phng.
Cng tng t nh bi ton trn ta bit tng ca n s chnh phng u tin. Do ta
cng c bi ton tip theo
76
Bi ton 12. Tm tt c cc s nguyn dng n sao cho trung bnh cng ca n s chnh phng
u tin cng l mt s chnh phng.
Bi ton 13. Tm cc s nguyn dng m tam gic vi 3 cnh :
1
2
(m
3
+m
2
) 1;
1
2
(m
3
m
2
) + 1; m
2
hoc m
3

1
2
(m1); m
3

1
2
(m+ 1); m c din tch l s
chnh phng.
Bi ton 14. Tm tt c cc s nguyn dng n c tnh cht n
2
+ (n + 1)
2
l s chnh phng.
Bi ton 15. Chng minh rng tn ti v hn nhng s nguyn dng n sao cho n! chia ht
cho n
2
+ 1.
Gii.
Xt phng trnh Pell loi II :
x
2
5y
2
= 1 (23)
Phng trnh Pell loi I lin kt vi (23) l phng trnh sau:
x
2
5y
2
= 1 (24)
Phng trnh (24)c nghim nguyn dng nh nht l (9; 4). Xt h phng trnh
_
u
2
+ 5v
2
= 9
2uv = 4
H ny c nghim nguyn dng nh nht l (u, v) = (2, 1). V th phng trnh (23) c dy
nghim l:
_
x
0
= 2 ; x
1
= 38 ; x
n+2
= 18x
n+1
x
n
; n = 0, 1, 2,
y
0
= 2 ; y
1
= 17 ; y
n+2
= 18y
n+1
y
n
; n = 0, 1, 2,
Cc dy nghim c tnh cht 5 < y
k
< 2y
k
< x
k
,vi mi k = 1, 2,
Tht vy, ta c 5 < y
k
< 2y
k
. Do y
k
> 5 nn 4y
2
k
< 5y
2
k
1 = x
2
k
suy ra 2y
k
< x
k
.
Do vy 5 < y
k
< 3y
k
< x
k
, k N

.
V th (x
k
)! = 1.2.3.4.5 y
k
(2y
k
) (x
k
).
T suy ra (x
k
)!
.
.
.5y
k
(2y
k
) = 2(x
2
k
+ 1) nn x
k
.
.
.(x
2
k
+ 1), k N

.
Bi ton c chng minh.
Bi ton 16. Xt dy s {u
n
}

n=0
=
__
_
n
2
+ (n + 1)
2
__

n=0
. Chng minh rng tn ti v hn
cc ch s n sao cho ta c ng thi
u
n
u
n1
> 1; [u
n+1
] [u
n
] = 1.
Bi ton 17. Chng minh rng phng trnh 5
x
3
y
= 2 c mt nghim dng duy nht l
(x, y) = (1, 1).
77
Ti liu tham kho
[1] Phan Huy Khi, "Cc chuyn s hc bi dng hc sinh gii ton trung hc ", Phng
trnh nghim nguyn, NXB Gio dc.
[2] Phan Huy Khi(2004), "Chuyn bi dng hc sinh gii ton trung hc ph thng ",
Cc bi ton c bn ca s hc, NXB Gio dc.
[3] H Huy Khoi, "Chuyn bi dng hc sinh gii ton trung hc ph thng",S hc,
NXB Gio dc.
[4] Nguyn Vn Mu (ch bin), Mt s vn s hc chn lc, NXB Gio dc.
[5] ng Hng Thng (1995), Bi ging s hc, NXB Gio dc.
[6] ng Hng Thng, Nguyn Vn Ngc, V Kim Thy(1997), Bi ging s hc,Tuyn tp
30 nm tp ch ton hc v tui tr, NXB Gio dc.
[7] V Dng Thy (ch bin)(2006), Tuyn tp 40 nm Olympiads Ton hc quc t(IMO
1959-2000), NXB Gio dc.
[8] Barbeau Edward J.(2003), Pells Equation, Problem Books in Mathematics, Springer.
78
PHP TH LNG GIC L CNG C GII TON
TRONG CC BI THI CHN HC SINH GII
Hunh B Lc - S Gio dc v o to Khnh Ha
Vic chn php th lng gic linh hot trong mt s ln cng thc lng gic s lm cho bi
ton n gin hn. Php th thng c cho di dng biu thc i s, y vic lng gic
ha bi ton c xt di nhiu dng ton thng gp sau y
1 Chng minh ng thc
Bi ton 1. (IMO 1985) Cho x, y, z R, sao cho x +y +z = xyz. Chng minh
x(1 y
2
)(1 z
2
) + y(1 z
2
)(1 x
2
) + z(1 x
2
)(1 y
2
) = 4xyz.
Li gii. R rng ng thc ng vi xyz = 0, nn chng ta ch cn chng minh vi x, y, z = 0.
Chia 2 v cho 4xyz ta c
1 y
2
2y
1 z
2
2z
+
1 z
2
2z
1 x
2
2x
+
1 x
2
2x
1 y
2
2y
= 1
T iu kin x + y +z = xyz ta ngh n vic lng gic ha bi ton bng cch
t x = tan A, y = tan B, z = tan C vi A, B, C l 3 gc ca mt tam gic, ta a bi ton tr
thnh
cot 2Bcot 2C + cot 2C. cot 2A + cot 2A. cot 2B = 1
tan 2A + tan 2B + tan 2C = tan 2A. tan 2B. tan 2C
y r rng l ng thc ng v tan(2A + 2B + 2C) = tan 2 = 0.
Bi ton c chng minh.
Bi ton 2. Cho a, b, c l 3 s thc khc
1

3
. Chng minh rng abc = a +b +c khi v ch khi
3a a
3
3a
2
1

3b b
3
3b
2
1

3c c
3
3c
2
1
=
3a a
3
3a
2
1
+
3b b
3
3b
2
1
+
3c c
3
3c
2
1
.
Li gii. t a = tan x; b = tan y; c = tan z vi x, y, z 0;

2
_
. T ng thc
tan(x +y +z) =
tan x + tan y + tan z tan x tan y tan z
x tan x tan y tan y tan z tan x tan z
Ta thy abc = a + b + c khi v ch khi x +y + z = k(k Z)
Suy ra tan(3x + 3y + 3z) = tan 3k = 0
Hay
tan 3x tan 3y tan 3z = tan 3x + tan 3y + tan 3z

3a a
3
3a
2
1
3b b
3
3b
2
1
3c c
3
3c
2
1
=
3a a
3
3a
2
1
+
3b b
3
3b
2
1
+
3c c
3
3c
2
1
.
79
Bi ton 3. Chng minh rng
n

k=0

1
3
_
k
cos
3
_
3
k

_
=
3
4
_
_

1
3
_
n+1
+ cos
3
3
n
_
.
Li gii. T ng thc cos 3x = 4 cos
3
x 3 cos x, ta c cos
3
x =
1
4
(cos 3x + 3 cos x)
Suy ra
n

k=0
_

1
3
_
k
cos
3
3
k
a
_
=
1
4
n

k=0
_
_

1
3
_
k
cos 3
k+1
a
_

1
3
_
k1
cos 3
k
a
_
_
Cho a = 3
n
ta c kt qu ca bi ton.
Bi ton 4. Chng minh rng
27 sin
3
9
0
+ 9 sin
2
27
0
+ 3 sin
3
81
0
+ sin
3
243
0
= 20 sin 9
0
.
Li gii. T sin
3
x =
1
4
(3 sin x sin 3x) ta c
27
3 sin 9
0
sin 27
0
4
+ 9
3 sin 27
0
sin 81
0
4
+ 3
3 sin 81
0
sin 243
0
4
+
3 sin 243
0
sin 729
0
4
=
81 sin 9
0
sin 729
0
4
=
81 sin 9
0
sin 9
0
4
= 20 sin 9
0
.
Bi ton 5. Tnh (1 cot 1
0
)(1 cot 2
0
) . . . (1 cot 44
0
).
Li gii. Ta c ((1 cot 1
0
)(1 cot 2
0
) . . . (1 cot 44
0
)
=
_
1
cos t
0
sin t
0
__
1
cos 2
0
sin 2
0
_
....
_
1
cos 44
0
sin 44
0
_
=
(sin t
0
cos t
0
) (sin 2
0
cos 2
0
) ... sin 44
0
cos 44
0
_
sin t
0
sin 2
0
... sin 44
0
Dng ng thc sin a cos a =

2 sin a 45
0
_
ta a v

2 sin 1
0
45
0
_
2 sin 2
0
45
0
_
...

2 sin 44
0
45
0
_
sin 1
0
sin 2
0
... sin 44
0
=

2
_
44
(1)
44
sin 44
0
sin 43
0
...... sin 2
0
sin 1
0
sin 44
0
sin 43
0
... sin 2
0
sin 1
0
= 2
22
.
Bi ton 6. Chng minh
a)
1
2
cos

7
_ _
1
2
cos
3
7
_ _
1
2
cos
9
7
_
=
1
8
.
b)
1
2
+ cos

20
_ _
1
2
+ cos
3
20
_ _
1
2
+ cos
9
20
_ _
1
2
+ cos
27
20
_
=
1
16
.
80
Li gii.
a) rng 1 2 cos 2x 1 2(2 cos
2
x 1) = 3 4 cos
2
x =
cos 3x
cos x
Tch trn c bin i li thnh
_

1
2
_
3
cos
3
7
cos

7
cos
9
7
cos
3
7
cos
27
7
cos
9
7
=
1
8
cos
27
7
cos

7
=
1
8
.
b) Ta cng c 1 + 2 cos 2x = 1 + 2(1 2 sin
2
x) = 3 4 sin
2
x =
sin 3x
sin x
Tch trn c vit li thnh
1
2
4
sin
3
20
sin

20
sin
9
20
sin
3
20
sin
27
20
sin
9
20
sin
81
20
sin
27
20
=
1
16
sin
81
20
sin

20
=
1
16
.
Bi ton 7. Chng minh rng
a)
24

n=1
1
cos (2
n
)
0
= 2
24
. tan 2
0
b)
25

n=2
_
2 cos (2
n
)
0

1
cos (2
n
)
0
_
= 1.
Li gii.
a) Ta c
1
cos x
=
2 sin x
2 sin xcos x
= 2
sin x
sin 2x
p dng ta c
24

n=1
1
cos (2
n
)
0
= 2
24
24

n=1
sin (2
n
)
0
sin (2
n+1
)
0
= 2
24
sin 2
0
sin (2
25
)
0
Ta cn chng minh sin(2
2
5)
0
= cos 2
0
hay cn chng minh
2
2
5 2 90
.
.
. 180 2
2
3 23
.
.
. 45 = 5.9
Ta c 2(2
2
)
1
1 3 = 2(1)
1
1 3 = 0 (mod 5)
4(2
3
)
7
5 = 4(1)
7
5 = 0(mod 9) 2
2
3 23
.
.
. 45 pcm.
b) Ta c 2 cos x
1
cos x
=
2cos
2
x1
cos x
=
cos 2x
cos x
Suy ra
25

n=2
_
2 cos (2
n
)
0

1
cos (2
n
)
0
_
=
25

n=2
cos (2
n+1
)
0
cos (2
n
)
0
=
cos (2
26
)
0
cos 4
0
Vy ta cn chng minh
cos(2
2
6)
0
= cos 4
0
2
6
4 4
.
.
. 180 v 2
2
6
4
360
V 2
2
6 4 = 4(2
2
4 1)nn 2
2
6 4 chia ht cho 2
6
4 1 v cng chia ht cho 2
6
1. T ta c
c 2
2
6 4 = k(4.5.9) = k180 vi k l mt s l pcm.
81
2 Chng minh bt ng thc - Tm gi tr ln nht, gi
tr nh nht
Bi ton 8. Chng minh

1
2

(x + y) (1 xy)
(1 + x
2
) (1 +y
2
)

1
2
.
Li gii. t x = tan a v y = tan b. Ta c ngay 1 sin 2 (a +b) 1.
Bi ton 9. (USA MO 2002) Tm GTLN ca biu thc
S = (1 x
1
)(1 y
1
) + (1 x
2
)(1 y
2
)
Vi x
1
2
+x
2
2
= y
1
2
+ y
2
2
= c
2
vi c > 0.
Li gii. Ta thy 2 im c ta (x
1
; x
2
) v (y
1
; y
2
) nm trn ng trn (O;c) Ta c th
t (x
1
; x
2
) = (c. cos ; c sin ) v (y
1
; y
2
) = (c. cos ; c. sin ). Khi
S = 2 c(cos + sin + cos + sin ) + c
2
(cos . cos + sin . sin )
S = 2 +c

2
_
sin
_
+

4
_
sin
_
+

4
__
+c
2
cos ( )
S 2 + 2c

2 + c
2
=
_
c +

2
_
2
Du = xy ra khi = =
5
4
Vy max S = (c +

2)
2
khi x
1
= x
2
= y
1
= y
2
=

2
2
.c.
Bi ton 10. Chng minh a, b, c R, ta c
|a b|

1 + a
2

1 + b
2

|a c|

1 + a
2

1 + c
2
+
|b c|

1 + b
2

1 + c
2
.
Li gii. t a = tan ; b = tan ; c = tan ; , ,

2
;

2
_
Khi a
2
+ 1 =
1
cos
2

; b
2
+ 1 =
1
cos
2

; c
2
+ 1 =
1
cos
2

Bt ng thc tr nn n gin hn
|sin ( )| |sin ( )| +|sin ( )|
Ta chng minh bt ng thc ny nh sau
|sin ( )| = |sin ( + )|
= |sin ( )| cos( ) + sin( ) cos( )
|sin ( )| |cos( ) + sin( )| |cos( )|
|sin ( )| +|sin( )| .
( FIZMATLTT, Moscow, 2002)
82
Bi ton 11. Cho a, b, c R. Chng minh rng
(ab +bc +ca 1)
2
a
2
+ 1
_ _
b
2
+ 1
_ _
c
2
+ 1
_
.
Li gii. T a
2
+ 1; b
2
+ 1; c
2
+ 1 ta ngh n vic t a = tan u, b = tan v c =
tan vi u, , w

2
;

2
_
Bt ng thc tr thnh
1 (tan u tan + tan tan w + tan w tan u 1) cos u. cos . cos w 1
(103 Trigonometry Problems)
Bi ton 12. Chng minh rng
x

1 + x
2
+
y
_
1 + y
2
+
z

1 + z
2

3
2
Vi x + y +z = xyz v x, y, z R.
Li gii. t x = tan A, y = tan B, z = tan C vi A, B, C l 3 gc ca 1 .
Bi ton c a v sin A + sin B + sin C
3

3
2
.
Bi ton c gii quyt d dng bng BT Jensen vi f(x) = sin x, x 0;

2
_
.
Bi ton 13. Chng minh rng
x
1 x
2
+
y
1 y
2
+
z
1 z
2

3
2
Vi 0 < x, y, z < 1 v xy +yz +xz = 1.
Li gii. Nhn 2 cho hai v, BT tr thnh
2x
1x
2
+
2y
1 y
2
+
2z
1 z
2
3

3
T iu kin bi ton xy +yz +yz = 1 v 0 < x, y, z < 1, ta ngh n vic t x = tan
A
2
; y =
tan
B
2
; z = tan
C
2
vi A, B, C l 3 gc ca mt tam gic nhn. Bi ton c a v dng
tan A + tan B + tan C 3

3 (ng theo BT Jensen).


3 Gii phng trnh - Bt phng trnh
Bi ton 14. Tm a sao cho tn ti x tha mn

1 x
2
a x.
83
Li gii. t x = cos t, t [0, ]

1 x
2
= sin t
Biu thc tr thnh: sin t + cos t a, m sin t + cos t =

2cos t

4
_
Vy a khng vt qu

2
Bi ton 15. Cho 4 s phn bit trong khong (0; 1). Chng minh rng tn ti 2 s x, y tha
mn
0 < x
_
1 y
2
y

1 x
2
<
1
2
.
Li gii. Cho 4 s a
i
= sin b
i
, gc b
i
trong phn t th nht
Bi ton ch ra 2 ch s k, m sao cho 0 < sin b
k
cos b
m
sin b
m
cos b
k
<
1
2
Vy ch cn chng minh tn ti 2 ch s k, m sao cho b
k
> b
m
v b
k
b
m
<

6
iu ny c c do nguyn l Dirichlet s c 2 s trong 4 s nm trong 1 khong (0,

6
], (

6
,

6
],(

6
,

6
].
Bi ton 16. Gii phng trnh x
3
3x =

x + 2 trn tp s thc.
Li gii. H s ca x
3
l 1, biu thc x
3
3x lm ta lin tng 4 cos
3
x 3 cos x
Ta thy x > 2 th x
3
3x > 4x 3x = x >

x + 2
Vy 2 x 2. Ta t x = 2 cos vi [0; ]
Phng trnh tr thnh
2 cos 3 =
_
2(1 + cos ) = 2 cos

2
2 sin
72
4
sin
52
4
= 0
Suy ra = 0;
4
7
;
4
5
. Vy x = 2, 2 cos
4
7
;
1
2
(1 +
_
5).
Bi ton 17. (IMO Shortlisted 1995) Cho cc s thc dng a, b, c. Xc nh tt c cc s thc
dng x, y, z sao cho
x +y +z = a +b +c v 4xyz (a
2
x +b
2
y +c
2
z) = abc.
Li gii. T gi thit ta c
4 =
a
2
yz
+
b
2
zx
+
c
2
xy
+
abc
xyz
t x
1
=
a

yz
, y
1
=
b

zx
, z
1
=
c

xy
, ta c
4 = x
2
1
+y
2
1
+z
2
1
+x
1
y
1
z
1
Vi 0 < x
1
; y
1
; z
1
< 2. Xt phng trnh trn nh l mt phng trnh bc 2 theo bin x
1
, ta
c nh thc
4x
2
1
4y
2
1
iu ny gi ta t x
1
= 2 sin u, y
1
= 2 sin v; 0 < u, v <

2
. Khi phng trnh tr thnh
4 = 4sin
2
u + 4sin
2
v +z
2
1
+ 4 sin usin v.z
1
(z
1
+ 2 sin usin v)
2
= 4(1 sin
2
u)(1 sin
2
v)
|z
1
+ 2 sin usin v| = |2 cos ucos v| .
84
V z
1
, sin u, sin v u l nhng s dng nn ta suy ra
z
1
= 2(cos ucos v sin usin v) = 2 cos(u +v)
Nh vy ta c
a = 2

yz sin u, b = 2

xz sin v, c = 2

xycos(u +v)
T gi thit x + y +z = a +b +c ta c :

x cos v

y cos u
_
2
+

x sin v

y sin u

z
_
2
= 0
Suy ra

z =

x sin v +

y sin u =

x
y
1
2
+

y
x
1
2
.
Do

z =

x
b
2

xz
+

y
a
2

yz
z =
a+b
2
.
Tng t ta cng c y =
c+a
2
, x =
b+c
2
.
R rng (x, y, z) =
b+c
2
,
c+a
2
,
a+b
2
_
tha mn cc iu kin ca bi v cng l nghim duy
nht tm c.
Bi ton 18. Gii h phng trnh
_

_
x
3
3x = y
y
3
3y = z
z
3
3z = x
Li gii. t x = 2 cos u, y = 2 cos v, z = 2 cos w; u, v, w [0, ]
H phng trnh cho ta cos 27u = cos u c nghim
u =
k
14
, k = 0, 1, , 14, u =
k
13
, k = 1, , 12 nn
x = 2 cos
k
14
, y = 2 cos
3k
14
, z = 2 cos
9k
14
, k = 0, 1, , 14
V x = 2 cos
k
13
, y = 2 cos
3k
13
, z = 2 cos
9k
13
, k = 1, , 12
V h phng trnh cho ch c ti a 27 nghim, nn 27 nghim trn l nghim ca h.
Bi ton 19. Gii h phng trnh
_

_
2x +x
2
y = y
2y + y
2
z = z
2z + z
2
x = x
Li gii. Xt x = 1, x = 1,
Xt x = 1. t x = tan a, a n phng trnh tan 8a = tan a, a (/2, /2)
Kt qu nghim ca h l (tan
k
7
, tan
2k
7
, tan
4k
7
), k = 3, 2, 1, 0, 1, 2, 3.
Bi ton 20. Gii h phng trnh
_

_

_
x
1
1/x
1
= 2x
2
x
2
1/x
2
= 2x
3
x
3
1/x
3
= 2x
4
x
4
1/x
4
= 2x
1
85
Li gii. Lin h cng thc 2 cot 2a = cot a
1
cot a
t x
1
= cot a, a n phng trnh cot 16a = cot a, a (0, )
Kt qu nghim ca h l
x
1
= cot
k
15
, x
2
= t cot
2k
15
, x
3
= cot
4k
17
, x
4
= cot
8k
15
), k = 1, 2, , 14.
Bi ton 21. Gii h phng trnh
_


_


_
3x y
x 3y
= x
2
3y z
y 3z
= y
2
3z x
z 3x
= z
2
Li gii. Nu x = 0 x = y = z = 0 (loi)
Vy x, y, z = 0. H phng trnh c vit li thnh
_

_
y =
3xx
3
13x
3
z =
3vy
3
13y
2
x =
3zz
3
13z
2
t x = tan u(u

2
;

2
_
Suy ra x = tan u = tan 27u, suy ra u =
k
26
.Vi

2
<
k
26
<

2
hay k = 1; 2; .
..............................................................................................................................; 12
Vy h phng trnh c nghim
x = tan
k
26
; y = tan
3k
26
; z = tan
9k
26
Vi k = 1; 2; . ...; 12.
4 Lng gic v dy s
Bi ton 22. (Romani TST 1986) Cho dy a
n
tha

a
n+2
+ 2 a
n
2, n 1. Tm a
1986
.
Li gii. t a
n
= 2 cosb
n
, 0 b /2, do 0 a
n
2
T bt ng thc bn tri ta lin h cng thc cos 2a + 1 = 2 cos
2
a, ta c cos
b
n+2
2
cos b
n
suy
ra
b
n+2
2
b
n

b
n+2k
2
k
b
n
do quy np, v hm cos gim
V k tin n v hn nn b
n
= 0 mi n suy ra a
n
= 2, mi n.
Bi ton 23. (AIME 1996) Cho dy x
n
tha x
1
= x, x
n+1
=
1
1xn

1
1+xn
, n N
Khi x
n
= 1 hay x
n
= 1 th dy kt thc. Hi c bao nhiu dy c 8 s hng nh th?
p s: x = tan b, x8 = tan 128 b, C 256 dy tha iu kin.
Bi ton 24. Cho dy x
n
tha
x
n+1
=

3x
n
1
x
n
+

3
, n 1.
Chng minh dy tun hon.
86
Li gii. Lin h cng thc tan(a b), t x
1
= tan t suy ra x
n+6
= x
n
.
Bi ton 25. Cho a
0
=

2; b
0
= 2 v
a
n+1
=
_
2
_
4 a
2
n
; b
n+1
=
2bn
2+

4+b
2
n
; n 0
a) Chng minh dy (a
n
)
n
v (b
n
)
n
gim v tin v 0.
b) Chng minh dy (2
n
a
n
) tng, dy (2
n
b
n
) gim v hai dy ny tin n cng mt gii hn.
Li gii.
a) t a
n
= 2 sin t
n
, ta c
2 sin t
n+1
= a
n+1
=
_
2
_
4 4 sin t
2
n
=

2 2 cos t
n
= 2 sin
t
n
2
Suy ra t
n+1
=
tn
2
vi t =

4
, t t
n
=

2n+2
; n 0. Vy a
n
= 2 sin

2n+2
vi n 0
t b
n
= 2 tan u
n
(n 0, u
n

_
0;

2

).
Ta c 2 tan u
n+1
= b
n+1
=
2 tan un
2+

4+4tan
2
un
=
4 tan un
2+
2
cos un
= 2
sin un
1+cos un
= 2 tan
un
2
Suy ra u
n+1
=
vn
2
v n
0
=

4
, t c u
n
=

2
n+2
n 0.
Vy b
n
= 2 tan

2
n+2
vi n 0.
Tr li bi ton ta thy 2 dy(a
n
)
n
v (b
n
)
n
u gim v lima
n
= limb
n
= 0.
b) Ta c hm
sin x
x
tng v
tan x
x
gim trn khong xt
Do 2
n
a
n
=

2
.
sin

2
n+2

2
n+2
tng v 2
n
b
n
=

2
.
tan

2
n+2

2
n+2
gim.
Ta cng c lim
n>
2
n
a
n
=

2
lim
n>
.
sin

2
n+2

2
n+2
=

2
lim
n>0
sin x
x
=

2
Tng t lim
n>
2
n
b
n
=

2
.
Bi ton 26. Cho 2 dy (x
n
) v (y
n
)
(x
1
) = y
1
=

3; x
n+1
= 2
n
+
_
1 + x
2
n
; y
n+1
=
yn
1+

1+y
2
n
; n 1.
Chng minh rng 2 < x
n
y
n
< 3 n > 1.
Li gii. t x
n
= tan a
n
+
_
1 + tan
2
a
n
= tan a
n
+
1
cos an
=
1+sin an
cos an
= tan
_
90
0
+an
2
_
V a
1
= 60
0
, a
2
= 75
0
; a
3
= 82, 5
0
t ta c a
n
= 90
0

30
0
2
n1
Nn x
n
= tan(90
0

30
0
2
n1
) = cot(
30
0
2
n1
) = cot
n
vi
n
=
30
0
2
n1
Tng t ta cng c y
n
= tan 2
n
=
2 tan n
1tan
2
n
Suy ra x
n
y
n
=
z
1tan
2
n
V 0
0
<
n
< 45
0
nn 0 < tan
2

n
< 1 v x
n
y
n
> 2
Vi n > 1, ta c
n
< 30
0
nn tan
2

n
<
1
3
suy ra x
n
y
n
< 3. Vy 2 < x
n
y
n
< 3.
Bi ton 27. Vi n 0, cho U
n
= arcsin

n+1

n+2

n+1
.
Chng minh rng tng S = u
0
+u
1
+u
2
+. . . +u
n
c gii hn xc nh khi n tin n v cng.
87
Li gii. t b
n
=
1

n+1
, ta c sin u
n
=

n+1

n+2

n+1
= b
n
_
1 b
2
n+1
b
n+1
_
1 b
2
n
Vy S = lim
n
N

n=0
_
arcsin
1

n+1
arcsin
1

n+2
_
= arcsin 1 lim
n
arcsin
1

N+2
=

2
.
Bi ton 28. (MOSP 1996) Cho dy a
n
khng gim trong [-1, 1]. Chng minh
n1

i=1
_
1 a
i
a
i+1

_
(1 a
2
i
)(1 a
2
i+1
) <

2
2
.
Li gii. t a
i
= cos a
i
= cos x
i
; x
i
[0,] , i = 1, . . . , n.
Bin i v tri thnh

2
n1

i=1
sin
x
i+1
x
i
2
Do hm sin li xung trn on [0,], nn p dng bt ng thc Jensen ta c

2
n1

i=1
sin
x
i+1
x
i
2
(n 1)

2 sin
x
n
x
1
2(n 1)

2(n 1) sin

2(n 1)
, do x
n
x
1
(0; )
M khi x > 0, ta li c sin x < x nn ta c kt qu.
Bi ton 29. (China MO 1996) Cho x
0
, x
i
> 0, i = 1, .., n c tng cc s hng bng 1. Chng
minh
n

k=1
1

1+x
0
++x
k1

x
k
+xn
<

2
.
Li gii. t x
0
+x
1
+ +x
k
= sin a
k
, k = 0, 1, , n, a
0
= 0 < a
1
< < a
n
= /2
n

k=1
sin a
k
sin a
k1

1 + sin a
k1

1 sin a
k1
=
n

k=1
2 sin
a
k
a
k1
2
cos
a
k
+a
k1
2
cos a
k1
<

2
Hm cos x gim trong phn t th nht v sin x < x nn ta c kt qu.
Bi ton 30. Chng minh
N

n=0
1
1tan
2
2
n
= tan 1.
Li gii. Ta c tan 2x =
2 tan x
1tan
2
x
= tan 1.
Suy ra
N

n=0
1
1tan
2
2
n
=
N

n=0
tan 2
2 tan 2
n
=
2
N
tan 2
N
tan 1
V lim
x0
tan x
x
= 1 nn
2
N
tan 2
N
> 1 pcm.
Ti liu tham kho
[1] T. Andresscu,R, Gelca ,2000, Mathematical olympiad challenges, Birkhauser.
[2] T. Andresscu,R, Gelca ,2007, Putnam and beyond, Springer.
88
S DNG VNH CC S NGUYN GII MT
S BI TON S HC
Nguyn Trung Hng, Trng THPT Chuyn L Qy n Khnh Ha
Trong tp s nguyn, khi nim chia ht v khi nim s nguyn t l hai khi nim quan trng
nht. Bi vit ny quan tm n vic gii mt s bi ton S hc ca tp s nguyn nhng
khng ng trong n m da trn vic xy dng tnh cht s hc trn cc vnh. Ni dung chnh
l s dng tnh cht: s phn tch duy nht thnh cc phn t bt kh qui ca cc phn t
trn cc vnh Z[i] v vnh cc s nguyn ca Q
_

d
_
gii cc bi ton lin quan.
1 Cc khi nim v mt s tnh cht
1.1 Nhc li v l thuyt vnh
nh ngha 1. Mt vnh l mt tp hp cng vi hai php ton hai ngi, php cng v php
nhn tha: a) (R; +) l mt nhm Abel; b) (R; .) l mt v nhm; c) (a +b) c = ac+bc, a, b, c
K. Vnh R c gi l vnh giao hon nu php nhn c tnh chn giao hon.
nh ngha 2. Mt trng F l mt vnh giao hon trong mi phn t khc khng u c
phn t kh nghch, tc l vi mi x F, x = 0, tn ti x
1
F sao cho:
x.x
1
= 1.
nh ngha 3. Cho vnh R. Phn t a R, a = 0 c gi l c ca khng nu tn ti
a R, a = 0 sao cho: ab = 0 hoc ba = 0. Vnh giao hon R c gi l mt min nguyn nu
n khng c c ca khng. T y ta c khi nim chia ht trn mt min nguyn R:
nh ngha 4. Cho a, b R (b = 0) . Ta ni a chia ht cho b, k hiu a
.
.
. b (hoc b|a)) nu tn
ti q R sao cho: a = b.q . Hai phn t a = 0, b = 0 c gi l lin kt, k hiu a b , nu
v ch nu a|b v b|a.
nh ngha 5. Mt phn t p = 0 ca R c gi l bt kh qui nu n khc c ca n
v v khng c c thc s trong R, tc l nu p = ab vi a, b R th hoc a p, b 1 hoc
a 1, b p.
nh ngha 6. Mt phn t p = 0 ca R c gi l nguyn t nu n khc c ca n v
v p|ab p|a p|b.
89
nh ngha 7. Cho cc phn t a
1
, a
2
, ..., a
n
R . Mt c chung ca chng l mt phn t
chia ht mi phn t cho. c chung d ca a
1
, a
2
, ..., a
n
c gi l c chung ln nht ca
chng, k hiu d = (a
1
, a
2
, ..., a
n
) , nu n chia ht cho mi c chung ca a
1
, a
2
, ..., a
n
. T y
ta rt ra, nu d = (a
1
, a
2
, ..., a
n
) , th ch c cc phn t lin kt vi d l c chung ln nht ca
a
1
, a
2
, ..., a
n
.
nh ngha 8. Cc phn t a
1
, a
2
, ..., a
n
. ca R c gi l nguyn t cng nhau nu c
chung ln nht ca chng lin kt vi n v 1 ca R. Tip theo ta s nu mt s loi vnh c
bit:
nh ngha 9. Mt min nguyn R c gi l mt vnh nhn t ha nu mi phn t khc
khng u c s phn tch duy nht thnh cc phn t bt kh qui, ngha l vi mi a R,
tn ti c ca n v u v cc phn t bt kh qui p
i
(i = 1, 2, ..., r) sao cho:a = u
r

i=1
p
i
.
Ch : Trong vnh nhn t ha, nu ab = c
n
, vi nguyn t cng nhau th tn ti cc c
u
1
, u
2
ca n v v cc phn t c
1
, c
2
sao cho: a = u
1
c
n
1
, b = u
2
c
n
2
vi u
1
.u
2
= 1
nh ngha 10. Mt vnh chnh C l mt min nguyn m tt c cc ideal u l chnh, tc
l u c dng xC = (x) vi x C.
nh ngha 11. Min nguyn E c gi l mt vnh Euclide nu tn ti mt nh x g : E


sao cho:
a) g (ab) g (a) , a, b E

b) Vi hai phn t a v b = 0 ca E, tn ti hai phn t q v


r cng ca E sao cho:a = bq + r vi r = 0 hoc g (r) < g (b) . T y ta c cc kt qu quan
trng:
nh l 1. Mi vnh chnh u l vnh nhn t ha.
nh l 2. Mi vnh Euclide u l vnh chnh.
1.2 Vnh Z[i]
Tp [i] = {a + ib|a, b } c gi l tp cc s nguyn Gauss a + ib . Khi , Z[i] l mt
min nguyn. Trn Z[i] ta xy dng hm chun N : [i] cho bi: N (z) = z.z vi mi Do ,
vi z = a +ib th N (z) = a
2
+b
2
. T y ta rt ra:
N (z
1
.z
2
) = N (z
1
) .N (z
2
) , z
1
, z
2
Z[i]
Mt s kt qu trong vnh Z[i]
nh l 3. Nu z
1
, z
2
Z[i] , z
2
= 0 th
a) N (z
1
z
2
) N (z
1
)
b) Tn ti q, r Z[i] sao cho z
1
= qz
2
+r v N (r) < N (z
2
) .
V vy vnh Z[i] l mt vnh Euclide.
nh l 4. Tp cc c ca n v trong Z[i] l U ([i]) = {i; 1; 1; i} .
90
nh l 5. Cc s nguyn t trong Z[i] l:
a) 1 + i;
b) Cc s nguyn t p ca N c dng 4k + 3
c) Cc s a + bi vi a, b Z v a
2
+ b
2
l s nguyn t.
1.3 Vnh cc s nguyn ca trng Q
_

d
_
Xt trng Q
_

d
_
vi d l s nguyn dng khng chnh phng. Vi z = a + b

d, ta k
d hiu z = a b

d. Khi , hm chun N : Q
_

_
N cho bi:
N (z) = |z.z| =

a
2
db
2

.
T y ta rt ra:
a) N (z
1
z
2
) = N (z
1
) .N (z
2
) , z
1
, z
2
Q
_

d
_
b) z
1
z
2
= z
1
.z
2
, z
1
, z
2
Q
_

d
_
.
Ta c kt qu quan trng sau y:
nh l 6. Nu d 2, 3 (mod 4) , th vnh cc s nguyn trong Q
_

d
_
l
_

d
_
= Q+Q

d.
Nu d 1 (mod 4) , th vnh cc s nguyn trong Q
_

d
_
l Z
_
1+

d
2
_
= Z + Z
_
1+

d
2
_
.
Trng hp d < 0 , ta c kt qu sau:
nh l 7. Vnh cc s nguyn trong Q
_

d
_
, vi d < 0 , l mt vnh nhn t ha khi
d {1; 2; 3; 7; 11; 19; 43; 67; 163} .
i vi d > 0 ta cha bit ht cc trng hp. Ta c th nu i din y mt s trng hp
chng hn d = 2, 3, 5, 6, 7, 11, ...
2 Mt s bi ton s hc
2.1 S dng vnh Z[i]
Bi ton 1. Gii phng trnh tm nghim nguyn: y
7
2x = x
2
+ 2
Li gii. Vit phng trnh dng:
(x + 1 + i) (x + 1 i) = y
7
91
Ta c nhn xt rng y l v nu ngc li th (x + 1)
2
1 (mod 4) : mu thun. Do , x l
v ko theo x + 1 + i, x + 1 i l nguyn t cng nhau trong vnh Z[i] .
V (x + 1 + i) (x + 1 i) = y
7
nn tn ti a, b Z sao cho:
x + 1 + i = u(a + ib)
7
= u
_
a
_
a
6
C
2
7
a
4
b
2
+ C
4
7
a
2
b
4
C
6
7
b
6
_
+ib C
1
7
a
6
C
3
7
a
4
b
2
+C
5
7
a
2
b
4
C
7
7
b
6
_
,
vi u l c ca n v. Suy ra |a (a
6
C
2
7
a
4
b
2
+C
4
7
a
2
b
4
C
6
7
b
6
)| = 1 hoc

b
_
C
1
7
a
6
C
3
7
a
4
b
2
+C
5
7
a
2
b
4
C
7
7
b
6
_
= 1
a = 0; b = 1 hoc b = 0; a = 1. Vy nghim nguyn ca phng trnh l: (1, 1) .
Nhn xt: im mu cht gii bi ton ny l chng minh cx + 1 + i v x + 1 i l
nguyn t cng nhau. Do , s dng c s phn tch duy nht thnh cc phn t bt kh
qui trong Z[i] t rt ra dng x + 1 + i = u(a +ib)
7
.
Bi ton 2. Gii phng trnh tm nghim nguyn: x
2
+ 9 = y
p
, vi p l mt s nguyn t c
dng 4k + 3.
Li gii. Vit phng trnh dng:
(3 + xi) (3 xi) = y
p
.
V (x + 3i, x 3i) = 1 nn 3 + xi = (a + ib)
p
. Khai trin nh thc ta c:
3 = a
p
C
2
p
a
p2
b
2
+... + (1)
p1
2
C
p1
p
ab
p1
.
Suy ra a|3 v 3 a
p
(mod p) .
Theo nh l Fermat nh a
p
a (mod p) , suy ra a = 3. T ,
1 = 3
p1
C
2
p
3
p3
b
2
+... + (1)
p1
2
C
p1
p
b
p1
.
Suy ra, 3
p1
1 = C
2
p
3
p3
b
2
C
4
p
3
p5
b
4
+... (1)
p1
2
C
p1
p
b
p1
. (*)
Do p = 4k + 3 , nn t (*) suy ra b
2
chn. Ta li c 3
p1
1 chia ht cho 2
3
v khng chia ht
cho 2
4
v l. Do , phng trnh v nghim.
Bi ton 3. Cho a, b, c l cc s nguyn tha: a = b
2
c v c khng chia ht cho bt k s nguyn
t p 3 (mod 4) . Chng minh rng a l tng ca hai s chnh phng.
Li gii. Xt s nguyn t p trong Z v gi = a +ib l s nguyn t trong Z[i] sao cho |p.
Khi , N () |N (p) = p
2
. Suy ra, N () = p hoc N () = p
2
V do ,
a
2
+ b
2
= p hoc a
2
+b
2
= p
2
Trng hp p 1 (mod 4) , gi s p = 4k +1, th s dng nh l Wilson ta chng minh c
p| (n
2
+ 1) , vi n = (2k)! . Do , | (n +i) (n i) . Suy ra:
|n +i hoc |n i.
92
Mt khc, nu p|(n i) th p|n v p|1 : v l. Do , N (p) = N () . Suy ra,
a
2
+b
2
= p.
Trng p = 2 th hin nhin p l tng hai s chnh phng. Do c khng chia ht cho bt k s
nguyn t p 3 (mod 4) , nn c l tch cc s nguyn t p 1; 2 (mod 4) . Suy ra, tn ti
t, r Z sao cho: c = N (t +ir) = t
2
+r
2
. T ,
a = b
2
c = N (b) .N (t +ir) = N (tb + irb) = (tb)
2
+ (rb)
2
.
2.2 S dng vnh cc s nguyn ca Q
_

d
_
Bi ton 4. Tm tt c cc cp s nguyn dng (x, y) sao cho: 13
x
+ 3 = y
2
Li gii. Bin i phng trnh v dng:
_
y

3
__
y +

3
_
=
_
4

3
_
x
_
4 +

3
_
x
.
Trong vnh Z
_
3

, gi s c s nguyn t p Z
_
3

sao cho
p|
_
y

3
_
, p|
_
y +

3
_
,
suy ra:
N (p) |N
_
y +

3
_
=

y
2
3

= 13
x
Mt khc, p|2

3 nn N (p) |N 2

3
_
= 12. T , N (p) | (12, 13
x
) = 1. Suy ra:N (p) = 1 : v
l. V vy, y

3, y +

3
_
= 1 , v do y +

3 l s bc x.
Ngoi ra, 4

3, 4 +

3
_
= 1 nn
y +

3 =
_
4 +

3
_
x
=
x

k=0
C
k
x
4
xk
_

3
_
k
1 =

k
C
2k+1
x
3
k
4
x(2k+1)
= x4
x1
+

k=0
C
2k+1
x
3
k
4
x(2k+1)
x = 1 y = 4.
Vy nghim nguyn dng ca phng trnh l: (1, 4) .
Bi ton 5. Gii phng trnh tm nghim nguyn: x
2
+x + 2 = y
3
Li gii. Vit phng trnh dng:
2x + 1

7
2

2x + 1 +

7
2
= y
3
93
V
_
2x+1

7
2
,
2x+1+

7
2
_
= 1 trong vnh cc s nguyn ca Q
_
7

nn c a, b Z sao cho:
2x + 1 +

7
2
=
_
a +b

7
2
_
3

Do , 3a
2
b 7b
3
= 4 b|4 b = 1; a = 1. Vy tt c cc nghim nguyn ca phng
trnh l: (2, 2) v (3, 2)
Ch : Do 7 1 (mod 4) nn vnh cc s nguyn ca Q
_
7

l Z
_
1+

7
2
_
Bi ton 6. (VMO, 2010) Chng minh rng vi mi s nguyn dng n, phng trnh: x
2
+
15y
2
= 4
n
c t nht n nghim t nhin (x, y) .
Li gii. Ta chng minh bi ton bng phng php quy np: Vi n = 1 phng trnh c
nghim (2, 0)
Vi n = 2 phng trnh c nghim (4, 0), (1, 1)
Ch rng, nu (x
0
, y
0
) l nghim ca phng trnh x
2
+15y
2
= 4
n
th (2x
0
, 2y
0
) l nghim ca
phng trnh x
2
+15y
2
= 4
n+1
. Do , ta ch cn chng minh phng trnh c nghim t nhin
l vi n 2. Gi s vi n 2, c cp s nguyn dng l (x
n
, y
n
) sao cho: x
2
n
+ 15y
2
n
= 4
n
. Xt
vnh Z
_
1+

15
2
_
, ta c:
N x
n
+

15y
n
_
= N x
n

15y
n
_
= x
2
n
+ 15y
2
n
= 4
n
.
Mt khc, N
_
1+

15
2
_
= 4
N
_
_
x
n
+

15y
n
_

1+

15
2
_
= N
_
_
x
n

15y
n
_

1+

15
2
_
= 4
n+1

xn15yn
2
_
2
15
xn+yn
2
_
2
=
xn+15yn
2
_
2
15
xnyn
2
_
2
= 4
n+1
Do x
n
, y
n
cng l nn mt trong 2 s
xn+yn
2
,
xnyn
2
phi l. Nu
xn+yn
2
l th
xnyn
2
chn v do
xn15yn
2
l, tng t cho trng hp cn li. Vy trong cc trng hp, hoc

xn15yn
2

,

xn+yn
2
_
hoc

xn+15yn
2

,

xnyn
2
_
l nghim t nhin l ca phng trnh:
x
2
+ 15y
2
= 4
n+1
.
Vy phng trnh cho c t nht n nghim t nhin.
Mt s bi ton khc
Bi ton 7. Gii cc phng trnh nghim nguyn sau:
a)x
5
1 = y
2
b)x
2
+ 2 = y
3
Bi ton 8. Gii cc phng trnh sau tm nghim nguyn vi n l mt s nguyn ln hn 1:
a)x
2
+ 1 = y
n
b)x
2
+ 4 = y
n
c)x
2
+ 11 = 3
n
94
Bi ton 9. Chng minh rng phng trnh x
2
+y
2
= p c nghim khi v ch khi p 1 (mod 4)
Bi ton 10. Gi s rng x, y, z l cc s t nhin tha mn xy = z
2
+ 1. Chng minh rng
tn ti cc s nguyn dng a, b, c, d sao cho x = a
2
+b
2
, y = c
2
+ d
2
v z = ac + bd.
Bi ton 11. Tm tt c cc nghim nguyn dng ca phng trnh x
2
+y
2
= z
2
.
Bi ton 12. (American Mathematical Monthly) Cho p = 4m1 l mt s nguyn t v x, y
l cc s nguyn t cng nhau sao cho
x
2
+ y
2
= z
2m
.
vi s nguyn z no . Chng minh rng p|xy.
Bi ton 13. (Romanian Mathematical Olympiad) Cho S l tp cc s nguyn dng c dng
a
2
+ 2b
2
, vi a, b l cc s nguyn v b = 0 . Chng minh rng nu l mt s nguyn t v
p
2
S th p S
Bi ton 14. Cho p 1 (mod 6) , chng minh rng tn ti a, b Z sao cho
p = a
2
ab +b
2
.
Bi ton 15. (IMO, 2001) Cho a > b > c > d l cc s nguyn dng v gi s rng:
ac + bd = (b +d + a c) (b + d a +c) .
Chng minh rng ab +cd khng l s nguyn t.
Bi ton 16. Xt dy s (u
n
) cho bi: u
1
= 2 v u
n+1
= 2u
2
n
1, n 1. Chng minh rng
nu c mt s l p sao cho p|a
n
, th p 1 (mod 2
n+2
) .
Ti liu tham kho
[1] M. Ram Murty, J. Esmonde, in Algebraic Number Theory, Springer, 2004.
[2] Serge Lang, i s, Phn I, Nh xut bn i hc v Trung hc Chuyn nghip, 1978.
[3] Ng Thc Lanh, i s v s hc, Tp 2, Nh xut bn Gio dc, 1986.
[4] T. Andreescu, D. Andrica, I. Cucurezeanu, An introduction to Diophantine Equations, ,
2010.
95
NI SUY THEO YU T HNH HC CA TH
Phm Th Thy Hng, Trng THPT So Nam, Qung Nam
Bi ton xc nh biu thc ca mt hm s khi bit gi tr ca hm s (hoc gi tr ca cc
o hm ca hm s) ti mt s im cho trc gi l bi ton ni suy. Cc bi ton ni suy
v cc vn lin quan n n l mt phn quan trng ca i s v gii tch ton hc. Cc
bi ton ni suy c v tr c bit quan trng trong ton hc khng ch nh nhng i tng
nghin cu m cn ng vai tr nh mt cng c c lc ca cc m hnh lin tc cng nh
cc m hnh ri rc ca gii tch trong l thuyt phng trnh, l thuyt xp x,. . .
S dng hm (a thc) ni suy P(x), ta d dng tnh c gi tr ca hm s f(x) ti x R
bt k tng i chnh xc. T ta c th tnh gn ng o hm v tch phn ca n trn
R. V cc a thc i s l hm s n gin nht, nn trc tin ta ngh n vic xy dng
P(x) dng a thc i s.
Tuy nhin, trng ph thng th cc bi ton ni suy cn rt mi m v b ng ngay c
i vi gio vin ging dy ton. V vy, vic i tm li gii bi ton ni suy l nim say m ca
khng t ngi, c bit l nhng ngi dy ton v hc ton. Cc bi ton ni suy a dng v
ti, phong ph v chng loi, v ph hp cho mi i tng mi cp hc.
1 Cc bi ton ni suy c in
1.1 Ni suy Lagrange
nh l 1 (ng nht thc Lagrange). Nu x
1
, x
2
, . . . , x
m
l m (m > 1) gi tr ty , i mt
khc nhau v f(x) l a thc bc nh thua m th ta c ng nht thc sau
f(x) = f(x
1
)
(x x
2
)(x x
3
) . . . (x x
m
)
(x
1
x
2
)(x
1
x
3
) . . . (x
1
x
m
)
+
+f(x
2
)
(x x
1
)(x x
3
) . . . (x x
m
)
(x
2
x
1
)(x
2
x
3
) . . . (x
2
x
m
)
+ +f(x
m
)
(x x
1
)(x x
2
) . . . (x x
m1
)
(x
m
x
1
)(x
m
x
2
) . . . (x
m
x
m1
)
(1)
1.2 Bi ton ni suy Lagrange
Bi ton 1 (Bi ton ni suy Lagrange). Cho x
0i
, a
0i
R, vi x
0i
= x
0j
i = j, (i, j =
1, 2, . . . , N). Hy xc nh a thc L(x) c bc khng qu N 1 (deg L(x) N 1) tha mn
iu kin
L(x
0i
) = a
0i
, i = 1, 2 . . . , N. (2)
96
1.3 Ni suy Taylor
Bi ton 2 (Bi ton ni suy Taylor). Cho x
0
, a
k
R, vi k = 0, 1, . . . , N 1. Hy xc nh
a thc T(x) c bc khng qu N 1 (deg P(x) N 1) v tha mn cc iu kin:
T
(k)
(x
0
) = a
k
, k = 0, 1, . . . , N 1. (3)
nh l 2 (Taylor). Gi s f : U(a, ) R l hm kh vi lin tc n cp n 1 trong - ln
cn U(a, ) ca im a v c o hm hu hn cp n ti im a. Khi , hm f c th biu
din di dng
f(x) =
n

k=0
f
(k)
(a)
k!
(x a)
k
+o((x a)
n
) (4)
khi x a, trong 0! = 1, f
(0)
(a) = f(a).
1.4 Ni suy Newton
Bi ton 3 (Bi ton ni suy Newton). Cho x
i
, a
i
R, vi i = 1, 2, . . . , N. Hy xc nh a
thc N(x) c bc khng qu N (deg N(x) N 1) v tha mn cc iu kin:
N
(i1)
(x
i
) = a
i
, i = 1, 2, . . . , N. (5)
1.5 Ni suy Hermite
Trong mt s trng hp, ta cn tm hm a thc khng nhng i qua cc im cho trc
m cn phi tha mn iu kin v o hm ti cc im . Ta gi a thc nh vy l a thc
ni suy Hermit.
Bi ton 4 (Ni suy Hermite). Cho x
i
, a
ki
R, vi i = 1, 2, . . . , n; k = 0, 1, . . . , p
i
1
v x
i
= x
j
i = j, trong p
1
+ p
2
+ + p
n
= N. Hy xc nh a thc H(x) c bc
deg H(x) N 1 tha mn iu kin
H
(k)
(x
i
) = a
ki
, i = 1, 2, . . . , n; k = 0, 1, . . . , p
i
1. (6)
2 Ni suy theo h thng cc im cc tr ca th
nh l 3. Gi s f(x) kh vi n cp 2 lin tc trn mt khong cha x
0
, f

(x
0
) = 0 v
f

(x
0
) = 0. Khi
Nu f

(x
0
) > 0 th hm s t cc tiu ti x
0
.
Nu f

(x
0
) < 0 th hm s t cc i ti x
0
.
Bi ton 5 (M rng nh l). Gi s f(x) kh vi n cp n ti x
0
v gi s
f

(x
0
) = f

(x
0
) = = f
n1
(x
0
) = 0, f
n
(x
0
) = 0.
Khi
97
Nu n chn th f(x) t cc tr ti x
0
. Hn na nu f
n
(x
0
) > 0 th f(x) t cc tiu ti
x
0
, nu f
n
(x
0
) < 0 th f(x) t cc i ti x
0
.
Nu n l th f(x) khng t cc tr ti x
0
.
Nhn xt. Hm f(x) t cc tr ti x = x
0
th f

(x
0
) = 0 hoc f

(x
0
) khng tn ti. Nhng
im nh th c gi l im dng ca hm s.
2.1 Ni suy theo h thng cc im dng bc mt
Bi ton 6 (im dng n). Xc nh a thc P(x) sao cho th ca n c cc im dng
n ti x
1
, x
2
, . . . , x
n
(x
1
< x
2
< < x
n
)
P

(x
1
) = P

(x
2
) = = P

(x
n
) = 0.
(P

(x
1
) = 0, . . . , P

(x
n
)),, tc l
P

(x) = a(x x
1
)(x x
2
) . . . (x x
n
).
Gii. Ta vit
P

(x) = a(x x
1
)(x x
2
) . . . (x x
n
)
= a
0
+a
1
x + +a
n
x
n
=
n

k=0
a
k
x
k
,
trong
_



_
a
n
= a
a
n1
= (1)a(x
1
+x
2
+ +x
n
)
a
n2
= (1)
2
(x
1
x
2
+ x
1
x
3
+ +x
n1
x
n
)
.
.
.
a
nk
= (1)
k
a(x
1
x
2
. . . x
k
+x
1
x
2
. . . x
k1
x
k+1
+ +
+x
1
x
k+2
. . . x
n1
x
n
+ x
2
x
3
. . . x
k+1
+ + x
k+1
x
k+2
. . . x
n
)
.
.
.
a
1
= (1)
n1
a(x
1
x
2
. . . x
n1
+ x
1
x
2
. . . x
n2
x
n
+ + x
2
x
3
. . . x
n
)
a
0
= (1)
n
ax
1
x
2
. . . x
n
.
Suy ra
P(x) = a
0
x +a
1
x
2
2
+ +a
n
x
n+1
n + 1
+ C
1
=
n

k=0
1
k + 1
a
k
x
k+1
+C
1
, C
1
: hng s.
98
Bi ton 7 (im dng bi). Xc nh a thc P(x) sao cho th ca n c cc im dng
ti x
1
, x
2
, . . . , x
n
(x
1
< x
2
< < x
n
), sao cho
P

(x) = a(x x
1
)
2
1
+1
(x x
2
)
2
2
+1
. . . (x x
n
)
2n+1
.
Gii. t (2
1
+ 1) + (2
2
+ 1) + + (2
n
+ 1) = 2(
1
+ +
n
) + n = N. Ta vit
P

(x) = a(x x
1
)
2
1
+1
(x x
2
)
2
2
+1
. . . (x x
n
)
2n+1
= a(x t
1
) (x t
2
1
+1
) (x t
2
1
+2
) (x t
2(
1
+
2
)+2)
)
(x t
2(
1
+
2
)+3)
) (x t
2(
1
+
2
+
3
)+3)
)
(x t
2(
1
+
n1
)+n
) (x t
N
)
= a
0
+a
1
x + +a
N
x
N
=
N

k=0
a
k
x
k
,
trong
_

_

_
t
1
= t
2
= = t
2
1
+1
= x
1
t
2
1
+2
= t
2
1
+3
= = t
2(
1
+
2
)+2
= x
2
.
.
.
t
2(
1
+
n1
)+(n1)+1
= t
2(
1
+
n1
)+(n1)+2
= = t
N
= x
n
Suy ra
P(x) =
N

k=0
a
k
k + 1
x
k+1
+ C, C : hng s,
vi _




_




_
a
0
= (1)
N
at
1
. . . t

1
. . . t
N
a
1
= (1)
N1
a(t
1
t
2
. . . t
N1
+ t
1
t
2
. . . t
N2
t
N
+ +t
2
t
3
. . . t
N
)
.
.
.
a
k
= (1)
Nk
a(t
1
t
2
. . . t
Nk
+t
1
t
2
. . . t
Nk1
t
Nk+1
+ +t
2
t
3
. . . t
N
)
.
.
.
a
N1
= (1)a(t
1
+ t
2
+ + t

1
+ + t
N1
+t
N
) =
= (1)a(
1
x
1
+
2
x
2
+ +
n
x
n
)
a
N
= 1
2.2 p dng
Bi ton 8. Xc nh a thc P(x) bc nh nht vi h s bc cao nht bng 1 sao cho th
hm s y = P(x) nhn x = 1, x = 2 l cc im cc tr.
Bi ton 9. Xc nh a thc P(x) bc nh nht vi h s bc cao nht bng 1 sao cho th
hm s y = P(x) nhn x = 1, x = 2, x = 3, x = 4 l cc im cc tr.
99
Bi ton 10. Xc nh a thc P(x) bc nh nht vi h s bc cao nht bng 1 sao cho
th hm s y = P(x) nhn x = 1, x = 2, x = 3, x = 4 l cc im cc tr v i qua im A(0, 1).
Bi ton 11. Xc nh a thc P(x) bc nh nht sao cho th hm s y = P(x) c cc
im cc i, cc tiu ln lt ti A(0, 1) v B(1, 0).
Bi ton 12. Xc nh a thc bc ba P(x) tha mn iu kin nhn im M(1; 1) lm tm
i xng v A(0, 1) l im cc tiu.
Bi ton 13. Xc nh a thc P(x) bc nh nht sao cho th hm s y = P(x) c h s
cao nht bng 1 v cc im cc tiu ln lt ti A(0, 1) v B(1, 1).
Bi ton 14. Xc nh a thc P(x) bc nh nht sao cho th hm s y = P(x) ln lt
t cc i, cc tiu ti A(2, 1) v B(1, 3).
Bi ton 15. Xc nh a thc P(x) bc nh nht sao cho th hm s y = P(x) c h s
cao nht bng 1 v cc im cc tiu ln lt ti A(0, 1) v B(1, 1).
Bi ton 16. Xc nh a thc P(x) bc nh nht sao cho th hm s y = P(x) c cc
im cc i ln lt ti A(1, 0) v B(2, 0).
Bi ton 17. Xc nh a thc P(x) bc nh nht sao cho th hm s y = P(x) c h s
cao nht bng 1, i qua im A(2, 0) v c cc im cc i ln lt ti B(0, 0) v C(1, 0).
Bi ton 18. Xc nh a thc P(x) bc nh nht sao cho th hm s y = P(x), i qua
im A(2, 0) v c cc im cc i ln lt ti B(1, 0) C(0, 0) v D(1, 0).
3 Ni suy theo h thng cc im un ca th
nh ngha 1. Nu hm s f(x) c o hm cp hai trn mt khong cha im x
0
, f

(x
0
) = 0
v f

(x) i du khi x qua im x


0
th M(x
0
; f(x
0
)) l mt im un ca th hm s y = f(x).
Bi ton 19. Thay im dng bc 1 bi im dng bc 2 (P

(x
1
) = 0, P

(x
2
) = 0, . . . , P

(x
n
)) =
0.
TH1. im dng n.
P

(x) = a(x x
1
)(x x
2
) . . . (x x
n
).
Ta vit
P

(x) = a(x x
1
)(x x
2
) . . . (x x
n
)
= a
0
+ a
1
x + + a
n
x
n
=
n

k=0
a
k
x
k
,
100
trong
_



_



_
a
0
= (1)
n
ax
1
x
2
. . . x
n
a
1
= (1)
n1
a(x
1
x
2
. . . x
n1
+x
1
x
2
. . . x
n2
x
n
+ + x
2
x
3
. . . x
n
)
.
.
.
a
k
= (1)
nk
a(x
1
x
2
. . . x
nk
+x
1
x
2
. . . x
nk1
x
n
+ +x
2
x
3
. . . x
n
)
.
.
.
a
n1
= (a)(x
1
+x
2
+ +x
n
)
a
n
= a
Suy ra
P

(x) = a
0
x +a
1
x
2
2
+ + a
n
x
n+1
n + 1
+C
1
=
n

k=0
1
k + 1
a
k
x
k+1
+C
1
,
hay
P(x) =
n

k=0
a
k
1
k + 1
1
k + 2
x
k+2
+C
1
x + C
2
TH2. im dng bi
P

(x) = a(x x
1
)
2
1
(x x
2
)
2
2
. . . (x x
n
)
2n
.
Gii. t 2
1
+ 2
2
+ + 2
n
= 2(
1
+ +
n
) = N.
Ta vit
P

(x) = a(x x
1
)
2
1
(x x
2
)
2
2
. . . (x x
n
)
2n
= a(x t
1
) (x t
2
1
) (x t
2
1
+1
) (x t
2(
1
+
2
)
)
(x t
2(
1
+
2
)+1)
) (x t
2(
1
+
2
+
3
)
)
(x t
2(
1
+
n1
)+1
) (x t
N
)
= a
0
+a
1
x + +a
N
x
N
=
N

k=0
a
k
x
k
,
trong
_

_

_
t
1
= t
2
= = t
2
1
= x
1
t
2
1
+1
= t
2
1
+2
= = t
2(
1
+
2
)
= x
2
.
.
.
t
2(
1
+
n1
)+1
= t
2(
1
+
n1
)+2
= = t
N
= x
n
Suy ra
P

(x) =
N

k=0
a
k
k + 1
x
k+1
+C
1
,
101
hay
P(x) =
N

k=0
a
k
1
k + 1
1
k + 2
x
k+2
+C
1
x + C
2
,
vi
_




_




_
a
0
= (1)
N
at
1
. . . t

1
. . . t
N
a
1
= (1)
N1
a(t
1
t
2
. . . t
N1
+ t
1
t
2
. . . t
N2
t
N
+ +t
2
t
3
. . . t
N
)
.
.
.
a
k
= (1)
Nk
a(t
1
t
2
. . . t
Nk
+t
1
t
2
. . . t
Nk1
t
Nk+1
+ +t
2
t
3
. . . t
N
)
.
.
.
a
N1
= (1)a(t
1
+t
2
+ + t

1
+ +t
N1
+ t
N
) =
= (1)a(
1
x
1
+
2
x
2
+ +
n
x
n
)
a
N
= 1
Bi ton 20. Xc nh a thc P(x) bc nh nht sao cho th hm s y = P(x) c h s
cao nht bng 1, i qua A(0, 1) v im un ti A(1, 1).
Bi ton 21. Xc nh a thc P(x) bc nh nht sao cho th hm s y = P(x) i qua
im A(1, 0) v im un ti A(1, 0).
Bi ton 22. Xc nh a thc P(x) bc nh nht sao cho th hm s y = P(x) i qua
im A(1, 0) v c cc im un ln lt ti B(0, 0) v C(0, 1).
Bi ton 23. Xc nh a thc P(x) bc nh nht sao cho th hm s y = P(x) c cc
im un ln lt ti A(2, 0), B(0, 0), C(3, 0) v i qua D(1

0).
4 Ni suy theo h thng cc cc tr v im un ca
th
Bi ton 24. Xc nh a thc P(x) bc nh nht sao cho th hm s y = P(x) c cc
im un v im cc tiu ln lt ti A(0, 0) v B(1, 0).
Gii. V th hm s y = P(x) c im un v im cc tiu ln lt ti x = 0, x = 1 v t
gi thit P(0) = P(1) = 0 nn theo nh l Rolle, ta tm c trong khong (0, 1) im x
0
sao
cho P

(x
0
) = 0. Do vy phng trnh P

(x) = 0 phi c t nht 4 nghim. Suy ra deg P

(x) 4.
Nu deg P

(x) = 4, th phng trnh P(x) = 0 c 5 nghim. Khi , theo gi thit th


_
P(0) = P

(0) = 0
P(1) = P

(1) = 0
Khi theo bi ton ni suy Hermite
P(x) = ax
3
(x 1)
2
, a = 0.
102
th hm s c im un v cc tiu ln lt ti A, B th a > 0.
Vy a thc bc nm c dng
P(x) = ax
3
(x 1)
2
, a > 0.
Th li, ta thy cc iu kin m bo th hm s y = P(x) c im un v im cc
tiu ln lt ti A(0, 0) v B(1, 0) l tha mn. Vy a thc P(x) bc nh nht sao cho th
hm s y = P(x) c im un v im cc tiu ln lt ti A(0, 0) v B(1, 0) l a thc
P(x) = ax
3
(x 1)
2
, a > 0.
Bi ton 25. Xc nh a thc P(x) bc nh nht sao cho th hm s y = P(x) c cc
im un v im cc i ln lt ti A(0, 0) v B(1, 0).
Gii. V th hm s y = P(x) c cc im un v im cc i ln lt ti x = 0, x = 1 v
t gi thit P(0) = P(1) = 0 nn theo nh l Rolle, ta tm c trong khong (0, 1) im x
0
sao
cho P

(x
0
) = 0. Do vy phng trnh P

(x) = 0 phi c t nht 4 nghim. Suy ra deg P

(x) 4.
Nu deg P

(x) = 4, th phng trnh P(x) = 0 c 5 nghim. Khi , theo gi thit th


_
P(0) = P

(0) = 0
P(1) = P

(1) = 0
Khi theo bi ton ni suy Hermite
P(x) = ax
3
(x 1)
2
, a = 0.
th hm s c im un v cc tiu ln lt ti A, B th a < 0.
Vy a thc bc nm c dng
P(x) = ax
3
(x 1)
2
, a < 0.
Th li, ta thy cc iu kin m bo th hm s y = P(x) c cc im un v im
cc tiu ln lt ti A(0, 0) v B(1, 0) l tha mn. Vy a thc P(x) bc nh nht sao cho
th hm s y = P(x) cc im un v im cc tiu ln lt ti A(0, 0) v B(1, 0) l a thc
P(x) = ax
3
(x 1)
2
, a < 0.
Bi ton 26. Xc nh a thc P(x) bc nh nht sao cho th hm s y = P(x) c cc
im un ti A(2, 0), B(1, 0) v t cc i ti C(0, 0).
Bi ton 27. Xc nh a thc P(x) bc nh nht sao cho th hm s y = P(x) c cc
im un ti A(2, 0), B(1, 0) v t cc tiu ti cc im C(1, 0), D(0, 0).
103
5 Mt s bi ton lin quan
Bi ton 28. Cho hm s f(x) lin tc trn [a, b], kh vi trn (a, b) v f(a) = f(b) = 0. Chng
minh rng vi mi k R

, phng trnh
f(x) + kf

(x) = 0 (7)
lun c t nht mt nghim x (a, b).
Chng minh. Xt hm s
g(x) = e
x
k
f(x).
Ta c g(x) lin tc trn [a, b], kh vi trn (a, b) v g(a) = g(b) = 0. Theo nh l Rolle, phng
trnh g

(x) = 0 lun c t nht mt nghim x (a, b). M


g

(x) =
1
k
e
x
k
f(x) + e
x
k
f

(x) =
1
k
e
x
k
[f(x) + kf

(x)].
V
1
k
e
x
k
= 0, x R nn
g

(x) = 0 f(x) + kf

(x) = 0.
Vy phng trnh (3.1) lun c t nht mt nghim x (a, b). y l iu cn chng minh.
Bi ton 29. Chng minh rng nu lim
x
f(x) = 0 th f

(x) c s lng cc khng im trong


khong (a, ) khng t hn so vi f(x) trn khong y. Kt qu vn ng nu thay + bi
.
Bi ton 30. Gi s hm s f(x) c n khng im trong khong (a, +). Chng minh rng
vi mi s thc hm s
f(x) + f

(x)
c t nht n 1 khng im trong khong . Hn na, nu tha mn iu kin
lim
x+
e
x
f(x) = 0
th hm nu c t nht l n khng im.
Bi ton 31. Nu a thc f(x) R[x] c k nghim thc th a thc f

(x) c t nht (k 1)
nghim thc.
Bi ton 32. Cho 0 x
1
< x
2
< x
3
< x
4
. Chng minh bt ng thc sau:
3
_
1
4
(x
1
x
2
x
3
+x
1
x
2
x
4
+x
1
x
3
x
4
+ x
2
x
3
x
4
)
<
_
1
6
(x
1
x
2
+ x
1
x
3
+x
1
x
4
+x
2
x
3
+x
2
x
4
+ x
3
x
4
)
104
Bi ton 33. Cho hm s f(x) c o hm lin tc trn on [a, b], tha mn iu kin
f(a) = f(b) = 0, f(x) = 0, x (a, b).
Chng minh rng tn ti dy {x
n
} vi x
n
(a, b) sao cho
lim
n+
f

(x
n
)
(
n

e 1)(f(x
n
)
= 2011 (8)
Chng minh. Xt hm s
g
n
(x) = e

2011x
n
f(x), x [a, b], n N.
R rng hm s g
n
(x) xc nh v lin tc trn on [a, b], kh vi trong khong (a, b). Hn
na, ta c g
n
(a) = g
n
(b) nn theo nh l Rolle, tn ti x
n
(a, b) sao cho g

n
(x
n
) = 0.
Vi mi n, tn ti mt x
n
nh th. iu c ngha l ta xy dng mt dy {x
n
}. Ta s
chng minh rng dy ny tha mn ng thc (3.3).
Tht vy, ta c
g

n
(x
n
) =
2011
n
e

2011xn
n
f(x
n
) + e

2011xn
n
f

(x
n
) = 0.
V th
f

(x
n
)
f(x
n
)
=
2011
n
.
Suy ra
lim
n+
f

(x
n
)
(
n

e 1)f(x
n
)
= lim
n+
2011
e
1
n
1
= 2011 (pcm).

Bi ton 34. Gi s
f(x) = (x x
1
)(x x
2
)(x x
3
),
vi x
1
< x
2
< x
3
. Chng minh rng
f

(x
1
)
f

(x
1
)
+
f

(x
2
)
f

(x
2
)
+
f

(x
3
)
f

(x
3
)
= 0.
Bi ton 35. Cho c
0
, c
1
, . . . , c
n
l cc s thc tha mn iu kin
c
0
+
c
1
2
+
c
2
3
+ +
c
n
n + 1
= c
0
+c
1
+
c
2
.2
2
3
+
c
3
.2
3
4
+ +
c
n
.2
n
n + 1
= 0
Chng minh rng phng trnh
c
1
+ 2c
2
x + +nc
n
x
n1
= 0
c t nht mt nghim thuc khong (0, 2).
105
Bi ton 36 (Olympic Nga). Cho phng trnh
a
0
x
n
+a
1
x
n1
+ + a
n1
x + a
n
= 0, a
0
= 0
c n nghim phn bit. Chng minh rng
(n 1)a
2
1
> 2na
0
a
2
.
Bi ton 37 (Olympic 30.4 - 2003). Tn ti hay khng cc s thc a, b, c phng trnh sau
c bn nghim thc phn bit.
x +
5
6
= a.e
3x
+b.e
2x
+c.e
x

1
4
.e
x
.
Bi ton 38. Gi s hm f(x) kh vi lin tc n ln trn [a, b], v trn on ny c khng t
hn n khng im tnh c bi. Chng minh rng:
max
[a,b]
|f(x)|
(b a)
n
n!
max
[a,b]

f
(n)
(x)

Bi ton 39 (Olympic sinh vin ton quc- 1993 vng 1). Cho hm s f(x) xc nh v c
o hm bc hai lin tc v khng ng nht bng 0 trn bt k on no ca R. Bit rng
th ca hm s y = f(x) ct ng thng ax + by + c = 0 ti ba im phn bit. Chng minh
rng tn ti x
0
R sao cho f

(x
0
) = 0 v f

(x) i du qua x = x
0
.
Bi ton 40 (Olympic sinh vin ton quc- 1994). Cho n l s nguyn dng, a
k
, b
k
R(k =
0, 1, . . . , n). Chng minh rng phng trnh
x +
n

k=1
(a
k
sin kx +b
k
cos kx) = 0
c nghim trong khong (, ).
Bi ton 41 (Olympic sinh vin ton quc - 1999). Gi s a thc vi h s thc
P(x) = a
0
+a
1
x + +a
n
x
n
c n nghim thc phn bit. chng minh rng
a
k1
a
k+1
< (a
k
)
2
, k = 1, 2, . . . , n 1.
Bi ton 42 (Olympic sinh vin ton quc - 2001). Chng minh rng tn ti s thc x (0, 1)
sao cho
1
_
x
t
2000
dt
(1 + t)(1 + t
2
) (1 + t
2001
)
=
x
2000
(1 + x)(1 + x
2
) (1 + x
2001
)
.
106
Bi ton 43 (Olympic sinh vin ton quc - 2003). Cho a thc vi h s thc P(x) bc
n(n 1) c m nghim thc. Chng minh rng a thc
f(x) = (x
2
+ 1)P(x) + P

(x)
c t nht m nghim thc.
Bi ton 44 (Olympic - 2006). Tm tt c cc dy s thc a
1
, a
2
, . . . , a
n
, vi n 1, a
n
= 0,
tha mn tnh cht sau:
Nu f : R R l hm s kh vi cp n v x
0
< x
1
< < x
n
l cc s thc tha mn
f(x
0
) = f(x
1
) = = f(x
n
) = 0
th tn ti (x
0
, x
n
) sao cho
a
0
f() + a
1
f

() + + a
n
f
(n)
() = 0.
Bi ton 45. K hiu T l tp hp tt c cc a thc bc 2011 c ng 11 nghim thc k c
nghim bi. Vi mi P(x) T, t
Q
P
(x) = (x
20
+ 1)P(x) P

(x)
v gi S
P
l s nghim thc ca a thc Q
P
(x). Tm min
PT
S
P
.
Ti liu tham kho
[1] Nguyn Vn Mu, 2005, a thc i s v phn thc hu t, NXB Gio Dc.
[2] Nguyn Vn Mu, 2006, a thc v p dng, NXB Gio Dc.
[3] Nguyn Vn Mu, 2007, Ni suy v p dng, NXB Gio Dc.
[4] Nguyn Vn Mu,2006, Tuyn tp Olympic sinh vin ton quc, NXB Gio Dc.
[5] Mt s bi bo lin quan.
[6] Nguyn Gia T - Phi Nga, Bi ging ton cao cp A1.
107
BT BIN NH L MT PHNG PHP CHNG
MINH V NG DNG TRONG GII TON
L Sng V c Thch Sn, Trng THPT Chuyn L Qu n Khnh Ha
Ta thng lm quen vi cc phng php chng minh trong ton hc nh: trc tip, quy np,
phn chng, nguyn tc Dirichlet ,tp th t v nguyn tc cc hn. Bt bin l s khng i
ca mt i lng qua mt hoc nhiu php bin i qua hu hn bc thc hin. Chnh yu
t ny cng gip ta c th phn bc nhng tnh hung khng th xy ra ca mt i lng
no trong bi ton. Yu t bt bin rt thng gp trong cc cuc thi ton hc v thng
l cc bi ton i hi s nhy bn v lp lun cht ch, di y l mt s bi ton nh vy.
y bt bin v na bt bin c xt c xem nh l mt phng php chng minh quan
trng, trong cc dng ton thi chn hc sinh gii c cp tng i phong ph gip c
ci nhn r hn trong bi ton t hp, y l bi ging cho i tuyn Khnh Ha.
1 Phng php ng d:
Mc ch ca tnh cht bt bin trong cc dng ton ny l nghin cu tnh chn l hoc
ng d vi 2, 3, 4 ca cc trng thi bi ton khi thay i theo yu cu
Bi ton 1. Trn mt hn o c ba ging thn ln: 133 con mu xm, 155 con mu v 177
con mu xanh. Nu hai con thn ln khc mu gp nhau th chng s ng thi chuyn sang
mu th ba (v d con mu xanh gp con mu xm th c hai con s cng chuyn sang mu )
cn nu hai con cng mu gp nhau th khng chuyn mu. Liu c khi no xy ra tnh hung
tt c cc con thn ln trn o cng mt mu khng?
Li gii. Nh vy bt bin y chnh l s d khi chia cho 3 ca s thn ln mi mu l
0-1-2. Tng s thn ln l 133 + 155 + 177 = 465 chia ht cho 3 do nu xy ra tnh hung
tt c con thn ln u cng mu th s d khi chia cho 3 ca ba loi thn ln l 0-0-0 tri vi
tnh bt bin 0-1-2. Nh vy khng th xy ra tnh hung .
Phng php ng d chnh l s pht hin ra i lng bt bin gn vi s d khi chia
cho mt s m ch yu l modul 2 v modul 3. Bi ton thn lntrn y chnh l modulo 3,
cn sau y l mt s bi ton s dng phng php ng d:
Bi ton 2. Trn bng c 2011 du cng v 2012 du tr . Mt hc sinh thc hin tr chi nh
sau : thay hai du bt k trn bng du cng nu hai s b xa i cng du v thay bng du tr
nu chng tri du. Hi sau 4022 ln thc hin nh vy du cn li l du cng hay du tr ?
Li gii. Nu thay :
(-) v (-) (+)
(+) v (+) (+)
108
(-) v (+) (-)
Ta pht hin c s bt bin y l s khng i u ca tch cc du trn bng.
M lc u trn bng c 2011 (+) v 2012 (-) cho nn tch s l du (+). Do du cn
li duy nht trn bng phi l du (+).
Bi ton 3. Trn bng cho ba s 2,2,2. Ta xa i mt trong ba s v thay vo tng ca hai
s cn li tr i 1. Hi sau mt s ln thc hin ta c th thu c cc s 11,1,2011 c hay
khng?
Li gii. Hy th kim tra: 2,2,2 2,3,2
Lc ny trong b ba s ny lun c 1 s l v 2 s chn. Nu ta tip tc thc hin th:
Chn + L - 1 Chn
Chn + Chn - 1 L
Nh vy trong mi trng hp th trong b ba s thu c lun c 1 s l v 2 s chn. Th
nhng 11,1,2011 li l 3 s l do khng th xy ra trng hp ny.
Bi ton 4. Trong mt gii u bng , cc i u vng trn mt lt vi nhau theo quy
nh mt trn thng c 2 im, ha 1 im,thua 0 im. Hi c th c hai i ln lt c
7 v 10 im trong cng mt thi im ca gii u khng?
Li gii. Ta thy rng sau mt trn u bt k nu c mt i thng th s im hai i t
c l 0 v 2. Khi hiu hai s im hai i t c l 2 cn nu ha nhau th hai i c
1 im mi i khi hiu s l 0. Vy th r rng hiu hai im s bt k ca hai i phi l
s chn. Th nhng 10 7 = 3 l s l do s khng tn ti hai i cng lc c 7 v 10
im.
Bi ton 5. Vit 2012 s t 1 n 2012, c mi ln xa i 2 s ri thay bi tr tuyt i ca
hiu 2 s . Hi s cui cng l s chn hay l?
Li gii. Gi S(n) l tng cc s cn li sau ln th n. S(0)l tng 2012 s u tin l 1 s
chn. M S(n) bt bin modulo 2 do s cui cng phi l s chn.
Bi ton 6. Trn bng ta vit ba s nguyn. Sau ta xa i mt s v vit vo tng hai s
cn li tr i 1. Thao tc nh vy lp li mt s ln v cui cng ta nhn c 3 s 29, 1876,
2011. Hi ba s u tin c th l 2,2,2 khng?
Li gii. Sau bc u tin t ba s 2,2,2, ta nhn c 2,2,3, ba s ny c hai chn v mt
l. T bc th hai tr i th kt qu lun lun c hai s chn v mt s l d ta thc hin bt
u t bt c s no (v nhng s chn bng tng ca mt s chn v mt s l tr i 1; s l
l tng ca hai s chn tr i 1). Nhng trong kt qu cho c hai s l, mt s chn nn vi
thao tc cho v xut pht t 2,2,2 khng th cho kt qu.
Bi ton trn c gii nh pht hin ra tnh chn l ca ba s khng thay i, nn t trng
thi xut pht khng th nhn c trng thi kt qu.
Bi ton 7. Trn ng trn theo mt th t bt k vit 4 ch s 1, 5 ch s 0, thc hin vit
ch s 1 vo gia hai ch s ging nhau v ch s 0 vo gia hai ch s khc nhau, cc ch s
ban u b xa i. Hi c th a ng trn v ton ch s 0 c khng?
109
Li gii. Nhn thy trn ng trn sau mi qu trnh trn lun cn li 9 ch s. Do vy s
ch s 0 v s ch s 1 lun khc tnh chn l. Gi s sau 1 s ln lp li qu trnh trn. ng
trn cn li ton ch s 0 suy ra trc trn ng trn khng c 2 ch s no ging nhau
ng cnh nhau nn s ch s 0 bng s ch s 1 ( v l ! ), suy ra pcm.
Bi ton 8. ( Corolado MO 1997) Mi ca bng 1997 X 1997 in cc s (+1) hoc (-1).
Mi hng ta tnh tch R
i
cc s trong hng . Mi ct ta tnh tch C
i
cc s trong ct .
Chng minh rng
1997

i=1
(R
i
+C
i
) lun khc khng.
Li gii. Bt bin c s dng y l s d
1997

i=1
(R
i
+C
i
) cho 4, ni cch khc ta ang s
dng bt bin theo modulo 4. i lng ny khng i vi bt k mt c thay i du no ca
mt trong nhng du c vit trn bng. Tht vy, gi s c s thay i phn t hng th
i v ct th j cho ta (R
i
+C
i
) thay v (R
i
+C
i
) . V (R
i
+C
i
) c gi tr 2, 0 hoc -2 theo
modulo 4 nn tng ban u thay i mt bi s ca 4. Do bt bin khng ph thuc vo
cch chn cc s (+1) hoc (-1). Cho nn ta ch cn xt trng hp tt c cc u in (+1).
Khi
1997

i=1
(R
i
+C
i
) 2 (mod 4). Vy vi mi cch in cc s (+1) hoc (-1) th
1997

i=1
(R
i
+C
i
)
lun khc khng.
Bi ton 9. ( Hungary MO 1989) Mi nh ca mt hnh vung t mt hn si. Thc hin
thay i s si theo quy lut sau: ta c th ly i mt s si mt nh v thm vo mt trong
hai nh k bn mt s si gp i. Hi c th nhn c 1989, 1988, 1990, 1989 vin si ti
cc nh lin tip ca hnh vung c hay khng?
Li gii. Gi 4 nh lin tip ca hnh vung l A, B, C, D ng vi s si l a, b, c, d. Khi
bc tip theo gi x l s si ly i, gi s nh A, do s si 4 nh l: a x, b +2x, c, d
hoc a x, b, c, d + 2x. Ta c (b +d + 2x) (a +c x) = b +d a c + 3x M ban u s si
mi nh l (1 , 1 , 1 , 1 ) nn t y ta c bt bin ca bi ton ny: hiu gia tng s si hai
nh A, C v hai nh B, D lun l bi ca 3. M vi ( 1989, 1988, 1990, 1989) th hiu ny c
s d l 2 khi chia cho 3 do khng xy ra.
Bi ton 10. (VMO 1992) Cho bng hnh ch nht 1991 1992 vi 1991 hng v 1992 ct. K
hiu vung nm giao ca hng th m (k t trn xung) v ct th n(k t tri sang phi)
l (m ; n). T mu cc vung ca bng theo cch sau: ln th nht t 3 (r ; s), (r + 1 ; s +
1), (r + 2; s + 2) vi r; s l hai s t nhin cho trc tha mn 1 r 1989 v 1 s 1991;
t ln th hai mi ln t ng 3 cha c mu nm cnh nhau hoc trong cng mt hng hoc
trong cng mt ct. Hi bng cch c th t mu c tt c cc vung ca bng cho
hay khng?
Li gii. Ta hy ghi vo mi vung ca bng mt s t nhin theo quy tc sau: mi hng,
ln lt t tri sang phi, ghi cc s t nhin t 1 n 1992. Nh vy ba s c ghi vo ba
cnh nhau trong cng mt hng s l ba s t nhin lin tip, trong cng mt ct s l ba s
t nhin bng nhau. Suy ra, k t ln th hai, mi ln t mu ta s xa i ba s c tng chia
110
ht cho 3.Nh th ba s c ghi vo ba (r ; s), (r + 1 ; s + 1), (r + 2 ; s + 2) s l s, s + 1
v s + 1 m tng ca chng l mt s chia cho 3 d 2. Vy nu t mu c ht cc vung
ca bng cho th tng S ca tt c cc s c ghi vo bng phi l mt s chia cho 3
d 2. Nhng S = 1991.(1 + 2 + . . . +1992) = 1991.1993.993 chia ht cho 3, mu thun!. Do
khng th t mu c tt c vung ca bng cho.
2 Bng biu :
Bi ton 11. Trn bn c 88 c 32 qun trng v 32 qun en, mi qun chim mt vung.
Ti mi bc i ngi chi thay tt c cc qun trng thnh qun en v tt c cc qun en
thnh qun trng trn mt hng hoc mt ct no . Hi sau hu hn bc c th cn li
chnh xc mt qun trng trn bn c khng?
Li gii. Nu trc khi chuyn c chnh xc k qun trng trn hng(ct) nh chuyn th s
qun en trn hng(ct)y l 8 k. Sau khi chuyn, 8 k qun en ny s tr thnh 8 k
qun trng v k qun trng li tr thnh k qun en. Nh vy s qun trng trn bn c sau
khi chuyn s thm vo 8 k v mt i k qun, tc l s qun trng thay i trn bn c l (8
k) k = 8 2k . S ny l mt s chn ,m s qun trng trn bn c lc u l 32 qun do
s qun trng trn bn c lun l s chn. Vy khng th cn li duy nht trn bn c mt
qun trng.
Bi ton 12. (IMO 2004) Ta nh ngha vin gch hnh mc cu l hnh gm 6 vung n v
nh hnh v di y, hoc hnh nhn c do lt hnh (sang tri, sang phi, ln trn, xung
di) hnh nhn c do xoay hnh i mt gc:
Hy xc nh tt c cc hnh ch nht mn trong m, n l cc s nguyn dng sao cho c
th lt hnh ch nht bng cc vin gch hnh mc cu?
Chng minh. D thy m,n / {1; 2; 5} . Chia hnh ch nht cho thnh cc m n vung
v nh s cc hng, cc ct t di ln trn, t tri sang phi. Ta gi (p; q) l nm giao
ca hng th p v ct th q. Hai vin gch hnh mc cu ch c th ghp li c mt trong
hai hnh di y:
111
Do , lt c hnh ch nht m x n th m.n phi chia ht cho 12. Nu t nht mt trong
hai s m, n chia ht cho 4 th c th lt c. Tht vy, gi s c m chia ht cho 4. Ta c
th vit n di dng: n = 3a + 4b, do c th lt c.
Xt trng hp mn u khng chia ht cho 4. Ta chng minh trng hp ny khng th
lt c. Gi s ngc li, khi m , n u chia ht cho 2 nhng khng chia ht cho 4. Ta to
bt bin nh sau: Xt (p ; q). Nu ch mt trong hai ta p , q chia ht cho 4 th in s 1
vo . Nu c hai ta p, q chia ht cho 4 th in s 2. Cc cn li in s 0. Vi cch
in s nh vy ta thu c bt bin l tng cc s trong Hnh 1 v tng cc s trong Hnh 2
u l s l. Do m, n chn nn tng cc s trong ton b hnh ch nht m n l s chn.
lt c th tng s Hnh 1 v Hnh 2 c s dng phi l s chn. Khi , m n chia ht
cho 24, v l !.
Bi ton 13. (VMO 2006).Xt bng vungm n (m, n 3). Thc hin tr chi sau: mi
ln t 4 vin bi vo 4 ca bng, mi mt vin bi, sao cho 4 to thnh mt trong cc
hnh di y:
Hi sau mt s ln ta c th nhn c bng m s bi trong cc bng nhau c khng
nu:
a) m = 2004, n = 2006 ?
b) m = 2005, n = 2006 ?
Bi ton 14. (VMO 1993)Cho a gic li A
1
A
2
...A
1993
m ti mi nh ghi mt du cng
(+) hoc mt du tr (-) sao cho trong 1993 du c c du (+) v (-)
Thc hin vic thay du nh sau: mi ln, thay du ng thi ti tt c cc nh A
i
(i =
1, 2, .., 1993) ca a gic theo quy tc:
Nu du ti A
i
v A
i+1
l nh nhau th du ti A
i
c thay bi du (+)
112
Nu du ti A
i
v A
i+1
l khc nhau th du ti A
i
c thay bi du (-) Quy c coi
A
1994
l A
1
.
Chng minh rng tn ti s nguyn k 2 sao cho sau khi thc hin lin tc k ln php thay
du ni trn, ta c a gic A
1
A
2
...A
1993
m du ti mi nh A
i
(i = 1, 2, .., 1993) trng vi
du ti chnh nh sau ln thay du th nht.
Bi ton 15. (Lin bang Nga 1998) Trn bng cho mt s nguyn, ngi ta ghi nh ch s
cui cng ca s ny, sau xa i v cng thm vo vi s cn li trn bng 5 ln ch s mi
xa. Gi s ban u ghi s hi sau mt s ln thc hin c th thu c s c hay khng?
3 Bt bin trong cc bi ton i s -Gii tch:
Cc i lng khng i ca cc s hng trong mt dy s c tnh cht no cn pht
hin . Da vo cc i lng ny m ta c th tm c cng thc tng qut ca dy s, chng
minh c cc tnh cht cng nh tm c gii hn , xt tnh hi t ca dy s .y cng
c tm gi l tnh bt bin. Sau y l mt s bi ton :
Bi ton 16. ( German MO 1996 ) T im ( 1 , 1) di chuyn 1 hn si trn mt phng ta
tha cc iu kin sau : (a) T im ( a , b ) c th n ( 2a , b ) hoc ( a , 2b ). (b) T
im ( a , b ) c th n ( a b , b ) nu a > b hoc ( a , b a ) nu a < b. Vi nhng s
nguyn dng x v y nh th no th hn si c th n im ( x , y)?
Li gii. Ta s chng minh rng iu kin cn v ca bi ton ny l (x , y) = 2
5
vi s
l s nguyn khng m, trong k hiu ( x , y ) l c chung ln nht ca hai s t nhin x
v y. Tht vy: iu kin cn : v ( p , q ) = ( p , q p) ta thy rng s c chung l l bt
bin sau hai php bin i. Ban u s lng ny l 1, s d l nh nhau nn ( x , y ) l mt
ly tha ca 2. iu kin : gi s (x , y) = 2
5
. Trong tt c cc cp ( p , q ) c th n
c (x , y) ta chn mt cp sao cho p + q nh nht. Nu p hoc q l s chn th mt trong
cc im
p
2
, q
_
hoc p,
q
2
_
cng tha mn , mu thun vi gi thit v tnh nh nht ca cp
p v q. Nu p > q th cng c th nhn c t im
p+q
2
, q
_
mu thun vi gi thit v tnh
nh nht ca tng p + q. Tng t vi trng hp p < q ,mu thun. Do p = q m (p , q)
l ly tha ca 2. T ta suy ra p = q = 1, nn (x , y ) l im tha mn. iu kin c
chng minh xong.
Bi ton 17. Cho dy s tha:
_
a
1
= 3, b
1
= 2
a
n+1
= a
2
n
+ 2b
2
n
, b
n+1
= a
n
b
n
, n N
Chng minh rng a
n
, b
n
l cc s nguyn t cng nhau.
Li gii. Ta chng minh bng quy np rng a
2
n
2b
2
n
= 1 .Tht vy: Vi n = 1, ta c:
a
2
1
2b
2
1
= 3
2
2.2
2
= 1
Vi n = k, gi s: a
2
k
2b
2
k
= 1
Vi n = k+1, ta c: a
2
k+1
2b
2
k+1
= (a
2
k
+ 2b
2
k
)
2
2(2a
k
b
k
)
2
= (a
2
k
2b
2
k
)
2
= 1
Vy a
2
n
2b
2
n
= 1, n . Gi d l UCLN ca a
n
, b
n
th ta suy ra d cng l c s ca 1 do
d = 1.
Cho nn a
n
, b
n
l cc s nguyn t cng nhau.
113
Bi ton 18. Cho dy s {u
n
} :
_

_

_
u
1
= 2
u
2
= 8
u
n
= 4u
n1
u
n2
> 0, n 3, S
n
=
n

i=1
arccot(u
2
i
)
Tm lim
n
S
n
.
Li gii. Trc ht ta chng minh u
2
n
u
n1
u
n+1
= 4, n 2. Tht vy ta c:
u
n
(4u
n1
) = u
n1
(4u
n
) u
n
(u
n
+u
n2
) = u
n1
(u
n+1
+ u
n1
)
u
2
n
u
n+1
u
n1
= u
2
n1
u
n
u
n2
u
2
n
u
n+1
u
n1
= ... = u
2
2
u
1
u
3
= 4
arc cot u
2
n
= arc cot
_
u
n
4un
4
_
= arc cot
un(u
n+1
+u
n1
)
u
2
n
u
n+1
u
n1
= arc cot
u
n+1
un
.
un
u
n2
+1
un
u
n1

u
n+1
un
= arc cot
u
n+1
un
arc cot
un
u
n2
S
n
= arc cot
u
n+1
un
Hn na u
n
= 4u
n1
u
n2
1 =
4u
n1
un

u
n2
u
n1
.
u
n1
un
Li c
_
0 <
u
n1
un
< 1
un
u
n+1
>
u
n1
un
lim
n
un
u
n+1
Chuyn qua gii hn ta c:
Do lim
n
u
n+1
un
= 2 +

3 lim
n
S
n
= arc cot(2 +

3) =

12
Vy lim
n
S
n
=

12
Bi ton 19. Chng minh rng phng trnh sau c v hn nghim nguyn dng
x
2
+y
2
xy1
= 5.
Li gii. y l mt bi ton S hc tuy nhin y ta s s dng tnh Bt bin trong
dy s chng minh phng v s nghim nguyn dng. Ta xt dy s (u
n
) xc nh bi:
u
0
= 1, u
1
= 1, u
n+1
= 5u
n
u
n1
, n 1.
Ta c
u
n+1
+u
n1
un
=
u
n+2
+un
u
n+1
= 5 (u
n+1
+u
n1
)u
n+1
= (u
n+2
+u
n
)u
n
u
n+2
u
n
u
2
n+1
= u
n+1
u
n1
u
2
n
u
n+1
u
n1
u
2
n
= u
3
u
1
u
2
2
= 5
(5u
n
u
n1
)u
n1
u
2
n
= 5 u
2
n
+ u
2
n1
5u
n
u
n1
+ 5 = 0

u
2
n
+u
2
n1
unu
n1
1
= 5.
Vy (x, y) = ( u
n1
, u
n
) l nghim ca phng trnh cho vi mi n 1. Cng vi cch
khai thc tnh bt bin trong dy s truy hi bc 2 nh trn chng ta s c c mt cch gii
khc rt c o cho cc bi ton s hc quen thuc sau y :
Bi ton 20. Chng minh rng phng trnh sau c v hn nghim nguyn dng : x
2
+y
2
+z
2
=
3xyz.
Li gii. Chn z = 1 ta c: x
2
+y
2
+ 1 = 3xy.
Xt dy s (u
n
) c xc nh nh sau :
u
0
= 0, u
1
= 1, u
n+1
= 3u
n
u
n1
, n 1.
114
Ta c u
n+1
u
n1
u
2
n
= u
3
u
1
u
2
2
= 1
Suy ra :
(3u
n
u
n1
)u
n1
u
2
n
= 1
u
2
n
+ u
2
n1
+ 1 = 3u
n
u
n1
Vy (x, y, z) = (u
n1
, u
n
, 1) l nghim ca phng trnh cho vi mi n 1.
Bi ton 21. Chng minh rng phng trnh sau c v hn nghim nguyn dng : x
2
+y
2
+z
2
=
xyz.
Li gii. t x = 3x
1
, y = 3y
1
, z = 3z
1
ta c x
2
1
+y
2
1
+z
2
1
= 3x
1
y
1
z
1
.
Xt dy s (u
n
) c xc nh nh sau :
u
0
= 1, u
1
= 1, u
n+1
= 3u
n
u
n1
, n 1.
Theo bi ton 4 th (x
1
, y
1
, z
1
) = (u
n1
, u
n
, 1) l nghim ca phng trnh
x
2
1
+y
2
1
+z
2
1
= 3x
1
y
1
z
1
.
Do (x, y, z) = (3u
n1
, 3u
n
, 3) l nghim ca phng trnh cho vi mi n 1.
Bi ton 22. Chng minh rng phng trnh sau c v hn nghim nguyn dng: x
2
+ y
2
+
z
2
+t
2
= 4xyzt.
Li gii. Chn z = t = 1 ta c x
2
+ y
2
+ 2 = 4xy.
Xt dy s (u
n
) c xc nh nh sau:
u
0
= 1, u
1
= 1, u
n+1
= 4u
n
u
n1
, n 1.
Ta c u
n+1
u
n1
u
2
n
= u
3
u
1
u
2
2
= 2
Suy ra
(4u
n
u
n1
)u
n1
u
2
n
= 2
u
2
n
+u
2
n1
+ 2 = 4u
n
u
n1
Vy (x, y, z, t) = (u
n1
, u
n
, 1, 1) l nghim ca phng trnh cho vi mi n 1.
Bi ton 23. Chng minh rng phng trnh sau c v hn nghim nguyn dng : x
2
+y
2
+
z
2
+t
2
= xyzt.
Bi ton 24. Chng minh rng phng trnh sau c v hn nghim nguyn dng : x
2
+y
2
=
4(x + 1)(y 1)
Bi ton 25. Chng minh rng phng trnh sau c v hn nghim nguyn dng :
x+2
y
+
3y+4
x
=
10
Bi ton 26. ( Bulgari MO) Chng minh rng vi mi n 3 phng trnh sau c t nht mt
nghim nguyn dng (x, y) tha mn x, y u l v 7x
2
+y
2
= 2
n
Bi ton 27. (VMO 2010) Chng minh rng vi mi s nguyn dng n phng trnh x
2
+
15y
2
= 4
n
c t nht n nghim t nhin.
115
Bi ton 28. Chng minh rng vi mi s nguyn dng n phng trnh sau c t nht mt
nghim nguyn dng tha mn (x, y, z) = 1:
x
2
+y
2
+z
2
= 7
2
n
.
Bi ton 29. Trn bng cho 4 s 3,4,5,6. Mi ln xa i 2 s x, y bt k trong 4 s v thay
vo bng hai s x +y +
_
x
2
+ y
2
v x +y
_
x
2
+ y
2
. Hi sau mt s ln thc hin trn bng
c th xut hin mt s nh hn 1 c khng?
4 Cc bi ton khc :
Bi ton 30. ( Rusian MO 1995 ) Cho ba ng si khc nhau. Sisyphus thc hin di chuyn
1 vin si t 1 trong ba ng si sang 1 trong 2 ng si cn li. Mi ln chuyn si, Sisyphus
nhn c t Zeus mt s tin bng hiu s gia s si ca ng si ly i v ng si nhn
thm trc khi di chuyn. Nu s chnh lch ny m th Sisyphus cng phi tr cho Zeus s
tin chnh lch . Sau mt s bc thc hin th s si mi ng s tr v nh ban u. Hi
khi s tin ti a m Sisyphus nhn c l bao nhiu?
Li gii. Ta chng minh tng sau l bt bin
a(a1)
2
+
b(b1)
2
+
c(c1)
2
+ s
Vi a, b, c l s si mi ng ban u v s l s tin Sisyphus nhn c ti mt thi im
no . Tht vy ta chuyn 1 vin si t ng si c a vin si sang ng si c b vin si. Khi
s tin Sisyphus nhn c ( hoc mt i ) l a b. Ta c :
(a1)(a2)
2
+
b(b+1)
2
+
c(c1)
2
+s +a b
=
a(a1)
2
+
b(b1)
2
+
c(c1)
2
+s
Do n khi s si tr v nh ban u th s tin Sisyphus nhn c bng s tin ban
u. M ban u Sisyphus khng c tin d n thi im ny Sisyphus cng khng c tin.
Bi ton 31. ( IMO Shortlisted 1994, Thy in ) C 1994 c gi ngi quanh mt bn trn,
h chi chung mt c bi gm n l. Ban u, mt c gi tt c cc l bi. C mi nc i, nu
c t nht mt c gi gi ti thiu 2 l bi, th mt trong cc c gi ny phi chuyn 1 l cho
mt trong hai c gi bn cnh c y. Tr chi kt thc khi v ch khi mi c gi ch gi nhiu
nht 1 l bi. a) Chng minh rng nu n 1994 th tr chi khng th no kt thc c. b)
Chng minh rng nu n < 1994 th tr chi bt buc phi kt thc.
Li gii. a) Nu n > 1994 theo nguyn tc irichlet c t nht 1 c gi gi 2 l bi, v th tr
chi khng th no kt thc c. Gi s n = 1994 . Gi cc c gi l G
1
, G
2
, ..., G
1994
v gi
s ban u G
1
gia tt c cc l bi. Ta nh ngha gi tr tm thi ca mt l bi l i nu G
i
ang gi n, 1 n 1994. Gi S l tng cc gi tr tm thi ca l bi. Ban u S = 1994.
Nu mt c gi khc vi G
1
hoc G
1994
chuyn l bi th S khng thay i. Nu G
1
hoc G
1994
chuyn l bi th S s tng ln hoc gim i 1994. Suy ra S = 1994h, vi h l s nguyn no .
Khi tr chi kt thc mi c gi s gi ng mt l bi, do lc ny S = 997.1995. Nhng
nh th th phi c 2h = 1995, iu ny l khng th. Tm li nu n 1994 th tr chi khng
th no kt thc c .
116
b) Khi mt c ny chuyn 1 l bi cho c kia ln th nht, c hai s nh du tn mnh
ln . Ln sau , nu mt trong hai ngi ny phi chuyn bi, c ta s a ng l c
nh du cho ngi kia. Nu lm nh th, l bi c nh du s kt li gia hai c gi
k nhau ni trn. Nu n < 1994, s c 2 c gi k nhau khng bao gi chuyn l bi cho nhau.
By gi, gi s khi lm th m tr chi khng kt thc c, th phi tn ti t nht 1 c gi
chuyn bi n v hn ln. T suy ra c mt c chuyn bi n v hn ln, trong khi mt
c k bn c ta ch chuyn hu hn ln m thi. Khi c k bn nay thc s khng chuyn na
( v ch chuyn hu hn ln ) th ng bi ca c y vn tip tc tng ln. iu ny r rng
mu thun.
Bi ton 32. ( Bungary MO 1999 ) Ba ng si c 51, 49 v 5 vin. Ta thc hin mt trong
hai nc i nh sau. Mt nc i l dn hai ng ty thnh mt ng. Nc i khc l chn
ng c s chn vin si chia thnh hai ng bng nhau. Hi c th thc hin mt dy cc
nc i nh th chia ba ng si thnh 105 ng m mi ng ch c mt vin si hay khng?
Li gii. Ban u s si trong ba ng l 51, 49 v 5 vin u l s l nn bc i u tin l
phi dn hai ng li.
Trng hp 1 : Dn hai ng c 5 v 49 vin ta c hai ng l 51 v 54 vin, mi ng u
l bi ca 3. Bc th hai ta chia ng c 54 vin thnh 2 dng c 27 vin. By gi s si trong
c ba ng l 51, 27, 27 cng chia ht cho 3. V c ba ng c s l vin nn bc th ba ta li
phi gp hai ng 27 v 51 vin thnh ng 78 vin. V hai s 27 v 51 u chia ht cho 3 nn
tng ( = 78 ) ca chng cng chia ht cho 3. Tc l khi thc hin cc nc i lun phin th
s si trong mi ng lun l bi ca 3. y chnh l bt bin ca trng hp ny. Tht vy
khi gp hai ng si c s si chia ht cho 3 th c mt ng si c s si chia ht cho 3 v
nu chia mt ng si ( l gp ca hai ng c s si l cng chia ht cho 3 ) c s chn vin
si chia ht cho 3 thnh hai phn bng nhau th s si trong mi phn vn chia ht cho 3. Do
s si trong mi ng vn chia ht cho 3 v nhiu nht c th c l 35 ng, mi ng 3
vin.
Trng hp 2 : Bc u tin dn hai ng c 5 v 51 vin, ta c hai ng c 49 v 56
vin, c hai u l bi ca 7. Khi thc hin cc bc i lun phin s si trong mi ng nhn
c lun l bi ca 7. Do s ng vi s si nh nht ch c th l 15 ng mi ng 7 vin.
Trng hp 3 : Bc u tin dn hai ng c 49 v 51 vin, ta c hai ng l 5 v 100
vin, s si trong mi ng u l bi ca 5. Khi thc hin mt trong hai nc i, s si trong
mi ng nhn c lun l bi ca 5. Do s ng vi s si nh nht l 21 ng, mi ng
5 vin.
Kt lun : Khng th chia ba ng si thnh 105 ng mi ng 1 vin si c.
Bi ton 33. ( Anh 2002 ) Mt tr chi dnh cho hai ngi bt u vi mt ct tin n ng
xu: Ngi chi th nht chia ct tin c thnh hai ct tin ty . Ngi chi th hai chn
mt ct tin ( t nhng ct tin c trn bn ) v li chia n ra thfnh hai ct tin ty . Tip
tc nh vy, ngi thng cuc l ngi chi n bc i lm cho tt c cc ct tin ch c mt
hoc hai ng tin. Vi gi tr ban u no ca n th ngi u tin s thng, gi s hai ngi
ti gii nh nhau?
117
Li gii. Ngi chi th nht thng khi v ch khi n = 3 hoc n chn. Ngi chi th hai
thng vi mi n l v n > 3. Ta c th kim tra kt lun trn n n = 6 v sau chng minh
bng quy np.
1) Nu n > 6 l chn, ngi th nht to ra ct c 1 ( c 1 ng xu ) v ct c n 1 ( c n
1 ng xu ). V n 1 l l nn ngi th nht s thng theo gi thit quy np ( v ngi chi
th nht tr thnh ngi chi th hai v v tr xut pht c s l ng xu ).
2) Nu l s l, ngi chi th nht to ra mt ct c s chn ng xu v mt ct c s l
ng xu. Ngi th hai li chia ct c s chn ng xu thnh hai ct c s l ng xu. Tip tc
nh vy, ngi chi th hai lun lun p li cc bc i ca ngi chi th nht v a ngi
th nht vo tnh th ch c s ct vi s l ng xu. Khi cc ct tin ti c 1 ( ch cn 1 ng
xu ), chng khng cn lin quan n bc chi. Vn nguy kch cho ngi chi l c c 3
mt ct tin: ch c mt cch duy nht ngi th hai c th thua l nu ngi chi th hai
to cho ngi chi th nht mt ct duy nht c 3 v rt nhiu ct c 1. Nhng trong trng
hp nh vy ngi chi th nht phi to ra hoc l mt ct duy nht hai ng xu v mt ct
3 ng xu, hoc l mt ct duy nht 4 ng xu. D trong trng hp no, ngi chi th hai
thng ngay bc i tip theo bng cch lm gim i: trong trng hp th nht, ct 3 ng
xu c chia ra thnh ct 1 ng xu v ct 2 ng xu v trong trng hp th hai, ngi th
hai chia ct 4 ng xu thnh hai ct mi ct 2 ng xu. Tm li chin thut thng ca ngi
th hai l lun lun to ra ton l nhng ct tin l tr khi dn n tnh th tt c cc ct
ch mt ng xu v mt ct duy nht c 3 ng xu, trong trng hp nh vy th chin thut
thng c thc hin nh m t t trn y.
Bi ton 34. ( Tp ch Kvant ) Cho dy s 1, 0, 1, 0, 1,. . . T s hng th 7 mi s bng ch
s tn cng ca tng 6 s hng trc . Chng minh rng dy s khng cha 6 s hng lin
tip 0, 1, 0, 1, 0, 1.
Li gii. Ta pht biu li bi ton nh sau :
Mt b 6 s (x
1
, x
2
, x
3
, x
4
, x
5
, x
6
) c bin i thnh b (x
2
, x
3
, x
4
, x
5
, x
6
, x
7
) vi x
7
l ch
s tn cng ca tng x
1
+x
2
+x
3
+x
4
+x
5
+x
6
. Hi c th nhn c b ( 0, 1, 0, 1, 0, 1 ) t
b ( 1, 0, 1, 0, 1, 0 ) bng cch p dng php bin i trn qua hu hn bc thc hin khng?
Ta s chng minh rng iu l khng th bng cch thit lp mt bt bin khng
i qua php bin i trn. Tht vy gi s (x
1
, x
2
, x
3
, x
4
, x
5
, x
6
) l ch s tn cng ca tng
2x
1
+ 4x
2
+ 6x
3
+ 8x
4
+ 10x
5
+ 12x
6
.
V s(x
2
, x
3
, x
4
, x
5
, x
6
, x
7
) - s (x
1
, x
2
, x
3
, x
4
, x
5
, x
6
)
= 2x
2
+ 4x
3
+ ... + 10x
6
+ 12 (x
1
+x
2
+ x
3
+x
4
+x
5
+ x
6
) 2x
1
4x
2
... 12x
6
10 (x
1
+x
2
+ x
3
+x
4
+x
5
+x
6
) 0(mod10)
T cho thy s (x
1
, x
2
, x
3
, x
4
, x
5
, x
6
) bt bin. V s(1, 0, 1, 0, 1, 0 ) = 18 v s(0, 1, 0, 1, 0,
1 ) = 24 nn khng th xut hin b ( 0, 1, 0, 1, 0, 1 ) trong dy s.
5 Mt s bi ton luyn tp v vn ny:
Bi ton 35. Chng minh rng mt hnh ch nht 4 11 khng th ph c bi cc mnh
ch L 3 2 nh hnh di y :
118
Bi ton 36. ( IMO Shortlist 1998, Iran ) Cho mt bng kch thc m x n. Mt b bi gm mn
qun bi, mi qun c hai mt trng v en. Ta thc hin nh sau : t tt c cc qun bi vo
bng, mt trng nga ln tt c cc ngoi tr duy nht mt gc bng nga mt
en. Mi bc i ta ly mt qun c mt en ra khi bng ng thi i mt tt c cc qun
nm trong cc chung nh vi va b loi b. Hy xc nh tt c cc cp ( m, n ) tt c
cc qun bi u b loi b khi bng.
Bi ton 37. ( IMO 1993 ) Trn mt bn c c v hn ngi ta quy c mt tr chi nh
sau : u tin, mnh c sp xp thnh mt khi n x n cc hnh vung k nhau, mi mnh t
trn mt hnh vung. Mt ln di chuyn ( mt nc i ) tc l mt ln nhy theo chiu ngang
hoc chiu ng bng qua hnh vung chim ch k n n mt hnh vung khng b chim
ch tip lin theo sau. Mnh no b nhy qua cng coi nh di ch. Tr chi kt thc khi
ch cn mt mnh duy nht trn bn c. Tm nhng gi tr n tr chi kt thc.
Bi ton 38. ( Anh 2000 ) Alice chi mt tr chi mt mnh trn mt bn c 20 X 20. Khi
u Alice tri trn bn c trong mi mt ng xu gm cc dng sau y: 100 penny, 100
nickel, 100 dime v 100 quarter. Alice chn 59 ng xu bt k v ly ra khi bn c. Sau mi
ln Alice ly mt ng xu theo nguyn tc sau y:
Mt ng penny c th ly i c nu c 4 hnh vung ( trn, di , phi , tri ) b trng.
Nhng ngoi bn c khng c tnh l 4 trng theo nguyn tc ny. V d nhng gc
bn c hoc bn cnh bn c, nhng ny thm ch c 3 bn cnh trng th ng xu ti
ny cng khng c tnh theo quy tc.
Mt ng Nikel c th ly i nu tn ti t nht 3 trng bn cnh ( nhng ngoi bn
c khng c tnh l trng ).
Mt ng Dime c th ly i nu nh c t nht 2 bn cnh trng ( nhng ngoi bn
c cng khng tnh l trng ).
Mt ng Quarter c th ly i ch khi c t nht 1 trng bn cnh ( nhng ngoi bn
c cng khng tnh l trng ). Alice thng nu nh Alice ly c tt c ng xu trn bn c.
Chng minh rng khng c kh nng thng ca Alice.
Ti liu tham kho
[1] T. Andreescu, R. Gelca, Mathematical Olympiad Challenges,, Birkhauser, 2000.
[2] Nguyn Hu in, Gii Ton bng phng php i lng Bt bin, NXB Gio dc, 2005.
[3] Hong Ngc Minh, Mt s phng php xy dng nghim cho phng trnh Diophante, Ti
liu tp hun gio vin Chuyn Ton, H Ni, 2011.
[4] T Duy Phng, L thuyt tr chi. K yu hi ngh, Nam nh, 2010.
[5] T. Andresscu, R. Gelca, Putnam and Beyond, Springer, 2007.
119
MT S DNG TON LIN QUAN N DY S
C QUY LUT
L Th Thanh Hng, Nh XBGD Vit Nam
Trong chng trnh s hc, ngoi cc bi tp tnh ton n gin da trn cc quy tc, tnh cht
c bn ca cc php tnh m cc hc sinh c rn luyn thng qua cc bi tp trong SGK
v SBT, cn c mt dng bi tp tnh ton trn cc dy s, dy phn s c quy lut m da
vo nhng quy lut tnh ton , hc sinh c th gii ton mt cch sng to, lgic, em li
nhiu hng th say m trong hc hc tp, pht trin t duy, tr tu, pht huy nng lc sng
to, nng khiu ton hc ca hc sinh.
Trong chuyn ny, cp mt s dng ton tnh ton trn cc dy s, dy phn s c
quy lut v mt vi tri nghim nh hng t duy hoc pht trin t duy hc sinh nhm bi
dng nng lc hc ton cho cc em hc sinh c kh nng hc gii ton.
1 Tm s cc s hng ca mt dy s c quy lut
Vi dng bi tp v dy cc s, dy cc phn s c quy lut, ta thng dng cc phng
php sau:
- Phng php phn tch s hng tng qut ri kh lin tip tnh tng cc dy s, dy
phn s c quy lut, gii ton tm x,v cc bi ton c lin quan.
- Phng php lm tri chng minh bt ng thc v cc bi ton lin quan. Vi phng
php ny ta thng dng tnh cht ca bt ng thc a mt v ca bt ng thc v
dng tnh c tng hu hn hoc tch hu hn.
tnh tng
S
n
= a
1
+ a
2
+a
3
+ + a
n
Ta biu din a
i
_
i = i, n
_
, , qua hiu hai s hng lin tip ca mt dy s khc. Chng hn
a
1
= b
1
b
2
; a
2
= b
2
b
3
; . . . ; a
n
= b
n1
b
n
S
n
= a
1
+a
2
+ a
3
+ +a
n
= b
1
b
n
tnh tch hu hn P
n
= a
1
.a
2
.a
3
. . . a
n
ta bin i cc a
k
v thng ca hai s hng lin tip
nhau :
a
1
=
b
1
b
2
; a
2
=
b
2
b
3
; . . . ; a
n
=
b
n1
b
n
P
n
= a
1
.a
2
, a
3
. . . . a
n
=
b
1
b
2
.
b
2
b
3
. . .
b
n1
b
n
=
b
1
b
n
Di y l mt s dng ton v dy phn s c quy lut ng thi cng nu mt s cc
dy s c quy lut vi cch tnh s hng tng qut.
120
Bi ton 1. Tm n sao cho tng ca 2n s hng
1
1.3
+
1
2.4
+
1
3.5
+ +
1
(2n 1).(2n + 1)
+
1
2n(2n + 2)
=
14651
19800
.
Gii. t
A =
1
1.3
+
1
2.4
+
1
3.5
+ +
1
(2n 1).(2n + 1)
+
1
2n(2n + 2)
Ta c
2
k(k + 2)
=
1
k

1
k + 2
vi k = 1; n
2A =
_
2
1.3
+
2
3.5
+ +
2
(2n 1).(2n + 1)
_
+
_
2
2.4
+
2
4.6
+ +
2
2n(2n + 2)
_
2A =
_
1
1
3
+
1
3

1
5
+ +
1
2n 1

1
2n + 1
_
+
_
1
2

1
4
+
1
4

1
6
+ +
1
2n

1
2n + 2
_
2A =
_
1
1
2n + 1
_
+
_
1
2

1
2n + 2
_
=
3
2
_
1
2n + 1
+
1
2n + 2
_
A =
14651
19800

3
2

_
1
2n + 1
+
1
2n + 2
_
=
14651
19800

1
2n + 1
+
1
2n + 2
=
199
9900

4n + 3
(2n + 1)(2n + 2)
=
199
9900
.
Do
(4n + 3; 2(2n + 1)) = (4n + 3; 4n + 2) = 1
v
(4n + 3; 2(2n + 2)) = (4n + 3; 4n + 4) = 1

4n + 3
(2n + 1)(2n + 2)
l phn s ti gin vi n l s t nhin bt k.

_
4n + 3 = 199
(2n + 1)(2n + 2) = 9900
n = 49.
Bi ton 2. Tm s nguyn dng n tha mn
2.2
2
+ 3.2
3
+ 4.2
4
+ 5.2
5
+ +n.2
n
= 2
n+10
Gii. Ta c
2.2
2
+ 3.2
3
+ 4.2
4
+ 5.2
5
+ +n.2
n
= 2
n+10
121
hay
1.2 + 2.2
2
+ 3.2
2
+ 4.2
4
+ 5.2
5
+ + n.2
n
= 2
n+10
+ 2.
V tri c th bin i nh sau :
2 + 2
2
+ 2
3
+ 2
4
+ + 2
n
= 2
n+1
2
2
2
+ 2
3
+ 2
4
+ + 2
n
= 2
n+1
2
2
2
3
+ 2
4
+ + 2
n
= 2
n+1
2
3
. . .
2
n
= 2
n+1
2
n
nn
1.2 + 2.2
2
+ 3.2
3
+ 4.2
4
+ 5.2
5
+ +n.2
n
= n.2
n+1
(2
n+1
2)
= 2
n+1
(n 1) + 2
2
n+1
(n 1) + 2 = 2
n+10
+ 2
n 1 =
2
n+10
2
n+1
= 2
9
n = 2
9
+ 1 = 513
Bi tp p dng
Bi ton 3. Cho
Q =
1

b
2

b
1

b
3

b
2
+
1

b
4

b
3
+ (1)
n
1

b
n

_
b
n1
Trong b
1
; b
2
; b
3
; b
4
; . . . ; b
n1
; b
n
; l cc s hng ca mt cp s cng (dy s cch u).
a) Tnh Q.
b) Bit cng sai (khong cch gia hai s hng lin tip nhau) l 17, s hng th nht l
bi ca 17 trong khong t 200 n 500, hy tnh n sao cho Q = 1 hoc Q = 1.
Gii.
a) Ta c
Q =
1

b
2

b
1

b
3

b
2
+
1

b
4

b
3
+ (1)
n
1

b
n

_
b
n1
Q =

b
2
+

b
1
b
2
b
1

b
3
+

b
2
b
3
b
2
+

b
4
+

b
3
b
4
b
3
+ (1)
n

b
n
+
_
b
n1
b
n
b
n1
Q =

b
2
+

b
1

b
3

b
2
+

b
4
+

b
3
+ (1)
n
(

b
n
+
_
b
n1
)
d
vi d = b
k
b
k1
Q =
_
b
n1
+

b
1
d
hoc Q =

b
1

_
b
n1
d
122
ty theo n chn hoc n l.
b) Do d = 17, b
1
l bi ca 17 trong khong t 200 n 500. Tnh n sao cho Q = 1 hoc
Q = 1.
Bi ton 4. Vit tt c cc phn s sau thnh dy.
1
1
;
2
1
;
1
2
;
3
1
;
2
2
;
1
3
;
4
1
;
3
2
;
2
3
;
1
4
; . . .
a) Hy nu quy lut vit ca dy v vit tip nm phn s na theo quy lut y.
b) Phn s
50
31
l s hng th my ca dy
Gii.
a) Quy lut ca dy l : Cc phn s theo nhm c tng ca TS v MS ln lt l cc s t
nhin lin tip.
1
1
;
2
1
;
1
2
;
3
1
;
2
2
;
1
3
;
4
1
;
3
2
;
2
3
;
1
4
;
5
1
;
4
2
;
3
3
;
2
4
;
1
5
; . . .
5 phn s tip theo ca dy l :
5
1
;
4
2
;
3
3
;
2
4
;
1
5
b) Phn s c tng TS v MS l 81. Ta nhn thy nu tng TS v MS l k th s cc phn
s vit c l (k 1). S cc phn s vit t u n khi c tng TS v MS l 80 l :
1 + 2 + 3 + + 79 =
(1 + 79).79
2
= 40.79 = 3160
Cc phn s c tng TS v MS l 81 c bt u nh sau :
Nh vy n phn s c 31 phn s c vit thm na. Vy phn s
50
31
l s hng th
(3160 + 31) = 3191
ca dy.
Bi ton 5. Cho
_
1 +
1
2
__
1 +
1
2
2
__
1 +
1
2
3
_
. . .
_
1 +
1
2
2
2009
_
= 2
_
1
1
2
2
n
_
.
Tm n?
2 Cc dng ton lin quan n bt ng thc
Bi ton 6. So snh
A =
1
2
2
+
1
3
2
+
1
4
2
+ +
1
n
2
vi 1 (n 2)
123
Gii. Da vo
1
n
2
<
1
n
2
1
vi (n 2)
A < B =
1
2
2
1
+
1
3
2
1
+ +
1
n
2
1
Do
B =
1
2
_
2
1.3
+
2
2.4
+
2
3.5
+ +
2
(n 1)(n + 1)
_
B =
1
2
_
1
1
3
+
1
2

1
4
+
1
3

1
5
+ +
1
n 1

1
n + 1
_
B =
1
2
_
1 +
1
2
+
1
3
+ +
1
n 1
_

1
2
_
1
3
+
1
4
+
1
5
+ +
1
n 1
+
1
n
+
1
n + 1
_
B =
_
1 +
1
2

1
n

1
n + 1
_
<
1
2
.
3
2
=
3
4
< 1 B < 1
Vy A < B < 1 hay A < 1
1
n
2
<
1
(n 1)n
A <
1
1.2
+
1
2.3
+ +
1
(n 1)n
Bi ton 7. So snh:
P =
1
2
2
+
1
4
2
+
1
6
2
+ +
1
(2n)
2
vi
1
2
.
Gii. Ta c
P =
1
2
2
+
1
4
2
+
1
6
2
+ +
1
(2n)
2
< Q =
1
2
2
1
+
1
4
2
1
+
1
6
2
1
+ +
1
(2n)
2
1
Do
1
2n 1

1
2n + 1
=
2
(2n 1)(2n + 1)
2Q =
2
1.3
+
2
3.5
+
2
5.7
+ +
2
(2n 1).(2n + 1)
2Q = 1
1
3
+
1
3

1
5
+
1
5

1
7
+ +
1
2n 1
+
1
2n + 1
2Q = 1
1
2n + 1
< 1 Q <
1
2
.
Do P < Q P <
1
2
(pcm).
Bi ton 8. So snh
1
2
.
3
4
.
5
6
.
7
8
. . .
999999
1000000
v
1
1000
.
124
Gii. Ta c
A =
1
2
.
3
4
.
5
6
.
7
8
. . .
999997
999998
.
999999
1000000
< A

=
2
3
.
4
5
.
6
7
.
8
9
. . .
999998
999999
1000000
1000000
A
2
< A.A

=
1
2
.
2
3
.
3
4
.
5
6
.
6
7
.
7
8
.
8
9
. . .
999997
999998
.
999998
999999
999999
1000000
1000000
1000000
A
2
=
1
1000
2
A <
1
1000
T bi tp ny ta c th chng minh BT cht ch hn :
1
2
.
3
4
.
5
6
.
7
8
. . .
2n 1
2n
<
1

3n + 1
vi n 1
Bi ton 9. Chng minh rng vi mi s t nhin n 1 ta c :
A = 1 +
1
2
2
+
1
3
2
+
1
4
2
+ +
1
n
2
>
n
n + 1
.
Bi ton 10. Chng minh rng vi mi s t nhin n 1 ta c :
1
5
+
1
13
+
1
25
+
1
41
+ +
1
n
2
+ (n + 1)
2
<
9
20
.
Bi ton 11. Chng minh rng
S =
1
2
+
1
2
2
+
1
2
3
+ +
1
2
7
< 1
Bi ton 12. Chng minh rng
A =
1
3
+
2
3
2
+
3
3
3
+ +
101
3
101
<
3
4
.
Gii. Ta c
2A = 3A A =
_
1 +
2
3
+
3
3
2
+
4
3
3
+ +
100
3
99
+
101
3
100
_

_
1
3
+
2
3
2
+
3
3
3
+ +
101
3
101
_
2A < 1 +
1
3
+
1
3
2
+
1
3
3
+ +
1
3
100
= 1 +
1
2
_
1
1
3
100
_
< 1 +
1
2
=
3
2
A <
3
4
Tng qut :
1
3
+
2
3
2
+
3
3
3
+ +
n
3
n
<
3
4
vi mi s t nhin n 1.
Bi ton 13. Chng minh rng
S =
1
3
2
+
2
3
3
+
3
3
4
+ +
100
3
101
<
1
4
125
Gii. Ta c
2S = 3S S =
_
1
3
+
2
3
2
+
3
3
3
+
4
3
4
+ +
100
3
100
_

_
1
3
2
+
2
3
3
+
3
3
4
+ +
100
3
101
_
2S =
1
3
+
1
3
2
+
1
3
3
+
1
3
4
+ +
1
3
100

100
3
101
2S <
1
3
+
1
3
2
+
1
3
3
+
1
3
4
+ +
1
3
100
=
1
2
_
1
1
3
100
_
<
1
2
S =
1
3
2
+
2
3
3
+
3
3
4
+ +
100
3
101
<
1
4
Tng qut : Vi mi n
mathbbN

, a
mathbbN

; a = 1 ta c th chng minh c :
a) S
1
=
1
a
+
1
a
2
+
1
a
3
+
1
a
4
+ +
1
a
n
<
1
a 1
b) S
2
=
1
a
+
2
a
2
+
3
a
3
+
4
a
4
+ +
n
a
n
<
a
(a 1)
2
c) S
3
=
1
a
2
+
2
a
3
+
3
a
4
+
4
a
5
+ +
n
a
n+1
<
a
(a 1)
2
Bi ton 14. Cho
S
1
= 1 +
1
5
;
S
2
= 1 +
1
5
+
1
5
2
;
S
3
= 1 +
1
5
+
1
5
2
+
1
5
3
; . . . ;
S
n
= S
n1
+
1
5
n
Chng minh rng : .
1
5S
2
1
+
1
5
2
S
2
2
+
1
5
3
S
2
3
+ +
1
5
n
S
2
n
<
35
36
Bi ton 15. Bit rng
a
2
1
+a
2
2
+ a
2
3
+a
2
4
+a
2
5
= 1.
Chng minh rng : gi tr nh nht ca (a
i
a
j
)
2
(1 i = j 5) khng th vt qu
1
10
.
Bi ton 16. Chng minh rng

n + 1

n >
1
2

n + 1
vi n mathbbN
T chng minh
1 +
1

2
+
1

3
+
1

4
+ + +
1

1994
< 2

1994
126
Gii. Ta c
1
2

n + 1
=
1

n + 1 +

n + 1
<
1

n + 1 +

n
=
(

n + 1

n)
(

n + 1 +

n)(

n + 1

n)
hay
1
2

n + 1
<

n + 1

n ( pcm)
Bi ton 17. Chng minh rng
1
5
+
1
13
+ +
1
2002
2
+ 2003
2
<
1
2
.
Bi ton 18. Chng minh rng
1
2

1
+
1
3

2
+ +
1
(n + 1)

n
vi n mathbbN; n 1.
Bi ton 19. Cho biu thc :
A =
3
4
+
8
9
+
15
16
+ +
9999
10000
.
Chng minh rng 98 < A < 99.
Tng qut ha bi ton, ta c
n 2 <
3
4
+
8
9
+
15
16
+ +
n
2
1
n
2
< n 1 vi n 2.
Bi ton 20. Cho tng
S
n
= 1 +
1
1 + 2
+
1
1 + 2 + 3
+ +
1
1 + 2 + 3 + +n
.
Tm s hu t s nh nht S
n
< a vi mi n
mathbbN

.
Bi ton 21. Cho
A =
1
14
+
1
29
+ +
1
n
2
+ (n + 1)
2
+ (n + 1)
2
+ +
1
1877
Chng minh rng
0, 15 < A < 0, 25
Ngoi ra c th chng minh bi ton tng qut :
1
6

1
3(k + 2)
<
1
1
2
+ 2
2
+ 3
2
+
1
2
2
+ 3
2
+ 4
2
+ +
1
k
2
+ (k + 1)
2
+ (k + 2)
2
<
1
4

1
2(k + 2)
127
Bi ton 22. Tm
A
B
bit
A =
1
2.32
+
1
3.33
+ +
1
n(n + 30)
+ +
1
1973.2003
B =
1
2.1974
+
1
3.1975
+ +
1
n(n + 1972)
+ +
1
31.2003
.
p dng (??) vi k = 30 ta c
30A =
30
2(2 + 30)
+
30
3(3 + 30)
+ +
30
1973(1973 + 30)
30A =
1
2

1
32
+
1
3

1
33
+ +
1
1973
+
1
2003
30A =
_
1
2
+
1
3
+ +
1
31
_

_
1
1974
+
1
1975
+ +
1
2003
_
(1)
p dng (??) vi k = 1972 ta c
1972B =
1972
2(2 + 1972)
+
1972
3(3 + 1972)
+ +
1972
31(31 + 1972)
1972B =
_
1
2

1
1974
_
+
_
1
3

1
1975
_
+ +
_
1
31

1
2003
_
1972B =
_
1
2
+
1
3
+ +
1
31
_

_
1
1974
+
1
1975
+ +
1
2003
_
(2)
T (1) v (2) ta c :
30A = 1972B
A
B
=
1972
30
=
986
15
.
Bi ton 23. Chng minh rng
A =
1
3
2

1
3
4
+
1
3
6
+
1
3
4n+2

1
3
4n
+
1
3
100
< 0, 1
Hy tng qut ha bi ton trn.
Bi tp vn dng
Bi ton 24. Chng t rng tng ca 100 s hng u tin ca dy sau nh hn
1
4
.
1
5
;
1
45
;
1
117
;
1
221
;
1
357
; . . .
Bi ton 25. Chng minh rng
A =
3
4
+
8
9
+
15
16
+ +
2499
2500
> 48.
128
Bi ton 26. Chng minh rng
a) n! > 2
n1
(n 3)
b) 1 + b + b
2
+ + b
n
=
1 b
n1
1 b
(b = 1)
c) 1 +
1
1!
+
1
2!
+ +
1
n!
< 3.
Bi ton 27. Cho cc s dng a
1
; a
2
; . . . ; a
n
. Chng minh rng
C
2
n
_
1
a
1
a
2
+
1
a
1
a
3
+
1
a
1
a
4
+ +
1
a
1
a
n
+
1
a
2
a
3
+
1
a
2
a
4
+ +
1
a
n1
a
n
_

4
_
1
a
1
+a
2
+
1
a
1
+a
3
+
1
a
1
+a
4
+ +
1
a
1
+a
n
+
1
a
2
+a
3
+
1
a
2
+a
4
+ +
1
a
n1
+a
n
_
Tm iu kin ca a
k
(k = 1; 2; 3; 4; . . . ; n) c ng thc.
Bi ton 28. Cho cc s t nhin a
1
< a
2
< < a
n
. Chng minh rng tng A:
A =

a
2
a
1
a
2
+

a
3
a
2
a
3
+ +

a
n
a
n1
a
n
< 1 +
1
2
+
1
3
+ +
1
n
2
Bi ton 29. Chng minh rng
1
n + 1
+
1
n + 2
+
1
n + 3
+ +
1
2n
>
1
2
vi n mathbbN; n > 1
Bi ton 30. Chng minh rng
1

1
+
1

2
+
1

3
+ +
1

n
>

n vi n mathbbN; n > 1
Bi ton 31. Chng minh rng
2(

n + 1 1) < 1 +
1

2
+
1

3
+ +
1

n
> 2

n 1 vi n mathbbN
Bi ton 32. Chng minh rng
1
9
+
1
25
+ +
1
(2n + 1)
2
<
1
4
vi n mathbbN; n 1
Bi ton 33. Chng minh rng
1
5
+
1
13
+
1
25
+
1
41
+ +
1
n
2
+ (n + 1)
2
<
1
2
129
Bi ton 34. Chng minh rng
1
65
<
1
5
3
+
1
6
3
+ +
1
n
3
+ +
1
2004
3
<
1
40
Bi ton 35. Chng minh rng
1
3
+
2
3
2
+
3
3
3
+
4
3
4
+ + +
n
3
n
<
3
4
Bi ton 36. Chng minh rng
S
n
=
3
1
2
.2
2
+
5
2
2
.3
2
+
7
3
2
.4
2
+ +
2n + 1
n
2
.(n + 1)
2
+
n
2
4(1
3
+ 2
3
+ +n
3
)
l mt hng s
Bi ton 37. Chng minh rng
a)
1
2

1
4
+
1
8

1
16
+
1
32

1
64
<
1
3
b)
1
3

2
3
2
+
3
3
3

4
3
4
+ +
99
3
99

100
3
100
<
3
16
c)
1
5
2

2
5
3
+
3
5
4

4
5
5
+ +
99
5
100

100
5
101
<
1
36
Bi ton 38. So snh tng A gm 11 s hng sau vi
1
16
A =
1
5
2
+
2
5
3
+
3
5
4
+ +
n
5
n+1
+ +
11
5
12
Tng qut bi ton, ta c: Vi a, n l s t nhin khc 0, a > 1 th
A =
1
a
2
+
2
a
3
+
3
a
4
+ +
n
a
n+1
<
1
(a 1)
2
.
Bi ton 39. Chng minh rng
1
1 + a
2

a
(1 + a
2
)
2
+
a
2
(1 + a
2
)
3

a
3
(1 + a
2
)
4
+ +
(1)
n+1
.a
n1
(1 + a
2
)
n
+
3
4
.
3 Cc bi ton tng hp
1. Ton chia ht
Bi ton 40. Cho
A = 1.2.3.4. . . . 1001;
B = 1002.1003.1004 . . . 2002.
Hi (A +B) c chia ht cho 2003 khng ?
130
Gii. Ta c
B = (2003 1001)(2003 1000)(2003 999) . . . (2003 1)
B = BS(2003) 1.2.3.4 . . . 1001 = BS(2003) A
A +B = BS(2003)
Vy (A +B) chia ht cho 2003.
Bi ton 41. Vit tng
2
1
+
2
2
2
+
2
3
3
+ +
2
n
n
v dng phn s
P
S
.
Chng minh rng p
.
.
.8 vi n > 3.
Bi ton 42. Cho tng
1 +
1
2
+
1
3
+ +
1
18
=
a
b
vi (a; b) = 1.
Cc mu s cc s hng ca tng l cc s t nhin lin tip t 2 n 18.
Chng minh rng :
b
.
.
.11.13.17
Tng qut bi ton, ta c : Nu p l s nguyn t, n l s t nhin tha mn p < n < 2p v
tng
1 +
1
2
+
1
3
+ +
1
n 1
+
1
n
=
a
b
vi (a; b) = 1
Th b
.
.
.p
2. Ton tm x
Bi ton 43. Tm x bit
_
2
11.13
+
2
13.15
+
2
15.17
_
.561 [3, 6 : (x 9, 52) : 1, 2] = 10
Gii.
_
2
11.13
+
2
13.15
+
2
15.17
_
.561 [3, 6 : (x 9, 52) : 1, 2] = 10

_
1
11

1
13
+
1
13

1
15
+
1
15

1
17
_
.561 10 = [3, 6 : (x 9, 52) : 1, 2]

_
1
11

1
17
_
.561 10 =
_
18
5
: (x 9, 52) :
6
5
_

6
187
.561 10 =
_
18
5
.
5
6
: (x 9, 52)
_
18 10 = [3 : (x 9, 52)] 8 = 3 : (x 9, 52) (x 9, 52) = 3 : 8
x 9, 52 = 0, 375 x = 9, 895
131
Bi ton 44. Tm x bit
a)
_
1
11.13
+
1
13.15
+
1
15.17
+ +
1
19.21
_
:
0, 75x + 4
x
=
4
231
b)
_
17
13

_
1
2
+
1
52
_
_
_
x
66
44
_
=
1
1.4
+
1
4.7
+
1
7.10
+
1
10.13
c)
1
5.8
+
1
8.11
+
1
11.14
+ +
1
x(x + 3)
=
101
1540
d) 1 +
1
3
+
1
6
+
1
10
+ +
1
x(x + 1) : 2
= 1
1991
1993
e)
_
1
21.22
+
1
22.23
+ +
1
29.30
_
.140 + 1, 08 : [0, 3.(x 1)] = 11
f)
_
2
31.33
+
2
33.35
+ +
1
39.41
_
.2542 [2, 04.(x + 1, 05)] : 0, 12 = 19
Bi ton 45. Tm t s ca hai s A v B bit :
A =
1
1.1981
+
1
2.1982
+
1
3.1983
+ +
1
n(1980 + n)
+ +
1
25.2005
B =
1
1.26
+
1
2.27
+
1
3.28
+ +
1
m(25 + m)
+ +
1
1980.2005
Trong A c 25 s hng, B c 1980 s hng.
Bi ton 46. Tm t s ca hai s A v B bit : A c (n 1) tha s v
A =
_
1
1
1 + 2
__
1
1
1 + 2 + 3
__
1
1
1 + 2 + 3 + 4
_
. . .
_
1
1
1 + 2 + +n
_
B =
n + 2
n
.
Bi ton 47. Tnh
M
N
bit
M =
1
3.5
+
1
5.7
+
1
7.9
+ +
1
29.31
N =
1
3.5.7
+
1
5.7.9
+
1
7.9.11
+ +
1
27.29.31
Bi ton 48. Tnh
A
B
bit
A =
1
2.32
+
1
3.33
+ +
1
n(n + 30)
+ +
1
1973.2003
B =
1
2.1974
+
1
3.1975
+ +
1
n(n + 1972)
+ +
1
31.2003
.
Ngoi cc bi tp nu trn , cn rt nhiu bi tp khc v dy s v dy cc phn s c quy
lut. Cc bn c th tham kho thm trong bo Ton hc tui tr, bo Ton tui th, cc sch
Chuyn ton tham kho.
132
Ti liu tham kho
[1] Phan Huy Khi(2004), "Chuyn bi dng hc sinh gii ton trung hc ph thng ",
Cc bi ton c bn ca s hc, NXB Gio dc.
[2] H Huy Khoi, "Chuyn bi dng hc sinh gii ton trung hc ph thng",S hc,
NXB Gio dc.
[3] Nguyn Vn Mu (ch bin), Mt s vn s hc chn lc, NXB Gio dc.
[4] ng Hng Thng (1995), Bi ging s hc, NXB Gio dc.
[5] ng Hng Thng, Nguyn Vn Ngc, V Kim Thy(1997), Bi ging s hc,Tuyn tp
30 nm tp ch ton hc v tui tr, NXB Gio dc.
[6] V Dng Thy (ch bin)(2006), Tuyn tp 40 nm Olympiads Ton hc quc t(IMO
1959-2000), NXB Gio dc.
133
VN DNG TNH N IU TRONG CC BI
TON TM GII HN DY S V GII PHNG
TRNH, BT PHNG TRNH, H PHNG
TRNH
Trng Vn im, Trng THPT Chuyn L Qu n - Khnh Ha
Trong nhiu nm qua, bi ton v gii hn dy s, phng trnh, h phng trnh l mt dng
thng gp trong cc k thi chn hc sinh gii quc gia cng nh cp tnh. y l mt dng
rt c bn v phn l thuyt cng rt n gin, bi vit ny cp n vic vn dng tnh n
iu gii quyt.
1 L thuyt c bn
Cc bi ton v dy s, gii phng trnh, bt phng trnh v h phng trnh c ni dung
kh a dng. y ta quan tm cc bi ton tm gii hn dy s (bn cht gii tch) v cc
bi ton gii phng trnh, bt phng trnh v h phng trnh c vn ng tnh n iu
ca hm s.
Vi bi ton gii hn dy s , ta cn nm vng nh ngha ca gii hn dy s v cc nh
l c bn v gii hn dy s, bao gm:
1. nh l Weierstrass: Dy n iu v b chn th hi t.
2. nh l kp: Nu x
n
y
n
z
n
vi mi n n
0
v lim
n
x
n
= lim
n
z
n
= a th lim
n
y
n
= a.
Mt trong nhng dng dy s thng gp nht l dy s xc nh bi x
0
= a, x
n+1
= f(x
n
)
vi f l mt hm s no . V vi loi dy s ny, cu hi thng gp nht l:
1. Chng minh dy s {x
n
} c gii hn hu hn
2. Tm tt c cc gi tr ca a sao cho dy s {x
n
} c gii hn hu hn
gii cc bi ton dng ny, ta c mt s tnh cht c bn sau
1. Nu f l hm s tng th dy {x
n
} s l dy n iu.
2. Nu f l hm s gim th cc dy {x
2n
} (dy vi ch s chn) v {x
2n+1
} (dy vi ch s
l) s l cc dy n iu.
3. Nu vi mi x, y ta c |f(x) f(y)| q|x y| vi q l hng s 0 < q < 1 v {x
n
} b chn
th {x
n
} hi t. c bit nu |f

(x)| q < 1 th ta lun c iu ny.


Vi dng bi ton gii phng trnh, bt phng trnh v h phng trnh ta ch cn nm
vng tnh n iu ca hm s ( vn dng o hm) ca mt hoc hai hm s lin quan m ta
phi nhn bit thng bi ton
134
2 Mt s bi ton minh ha
2.1 Cc bi ton v dy s
Bi ton 1. Cho dy s {x
n
} xc nh bi x
0
=

2 v x
n+1
=

2
xn
vi n = 0, 1, 2, . . . Chng
minh rng dy {x
n
} c gii hn hu hn v tm gii hn .
Li gii. t f(x) = (

2)
xn
th dy s c dng x
0
=

2 v x
n+1
= f(x
n
). Ta thy f(x) l
hm s tng v x
1
=

2
>

2 = x
0
. T , do f(x) l hm s tng nn ta c x
2
= f(x
1
) >
f(x
0
) = x
1
, x
3
= f(x
2
) > f(x
1
) = x
2
, ... Suy ra {x
n
} l dy s tng. Tip theo, ta chng minh
bng quy np rng x
n
< 2 vi mi n. iu ny ng vi n = 0. Gi s ta c x
k
< 2 th r
rng x
k+1
=

2
x
k
<

2
2
= 2. Theo nguyn l quy np ton hc, ta c x
n
< 2 vi mi n.
Vy dy {x
n
} tng v b chn trn bi 2 nn dy c gii hn hu hn. Gi a l gii hn
th chuyn ng thc x
n+1
=

2
xn
sang gii hn, ta c a =

2
a
. Ngoi ra ta cng c a 2.
Xt phng trnh x =

2
x

ln x
x
= ln(

2) . Kho st hm s
ln x
x
ta thy rng phng
trnh trn ch c 1 nghim < e v mt nghim ln hn e. V 2 l mt nghim ca phng trnh
nn r rng ch c 1 nghim duy nht ca phng trnh tho mn iu kin 2. T suy ra
a = 2.
Vy gii hn ca {x
n
} khi n dn n v cng l 2.
Bi ton 2. ( d b VMO 2008) Cho s thc a v dy s thc {x
n
} xc nh bi:
x
1
= a v x
n+1
= ln(3 + cos x
n
+ sinx
n
) 2008 vi mi n = 1, 2, 3, ...
Chng minh rng dy s {x
n
} c gii hn hu hn khi n tin n dng v cng.
Li gii. t f(x) = ln(3 + cos x
n
+sinx
n
) 2008 th
f

(x) =
cos x sin x
3 + sin x + cos x
T , s dng nh gi | cos x sin x|

2, | sin x + cos x|

2 ta suy ra
|f

(x)|

2
3

2
= q < 1.
p dng nh l Lagrange cho x, y thuc R, ta c
f(x) f(y) = f

(z)(x y)
T suy ra |f(x) f(y)| q|x y| vi mi x, y thuc R.
p dng tnh cht ny vi m > n N, ta c
|x
m
x
n
| = |f(x
m1
)f(x
n1
)| q|x
m1
x
n1
| q
n1
|x
mn+1
x
1
| q
N1
|x
mn+1
x
1
|.
Do dy {x
n
} b chn v q < 1 nn vi mi > 0 tn ti N ln q
N1
|x
mn+1
x
1
| < .
Nh vy dy {x
n
} tho mn iu kin Cauchy do hi t.
135
Bi ton 3. ( thi chn HSG Ngh An 2007) Chng minh rng vi mi s n nguyn dng
, lun tn ti duy nht mt s thc x
n
sao cho
1
2008
xn
x
n
+ n = 0. Xt dy s (x
n
), tm
lim(x
n+1
x
n
).
Li gii. Vi n N

, xt f(x) =
1
2008
x
x +n; x R.
f

(x) =
ln 2008
2008
x
1 x R. f(x) nghch bin trn R. (1).
Ta c
_
f(n) =
1
2008
n
> 0
f(x
n+1
) =
1
2008
n+1
1 < 0
f(x) = 0 c nghim x
n
(n, n + 1) (2).
T (1) v (2) suy ra pcm.
Ta c x
n
n =
1
2008
xn
> 0 x
n
> n 0 < x
n
n <
1
2008
n
.
Mt khc lim
1
2008
n
= 0 lim(x
n
n) = 0. Khi lim(x
n+1
x
n
) = lim{[x
n+1
(n + 1)]
(x
n
n) + 1} = 1.
Bi ton 4. Cho a R v xt dy s thc (x
n
), n = 0, 1, 2, 3, . . . xc nh bi
_
x
0
= a
x
n
=
3
_
6x
n1
6 sin(x
n1
)
Chng minh rng a R, dy s (x
n
) lun c gii hn hu hn khi n +. Hy tnh lim(x
n
).
Li gii.
+) Trng hp 1: Xt a = 0 th x
n
= 0 n = 0, 1, 2, 3, . . . Suy ra limx
n
= 0.
+) Trng hp 2: Xt a > 0. Ta lun c
x
x
3
6
sin x x, x 0 (Du = xy ra khi x = 0)
V sin x x x
n
0, n (Cm qui np)
V sin x x
x
3
6
sin x
n1
> x
n1

x
3
n1
6
, n 1
6x
n1
6 sin x
n1
< x
3
n1
n 1
x
n
=
3

6x
n1
6 sin x
n1
< x
n1
n 1
Suy ra (x
n
) l dy gim v b chn di bi 0, nn (x
n
) c gii hn hu hn khi n +
t limx
n
= 0 =
3

6 6 sin = 0
+) Trng hp 3: Xt a < 0. t b = a > 0.
Xt dy (y
n
) xc nh nh sau:
_
y
1
= b
y
n+1
=
3

6y
n
6 sin y
n
vi n = 0, 1, 2, . . .
Theo CM trn th (y
n
) c gii hn hu hn bng 0 . Suy ra limx
n
= 0
Kt lun: a R, dy (x
n
) c gii hn hu hn v limx
n
= 0.
Bi ton 5. ( chn HSG ca Thanh Ha 2006) Cho dy (x
n
) : x
0
= 1, x
n
=
1
1+x
n1
. Tnh
limx
n
.
136
Li gii. Nhn ra dy dng v: x
0
= 1, x
2
=
2
3
, x
4
=
5
8
. . . v x
1
=
1
2
, x
3
=
3
5
. . .
Chia dy cho thnh hai dy con (x
2n
) v x
2n+1
.
Ta chng minh bng quy np (x
2n
) gim.
Ta c: x
2n+2
=
1
1+x
2n+1
=
1+x
2n
2+x
2n
; x
2n
=
1+x
2n2
2+x
2n2
Gi thit x
2n
< x
2n2
, cho ta x
2n+2
< x
2n

1+x
2n
2+x
2n
<
1+x
2n2
2+x
2n2
x
2n
< x
2n2
ng.
Lc : x
2n+2
=
1+x
2n
2+x
2n
< x
2n
x
2n
>
1+

5
2
. Tng t vi dy (x
2n+1
), ta c 1 = x
0
> ... >
x
2n
>
1+

5
2
v
1
2
= x
1
< ... < x
2n1
<
1+

5
2
Suy cc dy (x
2n
) v (x
2n+1
) n iu, b chn. Theo du hiu Vaistrat, cc dy ny c gii
hn. Khi gi A = lim
n
x
2n
, B = lim
n
x
2n+1
.
Ta c h: A =
1
1+B
v B =
1
1+A
. Gii h ta c :A = B =
1+

5
2
Dy cho gm hp ca hai dy trn c cng gii hn. Suy ra lim
n
x
n
=
1+

5
2
Bi ton 6. ( ngh Olympic 30 4 nm 2001) Cho dy s {x
n
} tho mn x
1
=
5
2
v
x
n+1
=
1
2
x
2
n
+x
n
2 vi mi n l s nguyn dng. Chng minh rng dy {x
n
} hi t. Tm gii
hn ca n.
Li gii. T cng thc xc nh dy ta c: x
n+2
=
1
2
x
2
n+1
+ x
n+1
2 =
1
2
_
1
2
x
2
n
+x
n
2

2
+
1
2
x
2
n
+x
n
4 =
1
8
x
4
n
+
1
2
x
3
n
x
n
2
Xt hm s: f(x) =
1
8
x
4
+
1
2
x
3
x 2 x (2; 1) Ta c: f

(x) =
1
2
x
3
+
3
2
x
2
1 v
f

(x) =
1
2
x
3
+
3
2
x
2
1 < 0 x (2; 1)
Vy f

(x) > f

(1) = 0 (do f

(x) nghch bin trn (2; 1))


Do f(x) ng bin trn khong (2; 1), nn suy ra f(2) < f(x) < f(1), x (2; 1),
hay 2 < f(x) <
11
8
< 1, x (2; 1)
Nhng x
2n+2
= f(x
2n
) , t suy ra 2 < x
2n
< 1, vi n = 1, 2, 3, . . .
Mt khc do x
2
> x
4
ta suy ra f(x
2
) > f(x
4
) hay x
4
> x
6
,. . .
Hon ton tng t ta c: 1 > x
2
> x
4
> x
6
> ... > x
2n
> ... > 2.
Vy dy {x
2n
} gim v b chn di nn hi t.
t a = lim
n
x
2n
, t cng thc xc nh dy, chuyn qua gii hn ta c:
a =
1
8
a
4
+
1
2
a
3
a 2 (a 2)(a + 2)
3
= 0 a = 2
Hon ton tng t ta chng minh c dy {x
2n+1
} tng v b chn trn bi 2 v lim
n
x
2n
=
2. Nh vy dy {x
n
} hi t v c gii hn bng 2.
Bi ton 7. ( ngh Olympic 30 4 nm 2002) Cho dy s {x
n
} xc nh bi: x
0
=
2, 7; x
3
n+1
3x
n+1
(x
n+1
1) = x
n
+ 1. Chng minh rng dy s {x
n
} hi t.
Li gii. Ta c: x
3
n+1
3x
n+1
(x
n+1
1) = x
n
+ 1 (x
n+1
1)
3
= x
n
x
n+1
=
3

x
n
+ 1
Xt hm s g(x) = 1 +
3

x, ta c x
n+1
= g(x
n
). Ta c:
g

(x) =
1
3
3

x
2
0 < g

(x) =
1
3
3

4
= q < 1, x > 2
137
Do x
0
= 2, 7 x
n
> 2, n . Ta c x = g(x) x = 1+
3

x x1 =
3

x x
3
3x
2
+3x1 =
x f(x) = x
3
3x
2
+ 2x 1 = 0
Ta c: f

(x) = 3x
2
6x+2 = 3x(x2)+2 > 0; x > 2. Mt khc: f(2) = 1 < 0; f(3) = 5 > 0.
Do phng trnh x = g(x) c ng mt nghim x = r (2; 3). Ta chng minh: |x
n
r|
q
n
, n.
Vi n = 0 : |x
0
r| = |2, 7 r| < 1 = q
0
.
Gi s |x
k
r| q
k
, ta c |x
k+1
r| = |g(x
k
) g(r)|
p dng nh l Lagrange, ta c:
|x
k+1
r| = |g

(c)(x
k
r)| = |g

(c)| |x
k
r| |g

(c)| .q
k
Do x
k
, r > 2 c (x
k
; r) c (r; x
k
) c > 2 0 < g

(c) < q.
Vy |x
k+1
r| q
k
.q = q
k+1
. Theo nguyn l quy np ta c: |x
n
r| q
n
, n . Vy lim
n
x
n
=
r (do 0 < q < 1).
Bi ton 8. ( ngh Olympic 30 4 nm 2002) Cho dy s {x
n
} xc nh bi x
1
= x
2
= 1
v x
n+2
=
2
5
x
2
n+1
+
2
5
sin x
n
; n N

. Chng minh rng dy {x


n
} c gii hn v tnh gii
hn ca n.
Li gii. Trc ht ta chng minh: x
n
0;

2
_
, n N

. Tht vy: x
1
= x
2
= 1 0;

2
_
.
Gi s c x
n
0;

2
_
, k n. Khi : x
n+1
<
2
5

2
_
2
+
2
5
=

2
vi x
n
0;

2
_
v
x
n+1
> 0 x
n+1
0;

2
_
Theo nguyn l quy np: x
n
0;

2
_
, n N

.
Xt hm f(x) =
2
5
x
2
+
2
5
sin x x vi x 0;

2
_
.
f

(x) =
4
5
x +
2
5
cos 1 v f

(x) =
4
5

2
5
sin x
Ta c f

(x) = 0
4
5

2
5
sin x = 0 x = arcsin
2

2
= x
0
Ta c bng bin thin ca f

(x):
x 0 x
0

2
f

(x) + 0 0
f

(x)
2
5
1
3
5
T bng bin thin suy ra f

(x) = 0 c nghim duy nht x


1
x
0
;

2
_
v f

(x) > 0 vi x (0; x


1
)
,f

(x) < 0 vi x x
1
;

2
_
Khi ta c bng bin thin ca hm f(x):
x 0 x
1

2
f

(x) + 0 0
f(x) 0 0
138
T suy ra f(x) > 0, x 0;

2
_
v f(x) = 0 ti x = 0; x =

2
. Vy dy {x
n
} l dy b chn
(1)
Mt khc t cng thc xc nh dy v bng quy np, ta d dng chng minh c dy l
khng gim. T suy ra dy c gii hn.
Gi a = lim
n
x
n
1 a

2
v a tha mn f(a) = 0 suy ra a =

2
. Vy lim
n
x
n
=

2
.
Bi ton 9. ( Phan Huy Khi ) Dy s thc xc nh theo quy lut: x
1
= 2, 9; x
n+1
=

3 +
xn

x
2
n
1
vi n 1. Hy tm mt s thc nm bn tri dy con {x
1
, x
3
, ...} v bn phi dy con
{x
2
, x
4
, ...} ca dy s {x
n
}.
Li gii. Quy lut dy s suy ra rng x
n

3 vi mi s t nhin n 1. Yu cu ca bi
ton l tm s thc a sao cho x
2k
< a < x
2k1
vi mi s t nhin k = 1, 2, 3, . . .
D on a cn tm l gii hn ca dy s {x
n
} khi n . Khi a l nghim ca phng
trnh x =

3 +
x

x
2
1
vi x

3 ()
+) Gii phng trnh (*). V x

3 suy ra 0 <
1
x
< 1 nn c th t x =
1
sin
; 0;

2
_
. Khi
(*) tr thnh
1
sin
=

3 +
1
cos
sin cos +

3 sin cos = 0. Gii phng trnh lng


gic ny ta tm c sin =

3
6

5 1
_
v do sin > 0 nn chn sin =

3
6

5 1
_

x =

3
2

5 + 1
_
+) Ly a =

3
2

5 + 1
_
ta chng minh x
2k
< a < x
2k1
vi mi s t nhin k = 1, 2, 3, . . .
bng quy np
Da vo quy lut dy s, xt hm s y = f(x) =

3 +
x

x
2
1
(x

3).
Hm f lin tc v c o hm trn
_
3; +
_
; f

(x) =
1
(x
2
1)

x
2
1
< 0 ; x

3, suy ra
hm f nghch bin trn
_
3; +
_
. Ta c:
Vi k = 1 th x
1
= 2, 9. Khi a =

3
2

5 + 1
_
< 2, 9 = x
1
v x
2
= f(x
1
) =

3 +
2,9

2,9
2
1
<
f(a) = a. Vy vi k = 1 ta c x
2
< a < x
1
hay mnh ng vi k = 1.
Gi s mnh ng vi k = n ta chng minh n ng vi k = n + 1. Tht vy, do gi thit
quy np x
2n
< a < x
2n1
. Khi
x
2n+1
= f(x
2n
) > f(a) = a
x
2n+2
= f(x
2n+1
) < f(a) = a
Vy ta c iu phi chng minh: x
2n+2
< a < x
2n+1
.
Theo nguyn l quy np ta c: x
2k
< a =

3
2

5 + 1
_
< x
2k1
ng vi mi k = 1, 2, . . . tc
a =

3
2

5 + 1
_
l gi tr cn tm.
Bi ton 10. ( chnh thc - Olympic 30 4 nm 2002 )
Cho phng trnh x
n
+ x
n1
+ ... + x 1 = 0. Chng t rng vi mi n nguyn dng th
phng trnh c duy nht mt nghim dng x
n
v tm lim
n
x
n
.
Li gii. t f
n
(x) = x
n
+ x
n1
+ ... + x 1t h f
n
(x) l hm s lin tc trn ton trc s.
Xt:
f
n
(x
1
) f
n
(x
2
) = (x
n
1
x
n
2
) + x
n1
1
x
n1
2
_
+ (x
1
x
2
)
139
Nu 0 < x
2
< x
1
th f
n
(x
1
) f
n
(x
2
) > 0 hay f
n
(x
1
) > f
n
(x
2
) Do hm s f
n
(x) l hm ng
bin trn (0; +) (c th dng o hm chng minh)
Hn na f
n
(0) = 1 < 0, f
n
(1) = n1 > 0 vi n 2 nn phng trnh f
n
(x) = 0 c nghim
dng duy nht x
n
. V 1 = x
n
+x
2
n
+... +x
n
n
nn khi n tng th x
n
gim, tc l dy {x
n
} gim
v b chn di. Do tn ti x
0
= lim
n
x
n
Mt khc, 1 =
1x
n
n
1xn
x
n
v 0 < x
n
< 1 nn cho qua gii hn ta c
x
0
1x
0
= 1 hay x
0
=
1
2
. Vy
lim
n
x
n
=
1
2
.
Bi ton 11. (VMO -2005) Xt dy s thc (x
n
), n = 1, 2, 3, ..., xc nh bi: x
1
= a v
x
n+1
= 3x
3
n
7x
2
n
+ 5x
n
vi mi n = 1, 2, 3, ... ,trong a l mt s thc. Hy xc nh tt c
cc gi tr ca a dy s (x
n
) c gii hn hu hn khi n +. Hy tm gii hn ca dy s
(x
n
) trong cc trng hp .
Li gii. Xt hm s f(x) = 3x
3
7x
2
+ 5x. Khi , c th vit h thc xc nh dy (x
n
)
di dng x
n+1
= f(x
n
) vi mi n = 1, 2, 3, ....
Ta c f

(x) = 9x
2
14x + 5. T , ta c bng bin thin sau ca hm f(x) :
x
5
9
1 +
f

(x) + 0 0 +
f(x)
275
243
1
+
V f(x) x = 3x
3
7x2 + 4x = x(x 1)(3x 4) nn
f(x) = x x = 0 hoc x = 1 hoc x =
4
3
(1)
f(x) < 0 khi x < 0 (2)
f(x) > 0 khi x >
4
3
(3)
Hn na, do f(0) = 0, f(
4
3
) =
4
3
v
275
243
<
4
3
nn t s bin thin ca hm f trn R suy ra:
Vi mi x (; 0) lun c f(x) (; 0) (4)
Vi mi x (0;
4
3
) lun c f(x) (0;
4
3
) (5)
Vi mi x (
4
3
; ) lun c f(x) (
4
3
; ) (6)
Xt cc trng hp sau:
Trng hp 1: a < 0. Khi :
(4) x
n
(; 0) n 1.
(2) x
2
= f(x
1
) < x
1
.
T , do hm s f(x) ng bin trn khong (; 0), d dng chng minh c rng dy
(x
n
) l mt dy s gim. Kt hp iu ny vi (1), suy ra nu (x
n
) l dy hi t v limx
n
=
th phi c {0; 1;
4
3
} v < a. V a < 0 nn khng th c s tha mn ng thi hai iu
kin nu trn. iu ny chng t dy (x
n
) khng l dy hi t.
Trng hp 2: a >
4
3
. Khi :
(6) x
n
(
4
3
; +) n 1
(3) x
1
< f(x
1
) = x
2
.
140
T , do hm s f(x) ng bin trn khong (
4
3
; +), d dng chng minh c rng dy
(x
n
) l mt dy s tng. Kt hp iu ny vi (1), suy ra nu (x
n
) l dy hi t v limx
n
=
th phi c {0; 1;
4
3
} v > a.
V a > nn khng tn ti s tha mn ng thi hai iu kin nu trn. iu ny chng
t dy (x
n
) khng l dy hi t.
Trng hp 3: a = 0. Khi , dy (x
n
) l dy hng: x
n
= 0 n 1 . V vy, (x
n
) l dy hi
t v limx
n
= 0.
Trng hp 3: a =
4
3
. Khi , dy (x
n
) l dy hng: x
n
=
4
3
n 1 . V vy, (x
n
) l dy hi
t v limx
n
=
4
3
.
Trng hp 5: 0 < a <
4
3
. Khi , t (5) suy ra x
n
(0;
4
3
) n 2. Ta c:
|x
n+1
1| = |3x
3
n
7x
2
n
+ 5x
n
1| = (x
n
1)
2
|3x
n
1| n 1. (7)
V x
n
(0;
4
3
) n 1 nn |3x
n
1| < 1 n 1. Do , t (7) suy ra
|x
n+1
1| < (x
n
1)
2
n 1.
T , bng quy np theo n, d dng chng minh c rng
|x n 1| < (a 1)
2
n1
n 1 (8)
V a (0;
4
3
) nn |a 1| < 1. Do lim(a 1)
2
n1
= 0. V th, t (8) suy ra dy (x
n
) l dy
hi t v limx
n
= 1.
Vy tm li, dy (x
n
) l dy hi t khi v ch khi a [0;
4
3
]. V khi :
limx
n
= 0 nu a = 0.
limx
n
=
4
3
nu a =
4
3
.
limx
n
= 1 nu a (0;
4
3
)
2.2 Dng 2: Phng trnh, h phng trnh
1> Nu hm s y=f(x) lun lun tng( hoc gim) trn D th phng trnh f(x)=0 c nhiu
nht mt nhim trn D
2>Nu f(x)=0 c i du mt ln th phng trnh f(x)=0 c nhiu nht hai nghim trn
D.
Bi ton 1. Gii phng trnh 2x
3
+ 10x
2
17x + 8 = 2x
2
3

5x x
3
(1)
Li gii. x = 0 khng phi l nghim ca phng trnh nn (1) 2x
3
+
10
x

17
x
2
+
8
x
3
=
2
3
_
5
x
2
1
t t =
1
x
(t = 0).
8t
3
17t
2
+ 10t 2 = 2
3

5t
2
1
(2t 1)
3
+ 2(2t 1) = 5t
2
1 2
3

5t
2
1 (2)
141
Xt f(x) = x
3
+ 2x f

(x) = 3x
2
+ 2 > 0 x R
suy ra f lun tng trn R.
T (2), ta c:
f(2t 1) = f(
3

5t
2
1)
2t 1 =
3

5t
2
1.
8t
3
12t
2
+ 6t 1 = 5t
2
1.

_
t = 0 (loi)
t =
17

97
16
Vy nghim ca phng trnh x =
17

97
12
.
Bi ton 2. Gii phng trnh
1
2
log
2
(x + 2) + x + 3 = log
2
2x+1
x
+ 1 +
1
x
_
2
+ 2

x + 2
Li gii. iu kin : 2 < x <
1
2
; x > 0
Phng trnh tng ng
log
2

x + 2 2

x + 2 + x + 2 = log
2
_
2 +
1
x
_
+
_
2 +
1
x
_
2
2
_
2 +
1
x
_
t f(t) = log
2
t +t
2
2t (t > 0)
Ta c
f

(t) =
1
t ln 2
+ 2t 2
f

(t) = 2
1
t
2
ln 2
f

(t) > 0 2 >


1
t
2
ln 2
t >
1

ln 4
t 0
1

ln 4
+
f

(x) 0 +
f(x)
Vi f
_
1

ln 4
_
=

ln 4
ln 2
+
2

ln 4
2 =
ln 4+2 ln 22 ln 2

ln 4
ln 2.

ln 4
f

_
1

ln 4
_
=
2 ln 4ln 4
ln 2.

ln 4
=
ln 4(2

ln 4)
ln 2.

ln 4
> 0
f

(t) > 0 , t > 0 f(t) ng bin


Theo bi ra f

x + 2
_
= f 2 +
1
x
_

x + 2 = 2 +
1
x
()
t t =

x + 2 (t > 0) x = t
2
2. Phng trnh (*) tr thnh
t = 2 +
1
t
2
2
t
3
2t
2
2t + 3 = 0

_
t = 1
t =
1+

13
2
t =
1

13
2
(loi)
Vy nghim ca phng trnh:
_
x = 1
x =
3+

13
2
142
Bi ton 3. Gii phng trnh: x
2
+x 1 = xe
x
2
1
+ (x
2
1)e
x
()
Li gii.
() (x
2
1)(e
x
1) = x(e
x
2
1
1) (1)
Nu
_
x = 1
x = 0
th V T = 0 = V P
_
x = 1
x = 0
l nghim ca phng trnh.
Khi
_
x = 0
x = 1
th (1)
e
x
2
1
1
x
2
1
=
e
x
1
x
(2)
Xt hm s f(x) =
e
t
1
t
D = R\ {0}
f

(x) =
te
t
e
t
+1
t
2
Xt g(t) = te
t
e
t
+ 1
g

(t) = e
t
+te
t
e
t
= te
t
t 0 +
g

(t) 0 +
g(t)
+
0
+
g

(t) > 0, t = 0 f(t) tng trong tng khong xc nh


x 0 +
f

(x) + +
f(x) 0
1
1
+
Do (2) f(x
2
1) = f(x) x
2
x 1 = 0 x =
1

5
2
Bi ton 4. (Bnh nh) Gii phng trnh:
2x
3
+ 10x
2
17x + 8 = 2x
2
3

5x x
3
Li gii. D thy x = 0 khng phi l nghim ca phng trnh. Chia hai v ca phng trnh
cho x
3
, ta c
2 +
10
x

17
x
2
+
8
x
3
= 2
3
_
5
x
2
1 8t
3
17t
2
+ 10t 2 = 2
3

5t
2
1 (1)
143
vi t = 1/x (t = 0)
Ta bin i phng trnh (1) tip tc nh sau
(1) (2t 1)
3
+ 2(2t 1) = 5t
2
1 + 2
3

5t
2
1
Xt hm s f(x) = x
3
+ 2x th f

= 3x
2
+ 2 > 0 nn f l mt hm s tng trn R. Phng
trnh cui cng c th vit li thnh
f(2t 1) = f(
3

5t
2
1)
Do f l hm s tng nn phng trnh ny tng ng vi
2t 1 =
3

5t
2
1 8t
3
12t
2
+ 6t 1 = 5t
2
1
Gii ra ta c t = 0 (loi), t =
17

97
16
. Tng ng ta tm c x =
17

97
12
.
Bi ton 5. (Vnh Phc xut Olympic ng bng Bc b 2010) Gii phng trnh
(6
x
3
x
)(19
x
5
x
)(10
x
7
x
) + (15
x
8
x
)(9
x
4
x
)(5
x
2
x
) = 231
x
Li gii. Nhn xt1: Vi a > b > c > 1 th
_
a
x
b
x
nu x 0
a
x
b
x
nu x 0
Nhn xt 2: Hm s f(x) = a
x
b
x
xc nh ng bin v lin tc trn tp D = [0; +) do
f

(x) = a
x
ln a b
x
ln b > 0 a 0 vi a > b > 0 cho trc.
Nhn xt 3: Tch hai hm s ng bin, nhn gi tr dng trn tp D l hm ng bin tng
hai hm s ng bin trn D l hm s ng bin.
p dng
(+) nu x 0: V T = (6
x
3
x
)(19
x
5
x
)(10
x
7
x
) + (15
x
8
x
)(9
x
4
x
)(5
x
2
x
) 0;
V P = 231
x
> 0. Suy ra phng trnh khng c nghim dng.
(+) Vi x > 0 chia 2 v phng trnh cho 231
x
= (3.7.11)
x
ta c
(2
x
1)
__
19
11
_
x

_
5
11
_
x
___
10
7
_
x
1
_
+
__
15
11
_
x

_
8
11
_
x
___
9
7
_
x

_
4
7
_
x
___
5
3
_
x

_
2
3
_
x
_
= 1
T nhn xt (2) v nhn xt 3 hm s.
g(x) = (2
x
1)
__
19
11
_
x

_
5
11
_
x
___
10
7
_
x
1
_
+
__
15
11
_
x

_
8
11
_
x
___
9
7
_
x

_
4
7
_
x
___
5
3
_
x

_
2
3
_
x
_
l hm s ng bin trn D = (0; +) v g(1) = 1
phng trnh cho g(x) = g(1) x = 1 l nghim duy nht ca phng trnh.
Bi ton 6. ( Chn HSG Khnh Ha 2002) Gii h phng trnh :
_
(1 + 4
2xy
) 5
12x+y
= 1 + 2
2xy+1
(1)
y
3
+ 4x + 1 + ln(y
2
+ 2x) = 0 (2)
144
Li gii. t t = 2x y. Khi h (I) :
_
(1 + 4
2xy
) 5
12x+y
= 1 + 2
2xy+1
(1)
y
3
+ 4x + 1 + ln(y
2
+ 2x) = 0 (2)
Ta c (1) (1 + 4
t
) 5
t+1
= 1 + 2
t+1
5
_
_
1
5
_
t
+
4
5
_
t
_
= 1 + 2.2
t
t f(t) = 5
_
_
1
5
_
t
+
4
5
_
t
_
; g(t) = 1 + 2.2
t
Ta c: f(t) l hm s gim, g(t) l hm s tng v f(1) = g(1).
Do (3) t = 1 2x y = 1
Vy h (I)
_
2x = y + 1
y
3
+ 2y + 3 + ln(y
2
+y + 1) = 0
t h(y) = y
3
+ 2y + 3 + ln(y2 + y + 1)
Ta c h

(y) = 3y
2
+ 2 +
2y+1
y
2
+y+1
= 3y
2
+
2y
2
+4y+3
y
2
+y+1
= 3y
2
+
2(y+1)
2
+1
y
2
+y+1
> 0
h

(y) > 0 h(y) l hm s tng v h(1) = 0. Vy (I)


_
2x = y + 1
y = 1

_
x = 0
y = 1
Bi ton 7. (ng Nai) Gii h phng trnh
_
x
5
+xy
4
= y
10
+ y
6
(1)

4x + 5 +
_
y
2
+ 8 = 6 (2)
Li gii. . Nu y = 0 th t phng trnh (1) suy ra x = 0, v phng trnh (2) khng c
tho mn. Vy y = 0. Chia hai v ca phng trnh (1) cho y
5
, ta c
_
x
y
_
5
+
x
y
= y
5
+y (3).
Xt hm s f(x) = x
5
+ x, ta c f

(x) = 5x
4
+ 1 > 0, suy ra f l hm s tng trn R. Phng
trnh (3) c th vit li thnh f(x/y) = f(y) v do f l hm tng nn tng ng vi x/y = y,
suy ra x = y
2
. Thay vo phng trnh (2), ta c

4x + 5 +

x + 8 = 6 (4).). Gii ra ta
c x = 1 l nghim duy nht ca phng trnh (4). T h ban u c nghim duy nht
(x; y) = (1; 1) v (1; 1).
Bi ton 8. (Nguyn Tri- Hi Dng xut Olympic ng bng Bc b 2010) Gii h
phng trnh:
_
(x +y)
4
+ 3 = 4 (x + y)
x
4
y
4
64
+
9(x
2
y
2
)
32
+
7(xy)
8
+ 3 ln
_
x3
y3
_
= 0
Li gii. Theo bt Cauchy ta c (x +y)
4
+1+1+1 4
4
_
(x + y)
4
.1.1.1 = 4 |x + y| 4 (x +y)
Du bng xy ra x + y = 1()
T kt hp vi k
x3
y3
> 0 2 < x, y < 3
Pt th hai ca h
x
4
64
+
9x
2
32
+
7x
8
+ 3 ln (3 x) =
y
4
64
+
9y
2
32
+
7y
8
+ 3 ln (3 y)
Xt hm s f(x) =
x
4
64
+
9x
2
32
+
7x
8
+ 3 ln (3 x) (vi x < 3)
f

(x) =
x
3
16
+
9x
16
+
7
8
+
3
x3
=
(x
3
+9x+14)(x3)+48
16(x3)
=
x
4
3x
3
+9x
2
13x+6
16(x3)
=
(x1)
2
(x
2
x+6)
16(x3)
0 Suy hm s nghch bin trn (2; 3), vy f(x) =
f(y) x = y ( **).
T (*), (**) c x = y =
1
2
145
Bi ton 9. (THPT Chuyn Bin Ha - H Nam xut Olympic ng bng Bc b 2010)
Gii h phng trnh:
_
_
_
(x y) (x
2
+ xy +y
2
2) = 6 ln
_
y+

y
2
+9
x+

x
2
+9
_
x
5
y 3xy 1 = 0
Li gii. T (x y) (x
2
+xy +y
2
2) = 6 ln
_
y+

y
2
+9
x+

x
2
+9
_
x
3
2x + 6 ln
_
x +

x
2
+ 9
_
= y
3
2y + 6 ln
_
y +
_
y
2
+ 9
_
(1)
Xt f (t) = t
3
2t + 6 ln t +

t
2
+ 9
_
t R
f

(t) = 3t
2
2 +
6

t
2
+ 9
= 3
_
t
2
+
2

t
2
+ 9

2
3
_
Ta c
t
2
+
2

t
2
+9

2
3
= t
2
+ 9 +
2

t
2
+9

29
3
=
t
2
+9
27
+
1

t
2
+9
+
1

t
2
+9
+
26
27
(t
2
+ 9)
29
3
1 +
26
27
(t
2
+ 9)
29
3
1 +
26
3
=
29
3

29
3
= 0
Suy ra f

(t) 0 t hm s ng bin v lin tc trn R. M (1) f (x) = f (y) x = y.


Thay vo phng trnh cn li ca h ta c x
6
3x
2
1 = 0 (2) . t x
2
= u(u 0) suy
ra u
3
3u = 1 (3)
Xt g(u) = u
3
3u 1 vi u 0.
g

(u) = 3u
2
3, c g

(u) = 0 u = 1
Cn c vo BBT phng trnh (3) c nghim duy nht thuc (0; 2).
t u = 2 cos vi 0;

2
_
(3) tr thnh cos3=
1
2
=

9
x =
_
2 cos

9
Vy h c nghim
_
2 cos

9
;
_
2 cos

9
_
;
_
2 cos

9
;
_
2 cos

9
_
Bi ton 10. ( do Hi phng xut Olympic ng bng Bc b 2010)
Gii h phng trnh:
_

_

_
x = cos
_

3

3
y
_
y = cos
_

3

3
z
_
z = cos
_

3

3
x
_
Li gii. Xt hm s
f (x) = cos
_

3

3
x
_
f

(x) =

3

3
sin
_

3

3
x
_
|f

(x)| < 1
T |x y| = |f (y) f (z)| = |f

()| |(y z)| |y z|


146
Tng t ta c |x y| |y z| |z x| |y x| |x y| = |y z| = |z x|
Gi s x = max {x, y, z} x = y = z
T c f (x) = x. Xt hm s:
g (x) = x cos
_

3

3
x
_
g

(x) = 1

3

3
sin
_

3

3
x
_
> 0
Vy g(x) ng bin m g
_

3
2
_
= 0 nn h phng trnh cho c nghim duy nht x = y =
z =

3
2
.
Bi ton 11. (Chuyn Hng Yn xut Olympic ng bng Bc b 2010) Tm nghim ca
h phng trnh vi x 0
_
2x 2y +

2x +y + 2xy + 1 = 1
3

3y + 1 = 8x
3
2y 1
Li gii.
_
2x 2y +

2x +y + 2xy + 1 = 1 (1)
3

3y + 1 = 8x
3
2y 1 (2)
(1) (2x + 1) 2 (y + 1) +
_
(2x + 1) (y + 1) = 0
K: (2x + 1)(y + 1) 0. M x 0 nn
_
2x + 1 > 0
y + 1 0
(1)
_

2x + 1
_
y + 1
__

2x + 1 + 2
_
y + 1
_
= 0

2x + 1
_
y + 1 = 0
y = 2x
Thay vo (2):
3

6x + 1 = 8x
3
4x 1
(6x + 1) +
3

6x + 1 = (2x)
3
+ 2x
Hm s f(t) = t
3
+t ng bin trn R
(2)
3

6x + 1 = 2x
4x
3
3x =
1
2
Nhn xt: x > 1 khng l nghim ca phng trnh Xt 0 x 1. t x = cos vi 0

2
cos 3 =
1
2

_
=

9
+k
2
3
=

9
+k
2
3
(k Z)
Do 0

2
=

9
Vy h c nghim cos

9
; 2 cos

9
_
147
Bi ton 12. Gii h phng trnh sau:
_
_
_

x
2
2x + 6 . log
3
(6 y) = x
_
y
2
2y + 6 . log
3
(6 z) = y

z
2
2z + 6 . log
3
(6 x) = z.
(I)
Li gii. iu kin xc nh: x, y, z < 6.
Vit li h cho di dng tng ng
_

_
x

x
2
2x+6
= log
3
(6 y)
y

y
2
2y+6
= log
3
(6 z)
z

z
2
2z+6
= log
3
(6 x)
Xt cc hm s f(u) =
u

u
2
2u+6
v g(u) = log 3(6 u) trn (; 6).
Ta c: f

(u) =
6u
(u
2
2u+6)

u
2
2u + 6 > 0, vi u (; 6) v g

(u) =
1
(6u
) ln 3 < 0, vi
u (; 6).
Suy ra f(u) ng bin v g(u) nghch bin trn (; 6). (1)
Gi s (x, y, z) l mt nghim ca h (I). Do tnh i xng vng quanh ca h (I) i vi
x, y, z, khng mt tng qut c th gi s x = max x, y, z. Khi , t (I) v (1) ta c
g(y) = f(x) = maxf(x), f(y), f(z) = maxg(x), g(y), g(z)
y = minx, y, z g(z) = f(y) = minf(x), f(y), f(z) = ming(x), g(y), g(z)
z = maxx, y, z z = x f(z) = f(x) g(x) = g(y) x = y x = y = z.
Nh vy, h (I) ch c th c nghim dng x = y = z.
Vi x = y = z, t h (I) ta c h
_
_
_
f (x) = g (x)
f (y) = g (y)
f (z) = g (z)
(II)
Xt phng trnh f(u) = g(u). (2)
Do (1) nn phng trnh (2) ch c ti a mt nghim trn (; 6). Hn na, bng cch th
trc tip d thy u = 3 l nghim ca (2). Suy ra pt (2) c nghim duy nht u = 3. Do h
(II) c duy nht nghim x = y = z = 3. Vy, h cho c duy nht nghim x = y = z = 3.
Bi ton 13. Cho h
_
m(x
2
+
3

x
4
+
3

x
2
+ 1) = yx
m(
3

x
8
+ x
2
+
3

x
2
+ 1) + (m1)
3

x
4
= 2y
3

x
4
Tm m h c nghim
148
Li gii. Nu m = 0 th
_
yx = 0
2y
3

x
4
=
3

x
4
(x; y) = (0, c)c R
m = 0 t t =
3

x ta c h
_
m(t
6
+t
4
+ t
2
+ 1) = yt
3
m(t
8
+t
6
+ t
4
+t
2
+ 1) = (2y + 1)t
4
Nhn xt t=0 khng phi l nghim ca h t u = t +
1
t
(|u| 2)
_
m(t
3
+t +
1
t
+
1
t
3
) = y
m(t
4
+t
2
+ 1 +
1
t
2
+
1
t
4
) = 2y + 1

_
m(u
3
2u) = y
m(u
4
3u
2
+ 1) = 2y + 1

_
m(u
3
2u) = y
m(u
4
3u
2
+ 1) = 2m(u
3
2u) + 1()
Phng trnh (*) m(u
4
2u
3
3u
2
+ 4u + 1) = 1
u
4
2u
3
3u
2
+ 4u + 1 =
1
m
Xt hm s
f(u) = u
4
2u
3
3u
2
+ 4u + 1 (|u| 2)
f

(u) = 4u
3
6u
2
6u + 4
Vy h c nghim
1
m
3
_
m
1
3
m > 0
Bi ton 14. Gii h
_
e
x
= ey x + 1
e
x
= ex y + 1
(I)
Li gii.
(I)
_
e
x
= ey x + 1
e
x
e
y
= e(y x) + (y x)

_
e
x
= ey x + 1 (1)
e
x
e
y
= (y x)(e + 1) (2)
Nu x > y th e
x
e
y
> 0 > (y x)(e + 1)
Nu x < y th e
x
e
y
< 0 < (y x)(e + 1)
Nn t (2) x = y, thay vo (1):
e
x
ex + x 1 = 0
Xt f(x) = e
x
ex +x D = R
f

(x) = e
x
e + 1
f

(x) = 0 e
x
= e 1 x = ln(e 1)
x ln(e 1) +
f

(x) 0 +
f(x) CT
149
Do phng trnh f(x) = 0 c nhiu nht hai nghim, ngoi ra f(0) = 0 , f(1) = 0.
Vy h c ng hai nghim
_
x = 0
y = 0
_
x = 1
y = 1
2.3 Dng 3: Bt phng trnh, h bt phng trnh
Bi ton 1. ( Phan Huy Khi) Tm m h bt phng trnh sau c nghim dng
_
_
_
x(3 y
2
) > m
y(3 z
2
) > m
z(3 x
2
) > m
Li gii. Xt hm s f(t) = 3t t
3
vi t 0 f

(t) = 3 3t
2
x 0 1 +
f

(x) + 0 0
f(x) 0
2

T suy ra nu m < 2 th bpt: t(3 t


2
) > m c nghim t > 0. Do h cho c nghim
x = y = z = t
0
vi t
0
l nghim ty ca bpt : 3t t
3
> m.
Xt khi m 2 , gi s h c nghim l (a; b; c) th :
a > 0; b > 0; c > 0 v : a(3 b
2
) > m; b(3 c
2
) > m; c(3 a
2
) > m (1)
suy ra : 3 a
2
> 0; 3 b
2
> 0; 3 c
2
> 0. Nn: 0 < a; b; c <

3
Nhn v theo v cc bt ng thc dng cng chiu ca (1) Ta c : a(3a
2
)b(3b
2
)c(3c
2
) >
m
3
(2)
Mt khc v: 0 < a; b; c <

3 , nn : 0 < f(a) 2; 0 < f(b) 2; 0 < f(c) 2


suy ra : 0 < f(a).f(b).f(c) 2
3
m
3
( 3)
T (2) v (3) suy ra v l. Vy h c nghim khi v ch khi m < 2.
Bi ton 2. Cho , , [0;

2
]. Chng minh rng: 2
cos sin
+ 2
cos cos
+ 2
sin
4 .
Li gii. Xt hm s f(x) = 2
x
x 1 x [0; 1]
Ta c: f

(x) = 2
x
ln 2 1 l hm lin tc trn R.
f

(x) = 2
x
(ln 2)
2
> 0 x R nn f

(x) l hm tng trn R. Suy ra tn ti duy nht x


0

f

(x
0
) = 0. Ta c bng bin thin nh sau:
x 0 x
0
1
f

(x) + 0 0
f(x) 0
C
0
150
f(x) 0, x [0; 1] hay 2
x
x + 1, x [0; 1]
rng khi , , [0;

2
]. th cos sin , cos cos , sin [0; 1]
p dng b suy ra
2
cos sin
+ 2
cos cos
+ 2
sin
cos sin + cos cos + sin + 3
cos
2
sin
2
+ cos
2
cos
2
+ 3 = 4
Du bng xy ra chng hn: cos = 0, sin = 1 tc l =

2
.
Bi ton 3. ( chn HSG Quc Hc Hu -2005)
Gii bt phng trnh: e
x
+ (x
3
x) ln(x
2
+ 1) 3
3

x
()
Li gii. Biu thc ln(x2 + 1) lun xc nh .
x = 0; x = 1; x = 1 l cc gi tr tha mn bt phng trnh .
Ta c : x
3
x = (x
3

x)(x
2
+x
3

x +
3

x
2
).
Khi x / {0; 1; 1} th x =
3

x. Theo nh l Lagrange th tn ti s c nm gia x v


3

x sao
cho: e
x
e
3

x
= (x
3

x)e
c
Vy
() (x
3

x)[e
c
+ (x
2
+x
3

x +
3

x
2
)] ln(x
2
+ 1) 0
x
3

x 0 (V [e
c
+ (x
2
+x
3

x +
3

x
2
)] ln(x
2
+ 1) > 0)
x
3
x 0
Nghim ca bt phng trnh cho l :x (; 1] [0; 1]
Bi ton 4. Cho 1 = a > 0, chng minh rng :
ln a
a1

1+
3

a
a+
3

a
Li gii.
ln a
a1

1+
3

a
a+
3

a
(1) vi 1 = a > 0
Trng hp 1: a > 1
(1) (a +
3

a) ln a (1 +
3

a)(a 1) (2) t x =
3

a x > 1
(2) 3(x
3
+ x) ln x (1 + x)(x
3
1) x > 1
x
4
+ x
3
x 1 3(x
3
+x) ln x 0 (3) x > 1
t f(x) = x
4
+x
3
x 1 3(x
3
+ x) ln x x [1; +)
Ta c f

(x) = 4x
3
+ 3x
2
1 3[(3x
2
+ 1) ln x + (x
3
+ x)
1
x
] = 4x
3
4 3(3x
2
+ 1) ln x
f

(x) = 3(4x
2
3x 6x ln x
1
x
); f
(3)
(x) = 3(8x +
1
x
2
6 ln x 9)
f
(4)
(x) = 3(8
6
x

2
x
3
) =
6(4x
3
3x1)
x
3
=
6(x1)(4x
2
+4x+1)
x
3
0, x 1
Suy ra f
(3)
(x) ng bin trn [1; +)
f
(3)
(x) f
(3)
(1) = 0 . . . tng t f

(x) 0 x 1
f(x) > f(1) = 0 x > 1 suy ra (3) ng.
Trng hp 2: 0 < a < 1, t a =
1
a
1
, a
1
> 1 quay v trng hp 1.
Bi ton 5. Cho n N

vi n 7 ; 2 k < n. Chng minh k


n
> 2.n
k
.
151
Li gii.
k
n
> 2n
k
nln k > ln 2 +k ln n
nln k k ln n > ln 2()
Xt f(x) = nln x x ln n Trn [1; +)
f

(x) =
n
x
ln n; f

(x) > 0 x <


n
ln n
_
x 1
n
ln n
+
f

(x) + 0
f(x)
Do k 2 k n1 nn khi f(x) xt [2; n1] th gi tr nh nht t ti f(2) hay f(n1).
Ta chng minh
_
f(2) > ln 2
f(n 1) > ln 2
f(2) > ln 2 nln 2 2 ln n > ln 2 (n 1) ln 2 > 2 ln n
2
n1
> n
2
(bng quy np vi n 7 lun ng)
f(n 1) > ln 2 nln(n 1) (n 1) ln n > ln 2
nln(n 1) > (n 1) ln n + ln 2
(n 1)
n
> 2.n
n1
(2)
t n 1 = t (t 6) Th (2) tr thnh:
t
t+1
2(t + 1)
t
(3)
t 2
t+1
t
_
t
= 2 1 +
1
t
_
t
Ta c: 1 +
1
t
_
t
< 3 , t.
Do 1 +
1
t
_
t
< 6 t vi t 6 Vy (3) ng (2)ng (1) c chng minh.
3 Bi tp ngh
Bi ton 1. ( VMO 2007) Cho s thc a > 2. t f
n
(x) = a
1
0x
n+10
+ x
n
+ + x + 1,
(n = 1, 2, ...). Chng minh rng vi mi n phng trnh f
n
(x) = a c ng mt nghim . Chng
minh dy s (x
n
) c gii hn hu hn khi n .
Bi ton 2. (H Tnh 2009) Cho dy {x
n
} bit x
1
=
1
2
, x
n+1
=
x
2
n
1
2
vi mi n = 1, 2, 3, . . .
Tm gii hn ca dy {x
n
} khi n dn ti v cng.
Bi ton 3. (B Ra Vng Tu 2009) Cho dy s xc nh bi x
1
= 1, x
n+1
=
1
2(x
2
n
+1)
2008.
Chng minh rng {x
n
} c gii hn hu hn khi n dn n v cng.
Bi ton 4. (PTNK 1999) Cho a > 1 v dy s {x
n
} c xc nh nh sau: x
1
= a, x
n+1
= a
xn
vi mi n 1. Hy xc nh tt c cc gi tr ca a dy {x
n
} hi t.
152
Bi ton 5. (Bc Ninh 2009) Cho dy s {x
n
} xc nh bi
_
x
1
=

2
x
n+1
=
+cos 2xn
4
n N

Bi ton 6. Chng minh rng vi mi s nguyn dng n cho trc, phng trnh x
2n+1
= x+1
c ng mt nghim thc. Gi nghim l x
n
. Tm lim
n+
x
n
.
Bi ton 7. Cho phng trnh: x
13
x
6
+ 3x
4
3x
2
+ 1 = 0.
1) Chng minh rng phng trnh c ng mt nghim thc.
2) t x
1
= 1 v x
n+1
=
_
x
7/3
n
+ 1
_
3/13
vi mi s nguyn dng n. Chng minh rng
dy s {x
n
} c gii hn v khi t x
0
= lim
n+
x
n
th x
0
l nghim ni trn.
Bi ton 8. Cho tam gic ABC khng c gc t tho mn h thc:
1
3
(cos3A + cos3B)
1
2
(cos2A + cos2B) + cosA +cosB =
5
6
.
Hy tnh cc gc ca tam gic .
Bi ton 9. Gii phng trnh: 2x
2
6x + 2 = log
2
_
2x+1
(x_1)
2
_
Bi ton 10. Gii phng trnh: 2
2
x
+ 3
2
x
= 2
x
+ 3
x+1
+x + 1
Bi ton 11. Gii phng trnh 2
x

x
2
1 = 0
Bi ton 12. Gii phng trnh 3
x

2x
3
1 = 0
Bi ton 13. Gii phng trnh log
2

2+

3
(x
2
2x 2) = log
2+

3
(x
2
2x 3)
Bi ton 14. Tm a phng trnh c nghim
a
_
x 3
x
+

x 1 +

x 3

x + 4 = 0
Bi ton 15. Gii h
_
(4x
2
+ 1)x + (y 3)

5 2y = 0
4x
2
+y
2
+ 2

3 4x = 7
Bi ton 16. Gii h
_
_
_
2x + ln(x
2
+ x + 1) = y
2y + ln(y
2
+y + 1) = z
2z + ln(z
2
+ z + 1) = x
Bi ton 17. Gii h
_
_
_
x
2
+ 3x + 2 + ln(x
2
+x + 1) = y
y
2
+ 3y + 2 + ln(y
2
+y + 1) = z
z
2
+ 3z + 2 + ln(z
2
+z + 1) = x
153
Ti liu tham kho
[1] Cc thi HSGQG (VMO).
[2] Cc thi Ton quc t v cc nc.
[3] Phan Huy Khi, Dy s v gii hn, Nh xut bn H Ni, (1996).
[4] GS-TSKH Nguyn Vn Mu, Nguyn Thy Thanh Dy s v gii hn, Nh xut bn Gio
dc, (2002).
[5] GS-TSKH Nguyn Vn Mu, Phng php gii phng trnh v bt phng trnh, Nh xut
bn Gio dc, (1996).
[6] TS Trn Nam Dng, Tp cc bi dy s trong thi Olympic 30/4 ca Cc Tnh pha
Nam.
[7] Cc thi Olympic ca Cc Tnh Duyn Hi Bc b
[8] Bo Ton hc Tui tr.
[9] Ngun Internet.
154
NG DNG MT S NH L C BN CA GII
TCH
Hunh Tn Chu, Trng THPT Chuyn Lng Vn Chnh - Ph Yn
Trong chng trnh ton hc bc THPT cc nh l c bn ca gii tch v lp hm s lin tc
v kh vi c nhiu ng dng kh th v, nhng cha c cp nhiu trong cc sch tham
kho cho hc sinh chuyn ton. Trong mt s trng hp dng cc nh l trn gii quyt v
h tr cho cc bi ton v phng trnh, chng minh bt ng thc, cc bi ton lin quan n
dy s, . . . t ra kh hiu qu v th v so vi cc cch gii khc. Cn nhiu ng dng khc
ca cc nh l trn, mong c trao i t pha cc Thy c gio v cc em hc sinh yu thch
mn ton.
1 Mt s nh l v tnh lin tc ca hm s
nh l 1. Nu hm s f lin tc trn on [a; b] th hm s f b chn trn on [a; b], tc l
tn ti s M > 0 sao cho : |f (x)| M, x [a, b] .
nh l 2. Nu hm s f lin tc trn on [a; b] th f t c gi tr ln nht v t c
gi tr nh nht trn on , tc l tn ti cc im x
1
, x
2
[a; b] sao cho :
f (x
1
) f (x) f (x
2
) , x [a, b]
nh l 3. (nh l v gi tr trung gian ca hm s lin tc) Gi s hm s f lin tc trn
on [a; b] v f (a) = A, f (b) = B. Khi nu C l mt s bt k nm gia A v B th c t
nht mt im c (a; b) f(c) = C.
H qu 1. Nu hm s f lin tc trn on [a; b] th f nhn mi gi tr trung gian gia gi tr
nh nht m v gi tr ln nht M ca n trn on .
nh l 4. (NH L BOLZANO CAUCHY ) Nu hm s f lin tc trn on [a; b] v
f(a).f(b) < 0 th c t nht mt im c (a; b) f(c) = 0.
2 Mt s nh l c bn ca php tnh vi phn
nh l 5. (nh l Lagrange ) Nu hm s f(x) lin tc trn on [a; b], kh vi trn khong
(a; b). Khi tn ti t nht mt im c (a; b) sao cho f (b) f (a) = (b a) f

(c) .
H qu 2. Nu hm s f(x) lin tc trn on [a; b] v f

(x) = 0x [a; b] th f(x) l hng


s trn [a; b].
nh l 6. (nh l Rolle) Nu hm s f(x) lin tc trn on [a; b], kh vi trn khong (a; b)
v f(a) = f(b) th tn ti t nht mt im c (a; b) sao cho f

(c) = 0.
nh l 7. (nh l Cauchy) Cho v l hai hm s lin tc trn on [ab], kh vi trn
khong (a; b). Khi tn ti c (a; b) [ (b) (a)]

(c) = [(b) (a)]

(c)
155
3 Mt s bi ton p dng
3.1 Cc bi ton v phng trnh
Bi ton 1. Cho hm s f(x) lin tc trn on [a; b] v x
1
, x
2
, ..., x
n
l n gi tr bt k thuc
[a; b]. Chng minh rng tn ti s [a; b] sao cho : f() =
1
n
[f (x
1
) + f (x
2
) + ... +f (x
n
)].
Li gii. V hm s f(x) lin tc trn on [a; b] nn tn ti GTNN l m v GTLN l M trn
[a; b].
m
1
n
[f (x
1
) + f (x
2
) + ... +f (x
n
)] M
Do theo nh l v gi tr trung gian th tn ti s [a; b] sao cho :
f() =
1
n
[f (x
1
) + f (x
2
) + ... + f (x
n
)]
Bi ton 2. Cho hm s f(x) lin tc trn on [0; 1] tho mn iu kin f(0) = f(1) Chng
minh rng phng trnh : f (x) = f x +
1
2012
_
c nghim thuc on [0; 1].
Li gii. Xt hm s g (x) = f x +
1
2012
_
f (x)
Hm s ny xc nh v lin tc trn
_
0;
2011
2012

Ta c g (0) = f
1
2012
_
f (0)
g
_
1
2012
_
= f
_
2
2012
_
f
_
1
2012
_
; ; g
_
2011
2012
_
= f (1) f
_
2011
2012
_
Vy g (0) + g
1
2012
_
+ ... +g
2011
2012
_
= f (1) f (0) = 0
Suy ra tn ti i, j N v i, j 2011 sao cho g
i
2012
_
0 v g
j
2012
_
0
V g l hm s lin tc trn
_
0;
2011
2012

, nn theo nh l Bolzano Cauchy th phng trnh


g (x) = f x +
1
2012
_
f (x) = 0 c nghim trn
_
0;
2011
2012

do phng trnh : f (x) = f x +


1
2012
_
c nghim thuc on [0; 1].
Bi ton 3. Tm mi cp s thc (b; c) sao cho vi bt k s thc a th phng trnh a cos 2x +
b cos x + c = 0 c nghim thc thuc khong 0;

2
_
Li gii. t f (x) = a cos 2x + b cos x + c = 2acos
2
x +b cos x + ca
iu kin cn : Gi s (b, c) l cp s thc tha mn yu cu bi ton.
- Nu b = 0, ta ly a = 0, suy ra c = 0.
- Nu b = 0, ly a =
b
2

2
f (x) = 0
1

2
_
cos x
1

2
_
2
+
1

2
+
c
b
= 0

2
+
c
b
0
c
b

1

2
Ly a =
b
2
f (x) = 0 cos x (1 cos x) +
1
2
+
c
b
= 0

1
2
+
c
b
= cos x (1 cos x) < 0, x 0;

2
_

c
b
<
1
2
Nh vy ta c : hoc b = c = 0, hoc b = 0 v
1

2

c
b
<
1
2
(1)
iu kin :
156
- Nu b = c = 0 ta c : f

4
_
= 0, a R
- Nu b = 0 v
1

2

c
b
<
1
2
, ta c bf

4
_
= b
2
_
1

2
+
c
b
_
0 (2)
Vi x 0;

2
_
ta c :
b
_
f (x) + f

2
x
_
= 2b
2
_
c
b
+
1
2
(sinx + cos x)

2b
2 c
b
+
1
2
_
khi x 0, do tn x
0
0;

2
_
m bf (x
0
) < 0 (3) .
T (2) v (3), theo nh l Bolzano Cauchy i vi hm s lin tc ta suy ra tn ti nghim
x 0;

2
_
m f(x) = 0.
Vy cc cp s (b, c) tha mn (1) l cc cp s cn tm.
Bi ton 4. (VMO 2006) Cho hm s f (x) = x+
_
(x + a) (x + b), trong a, b l hai s
thc dng cho trc. Chng minh rng vi mi s thc s thuc khong (0; 1) u tn ti duy
nht mt s thc dng sao cho : f () =
a
s
+b
s
2
_1
s
.
Li gii. Ta c f(x) l hm s lin tc trn [0; +) . Ta s chng minh cc khng nh sau :
1. f(x) tng thc s trn [0; +)
Vi a = b v 0x ta c : f

(x) = 1 +
2x+a+b
2

(x+a)(x+b)
=
(

x+a

x+b)
2
2

(x+a)(x+b)
> 0
Do f(x) l hm tng trn [0; +) .
2. f (0) =

ab; lim
x+
f (x) =
a+b
2
Mt khc lim
x+
f (x) = lim
x+
x
2
+(x+a)(x+b)
x+

(x+a)(x+b)
= lim
x+
a+b+
ab
x
1+

(
a
x
+1)(
b
x
+1)
=
a+b
2
3. Vi mi 0 < s < 1, ta c :

ab <
a
s
+b
s
2
_1
s

a+b
2
BT bn tri l ng theo BT AM GM.
t m =
a
s
+b
s
2
_1
s
, x =
a
m
, y =
b
m
th x
5
+y
5
= 1.
Theo BT Bernoulli ta c :
x = (1 +x
s
1)
1
s
1 +
x
s
1
s
; y = (1 +y
s
1)
1
s
1 +
y
s
1
s
;
(khng ng thi xy ra ng thc).
Cng theo tng v ta c x + y > 2, suy ra BT bn phi ca 3.
T cc kt qu trn suy ra ! (0; 1) sao cho :
f () =
_
a
s
+b
s
2
_1
s
Bi ton 5. (OLYMPIC SINH VIN VIT NAM 1994) Cho n l s nguyn dng, a
k
, b
k

R(k = 1, 2, ..., n) . Chng minh rng phng trnh x +
n

k=1
(a
k
sin kx +b
k
cos kx) = 0 c nghim
trong khong (; ) .
Li gii. Xt hm F (x) =
x
2
2
+
n

k=1

a
k
k
cos kx +
b
k
k
sin kx
_
, x mathbbR.
Nhn xt F(x) l hm kh vi trn R.
157
Xt F

(x) = x +
n

k=1
(a
k
sin kx +b
k
cos kx) = 0
Khi F () =

2
2
+
n

k=1
_

a
k
k
(1)
k
_
; F () =

2
2
+
n

k=1
_

a
k
k
(1)
k
_
Do F () = F () . S dng nh l Rolle trong khong (; ) , ta nhn c iu phi chng
minh.
Bi ton 6. Cho a
0
, a
1
, ..., a
n
l cc s thc v tha mn iu kin
a
0
+
a
1
2
+
a
2
3
+... +
a
n
n + 1
= a
0
+a
1
+
a
2
.2
2
3
+
a
3
.2
3
4
+ ... +
a
n
.2
n
n + 1
= 0
Chng minh rng phng trnh : a
1
+ 2a
2
x + 3a
3
x
2
+ ... + na
n
x
n1
= 0 c t nht mt nghim
thuc khong (0; 2)
Li gii. Xt hm s f (x) = a
0
x +
1
2
a
1
x
2
+
1
3
a
2
x
3
+... +
1
n+1
a
n
x
n+1
Ta c f (1) = a
0
+
a
1
2
+
a
2
3
+... +
an
n+1
f (2) = 2
_
a
0
+a
1
+
a
2
.2
2
3
+
a
3
.2
3
4
+... +
an.2
n
n+1
_
Theo gi thit ta c f (1) = f (2) = 0. Hin nhin f(0) Theo nh l Rolle, tn ti cc s
c
1
, c
2
: 0 < c
1
< 1 < c
2
< 2 sao cho :
f

(c
1
) = f

(c
2
) = 0
p dng nh l Rolle vi hm s f

(x) trn [c
1
; c
2
], ta thy tn ti x
0
(c
1
; c
2
) sao cho :
f

(x
0
) = 0
Do x
0
(c
1
; c
2
) (0; 2) l nghim ca phng trnh f

(x) = a
1
+ 2a
2
x + 3a
3
x
2
+ ... +
na
n
x
n1
= 0. (pcm)
Bi ton 7. Gii phng trnh : 3
x
= 1 +x + log
3
(1 + 2x) (1)
Li gii. K: 1 + 2x > 0 x >
1
2
(1) x + 3
x
= (1 + 2x) + log
3
(1 + 2x)
3
x
+ log
3
3
x
= (1 + 2x) + log
3
(1 + 2x(2)
t (t) = t + log
3
t, t > 0
] Ta c (t) l hm ng bin trn (0; +)
(2) (3
x
) = (1 + 2x)
3
x
= 1 + 2x 3
x
2x 1 = 0
Xt hm s f(x) = 3
x
2x1, x >
1
2
f

(x) = 3
x
ln 32
Ta c f

(x) l hm ng bin, nn theo nh l Rolle phng trnh f(x) = 0 c khng qu 2


nghim. R rng f(0) = f(1) = 0.
Vy phng trnh cho ch c 2 nghim x = 0, x = 1.
158
Bi ton 8. Gii phng trnh : (1 + cos x) (2 + 4
cos x
) = 3.4
cos x
Li gii. t y = cos x, 1 y 1.
Ta c phng trnh: (1 + y) (2 + 4
y
) = 3.4
y

3.4
y
2+4
y
y 1 = 0
t f (y) =
3.4
y
2+4
y
y 1. Ta c f

(y) =
6 ln 4.4
y
(2+4
y
)
2
1
f

(y) = 0 6 ln 4.4
y
= (2 + 4
y
)
2
y l phng trnh bc hai theo 4
y
nn c khng qu 2 nghim.
Vy theo nh l Rolle th phng trnh f(y) = 0 c khng qu 3 nghim.
Mt khc ta thy y = 0; y =
1
2
; y = 1 l 3 nghim ca phng trnh f(y) = 0.
Vy phng trnh cho c cc nghim tng ng x = k.2; x =

2
+k.; x =

3
+k.2 (k )
Bi ton 9. Cho a, b, c R v n N

tha mn iu kin c =
6(a+b)
5(n+2)
. Chng minh rng
phng trnh sau c nghim trong khong 0;

2
_
:
asin
n
x +bcos
n
x + c sin x +c = 0
Li gii. t f (x) =
2a
n+2
sin
n+2
x
2b
n+2
cos
n+2
x +
2c
3
sin
3
x ccos
2
x
Ta c f(x) xc nh, lin tc v c o hm trn R f

(x) = 2asin
n+1
x cos x +2bcos
n+1
x sin x +
2csin
2
x cos x + 2c cos x sin cx
f

(x) = 2 sin x cos x (asin


n
x +bcos
n
x +c sin x +c)
Ta c f (0) =
2b
n+2
+
6(a+b)
5(n+2)
=
6a4b
5(n+2)
f

2
_
=
2a
n+2

2
3
.
6(a+b)
5(n+2)
=
2a
n+2

4(a+b)
5(n+2)
=
6a4b
5(n+2)
f (0) = f

2
_
, nn theo nh l Rolle th x
0
0;

2
_
: f

(x
0
) = 0
Hay x
0
0;

2
_
: f

(x
0
) = 2 sin x
0
cos x
0
(asin
n
x
0
+bcos
n
x
0
+c sin x
0
+c) = 0
x
0
0;

2
_
: asin
n
x
0
+ bcos
n
x
0
+c sin x
0
+c = 0
Do phng trnh a sin
x
+b cos
n
x + c sin x +c = 0 c nghim trong khong 0;

2
_
Bi ton 10. (VIT NAM TST 1997) Hy xc nh tt c cc cp s thc dng a, b sao
cho vi mi n N

v vi mi nghim thc x
n
ca phng trnh 4n
2
x = log
2
(2n
2
x + 1) ta lun
c:
a
xn
+b
xn
2 + 3x
n
Li gii. 4n
2
x = log
2
(2n
2
x + 1). t y = 2n
2
x,
PT tr thnh: 2y = log
2
(y + 1) 4
y
= y+ f(y) = 4
y
y 1 = 01
Ta c f

(y) = 4
y
ln 4 1, f(y) = 4
y
(ln 4)
2
> 0
Do phng trnh f(y) = 0 c khng qu 2 nghim phn bit.
Tht vy, nu phng trnh f(y) = 0 c 3 nghim phn bit th theo nh l Rolle, phng trnh
f

(y) = 0 c 2 nghim phn bit v phng trnh f(y) = 0 c nghim, mu thun.


Ta c f (0) = f
1
2
_
= 0 nn
_
y = 0
y =
1
2
x
n
= 0 x
n
=
1
4n
2
Nu x
n
= 0 th a, b > 0 ta
c a
0
+ b
0
2 + 3.0 l ng.
Nu x
n
=
1
4n
2
. Ta cn tm a, b > 0 a

1
4n
2
+b

1
4n
2
2
3
4n
2
, n N

()
t f(x) = a
x
+b
x
+ 3x.
159
Ta c : f

(x) = 3 a
x
ln a b
x
ln b, f(0) = 2
Hm (x) = t
x
ln t c

(x) = t
x
(ln t)
2
< 0 nn (x) gim (0; +)
f

(x) tng trn [0, +)


- Nu f

(0) < 0, do f(x) lin tc nn > 0 sao cho f

(x) < 0, x [0, )


Suy ra f(x) gim trn [0, )
Lun n N


1
4n
2
[0, ) nn f
1
4n
2
_
< f(0) = 2 v khng nh (*) khng tha mn.
- Xt f

(0) 0. Khi f

(0) 0, x [0; +) . Suy ra f(x) tng trn [0; +). Do


f
1
4n
2
_
f(0) = 2 v (*) tha mn.
Vy f

(0) 0 l iu kin cn v ta c (*).


Do 3 ln a + ln b = ln ab ab e
3
3.2 Chng minh bt ng thc
Bi ton 1. Cho a < b < c. Chng minh rng :
3a < a + b + c

a
2
+ b
2
+c
2
ab bc ca < a +b +c +

a
2
+b
2
+c
2
ab bc ca
Li gii. Xt hm s f (x) = (x a) (x b) (x c)
Hm s ny tha mn mi iu kin ca nh l Lagrange trn on [a; c]
Ta c f (a) = f (b) = f (c) = 0. Theo nh l Lagrange tn ti x
1
; x
2
: a < x
1
< b < x
2
< c sao
cho : f (b) f (a) = (b a) f

(x
1
) v f (c) f (b) = (c b) f

(x
2
)
f

(x
1
) = f

(x
2
) = 0
Mt khc f

(x) = 3x
2
2 (a + b + c) x + ab + bc + ca
V f

(x
1
) = f

(x
2
) = 0 nn suy ra x
1
, x
2
(x
1
< x
2
) l hai nghim ca phng trnh f

(x) = 0
x
1
=
a +b +c

a
2
+b
2
+c
2
ab bc ca
3
; x
2
=
a +b +c +

a
2
+b
2
+c
2
ab bc ca
3
T a < x
1
< b < x
2
< c suy ra pcm.
Bi ton 2. Chng minh rng vi mi s thc dng a, b, c, d ta u c :
3
_
abc +abd +acd + bcd
4

_
ab + ac + ad +bc +bd +cd
6
Li gii. Do a, b, c, d c vai tr nh nhau, khng mt tnh tng qut gi s a b c d.
Xt hm s f (x) = (x a) (x b) (x c) (x d)
Hm s ny tha mn mi iu kin ca nh l Lagrange trn [a; b] , [b, c] , [c, d]
Ta c f (a) = f (b) = f (c) = f (d) = 0. Theo nh l Lagrange tn ti x
1
, x
2
, x
3
:
a x
1
b x
2
c x
3
d sao cho f

(x
1
) = f

(x
2
) = f

(x
3
) = 0
Do f

(x) = 4 (x x
1
) (x x
2
) (x x
3
)
S hng khng cha x ca f

(x) l 4x
1
x
2
x
3
H s ca s hng cha x ca f(x) l : (abc +abd +acd + bcd)
Suy ra : 4x
1
x
2
x
3
= (abc + abd +acd +bcd) x
1
x
2
x
3
=
abc+abd+acd+bcd
4
H s ca s hng cha x ca f

(x) l : 4 (x
1
x
2
+ x
2
x
3
+x
3
x
1
)
160
] H s ca s hng cha x
2
ca f(x) l : ab +ac + ad +bc +bd +cd
Suy ra 4 (x
1
x
2
+x
2
x
3
+x
3
x
1
) = 2 (ab +ac +ad + bc + bd + cd)
x
1
x
2
+x
2
x
3
+ x
3
x
1
=
ab+ac+ad+bc+bd+cd
2
Theo bt ng thc AM GM :
x
1
x
2
+x
2
x
3
+x
3
x
1
3

3
_
(x
1
x
2
x
3
)
2
ab +ac +ad + bc + bd +cd
6

3

_
abc +abd + acd +bcd
4
_
2
Hay
3
_
abc+abd+acd+bcd
4

_
ab+ac+ad+bc+bd+cd
6
(pcm).
ng thc xy ra khi v ch khi a = b = c = d.
Bi ton 3. Cho t > 0. Chng minh : 1 +
1
t+1
_
t+1
> 1 +
1
t
_
t
Li gii. Xt hm s f (x) = x ln 1 +
1
x
_
= x [ln (x + 1) ln x]vi x > 0.
Ta c f

(x) = ln (x + 1) ln x + x
1
x+1

1
x
_
= ln (x + 1) ln x
1
x+1
(1)
Xt hm s g (y) = ln y trn on [x; x +1]. Theo nh l Lagrange tn ti c : x < c < x +1 sao
cho :
g (x + 1) g (x) = (x + 1 x) g

(c) ln (x + 1) ln x =
1
c
>
1
x+1
ln (x + 1) ln x
1
x+1
> 0 (2)
T (1) v (2) suy ra f

(x) > 0, x > 0.


Vy f(x) l hm s ng bin trn (0; +) .
Nh vy vi t > 0 ta c f (t + 1) > f (t) (t + 1) ln 1 +
1
t+1
_
> t ln 1 +
1
t
_
ln 1 +
1
t+1
_
t+1
> ln 1 +
1
t
_
t
1 +
1
t+1
_
t+1
> 1 +
1
t
_
t
(pcm)
Bi ton 4. Gi s S
1
=
4n
2

k=1
1
k
1
2
v S
2
=
n

k=1
1
k
1
3
. Vi nhng n nguyn dng no ta c S
1
< S
2
Li gii. Xt hm s f (x) = x
1
2
(x 1).
Theo nh l Lagrange trn on : [n; n + 1] tn ti s c (n; n + 1) ta c :
f (n + 1) f (n) = f

(c) =
1
2
c

1
2
<
1
2
n

1
2
n

1
2
> 2
_
(n + 1)
1
2
n
1
2
_
Cho n nhn cc gi tr t 1, 2, . . . , 4n
2
, cng li ta nhn c :
S
1
> 2
_
1 +

4n
2
+ 1
_
> 4n 2
Xt hm s g (x) = x
2
3
(x 1).
Theo nh l Lagrange trn on : [n; n + 1] tn ti s c (n; n + 1) ta c :
g (n + 1) g (n) = g

(c) =
2
3
c

1
3
>
2
3
(n + 1)

1
3
2(n + 1)

1
3
< 3
_
(n + 1)
2
3
n
2
3
_
Cho n nhn cc gi tr t 0, 2, . . . , n1, cng li ta nhn c :
2S
2
< 3n
2
3
< 3n < 8n 4
161
Suy ra 2S
2
< 2S
1
, n N

. Do khng tn ti s n nguyn dng no tha mn bi ton.


3.3 Cc bi ton dy s
Bi ton 1. Xt phng trnh : x
n
x
2
x 1 = 0 (n > 2)
1. Chng minh rng vi mi s nguyn n > 2 th phng trnh trn c mt nghim dng duy
nht. 2. Tm lim
n
n(x
n
1), trong x
n
l nghim dng ca phng trnh trn.
Li gii. 1. Xt hm f (x) = x
n
x
2
x 1 = 0 (n > 2).
Ta c f (1) = 2 < 0; f (2) = 2
n
7 > 0, nn theo nh l Bolzano Cauchy suy ra phng
trnh f (x) = 0 c t nht mt nghim thuc (1; 2)
R rng nu x l nghim dng ca phng trnh trn th x > 1. (do x
n
= x
2
= x + 1 > 1).
Vi x > 1 th f

(x) = nx
n1
2x 1 > 0, nn theo nh l Rolle phng trnh f(x) = 0 ch c
nhiu nht mt nghim. Suy ra iu phi chng minh. Hn na ta c x
n
(0; 2).
2. Trc ht ta chng minh lim
n
x
n
= 1.
Tht vy 1 < x
n
=
n
_
x
2
n
+x
n
+ 1
x
2
n
+xn+n
n
< 1 +
5
n
] (BT AM GM)
Suy ra lim
n
x
n
= 1 (1)
Ta c x
n
n
= x
2
n
+x
n
+ 1 n =
ln(x
2
n
+xn+1)
ln xn
n(x
n
1) =
(xn1)
ln xn
ln (x
2
n
+x
n
+ 1) (2)
Ta chng minh : lim
n
xn1
ln xn
= 1 (3)
Tht vy t x
n
1 = y
n
.
Ta lim
n
ln xn
xn1
= lim
n
(1 + y
n
)
yn
= ln e = 1 (do lim
n
y
n
= 0)
T (1), (2) v (3) suy ra : lim
n
n(x
n
1) = ln 3.
Bi ton 2. (VMO 2002) Xt phng trnh :
1
x1
+
1
4x1
+ ... +
1
k
2
x1
+ ... +
1
n
2
x1
=
1
2
] (1)
trong n l tham s nguyn dng
1. Chng minh rng vi mi s nguyn dng n, phng trnh nu trn c duy nht nghim
ln hn 1. K hiu nghim l x
n
. 2. Chng minh rng dy s {x
n
} c gii hn bng 4 khi
n +
Li gii. 1. (1)
1
2
+
1
x1
+
1
4x1
+... +
1
n
2
x1
= 0 (2)
t f
n
(x) =
1
2
+
1
x1
+
1
4x1
+ ... +
1
n
2
x1
Vi mi n N

, hm s f
n
(x) lin tc v nghch bin trn khong (1; +) v lim
x1
+
f
n
(x) = +,
lim
x+
f
n
(x) =
1
2
, nn theo nh l Bolzano Cauchy !x
n
> 1 : f
n
(x
n
) = 0
Vy phng trnh (1) c nghim duy nht ln hn 1.
2. Vi mi n N

, ta c :
f(4) =
1
2
+
1
2
2
1
+
1
4
2
1
+... +
1
(2n)
2
1
=
1
2
_
1 + 1
1
3
+
1
3

1
5
+... +
1
2n 1

1
2n + 1
_
=
1
2(2n + 1)
< 0 = f
n
(x
n
)
162
Do hm f
(
x) nghch bin trn (1; +) nn suy ra x
n
< 4, n N

(3)
Do vi mi n N

, hm f
n
(x) kh vi trn [x
n
; 4], nn theo nh l Lagrange vi mi n N

tn
ti t (x
n
; 4):
fn(4)fn(xn)
4xn
= f
/
n
(t) =
1
(t1)
2
+
4
(4t1)
2
+... +
n
2
(n
2
t1)
2
<
1
9
, n N

1
2(2n+1)(4xn)
<
1
9
, n N

x
n
> 4
9
2(2n+1)
, n N

(4)
T (3) v (4) ta c 4
9
2(2n+1)
< x
n
< 4
Do theo nh l v gii hn ca dy s kp gia hai dy s, ta c lim
n
x
n
= 4
Bi ton 3. Cho dy s thc (x
n
) vi n = 1, 2, . . . , tha mn ln (1 +x
2
n
) + nx
n
= 1] vi mi
s nguyn dng n. Tm lim
n+
n(1nxn)
xn
Li gii. Vi mi n N

ta t f
n
(x) = ln (1 + x
2
) + nx 1, x R.
Ta c f
n

(x) =
2x
1+x
2
+n =
(x+1)
2
1+x
2
+n 1 0
f
n

(x) = 0 n = 1, x = 1]. Do hm s f
n
(x)] l hm s tng thc s.
Ch f
n
(0) = 1 < 0; f
n
1
n
_
= ln 1 +
1
n
2
_
> 0, nn theo nh l Bolzano Cauchy, suy ra c
duy nht mt s x
n
R tha mn f
n
(x
n
) = 0 v 0 < x
n
<
1
n
. Bi vy
lim
n+
n(1 nx
n
)
x
n
= lim
n+
nln (1 +x
2
n
)
x
n
= lim
n+
_
nx
n
. ln 1 + x
2
n
_ 1
x
2
n
_
= 1
Do lim
x0
ln (1 +x
2
)
1
x
2
= 1 v nx
n
= 1 ln (1 +x
2
n
) 1 khi n +,
v x
n
0 khi n +. Ch
n(1+nxn)
xn
=
n
xn
+n
2
+ khi n +
Kt lun : lim
n+
n(1nxn)
xn
= 1
4 Bi tp p dng
Bi ton 1. Cho hm s f(x) lin tc trn on [a; b] v , l hai s dng bt k. Chng
minh rng phng trnh: f(x) =
f(a)+f(b)
+
c nghim trn on [a; b]
Bi ton 2. (ROMANIA 1998) Cho cc s thc a, b, c tho mn : 2a +10b +29c = 0. Chng
minh rng phng trnh : ax
3
+bx +c = 0 c nghim thuc on [0; 1]
Bi ton 3. Cho hm s f(x) lin tc trn on [0; 1] , nhn gi tr trong khong (0; 2). Chng
minh rng tn ti s c (0; 1) sao cho : (1 c)
2
+ [f (c) + 2
1c
]
2
= 1
Bi ton 4. Chng minh rng vi a, b, c l cc s thc tu cho trc, phng trnh a cos 3x+
a cos 2x +c cos x + sin x = 0 lun c nghim trong khong (0; 2)
Bi ton 5. Cho m > 0, cn a, b, c tho mn iu kin
a
m+2
+
b
m+1
+
c
m
= 0 Chng minh rng
khi phng trnh ax
2
+bx +c = 0 c t nht mt nghim thuc khong (0; 1).
Bi ton 6. Chng minh rng phng trnh : x
5
5x
4
+15x
3
x
2
+3x 7 = 0 c nghim duy
nht.
163
Bi ton 7. Xc nh s nghim ca phng trnh : 2e
2x
2
(x
6
3x
4
+ 5x
2
1) 2e 5 = 0
Bi ton 8. Chng minh rng vi mi a, b phng trnh a (25 sin 5x sin x)+b (49 sin 7x 9 sin 3x) =
0 c t nht 7 nghim trn [0; 2]
Bi ton 9. Cho P (x) = (x x
1
) (x x
2
) (x x
3
), vi x
1
< x
2
< x
3
. Chng minh rng :
P

(x
1
)
P

(x
1
)
+
P

(x
2
)
P

(x
2
)
+
P

(x
3
)
P

(x
3
)
= 0
Bi ton 10. (VIT NAM TST 1994) Cho a thc bc bn P(x) c 4 nghim dng. Chng
minh rng phng trnh :
14x
x
2
.P (x) + 1
14x
x
2
_
P

(x) P

(x) = 0 cng c 4 nghim dng.


Bi ton 11. Gii phng trnh :
_
1
2
_
2sin
2
x
+
1
2
= cos2x + log
4
4cos
3
2x cos6x 1
_
Bi ton 12. Gii phng trnh :
64
x
8.343
x1
= 8 + 12.4
x
.7
x1
Bi ton 13. Tm nghim dng ca phng trnh:
x ln
_
1 +
1
x
_
1+
1
x
x
3
ln
_
1 +
1
x
2
_
1+
1
x
2
= 1 x
Bi ton 14. (OLYMPIC SINH VIN VIT NAM 1999) Cho hm s f(x) kh vi trn [0; 1]
v tha mn iu kin f (0) = 0; f (1) = 1; 0 f (x) 1, x R. Chng minh rng tn ti
hai s a, b (0; 1), a = b sao cho f

(a) .f

(b) = 1
Bi ton 15. (OLYMPIC SINH VIN VIT NAM 1994) Cho hm s f(x) lin tc v c
o hm cp mt trn (0; +) v khng phi l hm hng. Cho a, b l hai s thc tha mn
iu kin 0 < a < b. Chng minh rng phng trnh : xf

(x) f (x) =
af(b)bf(a)
ba
c t nht mt
nghim thuc khong (a; b)
Bi ton 16. (OLYMPIC SINH VIN VIT NAM 2003) Cho hm s f(x) kh vi trn on
[a; b] v tha mn iu kin a) f (a) =
1
2
(a b) b) f (b) =
1
2
(b a) c) f
a+b
2
_
= 0 Chng minh
rng tn ti cc s i mt khc nhau c
1
, c
2
, c
3
(a; b) sao cho f

(c
1
) .f

(c
2
) .f

(c
3
) = 1
Bi ton 17. Chng minh rng nu 0 < b < a <

2
th ta c
ab
cos
2
b
< tan a tan b <
ab
cos
2
a
Bi ton 18. Chng minh rng vi x (0; 1) ; n N

, ta c x
n

1 x <
1

2ne
Bi ton 19. (VMO 1996) Cho bn s thc khng m a, b, c, d tha mn iu kin :
2(ab + ac + ad +bc +bd +cd) + abc +abd + acd +bcd = 16. Chng minh rng :
a + b +c +d
2
3
(ab +ac +ad +bc +bd + cd)
. Hi du bng xy ra khi no ?
164
Bi ton 20. (OLYMPIC 30 4 2002) Cho phng trnh : x
n
+x
n1
+... +x1 = 0 Chng
minh rng vi mi n nguyn dng th phng trnh trn c duy nht mt nghim dng x
n
.
Chng minh rng dy s (x
n
) c gii hn hu hn khi n +. Tm lim
n
x
n
Bi ton 21. Cho phng trnh vi tham s n nguyn dng : x + 2x
2
+... +nx
n
=
3
4
.
1. Chng minh rng phng trnh trn c nghim dng duy nht vi mi n nguyn dng, k
hiu l x
n
. 2. Chng minh rng dy s (x
n
) c gii hn hu hn khi n +. Tnh gii hn
.
Bi ton 22. Cho phng trnh
1
x
+
1
x1
+
1
x
2
1
+... +
1
x
n
1
= 0(n N

)
1. Chng minh rng vi mi n N

, phng trnh trn lun c nghim duy nht x


n
(0; 1)
2. Chng minh dy s (x
n
) , vi x
n
xc nh cu 1 c gii hn. Tm gii hn .
Bi ton 23. Tm lim
n+
_
1 +
1

2
+
1

3
+... +
1

n
_
Bi ton 24. (OLYMPIC SINH VIN VIT NAM 2002) Cho dy s thc {u
n
} c xc
nh nh sau : u
1
= a , u
n+1
=
1
2
ln (1 +u
2
n
) 2002, n 1. Chng minh rng dy {u
n
} l mt
dy hi t.
Bi ton 25. Cho dy s thc (x
n
) , n = 1, 2, 3, ... c xc nh nh sau :
_
x
1
= b
x
n+1
=
2006
3
ln x
2
n
+ 2006
2
_
2006
2
, n = 1, 2, 3, ...
Chng minh rng dy (x
n
) c gii hn hu hn khi n +.
Ti liu tham kho
1. Nguyn Vn Mu, Phng trnh v bt phng trnh, Nh xut bn Gio dc 1999.
2. Nguyn Vn Mu, Dy s v p dng, Nh xut bn Gio dc 2008.
3. Nguyn Vn Mu, Cc thi Olympic Ton sinh vin, Nh xut bn Gio dc 2006.
4. Phan Huy Khi, Ton nng cao gii tch, Nh xut bn H Ni 2002
5. Nguyn Khc Minh, Nguyn Vit Hi, Cc Bi Thi Olympic Ton THPT Vit Nam (1990
2006)
6. Tuyn tp cc thi Ton hc Olympic truyn thng cc Tnh pha nam
7. Cc ngun ti liu trn Internet.
8. Tp ch Ton hc v Tui tr
9. G.H. Hardy, J.E.Littlewood, G.Polya, Bt ng thc, Nh xut bn i Hc Quc Gia H
Ni 2002.
165
MT S PHNG PHP GII H PHNG TRNH
L Vn Thn, Trng THPT Chuyn L Qu n - Khnh Ha
H phng trnh i s l mng kin thc quan trng trong chng trnh ton hc ph thng,
n thng gp trong cc k thi tuyn sinh vo lp 10; tuyn sinh i hc, cao ng; thi hc sinh
gii. Mc d hc sinh c c st phn ny kh nhiu song phn ln cc em vn thng lng
tng trong qu trnh tm ra cch gii. Kinh nghim ca ti v mt hnh thc l khng mi.
Ci mi y chnh l s phn loi c tnh cht xuyn sut chng trnh nhng vn bm vo
cc k thut quen thuc, ph hp vi t duy ca hc sinh. Thm vo , vi mi bi ton u
c s phn tch lgic, c s tng qut v iu c bit l cho hc sinh tm ra ci gc ca bi
ton, cc bi ton t u m c, ngi ta to ra chng bng cch no. Thng qua cc vic
lm thng xuyn ny, hc sinh dn dn thch nghi mt cch rt tt, c t duy sng to, c
nng lc lm ton v to ra cc bi ton mi. Hc sinh thng hiu su v thch nghi khi hc
phn ny.
1 Phng php th
- C s phng php. Ta rt mt n (hay mt biu thc) t mt phng trnh trong h
v th vo phng trnh cn li.
- Nhn dng. Phng php ny thng hay s dng khi trong h c mt phng trnh l
bc nht i vi mt n no .
V d 1. Gii h phng trnh
_
2x + 3y = 5 (1)
3x
2
y
2
+ 2y = 4 (2)
Chng minh. T (1) ta c x =
53y
2
th vo (2) ta c 3
53y
2
_
2
y
2
+ 2y 4 = 0
3(25 30y + 9y
2
) 4y
2
+ 8y 16 23y
2
82y + 59 = 0 y = 1, y =
59
23
Vy tp nghim ca h phng trnh l
_
(1; 1) ;
31
23
;
59
23
__

V d 2. Gii h phng trnh
_
x
4
+ 2x
3
y +x
2
y
2
= 2x + 9 (1)
x
2
+ 2xy = 6x + 6 (2)
Phn tch. Phng trnh (2) l bc nht i vi y nn ta dng php th.
Chng minh. x = 0 khng tha mn (2) x = 0, (2) y =
6x+6x
2
2x
th vo (1) ta c
x
4
+ 2x
3
_
6x + 6 x
2
2x
_
+x
2
_
6x + 6 x
2
2x
_
2
= 2x + 9
166
x
4
+ x
2
(6x + 6 x
2
) +
(6x + 6 x
2
)
2
4
= 2x + 9 x(x + 4)
3
= 0
_
x = 0
x = 4
Do x = 0 nn h phng trnh c nghim duy nht 4;
17
4
_
.
Ch .
+ H phng trnh ny c th th theo phng php sau:
H
_
(x
2
+ xy)
2
= 2x + 9
x
2
+xy =
x
2
+6x+6
2

_ _
x
2
+6x+6
2
_
2
= 2x + 9
x
2
+xy =
x
2
+6x+6
2
+ Phng php th thng l cng on cui cng khi ta s dng cc phng php khc.
2 Phng php cng i s
- C s phng php. Kt hp 2 phng trnh trong h bng cc php ton: cng, tr,
nhn, chia ta thu c phng trnh h qu m vic gii phng trnh ny l kh thi hoc c
li cho cc bc sau.
- Nhn dng. Phng php ny thng dng cho cc h i xng loi II, h phng trnh
c v tri ng cp bc k.
V d 3. Gii h phng trnh
_
3y =
y
2
+2
x
2
3x =
x
2
+2
y
2
Chng minh. K:xy = 0
H
_
3x
2
y = y
2
+ 2 (1)
3y
2
x = x
2
+ 2 (2)
Tr v hai phng trnh ta c
3x
2
y 3xy
2
= y
2
x
2
3xy(x y) + (x y)(x +y) = 0
_
x y = 0
3xy + x + y = 0
TH 1. x y = 0 y = x th vo (1) ta c 3x
3
x
2
2 = 0 x = 1
TH 2. 3xy +x +y = 0 T 3y =
y
2
+2
x
2
y > 0, 3x =
x
2
+2
y
2
x > 0.
3xy +x +y > 0 Do TH 2 khng xy ra.
Vy h phng trnh c nghim duy nht (1; 1).
V d 4. Gii h phng trnh
_
_
_
1

x
+
_
2
1
y
= 2 (1)
1

y
+
_
2
1
x
= 2 (2)
Chng minh. K: x
1
2
, y
1
2
Tr v hai pt ta c
1

x

1

y
+
_
2
1
y

_
2
1
x
= 0

xy
+
2
1
y
2
1
x
_
_
2
1
y
+
_
2
1
x
= 0
y x

xy

x +

y
_ +
y x
xy
__
2
1
y
+
_
2
1
x
_ = 0
167
TH 1.y x = 0 y = x th vo (1) ta c
1

x
+
_
2
1
x
= 2
t t =
1

x
, t > 0 ta c

2 t
2
= 2t
_
2 t 0
2 t
2
= 4 4t +t
2

_
t 2
t
2
2t + 1 = 0
t = 1 x = 1 v y = 1
TH 2.
1

xy(

x+

y)
+
1
xy

2
1
y
+

2
1
x

= 0. TH ny v nghim do K
Vy h c nghim duy nht (1; 1).
V d 5. Gii h phng trnh
_
3x
2
+ 5xy 4y
2
= 38
5x
2
9xy 3y
2
= 15
Phn tch. y l h phng trnh c v tri ng cp bc hai nn ta s cn bng s hng
t do v thc hin php tr v.
Chng minh. H
_
45x
2
+ 75xy 60y
2
= 570
190x
2
342xy 114y
2
= 570
145x
2
+ 417xy + 54y
2
= 0
Gii phng trnh ny ta c y =
1
3
x, y =
145
18
x th vo mt trong hai phng trnh ca
h ta thu c kt qu.
Ch . - Cch gii trn c th p dng cho pt c v tri ng cp bc cao hn.
- Cch gii trn chng t rng h phng trnh ny hon ton gii c bng cch t
y = tx, x = 0 hoc t x = ty, y = 0.
V d 6. Tm m
_
3x
2
+ 2xy +y
2
= 11
x
2
+ 2xy + 3y
2
= 17 +m
c nghim
Phn tch. c kt qu nhanh hn ta s t ngay y = tx, x = 0
Chng minh. TH 1. x = 0
_
y
2
= 11
3y
2
= m+ 17

_
y
2
= 11
y
2
=
m+17
3
Vy h c nghim x = 0
m+17
3
= 11 m = 16
TH 2. x = 0. t y = tx. H
_
3x
2
+ 2tx
2
+t
2
x
2
= 11
x
2
+ 2tx
2
+ 3t
2
x
2
= 17 +m

_
(3 + 2t +t
2
)x
2
= 11
(1 + 2t + 3t
2
)x
2
= 17 +m

_
x
2
=
11
3+2t+t
2
(1 + 2t + 3t
2
).
11
3+2t+t
2
= 17 +m

_
x
2
=
11
3+2t+t
2
(m16)t
2
+ 2(m+ 6)t + 3m+ 40 = 0 ()
Ta c
11
3+2t+t
2
> 0, t nn h c nghim pt (*) c nghim. iu ny xy ra khi v ch khi
m = 16 hoc m = 16,

= (m+ 6)
2
(m16)(3m+ 40) 0
5

363 m 5 +

363
Kt lun. 5

363 m 5 +

363
168
V d 7. Tm m
_
5x
2
+ 2xy y
2
3
2x
2
+ 2xy +y
2

m
m1
c nghim
Chng minh. Nhn 2 v ca bpt th hai vi -3 ta c
_
5x
2
+ 2xy y
2
3
6x
2
6xy 3y
2
3
1
m1
Cng v hai bpt cng chiu ta c x
2
4xy 4y
2

1
m1
(x + 2y)
2

1
m1
iu kin cn h bpt c nghim l
1
m1
> 0 m > 1
iu kin . Vi m > 1. Xt h pt
_
5x
2
+ 2xy y
2
= 3
2x
2
+ 2xy +y
2
= 1
(II)
Gi s (x
0
; y
0
) l nghim ca h (II). Khi
_
5x
2
0
+ 2x
0
y
0
y
2
0
= 3
2x
2
0
+ 2x
0
y
0
+y
2
0
= 1

_
5x
2
0
+ 2x
0
y
0
y
2
0
3
2x
2
0
+ 2x
0
y
0
+y
2
0

m
m1
Vy mi nghim ca h (II) u l nghim ca h (I).
(II)
_
5x
2
+ 2xy y
2
= 3
6x
2
6xy 3y
2
= 3
x
2
4xy 4y
2
= 0 x + 2y = 0 x = 2y
Thay x = 2y vo pt th 2 ca h (II) ta c
8y
2
4y
2
+y
2
= 1 5y
2
= 1 y =
1

5
x =
2

5
H (II) c nghim, do h (I) cng c nghim. Vy m > 1.
V d 8. Gii h phng trnh
_
_
_

3x
_
1 +
1
x+y
_
= 2

7y
_
1
1
x+y
_
= 4

2
Phn tch.Cc biu thc trong ngoc c dng a + b v ab nn ta chia hai v pt th nht
cho

3x v chia hai v pt th hai cho



7y.
Chng minh. K: x 0, y 0, x + y = 0
D thy x = 0 hoc y = 0 khng tha mn h pt. Vy x > 0, y > 0.
H
_
_
_
_
1 +
1
x+y
_
=
2

3x
_
1
1
x+y
_
=
4

7y

_
2 =
2

3x
+
4

7y
2
x+y
=
2

3x

4

7y

_
1

3x
+
2

7y
= 1 (1)
1

3x

2

7y
=
1
x+y
Nhn theo v hai pt trong h ta c
_
1

3x
+
2

7y
__
1

3x

2

7y
_
=
1
x+y

1
3x

8
7y
=
1
x +y
7y
2
38xy 24x
2
= 0
_
y = 6x
y =
4
7
x
TH 1. y = 6x th vo pt (1) ta c
1

3x
+
2

21x
= 1 x =
11 + 4

7
21
y =
22 + 8

7
7
169
TH 2.y =
4
7
x khng xy ra do x > 0, y > 0.
Vy h pt c nghim duy nht (x; y) =
_
11+4

7
21
;
22+8

7
7
_
.
Ch . H phng trnh c dng
_
a +b = m
a b = n

_
m+n = 2a
mn = 2b
Trong trng hp ny, dng th nht c v phi cha cn thc nn ta chuyn v dng th
hai sau nhn v mt cn thc.
Tng qut ta c h sau:
_
a

bx
= m+
n
px+qy
c

dy
= m+
n
px+qy
V d 9. Gii h phng trnh
_
_
_
x
2
(y +z)
2
= (3x
2
+ x + 1)y
2
z
2
y
2
(z +x)
2
= (4y
2
+y + 1)z
2
x
2
z
2
(x +y)
2
= (5z
2
+z + 1)x
2
y
2
Phn tch. Nu chia hai v ca mi phng trnh cho x
2
y
2
z
2
th ta c h mi n gin
hn.
Chng minh. TH 1. xyz = 0. Nu x = 0 th h y
2
z
2
= 0
_
y = 0
z = t, t R
hoc
_
z = 0
y = t, t R
TH 2. xyz = 0. Chia hai v ca mi pt trong h cho x
2
y
2
z
2
ta c
_

_

_
_
1
z
+
1
y
_
2
= 3 +
1
x
+
1
x
2
(1)
1
x
+
1
z
_
2
= 4 +
1
y
+
1
y
2
(2)
_
1
y
+
1
x
_
2
= 5 +
1
z
+
1
z
2
(3)
Cng v 3 phng trnh ca h ta c
_
1
z
+
1
y
_
2
+
_
1
x
+
1
z
_
2
+
_
1
y
+
1
x
_
2
= 12 +
1
x
+
1
y
+
1
z
+
1
x
2
+
1
y
2
+
1
z
2

_
1
x
+
1
y
+
1
z
_
2

_
1
x
+
1
y
+
1
z
_
12 = 0
_
1
x
+
1
y
+
1
z
= 4(4)
1
x
+
1
y
+
1
z
= 3(5)
T (4) v (1) ta c 4
1
x
_
2
= 3 +
1
x
+
1
x
2

9
x
= 13 x =
9
13
.
T (4) v (2) ta c y =
3
4
. T (4) v (3) ta c z =
9
11
Tng t, t (5), (1), (2), (3) ta c x =
5
6
, y = 1, z =
5
4
Vy h c tp nghim l
S =
_
(t; 0; 0); (0; t; 0); (0; 0; t);
_
9
13
;
3
4
;
9
11
_
;
_

5
6
; 1;
5
4
_
, t R
_

170
Nhn xt. Qua v d trn ta thy: t mt h phng trnh n gin, bng cch i bin
s ( trn l php thay nghch o) ta thu c mt h phc tp. Vy i vi mt h phc tp
ta s ngh n php t n ph h tr nn n gin.
3 Phng php t n ph
V d 10. Gii h phng trnh
_
x + y +xy = 1
x
2
+y
2
xy = 7
Chng minh. y l h i xng loi I n gin nn ta gii theo cch ph bin.
H
_
(x +y) + xy = 1
(x +y)
2
3xy = 7
t
_
x +y = S
xy = P
(x, y S
2
4P) ta c
_
S +P = 1
S
2
3P = 7

_
S = 1, P = 2
S = 4, P = 3
TH 1.
_
S = 1
P = 2

_
x + y = 1
xy = 2

_
x = 1, y = 2
x = 2, y = 1
TH 2.
_
S = 4
P = 3

_
x + y = 4
xy = 3

_
x = 1, y = 3
x = 3, y = 1
Vy tp nghim ca h l
S = {(1; 2); (2; 1); (1; 3); (3; 1)}

Ch . - Nu h pt c nghim l x; y th do tnh i xng, h cng c nghim l y, x . Do vy,


h c nghim duy nht th iu kin cn l x = y .
- Khng phi lc no h i xng loi I cng gii theo cch trn. i khi vic thay i cch
nhn nhn s pht hin ra cch gii tt hn.
V d 11. Gii h phng trnh
_
x
2
+y
2
+x +y = 18
xy(x + 1)(y + 1) = 72
Phn tch. y l h i xng loi I
Hng 1. Biu din tng pt theo tng x +y v tch xy
Hng 2. Biu din tng pt theo x
2
+x v y
2
+ y. R rng hng ny tt hn.
Chng minh. H
_
(x
2
+x) + (y
2
+ y) = 18
(x
2
+x)(y
2
+ y) = 72
t
_
x
2
+x = a, a
1
4
y
2
+ y = b, b
1
4
ta c
_
a +b = 18
ab = 72

_
a = 6, b = 12
a = 12, b = 6
TH 1.
_
a = 6
b = 12

_
x
2
+x = 6
y
2
+y = 12

_
x = 2, x = 3
y = 3, y = 4
TH 2.i vai tr ca a v b ta c
_
x = 3, x = 4
y = 2, y = 3
Vy tp nghim ca h l
171
S = {(2; 3); (2; 4); (3; 3); (3; 4); (3; 2); (4; 2); (3; 3); (4; 3)}
Nhn xt. Bi ton trn c hnh thnh theo cch sau
Xut pht t h phng trnh n gin
_
a +b = 18
ab = 72
(I)
1. Thay a = x
2
+x, b = y
2
+ y vo h (I) ta c h
(1)
_
x
2
+y
2
+x +y = 18
xy(x + 1)(y + 1) = 72
chnh l v d 11
2. Thay a = x
2
+xy, b = y
2
xy vo h (I) ta c h
(2)
_
x
2
+y
2
= 18
xy(x
2
y
2
) = 72
3. Thay a = x
2
+ 2x, b = 2x + y vo h (I) ta c h
(3)
_
x
2
+ 4x +y = 18
x(x + 2)(2x +y) = 72
4. Thay a = x +
1
x
, b = y +
1
y
vo h (I) ta c h
(4)
_
(x +y)xy + x + y = 18xy
(x
2
+ 1)(y
2
+ 1) = 72xy
5. Thay a = x
2
+ 2xy, b = y
2
xy vo h (I) ta c h
(5)
_
x
2
+y
2
+xy = 18
xy(x + 2y)(y x) = 72
- Nh vy, vi h xut (I), bng cch thay bin ta thu c rt nhiu h pt mi.
- Thay h xut pht (I) bng h xut pht (II)
_
a +b = 7
a
2
b
2
= 21
v lm tng t nh trn
ta li thu c cc h mi khc. Chng hn
6. Thay a = x
2
+y
2
, b = xy vo h (II) ta c h
(6)
_
x
2
+y
2
+xy = 7
x
4
+y
4
+x
2
y
2
= 21
7. Thay a = x +
1
x
, b = y +
1
y
vo h (II) ta c h
(7)
_
x + y +
1
x
+
1
y
= 7
x
2
y
2
+
1
x
2

1
y
2
= 21
8. Thay a = x +
1
y
, b =
x
y
vo h (II) ta c h
(8)
_
xy +x + 1 = 7y
(xy + 1)
2
+x
2
= 21y
2
172
9. Thay a = x + y, b =
1
y
vo h (II) ta c h
(9)
_
(x +y)y + 1 = 9y
(x +y 2)
2
y
2
21y
2
= 1
10. Thay a = x
2
+ 2x, b = y
2
+ 2x vo h (II) ta c h
(10)
_
x
2
+y
2
+ 4x = 7
x
4
y
4
+ 4x(x
2
y
2
) = 21
Nh vy, nu chng ta bit cch to ra bi ton th chng ta c th ngh ra cch gii ca nhng
bi ton khc.
V d 12. Gii cc h pt sau
a)
_
x(x +y + 1) 3 = 0
(x +y)
2

5
x
2
+ 1 = 0
b)
_
x
2
+y +x
3
y +xy
2
+xy =
5
4
x
4
+y
2
+xy(1 + 2x) =
5
4
c)
_
x + y

xy = 3

x + 1 +

y + 1 = 4
d)
_
x
2
+y
2
+ 2(x + y) = 7
y(y 2x) 2x = 10
Chng minh. a) K. x = 0 H
_
x +y + 1 3.
1
x
= 0
(x + y)
2
5.
1
x
_
2
+ 1 = 0
t x +y = a,
1
x
= b ta c h
_
a + 1 3b = 0
a
2
5b
2
+ 1 = 0

_
a = 3b 1
(3b 1)
2
5b
2
+ 1 = 0

_
a = 2, b = 1
a =
1
2
, b =
1
2

_
x = y = 1
x = 2, y =
3
2
b) H
_
(x
2
+ y) + xy(x
2
+y + 1) =
5
4
(x
2
+ y)
2
+xy =
5
4
t x
2
+y = a, xy = b ta c
_
a + b(a + 1) =
5
4
a
2
+b =
5
4

_
a
2
a ab = 0
b =
5
4
a
2

_
a = 0, b =
5
4
a =
1
2
, b =
3
2
TH1.
_
a = 0
b =
5
4

_
x
2
+y = 0
xy =
5
4

_
_
_
x =
3
_
5
4
y =
3
_
25
16
TH2.
_
a =
1
2
b =
3
2

_
x
2
+y =
1
2
xy =
3
2

_
x
2

3
2x
=
1
2
y =
3
2x

_
x = 1
y =
3
2
Vy tp nghim ca h pt l S =
_
_
1;
3
2
_
;
_
3
_
5
4
;
3
_
25
16
__
c) K: x 1, y 1, xy 0
H
_
x +y

xy = 3
x +y + 2 + 2
_
(x + 1)(y + 1) = 16

_
x +y

xy = 3
x +y + 2

x +y +xy + 1 = 14
t x +y = a,

xy = b a 2, b 0, a
2
4b
2
ta c h pt
_
a b = 3
a + 2

a +b
2
+ 1 = 14

_
a = 3 +b
2

b
2
+b + 4 = 11 b

_
a = 3 +b
3b
2
+ 26b 105 = 0
173

_
b = 3
a = 6

_
x = 3
y = 3
(tha iu kin)
d) H
_
(x + 1)
2
+ (y + 1)
2
= 9
(y x)
2
(x + 1)
2
= 9
t a = x + 1, b = y + 1 b a = y x ta c h
_
a
2
+b
2
= 9
(b a)
2
a
2
= 9
a
2
+b
2
= (b a)
2
a
2
a
2
= 2ab a = 0 hoca = 2b
a = 0 b = 3 x = 1, y = 2 hoc x = 1, y = 4
a = 2b 5b
2
= 9 b =
3

5
a =
6

5
x = 1
6

5
, y = 1 +
3

5
hoc x = 1 +
6

5
, y = 1
3

5
Vy h c 4 nghim nh trn.
4 Phng php a v dng tch
- C s phng php. Phn tch mt trong hai phng trnh ca h thnh tch cc nhn
t. i khi cn t hp hai phng trnh thnh phng trnh h qu ri mi a v dng tch.
- Cch thnh lp h dng ny
_
(ax +by + c)f(x; y) = 0
g(x; y) = 0
trong f(x, y) c chn
sao cho
_
f(x; y) = 0
g(x; y) = 0
v nghim hoc
_
f(x; y) = 0
g(x; y) = 0
gii c; g(x, y) c chn sao cho
_
ax +by + c = 0
g(x; y) = 0
gii c v tha mn kt hp c vi f(x, y)
V d 13. Gii h phng trnh sau
_
xy + x + y = x
2
2y
2
(1)
x

2y y

x 1 = 2x 2y (2)
Phn tch. R rng, vic gii phng trnh (2) hay kt hp (1) vi (2) khng thu c kt
qu kh quan nn chng ta tp trung gii (1).
Chng minh. K: x 1, y 0
(1) y(x +y) + (x +y) = x
2
y
2
(x +y)(y + 1 x + y) = 0
TH1. x + y = 0 (loi dox 1, y 0 )
TH2. 2y + 1 x = 0 x = 2y + 1 th vo pt (2) ta c
(2y + 1)

2y y

2y = 4y + 2 2y (y + 1)

2y = 2(y + 1)

_
y + 1 = 0

2y = 2

_
y = 1
y = 2
Do y 0 y = 2 Vy h c nghim (x; y) = (5; 2)
Ch . Do c th phn tch c thnh tch ca hai nhn t bc nht i y (hay x) nn c
th gii pt (1) bng cch coi (1) l pt bc hai n y (hoc x).
V d 14. Gii h phng trnh sau
_
x
1
x
= y
1
y
(1)
2y = x
3
+ 1 (2)
174
Phn tch. T cu trc ca pt (1) ta thy c th a (1) v dng tch.
Chng minh. K: xy = 0 (1) xy
1
x
+
1
y
= 0 xy +
xy
xy
= 0 (xy)
_
1 +
1
xy
_
= 0
TH1. x = y th vo (2) ta c x
3
2x + 1 = 0 x = 1 hoc x =
1

5
2
(TM)
TH2. 1 +
1
xy
= 0 y =
1
x
th vo pt (2) ta c
x
4
+ x + 2 = 0 (x
2

1
2
)
2
+ (x +
1
2
)
2
+
3
2
= 0 Pt ny v nghim.
Vy h c nghim S =
_
(1; 1);
_
1+

5
2
;
1+

5
2
_
;
_
1

5
2
;
1

5
2
__

V d 15. Gii h phng trnh sau
_
x
1
x
3
= y
1
y
3
(1)
(x 4y)(2x y + 4) = 36 (2)
Chng minh.
x
1
x
3
= y
1
y
3
(x y) =
(yx)(y
2
+xy+x
2
)
x
3
y
3

_
x = y
y
2
+xy+x
2
x
3
y
3
= 1
TH1. x = y th vo (2) ta c x
2
+ 4x 12 = 0
_
x = 6
x = 2
TH2.
y
2
+xy+x
2
x
3
y
3
= 1 xy < 0
th vo pt (2) ta c
(2) 2x
2
+ 4y
2
9xy + 4x 16y = 36 2(x + 1)
2
+ 4(y 2)
2
9xy = 18
Trng hp ny khng xy ra do xy < 0 2(x + 1)
2
+ 4(y 2)
2
9xy > 0
Vy h c nghim S = {(2; 2); (6; 6)}
V d 16. Gii h phng trnh sau
_
x
2
+y
2
+
8xy
x+y
= 16 (1)

x + y = x
2
y (2)
Phn tch. R rng, vic gii phng trnh (2) hay kt hp (1) vi (2) khng thu c kt
qu kh quan nn chng ta tp trung gii (1)
Chng minh. K: x +y > 0 (1) (x
2
+y
2
)(x +y) + 8xy = 16(x +y)

_
(x +y)
2
2xy

(x +y) + 8xy = 16(x +y)


(x + y)
_
(x +y)
2
16

2xy(x +y 4) = 0
(x + y 4) [(x +y)(x +y + 4) 2xy] = 0
TH1. x + y 4 = 0 th vo (2) ta c x
2
+x 6 = 0
_
x = 3 y = 7
x = 2 y = 2
TH2.(x +y)(x +y + 4) 2xy = 0 x
2
+y
2
+ 4(x +y) = 0 Pt ny v nghim do iu kin.
Vy h c nghim S = {(3; 7); (2; 2)}
5 Phng php s dng tnh n iu ca hm s
* C s phng php. Nu f(x) n iu trn khong (a; b) v x, y (a; b) th f(x) =
f(y) x = y
175
* Cch xy dng h theo phng php ny.
- Ly hm s f(t) n iu trn khong (a; b) , u(x; y), v(x; y) (a; b)
- Ly g(x; y) sao cho h
_
u(x; y) = v(x; y)
g(x; y) = 0
gii c trn tp xc nh ca chng.
- Lp h phng trnh
_
f(u) = f(v)
g(x; y) = 0
V d 17. Gii h phng trnh sau
_
2
x
2
y
= (y x)(xy + 2)
x
2
+y
2
= 2
Phn tch. Nu thay 2 = x
2
+y
2
vo phng trnh th nht th ta s c ht
Chng minh. Thay 2 = x
2
+y
2
vo phng trnh th nht ta c
2
x
2
y
= (y x)(xy +x
2
+ y
2
) 2
x
2
y
= y
3
x
3
2
x
+x
3
= 2
y
+ y
3
(1)
Xt hm s f(t) = 2
t
+ t
3
, t R c f

(t) = 2
t
ln 2 + 3t
2
> 0, t R suy ra f(t) ng bin
trn R . (1) f(x) = f(y) x = y th vo pt th hai ta c x = y = 1. Vy tp nghim
ca h l S = {(1; 1); (1; 1)}
V d 18. Gii h phng trnh sau
_
(4x
2
+ 1)x + (y 3)

5 2y = 0 (1)
4x
2
+y
2
+ 2

3 4x = 7 (2)
Chng minh. K:
_
3 4x 0
5 2y 0

_
x
3
4
y
5
2
(1) (4x
2
+ 1)2x + (2y 6)

5 2y = 0

_
(2x)
2
+ 1

(2x) =
_
_
5 2y
_
2
+ 1
_

5 2y (2x)
3
+ 2x =

5 2y
_
3
+

5 2y
f(2x) = f(

5 2y) vi f(t) = t
3
+ t f

(t) = 3t
2
+ 1 > 0, t R, f(t) ng bin trn
R. Vy f(2x) = f(

5 2y) 2x =

5 2y y =
54x
2
2
, x 0.
Th vo phng trnh th hai ta c 4x
2
+
_
54x
2
2
_
2
+ 2

3 4x 7 = 0 g(x) = 0
Vi g(x) = 4x
2
+
_
54x
2
2
_
2
+ 2

3 4x 7, x
_
0;
3
4


V d 19. Gii h phng trnh sau
_
x
3
3x = y
3
3y (1)
x
2
+y
2
= 1 (2)
Phn tch. Ta c th gii h trn bng phng php a v dng tch. Tuy nhin ta mun
gii h ny bng phng php s dng tnh n iu ca hm s. Hm s f(t) = t
3
3t khng
n iu trn ton trc s, nhng nh c (2) ta gii hn c x v y trn on [1; 1]
Chng minh. T (2) ta c x
2
1, y
2
1 x, y [1; 1]
Hm s f(t) = t
3
3t c f

(t) = 3t
2
3 < 0, t (1; 1) f(t) ng bin trn on
[1; 1] .x, y [1; 1] nn (1) f(x) = f(y) x = y th vo pt (2) ta c x = y =

2
2
.
Vy tp nghim ca h l S =
__

2
2
;

2
2
_
;
_

2
2
;

2
2
__

Nhn xt. Trong trng hp ny ta hn ch min bin thin ca cc bin hm s
n iu trn on .
176
V d 20. Tm cc gi tr ca m h phng trnh sau c nghim
_
x
3
y
3
+ 3y
2
3x 2 = 0
x
2
+

1 x
2
3
_
2y y
2
+ m = 0
Chng minh. K: 1 x 1, 0 y 2
(1) x
3
3x = (y 1)
3
3(y 1)
Hm s f(t) = t
3
3t nghch bin trn on [1; 1].
x, y 1 [1; 1] nn f(x) = f(y 1) x = y 1 y = x + 1
Th vo pt(2) ta c x
2
2

1 x
2
= m (3)
H c nghim Pt (3) c nghim x [1; 1]
Xt g(x) = x
2
2

1 x
2
, x [1; 1] , g

(x) = 2x
_
1 +
1

1x
2
_
g

(x) = 0 x = 0.g(0) = 2, g(1) = 1. g(0) = 2, g(1) = 1


Pt (3) c nghim x [1; 1] 2 m 1 1 m 2
V d 21. Gii h phng trnh
_
x +

x
2
+ 1 = 3
y
y +
_
y
2
+ 1 = 3
x
Chng minh. Tr v hai pt ta c
x +

x
2
+ 1
_
y +
_
y
2
+ 1
_
= 3
y
3
x
x +

x
2
+ 1 + 3
x
= y +
_
y
2
+ 1 + 3
y
f(x) = f(y) nn f(t) = t +

t
2
+ 1 + 3
t
.f(t) = 1 +
t

t
2
+1
+ 3
t
ln 3 > 0, R.
f(t) ng bin trn R. Bi vy f(x) = f(y) x = y th vo pt th nht ta c
x +

x
2
+ 1 = 3
x
1 = 3
x

x
2
+ 1 x
_
g(0) = g(x)
Vi g(x) = 3
x

x
2
+ 1 x
_
.g

(x) = 3
x
ln 3

x
2
+ 1 x
_
+ 3
x
_
x

x
2
+1
1
_
= 3
x

x
2
+ 1 x
_
_
ln 3
1

x
2
+1
_
> 0, x R. do

x
2
+ 1 x > 0 v

x
2
+ 1 1
Suy ra g(x) ng bin trn R. Bi vy g(x) = g(0) x = 0
Vy h phng trnh c nghim duy nht x = y = 0.
V d 22. Chng minh h
_
e
x
= 2007
y

y
2
1
e
y
= 2007
x

x
2
1
c ng 2 nghim x > 0, y > 0
Chng minh. K:
_
x
2
1 > 0
y
2
1 > 0

_
x (; 1) (1; +)
y (; 1) (1; +)
Do
_
x > 0
y > 0
nn
_
x > 1
y > 1
Tr v hai pt ta c e
x
e
y
=
x

x
2
1

y

y
2
1
e
x

x
2
1
= e
y

y
2
1
Hay f(x) = f(y) vi f(t) = e
t

t
2
1
, t (1; +)
f

(t) = e
t
+
1
(t
2
1)

t
2
1
> 0, t (1; +) f(t) ng bin trn (1; +)
Bi vy f(x) = f(y) x = y th vo pt th nht ta c
e
x
= 2007
x

x
2
1
e
x
+
x

x
2
1
2007 = 0 g(x) = 0
Vi g(x) = e
x
+
x

x
2
1
2007, x (1; +). Ta c
g

(x) = e
x

1
(x
2
1)

x
2
1
; g

(x) = e
x
+
3x(x
2
1)
(x
2
1)
3

x
2
1
> 0, x (1; +)
177
Suy ra g

(x) ng bin trn (1; +). g

(x) lin tc trn (1; +). v c


lim
x1
+
g

(x) = , lim
x+
g

(x) = +
nn g

(x) = 0 c nghim duy nht trn x


0
(1; +) v
g

(x) > 0 g

(x) > g

(x
0
) x > x
0
. g

(x) < 0 1 < x < x


0
T BBT ca g(x) ta suy ra pt g(x) = 0 c ng 2 nghim x (1; +). Vy h phng trnh
cho c ng 2 nghim dng.
V d 23. Gii h phng trnh
_
ln(1 + x) ln(1 + y) = x y (1)
x
2
12xy + 20y
2
= 0 (2)
Chng minh. K: x > 1, y > 1
(1) ln(1 + x) x = ln(1 +y) y f(x) = f(y) vi
f(t) = ln(1 +t) t, t (1; +)
f

(t) =
1
1+t
1 =
t
1+t
= 0 t = 0 (1; +) f(t) ng bin trn (1; 0) v nghch
bin trn khong
TH 1. x, y (1; 0) hoc x, y (0; +) th f(x) = f(y) x = y
Th vo pt (2) ta c x = y = 0 (khng tha mn)
TH 2. x (1; 0),y (0; +) hoc ngc li th xy < 0 x
2
12xy + 20y
2
> 0
TH 3. xy = 0 th h c nghim x = y = 0 . Vy h c nghim duy nht x = y = 0.
Mt s xut
Mi bi ton thng c ci gc ca n, vic hc sinh pht hin ra bi ton gc s thy ton
hc rt thc t, t nhin v khng kh nh cc em ngh ng thi to nim tin v hng th
hc tp vi cc em. Vi tinh thn nh vy v theo hng ny cc thy c gio v cc em hc
sinh c th tm ra c nhiu kinh nghim hay vi nhiu ti khc nhau. Chng hn, cc bi
ton v tch phn, cc bi ton v t hp xc sut, cc bi ton v phng php ta trong
mt phng, trong khng gian.
Ti liu tham kho
[1] Nguyn Vn Mu, Phng trnh hm, Nh xut bn Gio dc, 1997.
[2] Pl. Kannappan, Functional Equations and Inequalities with Applications, Springer, 2009,
295-323.
[3] J. Bochnak and J. Sicial, Analytic functions in topological vector spaces, Studia Math., 39
(1971), 77-112.
178
MT S BI TON V A THC TRONG CC K
THI HC SINH GII
Hunh Kim Linh - T Hng Khanh, Trng THPT Chuyn L Qu n - Khnh Ha
1 Cc bi ton c li gii
Bi ton 1. Tm tt c cc a thc f(x) = x
3
+ ax
2
+ bx + c, vi a, b, c l cc s thc tha
phng trnh f(x) = 0 c cc nghim l a, b, c.
Gii. a thc f(x) = x
3
+ ax
2
+ bx + c c cc nghim l a, b, c c vit di dng :
f(x) = (x a)(x b)(x c).
ng nht cc h s ta c :
_
_
_
(a + b + c) = a
ab +bc +ca = b
abc = c
Vi c = 0, ta c 2 nghim a = b = c = 0 v a = 1, b = 2, c = 0.
Vi c = 0 ta c
_
_
_
(a +b +c) = a
ab + bc + ca = b
abc = c

_
_
_
a =
1
b
c =
2
b
b
b
4
+b
3
2b
2
+ 2 = 0
b
4
+ b
3
2b
2
+ 2 = 0 (b + 1)(b
3
2b + 2) = 0 b = 1 hoc b
3
2b + 2 = 0.
Vi b = 1 th a = 1, c = 1 Vi b
3
2b +2 = 0, bng cch t b = 2
_
2
3
x ta a phng trnh
v dng 4x
3
3x +
3

3
2

2
= 0, phng trnh c nghim :
x =
3
_

3
2

2
+
_
27
8
1 +
3
_

3
2

2

_
27
8
1
2
=
3
_

19

27
3
_

19 +

27
2

2
T suy ra b = 2
_
2
3

3

19

27
3

19+

27
2

2
=
3

19

27
3

19+

27

3
.
t b
0
=
3

19

27
3

19+

27

3
ta suy ra a
0
, c
0
tng ng.
Vy c tt c 4 a thc tha mn bi l f
1
(x) = x
3
; f
2
(x) = x
3
2x; f
3
(x) = x
3
+ x
2
x
1; f
4
(x) = x
3
+a
0
x
2
+ b
0
x +c
0
vi
_

_
b
0
=
3

19

27
3

19+

27

3
a
0
=
1
b
0
c
0
=
2
b
0
b
0
Bi ton 2. Cc a thc P(x) = x
5
x 1 v Q(x) = x
2
+ ax + b vi a, b Q c th c
nghim phc chung khng ?
179
Gii. Gi s l nghim chung ca P(x) v Q(x),
Ta c :
5
= + 1 v
2
= a b.
+1 =
5
=(
2
)
2
= (a b)
2
= [a
2
(a b) + (2ab + b
2
)].
= (2ab a
3
)(a b) + (b
2
a
2
b) = = (a
4
3a
2
b +b
2
) + a
3
b 2ab
2

_
a
4
3a
2
b +b
2
= 1 (1)
a
3
b 2ab
2
= 1 (2)
T (1) b
2
= 1 a
4
+ 3a
2
b thay vo (2) v rt gn b =
2a
5
2a1
5a
3
Thay vo (1) ta c a
10
+ 3a
6
11a
5
4a
2
4a 1 = 0 iu ny tri vi a Q. V phng
trnh a
10
+ 3a
6
11a
5
4a
2
4a 1 = 0 khng c nghim hu t.
Bi ton 3. Gi s a, b l 2 trong 4 nghim ca phng trnh : x
4
+ x
3
1 = 0 (1). Chng
minh rng ab l nghim ca phng trnh x
6
+ x
4
+x
3
x
2
1 = 0 (2)
Gii. Gi s a, b, c, d l 4 nghim ca phng trnh (1)
Suy ra P(x) = x
4
+x
3
1 = (x a)(x b)(x c)(x d) = 0
Ta chng minh : (ab)
3
+ (cd)
3
+ ab + cd + 1 = 0
Khi ta suy ra : (ab)
6
+ (ab)
4
+ (ab)
3
(ab)
2
1 = 0 ab
3

1
(ab)
3
+ ab
1
ab
+ 1 = 0
V abcd = 1 hay ab =
1
cd
.Tht vy P(a) = P(b) = 0
a
3
=
1
a + 1
, b
3
=
1
b + 1
(ab)
3
=
1
(a + 1)(b + 1)
=
(1 + c)(1 + d)
P(1)
Hay (ab)
3
= (1 + c)(1 + d). Tng t (cd)
3
= (1 + a)(1 + b) Suy ra
(ab)
3
+ (cd)
3
+ab +cd + 1 = (1 + c)(1 + d) (1 + a)(1 + b) + ab + cd + 1
= 1 a b c d = 0 (ng v a +b +c +d = 1)
Bi ton 4. Gi s a
n
l dy s Fibonaxi xc nh bi
_
a
1
= a
2
= 1
a
n+2
= a
n+1
+a
n
(n N)
Chng
minh rng nu a thc P(x) bc 1005 tha iu kin P(k) = a
k
vi k = 1007, . . . , 2012. Th
P(2013) = a
2
013 1.
Gii. Ta chng minh quy np theo n N. Khng nh tng qut :
Nu P(x) c bc n tha P(k) = a
k
vi k = n + 2, . . . , 2n + 2
Th P(2n + 3) = a
2n+3
1.
n = 1 ta c P(3) = 2, P(4) = 3 P(x) x 1 v P(5) = 4 = a
5
1.
Gi s khng nh ng vi n 1. Ta s chng minh n cng ng vi n.
Gi s a thc P(x) bc n tha P(k) = a
k
vi k = n + 2, . . . , 2n + 2.
Xt a thc Q(x) = P(x + 2) P(x + 1) c bc khng vt qu n 1 tha Q(k) = a
k
vi
k = n + 1, ..., 2n
V Q(k) = P(k + 2) P(k + 1) = a
k+2
a
k+1
= a
k
Ngha l Q(2n + 1) = a
2n+1
1 ( theo gi thit quy np)
Nhng Q(2n + 1) = P(2n + 3) P(2n + 2)
P(2n + 3) = P(2n + 2) + Q(2n + 1) = a
2n+2
+a
2n+1
1 = a
2n+3
1.
180
Bi ton 5. Tm s nguyn a sao cho a thc f (x) = x
13
+ x + 90 chia ht cho a thc
g (x) = x
2
x +a.
Gii. Gi s : f(x) = g(x).Q(x) x
13
+ x + 90 = (x
2
x + a) .Q(x)
V a Z nn ta xt :
a 0 khi f (x) = x
13
+x +90 v g (x) = x
2
x +a c nghim khng m. V l a > 0, cho
x = 1, x = 0, x = 1 ta c
_
_
_
(a + 2) .Q(1) = 88
a.Q(0) = 90
a.Q(1) = 92
2
.
.
.a
_
a = 1
a = 2
Nu a = 1 th t (1) suy ra 88
.
.
.3 v l
Nu a = 2 th ta c
f(x) = x
2
x +a
_
. x
11
+x
10
x
9
3x
8
x
7
+ 5x
6
+ 7x
5
3x
4
7x
3
+ 11x
2
+ 23x + 45
_
Vy a = 2 th f(x) chia ht cho g(x)
Bi ton 6. Lp a thc bc 3 c cc nghim x
1
, x
2
, x
3
tha :
_

_
1
x
1
+
1
x
2
+
1
x
3
= 2 (1)
1
x
2
1
+
1
x
2
2
+
1
x
2
3
= 1 (2)
1
x
4
1
+
1
x
4
2
+
1
x
4
3
= 1 (3)
Gii. Gi s a thc cn tm c dng P(x) = x
3
+ ax
2
+bx +c.
Theo nh l VIET :
_
_
_
x
1
+ x
2
+x
3
= a
x
1
x
2
+x
2
x
3
+ x
3
x
1
= b
x
1
x
2
x
3
= c
(1) 2 =
x
1
x
2
+x
2
x
3
+ x
3
x
1
x
1
x
2
x
3
=
b
c
b = 2c
(2) 1 =
x
2
1
x
2
2
+x
2
2
x
2
3
+ x
2
3
x
2
1
x
2
1
x
2
2
x
2
3
=
(x
1
x
2
+x
2
x
3
+x
3
x
1
)
2
2x
1
x
2
x
3
(x
1
+x
2
+x
3
)
x
2
1
x
2
2
x
2
3
=
b
2
2ac
c
2
b
2
2ac = c
2
(3) 1 =
x
4
1
x
4
2
+x
4
2
x
4
3
+x
4
3
x
4
1
x
4
1
x
4
2
x
4
3
= =
(b
2
2ac)
2
2c
2
(a
2
2b)
c
4
(b
2
2ac)
2
2c
2
(a
2
2b) = c
4
Vy a, b, c tha
_
_
_
b = 2c
b
2
2ac =c
2
(b
2
2ac)
2
2c
2
(a
2
2b) = c
4

_
_
_
a =
8
3
b =
32
9
c =
16
9
Bi ton 7. Tm tt c cc gi tr ca tham s m cc nghim x
1
, x
2
, x
3
, x
4
ca a thc :
P(x) = x
4
+ 3x
3
+ 6x
2
+ mx + 4 tha x
1
=
1
x
2
+
1
x
3
+
1
x
4
.
181
Gii. Ta c
_

_

_
x
1
+x
2
+ x
3
+x
4
= 3
x
1
x
2
+ x
1
x
3
+x
1
x
4
+x
2
x
3
+x
2
x
4
+ x
3
x
4
= 6
x
1
x
2
x
3
+x
1
x
2
x
4
+x
1
x
3
x
4
+x
2
x
3
x
4
= m
x
1
x
2
x
3
x
4
= 4
x
2
x
3
+x
2
x
4
+x
3
x
4
= x
1
x
2
x
3
x
4
Suy ra 4 = x
1
x
2
x
3
x
4
= x
2
x
3
+x
2
x
4
+x
3
x
4
= 6x
1
(x
2
+ x
3
+x
4
) = 6x
1
(3 x
1
) = 6+3x
1
+x
2
1
Hay x
2
1
= 3x
1
2(*)
x
3
1
= x
1
x
2
1
= x
1
(3x
1
2) = 3x
2
1
2x
1
= 3 (3x
1
2) 2x
1
= 7x
1
+ 6
x
4
1
= x
1
x
3
1
= 7x
2
1
+ 6x
1
= 7 (3x
1
2) + 6x
1
= 15x
1
14
Hay (m12) x
1
4 = 0 x
1
=
4
m12
(m = 12) .
Thay x
1
=
4
m12
vo (*) ta c :
16
(m12)
2
+ 3
4
m12
+ 2 = 0(**)
Gii phng trnh (**) tm c m = 8 hoc m = 10.
Bi ton 8. Cho a, b R. Tm a thc P(x) tho : xP(x a) = (x b)P(x), x R
Gii.
Xt a = b = 0 P(x) tu .
a = 0, b = 0 th P(x) = 0, x R.
a = 0, b = 0 th P(x) = const
a = 0 v b = 0 th xt 2 trng hp
b
a
/ N v
b
a
N.
Nu
b
a
/ N khi thay x = b th x = b a l nghim
Tng t khi thay x = b a th x = b 2a l nghim, . . .
Do P(x) = x, x R.
Nu
b
a
N th P(x) c x = a, x = 2a, . . . , x = (n 1)a l nghim
Do P(x) = (x a)(x 2a) . . . (x (n 1)a).Q(x)
Th vo iu kin bi ra ta c Q(x a) = Q(x), x R hay Q(x) = const
Vy P(x) = (x a)(x 2a) . . . (x (n 1)a).
Bi ton 9. Hi c tn ti hay khng a thc f(x) bc 2012 sao cho f (x
2
2011) chia ht
cho f(x).
Gii. Xt a thc f (x) = (x +a)
2012
Ta f (x
2
2011) = (x
2
2011 + a)
2012
=
_
(x +a)
2
2a (x +a) + a
2
+a 2011

2012
Nu ta chn a sao cho a
2
+ a 2011 = 0 a =
1

8045
2
. th f (x
2
2011) = (x
2
a
2
)
2012
=
(x a)
2012
(x +a)
2012
chia ht cho f(x).
Vy a thc f (x) =
_
x +
1

8045
2
_
2012
tha iu kin ca bi ton.
Cch 2 : Xt a thc f (x) =
2012

k=1
(x a
k
), a
k
R.
Ta c : f (x
2
2011) =
2012

k=1
(x
2
2011 a
k
)
Th khi f (x
2
2011) =
2012

k=1
(x
2
a
2
k
) = f(x)
2012

k=1
(x +a
k
) chia ht cho f(x).
182
Vy a thc f (x) =
_
x
1

8045
2
_
2012
tha iu kin ca bi ton.
Bi ton 10. Cho 0 1. Chng minh rng : vi mi s phc a phng trnh : z
3
az+a = 0
c t nht 1 nghim z tha : |z | 2 .
Gii. Gi z
1
, z
2
, z
3
l 3 nghim ca pt : z
3
az +a = 0.
Ta c :
_
_
_
z
1
+ z
2
+ z
3
= 0
z
1
z
2
+ z
2
z
3
+z
3
z
1
= a
z
1
z
2
z
3
= a
Suy ra 1 = 1 (z
1
+z
2
+z
3
) + (z
1
z
2
+ z
2
z
3
+z
3
z
1
) z
1
z
2
z
3
= (1 z
1
) (1 z
2
) (1 z
3
)
Hay |1 z
1
| |1 z
2
| |1 z
3
| = 1 z
i
: |z
i
1| 1
Khi 2 = 1 + 1 |z
i
1| +|1 | |z
i
|
Tc l c t nht 1 nghim z ca phng trnh tho : |z | 2 .
Bi ton 11. Cho a, b, c, d, e R. Chng minh rng nu phng trnh ax
2
+(b+c)x+d+e = 0
c nghim x
0
[1; +) th phng trnh ax
4
+bx
3
+cx
2
+ dx + e = 0 cng c nghim thc.
Gii. Ta c ax
2
0
+ cx
0
+e = (bx
0
+d)
Xt f(x) = ax
4
+bx
3
+cx
2
+dx +e suy ra
f

x
0
_
=(ax
2
0
+cx
0
+e) +

x
0
(bx
0
+d);
f

x
0
_
=(ax
2
0
+cx
0
+ e)

x
0
(bx
0
+d)
Hay
f

x
0
_
f

x
0
_
=(ax
2
0
+cx
0
+ e)
2
x
0
(bx
0
+ d)
2
= (ax
2
0
+cx
0
+ e)
2
x
0
(ax
2
0
+ cx
0
+e)
2
= (ax
2
0
+cx
0
+e)
2
(1 x
0
) 0
Vy phng trnh f(x) = 0 c t nht 1 nghim x
_

x
0
;

x
0

Bi ton 12. Cho a thc P(x) vi h s thc bc n (n 1) c m nghim thc. Chng minh
rng : a thc Q(x) = (x
2
+ 1) P(x) + P
,
(x) c t nht m nghim thc
Gii. Xt f(x) = e
x
3
3
+x
P(x)
D thy f(x) = 0 c tp hp tt c cc nghim thc trng vi tp cc nghim thc ca P(x).
Theo nh l Rolle th phng trnh f
,
(x) = e
x
3
3
+x
[P
,
(x) + (x
2
+ 1)P(x)] = 0 c t nht m1
nghim thc.
Nu m chn, n l th P(x) c t nht m+ 1 nghim thc, v l !
Do n chn. Khi P
,
(x) +(x
2
+1)P(x) c bc l n+2 l s chn v c m1 s l nghim
thc
Suy ra a thc Q(x) = (x
2
+ 1) P(x) + P
,
(x) c t nht m nghim thc.
Bi ton 13. Cho f(x) kh vi trn on [0; 1] v tho iu kin :
_
f(0) = 0, f(1) = 1
0 f(x) 1, x R
Chng minh rng tn ti a, b (0; 1) : f

(a).f

(b) = 1
183
Gii. Xt hm g(x) = f(x) + x 1 th g(x) kh vi trn on [0; 1] do f(x) kh vi trn on
[0; 1]
Do g(0) = 1 v g(1) = 1 nn theo nh l Lagrange c (0; 1) sao cho g(c) = 0
Suy ra f(c) + c 1 = 0 hay f(c) = 1 c
Mt khc theo nh l Lagrange i vi hm f(x) trn cc on [0; c] v [c; 1] ta c :
f(c)f(0)
c0
=
f
,
(a), a (0; c) v
f(1)f(c)
1c
= f
,
(b), b (c; 1)
Suy ra f
,
(a).f
,
(b) =
f(c)
c
.
1f(c)
1c
=
(1c)c
c(1c)
= 1
Bi ton 14. (IMO 2006) Cho P(x) l a thc bc n > 1 vi h s nguyn, gi k l mt s
nguyn dng. Xt a thc Q(x) = P(P(. . . P(x)) . . .)), P xut hin k ln. Chng minh rng
c ti a n s nguyn t sao cho Q(t) = t.
Gii. u tin , ta chng minh vi mi im c nh x ca Q trong thc t l 1 im c nh
ca P P.
Xt dy cho bi x
0
= x v x
i+1
= P (x
i
) vi i 0.
Gi s x
k
= x
0
, ta bit P(u) P(v) chia ht cho uv vi mi s nguyn u, v phn bit . Hn
na
d
i
= x
i+1
x
i
P (x
i+1
) P (x
i
) = x
i+2
x
i+1
= d
i+1
vi mi i , kt hp vi d
k
= d
0
ta suy ra |d
0
| = |d
1
| = ... = |d
k
| .
Gi s d
1
= d
0
= d = 0 th d
2
= d (nu khng th x
3
= x
1
v x
0
khng th xut hin trong dy
mt ln na ). Tng t d
3
= d, . . . , vy x
i
= x
0
+id = x
0
vi mi i, iu ny mu thun .
M d
1
= d
0
, suy ra x
2
= x
0
. Vy c th gi thit rng Q = P P
Nu mi s nguyn t vi P (P (t)) cng tho mn P (t) = t th d dng nhn thy s nghim
ca bi ton ti a l degP = n.
Gi s P (t
1
) = t
2
, P (t
2
) = t
1
, P (t
3
) = t
4
, P (t
4
) = t
3
, vi t
1
= t
2,3,4
khng nht thit t
3
= t
4
)
,v t
2
t
4
.
.
. (t
1
t
3
) v iu ngc li cng ng nn t
1
t
3
= (t
2
t
4
) .Gi s t
1
t
3
= t
2
t
4
hay ni cch khc t
1
t
2
= t
3
t
4
= u = 0. V ta cng c t
1
t
4
= (t
2
t
3
),
nn t
1
t
3
+u = (t
1
t
3
u), iu ny l v l !
Vy phi c t
1
t
3
= t
4
t
2
,hay p (t
1
) + t
1
= p (t
3
) + t
3
= c .Suy ra mi nghim nguyn t ca
phng trnh P (P (t)) = t u tho mn P(t)+, vy s s nguyn khng vt qu n.
Bi ton 15. A1. Tm tt c nhng a thc h s nguyn bc hai sao cho tn ti a thc q(x)
h s nguyn tha l mt a thc c tt c cc h s 1.
Li gii 1: Chng ta chng minh rng ch c nhng a thc tha mn yu cu l nhng a
thc x
2
x 1, x
2
1 v x
2
2x + 1.
Gi f(x) l a thc bc n c tt c cc h s 1. Gi s z l nghim ca phng trnh vi
|z| > 1 vy
|z|
n
=

z
n1
z
n2
... 1

z|
n
1
|z|
n1
+|z|
n2
+... + 1 =
|
|z| 1
184
iu ko theo |z|
n
(|z| 2) 1; vy |z| < 2. Nh th tt c cc nghim ca f(x) = 0 c
gi tr tuyt i b hn 2.
R rng, mt a thc p(x) tha yu cu phi c dng p(x) = x
2
+ax 1 vi a Z. Gi x
1
v
x
2
l cc nghim ca n( khng bt buc phi khc nhau). V x
1
x
2
= 1 nn ta c th gi s
rng |x
1
| 1 v |x
2
| 1. V x
1
v x
2
cn l nghim nguyn ca l a thc c h s 1, nn ta
c |x
1
| < 2. v nh vy |a| = |x
1
+x
2
| |x
1
| +|x
2
| < 2 + 1 a {2, 1, 0}.
Nu a = 1 th g(x) = 1 cho ta li gii.
Nu a = 0 th g(x) = x + 1 cho ta li gii.
Nu a = 2, c hai a thc x
2
2x 1 c nghim vi gi tr tuyt i ln hn 2 nn chng
khng tha yu cu. Cui cng, cc a thc p(x) = x
2
2x + 1 tha yu cu vi q(x) = 1.
Bnh lun: T ng nghim c th c hiu l nghim o v khng cn phi ni r. V
s phc khng cn phi ch r, p(x) = x
2
+ ax 1 c nghim thc nu |a| 2 v trng hp
|a| 1 phi c x l ring.
Ch cn quan tm l cho d c h s bng 0 th kt lun |z| < 2 vn ng. Tuy vy, c nhng
li gii c bit l x
2
v x
2
x
Li gii 2: Gi s a thc p(x) = a
0
+ a
1
x + x
2
v g(x) = b
0
+ b
1
x + ... + b
n
x
n
sao cho
p(x)g(x) = c
0
+c
1
x + ...c
n+2
x
n+2
vi mic
k
= 1.
Khi |a
0
| = |b
0
| = |b
n
| = 1 v a
0
b
1
= c
1
a
1
b
0
; a
0
b
k
= c
k
a
1
b
k1
b
k2
; vi k = 2, . . . , n
V nh vy |b
1
||a
1
| 1; |b
k
||a
1
b
k1
| |b
k2
| 1 vi k = 2, . . . , n
Gi s . Th r rng khng th hng, vi n 1, v ta c |b
1
| 2; |b
k
| 3|b
k1
| |b
k2
| 1; vi
k = 2, . . . , n
T bt ng thc cui( vit li) ta c
|b
k1
| |b
k2
| 2|b
k1
| |b
k2
| 1 2(|b
k1
| |b
k2
|) 1
Ta thy dy d
k
= |b
k
| |b
k1
| , (k = 1, . . . , n), tha d
k
2d
k1
1vik 2( qui np).
V d
1
= |b
1
| 1 1 ko theo gi thit qui np d
k
1 vi k = 1, . . . , n. Tng ng
|b
k
| |b
k1
| + 1vi k = 3, ..., n nn |b
n
| |b
0
| +n tri vi gi thit |b
0
| = |b
n
| = 1, n 1
iu ko theo phi c dng a
0
+a
1
x +x
2
vi |a
0
| = 1; |a
1
| 2.
Nu|a
1
| 1 hay|a
1
| = 2 v a
0
= 1 th a thc g(x) tng ng tn ti (xem 8 v d ca li gii
1)
Cn li trng hp a
0
= 1, a
0
= 1 . Gi s g(x) tn ti v khng mt tnh tng qut ta c th gi
s b
0
= 1 v a
1
= 1( nu b
0
= 1 th thay q(x) bi q(x) v vi a
1
= 2 th thay q(x) bi q(x).
Vi b
0
= 1 , a
0
= 1, a
1
= 2 cng thc quy np u tin thnh b
1
= 2c
1
, b
k
= 2b
k1
+b
k2
c
k
vi k = 2, . . . , n Vy b
1
1, b
2
2b
1
+ 1 c
2
2, v quy np cho thy b
k
2 vi k = 2, . . . , n
li tri vi gi thit |b
0
| = 1. Nh vy khng c a thc p(x) tha bi ton ngoi tr 8 a thc
trnh by trong li gii 1.
2 MT S BI TP T LUYN
Bi 1 Tm tt c cc a thc f(x) vi h s nguyn sao cho vi mi n nguyn dng ta c
f(n) l c ca 2n 1.
185
Hng dn. Nu f(x) l a thc khng hng th tn ti n sao cho |f(n)| > 1. Gi p
l c s nguyn t ca f(n). Ta c p|f(n)|2
n
1. Mt khc p|f(n +p)|2
n+p
1. Suy ra
p|2
n+p
2
n
= 2
n
(2
p
1). Do (2
n
1, 2
n
) = 1 nn t y suy ra p|2
p
1. Nhng theo nh l
Fermat th p|2
p
2. Nh vy t y suy ra p|1. Mu thun. Vy f(x) phi l a thc hng. p
s f(x) 1, f(x) 1.
Bi 2. Chng minh rng a thc P(x) = x
n
+ 29x
n1
+ 2009 vi n l s nguyn dng ln
hn hay bng 2 khng th phn tch thnh tch ca 2 a thc vi h s nguyn c bc ln hn
hay bng 1.
Hng dn. S dng tiu chun Eisenstein m rng nh sau
Cho a thc P(x) = a
n
x
n
+ a
n1
x
n1
+ ... + a
1
x + a
0
Z[x].Gi s tn ti s nguyn t p v
s nguyn dng k tho mn ng thi cc iu kin sau
1) a
n
khng chia ht cho p
2) a
0
chia ht cho p nhng khng chia ht cho p
2
3) a
1
, a
2
, . . . , a
nk
chia ht cho p
Khi , nu P(x) = Q(x).S(x) vi Q(x), S(x) l cc a thc vi h s nguyn th mt trong hai
a thc Q(x), S(x) c bc nh hn k.
Bi 3. Tm tt c cc a thc P(x) tho mn iu kin P
2
(x) P(x
2
) = 2x
4
.
Hng dn. t P(x) = a
n
x
n
+R(x) vi R(x) l a thc bc r < n. Khi
P
2
(x) P(x
2
) = (a
2
n
a
n
)x
2n
+ 2a
n
x
n
R(x) + R
2
(x) R(x
2
). T y suy ra P
2
(x) P(x
2
) c
bc l 2n nu a
n
= 1 v c bc n+r nu a
n
= 1. T suy ra 2 n 4. Hn na, nu
n = 4 th a
n
= 1 v r = 0
n = 3 th a
n
= 1 v r = 1
T y, dng phng php h s bt nh, d dng tm c cc nghim l: x
4
+ 1, x
3
+x, 2x
2
v x
2
.
Bi 4. Tm tt c cc a thc P(x) vi h s thc tho mn iu kin P
2
(x) = P(x
2
)2P(x).
Hng dn. t Q(x) = P(x) + 1 th Q
2
(x) = Q(x
2
). Chng minh Q(x) = x
n
l a thc bc
n duy nht tho mn phng trnh ny. T suy ra nghim ca bi ton l x
n
1 cng cc
a thc ng nht hng s P(x) 0 v P(x) 1.
Ti liu tham kho
[1] Nguyn Vn Mu, A THC I S V PHN THC HU T, Nh Xut Bn Gio
Dc, 2007.
[2] Nguyn Vn Mu(Ch bin), Trnh o Chin, Trn Nam Dng, Nguyn ng Pht, Nh
Xut Bn Gio Dc, 2008.
[3] Nguyn Hu in, A THC V NG DNG,Nh Xut Bn Gio Dc, 2003
[4] Tp ch Ton Hc v Tui Tr.
186
MT S BI TON V CHIA HT I VI CC
A THC I XNG
Nguyn Vn Ngc, Vin Ton hc
Trong ti liu ny gii thiu mt s bi ton v tnh chia ht ca cc a thc i xng v phn
i xng. Mt a thc c gi l i xng, nu gi tr ca n khng thay i khi ta i ch
hai bin bt k v c gi l phn i xng, nu n i du khi ta i ch hai bin bt k.
gii cc bi ton v tnh chia ht gia cc a thc ta thng s dng nh l Bzout, h qu
di y v cc k nng phn tch thnh nhn t.
nh l Bzout. S d trong php chia a thc f(x) cho x a bng f(a).
H qu. a thc f(x) chia ht cho x a khi v ch khi f(a) = 0, tc l, x = a l nghim ca
f(x).
Xt mt s bi ton sau y.
Bi ton 1. Chng minh rng x
2n
+x
n
y
n
+y
2n
chia ht cho x
2
+xy +y
2
khi v ch khi n khng
phi l bi ca 3.
Li gii. S dng cc cng thc
x
3
y
3
= (x y)(x
2
+ xy +y
2
),
x
k
y
k
= (x y)(x
k1
+ x
k2
y + ... + xy
k2
+y
k1
)
d dng thy rng x
3k
y
3k
chia ht cho x
2
+xy +y
2
. Xt cc trng hp sau
1) n = 3k. Ta c
x
2n
+x
n
y
n
+ y
2n
= x
6k
+x
3k
y
3k
+ y
6k
= (x
6k
y
6k
) + (x
3k
y
3k
) + 3y
6k
.
T suy ra x
2n
+x
n
y
n
+ y
2n
khng chia ht cho x
2
+xy +y
2
.
2) n = 3k + 1. Ta c
x
2n
+x
n
y
n
+y
2n
= x
6k+2
+x
3k+1
y
3k+1
+y
6k+2
=
= x
2
(x
6k
y
6k
) + xy
3k+1
(x
3k
y
3k
) + y
6k
(x
2
+xy +y
2
).
Suy ra trong trng hp ny x
2n
+ x
n
y
n
+y
2n
chia ht cho x
2
+xy + y
2
.
3) n = 3k + 2. Ta c
x
2n
+x
n
y
n
+y
2n
= x
6k+4
+x
3k+2
y
3k+2
+y
6k+4
= x
4
(x
6k
y
6k
)+
+x
2
y
3k+2
(x
3k
y
3k
) + y
6k
(x
4
+x
2
y
2
+ y
4
) =
= x
4
(x
6k
y
6k
) + x
2
y
3k+2
(x
3k
y
3k
) + y
6k
(x
2
+x
2
y
2
y
4
)(x
2
xy +y
2
).
Suy ra x
2n
+x
n
y
n
+y
2n
chia ht cho x
2
+xy +y
2
. Vy iu kin cn v x
2n
+x
n
y
n
+y
2n
chia ht cho x
2
+xy +y
2
l n khng phi l bi ca 3.
187
Bi ton 2. Chng minh rng vi mi n Z
+
, a thc x
2n
x
n
y
n
+ y
2n
khng chia ht cho
x
2
+ xy +y
2
.
Li gii. Gi s x
2n
x
n
y
n
+y
2n
chia ht cho x
2
+xy + y
2
, tc l
x
2n
x
n
y
n
+y
2n
= (x
2
+ xy +y
2
)q(x, y),
trong q(x, y) l a thc i xng vi h s nguyn (do h s chnh ca a thc chia bng 1,
cn cc h s ca a thc b chia v a thc chia l cc s nguyn). Trong ng thc trn cho
x = y = 1, ta c 1 = 3q(1, 1), v l v q(1, 1) l mt s nguyn. iu ny chng t a thc
x
2n
x
n
y
n
+y
2n
khng chia ht cho x
2
+xy +y
2
.
Bi ton 3. Vi nhng n Z
+
no, th x
2n
+x
n
y
n
+y
2n
chia ht cho x
2
xy +y
2
?
Li gii. Gi s
x
2n
+x
n
y
n
+ y
2n
= (x
2
xy +y
2
)q(x, y), (1)
trong q(x, y) l a thc i xng vi h s nguyn.
Ta xt hai trng hp:
1) n l s l. Trong ng thc (1) thay x bi x ta c
x
2n
x
n
y
n
+y
2n
= (x
2
+xy +y
2
)q(x, y).
Theo Bi ton 2 ng thc ny khng th xy ra.
2) n l s chn. Trong (7) thay x bi x, ta c
x
2n
+ x
n
y
n
+y
2n
= (x
2
+xy +y
2
)q(x, y).
Theo Bi ton 1 th ng thc trn ng khi v ch khi n = 3m+ 1 hoc n = 3m+ 2.
Nu n = 3m+ 1 , th do n l s chn, nn m phi l s l, hay m = 2k + 1, do n = 6k + 4.
Nu n = 3k + 2, th do n l s chn, nn m phi l s chn, hay m = 2k, do n = 6k + 2.
Vy x
2n
+ x
n
y
n
+ y
2n
chia ht cho x
2
xy + y
2
khi v ch khi n = 6k + 2 hoc n = 6k + 4,
vi k Z, n Z
+
.
Bi ton 4. Vi n Z
+
no th x
2n
x
n
y
n
+y
2n
chia ht cho x
2
xy +y
2
?
Li gii. Gi s
x
2n
x
n
y
n
+y
2n
= (x
2
xy +y
2
)q(x, y), (2)
trong q(x, y) l a thc i xng vi h s nguyn. Xt hai trng hp ca n.
1) n l s chn. Trong (2) thay x bi x, ta c
x
2n
x
n
y
n
+y
2n
= (x
2
+xy +y
2
)q(x, y).
Theo Bi ton 2 ng thc ny khng th xy ra.
2) n l s l. Trong (2) thay x bi x, ta c
x
2n
+ x
n
y
n
+y
2n
= (x
2
+xy +y
2
)q(x, y).
Theo Bi ton 1 th ng thc trn ng khi v ch khi n = 3m+ 1 hoc n = 3m+ 2.
Nu n = 3m+ 1, th do n l s l, nn m phi l s chn, tc l m = 2k v khi n = 6k + 1.
Nu n = 3m+ 2, th do n l s l, nn m phi l s l, tc l m = 2k 1. Khi n = 6k 1.
Vy x
2n
x
n
y
n
+y
2n
chia ht cho x
2
xy + y
2
khi v ch khi n = 6k 1, k Z, n Z
+
.
188
Bi ton 5. Xc nh n (x +y)
n
+x
n
+ y
n
chia ht cho x
2
+xy +y
2
Li gii. Gi s (x +y)
n
+x
n
+y
n
chia ht cho x
2
+xy + y
2
. Khi ta c
(x +y)
n
+x
n
+ y
n
= (x
2
+ xy +y
2
)q(x, y), (3)
trong q(x, y) l a thc i xng vi h s nguyn. Trong (3) thay x, y tng ng bi x
2
, y
2
,
ta c
(x
2
+y
2
)
n
+ x
2n
+y
2n
= (x
4
+x
2
y
2
+ y
4
)q(x
2
, y
2
),
= (x
2
+xy + y
2
)(x
2
xy +y
2
)q(x
2
, y
2
). (4)
ng thc (4) chng t (x
2
+y
2
)
n
+x
2n
+y
2n
phi chia ht cho x
2
xy + y
2
. Ta c
(x
2
+ y
2
)
n
(xy)
n
= (x
2
+ y
2
xy)[(x
2
+y
2
)
n1
+ (x
2
+y
2
)
n2
+...
+(x
2
+y
2
)(xy)
n2
+ (xy)
n1
]. (5)
Tip theo ta c
(x
2
+y
2
)
n
+x
2n
+y
2n
= [(x
2
+y
2
)
n
(xy)
n
] + (x
2n
+ x
n
y
n
+y
2n
). (6)
T (5) v (6) suy ra (x
2
+y
2
)
n
+x
2n
+y
2n
chia ht cho x
2
xy+y
2
khi v ch khi x
2n
+x
n
y
n
+y
2n
chia ht cho x
2
xy + y
2
. Theo Bi ton 3 iu ny c c khi v ch khi n = 6k + 2 hoc
n = 6k + 4 vi k Z, n Z
+
.
Ngc li, gi thit rng n = 2m, vi m = 3k + 1 hoc m = 3k + 2. Th th
(x +y)
n
+ x
n
+y
n
= (x + y)
2m
+ x
2m
+ y
2m
=
[(x +y)
2m
(xy)
m
] + (x
2m
+ x
m
y
m
+ y
2m
).
rng
(x +y)
2m
(xy)
m
= [(x +y)
2
]
m
(xy)
m
] =
[(x +y)
2
xy]p(x, y) = (x
2
+xy +y
2
)p(x, y),
trong p(x, y) l a thc i xng vi h s nguyn. Do (x + y)
2m
(xy)
m
chia ht cho
x
2
+ xy +y
2
.
Mt khc, v m = 3k +1, m = 3k +2 nn theo Bi ton 1, a thc x
2m
+x
m
y
m
+y
2m
chia
ht cho x
2
+xy +y
2
.
Kt lun: a thc (x + y)
n
+ x
n
+ y
n
chia ht cho x
2
+ xy + y
2
khi v ch khi n = 6k + 2
hoc n = 6k + 4, vi k Z, n Z
+
.
Bi ton 6. Chng minh rng x
2n
+ x
n
y
n
+ y
2n
chia ht cho x
2
+ xy + y
2
khi v ch khi m
khng phi l bi ca 3.
189
Li gii. S dng cc cng thc
x
3
y
3
= (x y)(x
2
+ xy +y
2
),
x
k
y
k
= (x y)(x
k1
+ x
k2
y + ... + xy
k2
+y
k1
)
d dng thy rng x
3k
y
3k
chia ht cho x
2
+xy +y
2
. Xt cc trng hp sau
1) n = 3k. Ta c
x
2n
+x
n
y
n
+ y
2n
= x
6k
+x
3k
y
3k
+ y
6k
= (x
6k
y
6k
) + (x
3k
y
3k
) + 3y
6k
.
T suy ra x
2n
+x
n
y
n
+ y
2n
khng chia ht cho x
2
+xy +y
2
.
2) n = 3k + 1. Ta c
x
2n
+x
n
y
n
+y
2n
= x
6k+2
+x
3k+1
y
3k+1
+y
6k+2
=
= x
2
(x
6k
y
6k
) + xy
3k+1
(x
3k
y
3k
) + y
6k
(x
2
+xy +y
2
).
Suy ra trong trng hp ny x
2n
+ x
n
y
n
+y
2n
chia ht cho x
2
+xy + y
2
.
3) n = 3k + 2. Ta c
x
2n
+x
n
y
n
+y
2n
= x
6k+4
+x
3k+2
y
3k+2
+y
6k+4
= x
4
(x
6k
y
6k
)+
+x
2
y
3k+2
(x
3k
y
3k
) + y
6k
(x
4
+x
2
y
2
+ y
4
) =
= x
4
(x
6k
y
6k
) + x
2
y
3k+2
(x
3k
y
3k
) + y
6k
(x
2
+x
2
y
2
y
4
)(x
2
xy +y
2
).
Suy ra x
2n
+x
n
y
n
+y
2n
chia ht cho x
2
+xy +y
2
. Vy iu kin cn v x
2n
+x
n
y
n
+y
2n
chia ht cho x
2
+xy +y
2
l n khng phi l bi ca 3.
Bi ton 7. Chng minh rng vi mi n Z
+
, a thc x
2n
x
n
y
n
+ y
2n
khng chia ht cho
x
2
+xy + y
2
.
Li gii. Gi s x
2n
x
n
y
n
+y
2n
chia ht cho x
2
+xy +y
2
, tc l
x
2n
x
n
y
n
+ y
2n
= (x
2
+xy + y
2
)q(x, y),
trong q(x, y) l a thc i xng vi h s nguyn (do h s chnh ca a thc chia bng 1,
cn cc h s ca a thc b chia v a thc chia l cc s nguyn). Trong ng thc trn cho
x = y = 1, ta c 1 = 3q(1, 1), v l v q(1, 1) l mt s nguyn. iu ny chng t a thc
x
2n
x
n
y
n
+ y
2n
khng chia ht cho x
2
+xy + y
2
.
Bi ton 8. Vi n Z
+
no th x
2n
+ x
n
y
n
+y
2n
chia ht cho x
2
xy + y
2
?
Li gii. Gi s
x
2n
+ x
n
y
n
+y
2n
= (x
2
xy + y
2
)q(x, y), (7)
trong q(x, y) l a thc i xng vi h s nguyn.
190
Ta xt hai trng hp:
1) n l s l. Trong ng thc (7) thay x bi x ta c
x
2n
x
n
y
n
+y
2n
= (x
2
+xy +y
2
)q(x, y).
Theo Bi ton 2 ng thc ny khng th xy ra.
2) n l s chn. Trong (7) thay x bi x, ta c
x
2n
+x
n
y
n
+y
2n
= (x
2
+xy +y
2
)q(x, y).
Theo Bi ton 1 th ng thc trn ng khi v ch khi n = 3m+ 1 hoc n = 3m+ 2.
Nu n = 3m+ 1 , th do n l s chn, nn m phi l s l, hay m = 2k + 1, do n = 6k + 4.
Nu n = 3k + 2, th do n l s chn, nn m phi l s chn, hay m = 2k, do n = 6k + 2.
Vy x
2n
+ x
n
y
n
+ y
2n
chia ht cho x
2
xy + y
2
khi v ch khi n = 6k + 2 hoc n = 6k + 4,
vi k Z, n Z
+
.
Bi ton 9. Vi n Z
+
no th x
2n
x
n
y
n
+ y
2n
chia ht cho x
2
xy +y
2
?
Li gii. Gi s
x
2n
x
n
y
n
+ y
2n
= (x
2
xy +y
2
)q(x, y), (8)
trong q(x, y) l a thc i xng vi h s nguyn. Xt hai trng hp ca n.
1) n l s chn. Trong (8) thay x bi x, ta c
x
2n
x
n
y
n
+y
2n
= (x
2
+xy +y
2
)q(x, y).
Theo Bi ton 2 ng thc ny khng th xy ra.
2) n l s l. Trong (8) thay x bi x, ta c
x
2n
+ x
n
y
n
+y
2n
= (x
2
+xy +y
2
)q(x, y).
Theo Bi ton 1 th ng thc trn ng khi v ch khi n = 3m+ 1 hoc n = 3m+ 2.
Nu n = 3m+ 1, th do n l s l, nn m phi l s chn, tc l m = 2k v khi n = 6k + 1.
Nu n = 3m+ 2, th do n l s l, nn m phi l s l, tc l m = 2k 1. Khi n = 6k 1.
Vy x
2n
x
n
y
n
+y
2n
chia ht cho x
2
xy + y
2
khi v ch khi n = 6k 1, k Z, n Z
+
.
Bi ton 10. Xc nh n (x +y)
n
+x
n
+ y
n
chia ht cho x
2
+xy +y
2
Li gii. Gi s (x + y)
n
+x
n
+y
n
chia ht cho x
2
+xy + y
2
. Khi ta c
(x +y)
n
+x
n
+ y
n
= (x
2
+ xy +y
2
)q(x, y), (9)
trong q(x, y) l a thc i xng vi h s nguyn. Trong (9) thay x, y tng ng bi x
2
, y
2
,
ta c
(x
2
+y
2
)
n
+ x
2n
+y
2n
= (x
4
+x
2
y
2
+ y
4
)q(x
2
, y
2
),
= (x
2
+xy + y
2
)(x
2
xy +y
2
)q(x
2
, y
2
). (10)
191
ng thc (10) chng t (x
2
+ y
2
)
n
+x
2n
+y
2n
phi chia ht cho x
2
xy +y
2
. Ta c
(x
2
+y
2
)
n
(xy)
n
= (x
2
+y
2
xy)[(x
2
+y
2
)
n1
+ (x
2
+y
2
)
n2
+ ...
+(x
2
+y
2
)(xy)
n2
+ (xy)
n1
]. (11)
Tip theo ta c
(x
2
+y
2
)
n
+ x
2n
+y
2n
= [(x
2
+y
2
)
n
(xy)
n
] + (x
2n
+x
n
y
n
+y
2n
). (12)
T (11) v (12) suy ra (x
2
+y
2
)
n
+x
2n
+y
2n
chia ht cho x
2
xy+y
2
khi v ch khi x
2n
+x
n
y
n
+y
2n
chia ht cho x
2
xy + y
2
. Theo Bi ton 3 iu ny c c khi v ch khi n = 6k + 2 hoc
n = 6k + 4 vi k Z, n Z
+
.
Ngc li, gi thit rng n = 2m, vi m = 3k + 1 hoc m = 3k + 2. Th th
(x +y)
n
+x
n
+y
n
= (x +y)
2m
+x
2m
+y
2m
= [(x +y)
2m
(xy)
m
]+
+(x
2m
+x
m
y
m
+y
2m
).
rng
(x + y)
2m
(xy)
m
= [(x + y)
2
]
m
(xy)
m
] = [(x +y)
2
xy]p(x, y) =
= (x
2
+ xy +y
2
)p(x, y),
trong p(x, y) l a thc i xng vi h s nguyn. Do (x + y)
2m
(xy)
m
chia ht cho
x
2
+xy + y
2
.
Mt khc, v m = 3k +1, m = 3k +2 nn theo v d 1, a thc x
2m
+x
m
y
m
+y
2m
chia ht
cho x
2
+ xy +y
2
.
Kt lun: a thc (x + y)
n
+ x
n
+ y
n
chia ht cho x
2
+ xy + y
2
khi v ch khi n = 6k + 2
hoc n = 6k + 4, vi k Z, n Z
+
.
Bi ton 11. Chng minh rng vi mi s nguyn dng n, a thc
f(x, y, z) = (x +y + z)
2n+1
x
2n+1
y
2n+1
z
2n+1
chia ht cho a thc
g(x, y, z) = (x +y + z)
3
x
3
y
3
z
3
.
Li gii. Trc ht ta phn tch g(x, y, z) thnh nhn t. V khi x = y, x = z, y = z th
g = 0, nn theo nh l Bezout a thc g(x, y, z) chia ht cho (x +y)(x +z)(y +z). Mt khc,
v bc ca g bng 3, nn n c dng
g(x, y, z) = a(x +y)(x +z)(y +z)
Cho x = y = z = 1 ta tm c a = 3. Vy ta c
g(x, y, z) = (x +y + z)
3
x
3
y
3
z
3
= 3(x +y)(x + z)(y + z).
Bng cch tng t ta thy f(x, y, z) chia ht cho (x + y)(x + z)(y + z) vi mi n nguyn
dng, tc l f(x, y, z) chia ht cho g(x, y, z).
192
Bi ton 12. Chng minh rng vi mi s nguyn dng n 2, a thc
f(x, y, z) = (x +y +z)
2n
+x
2n
+y
2n
+z
2n
(x +y)
2n
(y + z)
2n
(z +x)
2n
chia ht cho a thc
g(x, y, z) = (x +y + z)
4
+x
4
+y
4
+z
4
(x +y)
4
(y +z)
4
(z +x)
4
.
Li gii. t

1
= x +y +z,
2
= xy +yz +zx,
3
= xyz, s
k
= x
k
+y
k
+ z
k
.
Khi ta c:
g(x, y, z) =
4
(
4
1
4
2
1

2
+ 2
2
2
+ 4
1

3
)
4(
1

2
2

1

3
2
2
2
) 6(
2
2
2
1

3
) = 12
1

3
= 12(x + y +z)xyz.
Tip theo chng ta bin i f(x, y, z) nh sau
f(x, y, z) = (x +y +z)
4
+ [x
2n
(y +z)
2n
] + [y
2n
(x +z)
2n
] + [z
2n
(x +y)
2n
]
Ta c x
2n
(y +z)
2n
chia ht cho x
2
(y +z)
2
= (x +y +z)(x y z), do x
2n
(y +z)
2n
chia ht cho x + y +z. Tng t, y
2n
(x +z)
2n
v z
2n
(x +y)
2n
chia ht cho x +y + z.
Mt khc
f(0, y, z) = f(x, 0, z) = f(x, y, 0) = 0.
Do theo nh l Bezout, f(x, y, z) chia ht cho xyz. Do , f(x, y, z) chia ht cho (x + y +
z)xyz, suy ra f(x, y, z) chia ht cho g(x, y, z).
Bi ton 13. Chng minh rng, nu a thc i xng f(x, y, z) chia ht cho x y, th n chia
ht cho (x y)
2
(x z)
2
(y z)
2
.
Li gii. Gi s rng
f(x, y, z) = (x y)g(x, y, z).
V
f(x, y, z) = f(y, x, z) = (y x)g(y, x, z) = (x y)g(y, x, z),
nn
g(y, x, z) = g(x, y, z),
suy ra g(x, y, z) l a thc phn i xng theo hai bin x, y. Vy g(x, y, z) chia ht cho x y.
Do f(x, y, z) chia ht cho (x y)
2
. V f(x, y, z) l a thc i xng, nn vai tr ca x, y, z
l nh nhau, cho nn f(x, y, z) cng chia ht cho (x z)
2
v (y z)
2
. Vy f(x, y, z) chia ht
cho (x y)
2
(x z)
2
(y z)
2
.
Bi ton 14. Tm iu kin cn v , a thc x
3
+y
3
+z
3
+kxyz chia ht cho x +y +z.
193
Li gii. Xt a thc theo bin x
f(x) = x
3
+ (kyz)x + (y
3
+ z
3
).
Theo nh l Bezout, f(x) chia ht cho x +y +z = x (y z) khi v ch khi f(y z) = 0
Ta c
f(y z) = (y +z)
3
kyz(y +z) + (y
3
+z
3
) = (k + 3)yz(y +z) = 0, y, z.
T suy ra k = 3.
Vy, iu kin cn v x
3
+ y
3
+ z
3
+ kxyz chia ht cho x + y +z l k = 3.
Bi ton 15. Cho a, b, c l cc s nguyn. Chng minh rng, nu x +y +z chia ht cho 6, th
a
3
+b
3
+c
3
cng chai ht cho 6.
Li gii. Ta c
x
3
+y
3
+z
3
= (x +y + z)(x
2
+y
2
+z
2
xy yz zx) + 3xyz.
Theo gi thit x + y + z chia ht cho 6, nn trong ba s x, y, z phi c t nht mt s chia ht
cho 2. T suy ra 3xyz chia ht cho 6. Vy, theo ng thc trn, th x
3
+y
3
+x
3
chia ht cho
6.
Bi tp
1. Chng minh rng (x +y)
n
x
n
y
n
chia ht cho x
2
+xy + y
2
khi v ch khi n = 6k 1.
2. Chng minh rng (x +y)
2n+1
+x
n+2
y
n+2
chia ht cho x
2
+ xy +y
2
vi mi n Z
+
.
3. Chng minh rng, nu a thc i xng f(x, y) chia ht cho x
2
y
2
, th n chia ht cho
x
3
+ y
3
(x + y)xy.
4. Chng minh rng vi mi s t nhin p, q th a thc
x
p
y
q
+y
p
z
q
+z
p
x
q
x
q
y
p
y
q
z
p
z
q
x
p
chia ht cho (x y)(x z)(y z).
5. Chng minh rng, vi mi cc s t nhin k, m, n, th a thc
x
k
y
m
z
n
+y
k
z
m
x
n
+z
k
x
m
y
n
x
n
y
m
z
k
y
n
z
m
x
k
z
k
x
m
y
k
chia ht cho (x y)(x z)(y z).
6. Cho a thc
f(x, y, z) = x
m
y
n
+y
m
z
n
+z
m
x
n
x
n
y
m
y
n
z
m
z
n
x
m
.
Chng minh rng, nu f(x, y, z) chia ht cho (x + y)(x + z)(y + x), th n chia ht cho (x
2

y
2
)(x
2
z
2
)(y
2
z
2
).
7. Chng minh rng, a thc
a
4
(b
2
+ c
2
a
2
)
3
+b
4
(c
2
+a
2
b
2
)
3
+c
4
(a
2
+b
2
c
2
)
3
chia ht cho a
4
+b
4
+c
4
2a
2
b
2
2a
2
c
2
2b
2
c
2
.
194
NT P HM S TIM N TRONG BI TON
BT NG THC BI TON TM GI TR LN
NHT, GI TR NH NHT
Hunh Duy Thy, Trng THPT Tng Bt H - Bnh nh
Chng minh bt ng thc . . . Tm gi tr nh nht,gi tr ln nht ca biu thc. . . Nhng
cm t y hm cha mt mng kin thc trng tm, hc ba trong chng trnh ton hc
ph thng, m phn nhiu th sinh rt ngi khi va chm. Cn na cng l phn kin
thc lun thi s, cun ht, quyn r ngi hc nht l vi i tng kh, gii.
. . . v nh dng sng no cng bt ngun t nhng con sui nh, mi bi ton d kh n u
cng c ngun gc t nhng bi ton n gin . . . .
Tc gi ca bi vit ny rt mong gp mt cht suy ngh trong vic tm ra con ng n
vi nhng dng sui nh kia.
Bi vit c trnh by theo hng gii quyt nhng cu hi kinh in.
- Bt u t u? - Khai thc, khm ph, pht hin v kin to vn ra sao?
- Gii php no l kh thi? . . . . . .
im mu cht trong phng php vn dng tnh cht ca hm s, l xy dng c hm s
tng thch vi bi ton.
1 Loi 1.Chn trc tip mt tham s bin thin lm bin
s.
Bi ton 1 (USAMO). Cho a, b, c l cc s thc thuc on [0; 1]
Chng minh rng:
a
b+c+1
+
b
c+a+1
+
c
a+b+1
+ (1 a)(1 b)(1 c) 1(1)
Li gii. - Vai tr ca cc s a, b, c l nh nhau khng mt tnh toang quat, ta chn a l
bin s. Nh vy ta hnh thnh c hm s f bin s a.
y = f(a) vi a [0; 1]
Xt hm s f(a) =
a
b+c+1
+
b
c+a+1
+
c
a+b+1
+ (1 a)(1 b)(1 c) trn tp [0, 1]
f

(a) =
1
b+c+1

b
(c+a+1)
2

c
(a+b+1)
2
(1 b)(1 c)
f

(a) =
2b
(c+a+1)
3
+
2c
(a+b+1)
3
0, a [0; 1]
Suy ra hm s f

(a) ng bin trn on [0; 1].


* Trng hp 1: f

(a) 0, a [0; 1].


Suy ra hm s f(a) ng bin trn on [0; 1]
Do no f(a) f(1) =
1
b+c+1
+
b
c+1+1
+
c
1+b+1

1
b+c+1
+
b
c+b+1
+
c
c+b+1
=
1 + b +c
1 + b +c
= 1
195
* Trng hp 2: f

(a) < 0, a [0; 1]


Suy ra hm s f(a) nghch bin trn on [0; 1] Do :
f(a) f(0) =
b
c + 1
+
c
b + 1
+ (1 b)(1 c)
=
b(b + 1) + c(c + 1) + (1 b)(1 c)(c + 1)(b + 1)
(c + 1)(b + 1)
=
1 + b +c +b
2
c
2
1 + b + c + bc

1 + b +c +bc
1 + b +c +bc
= 1
* Trng hp 3: f

(a) i du trn on [0; 1]


Mt khc f

(a) l hm s lin tc v ng bin trn on [0; 1].


T suy ra phng trnh f

(a) = 0 c nghim duy nht a = , (0; 1).


Lp bng bin thin, ta nhn c f(a) 1.
Du ng thc xy ra ti (a, b, c) l cc hon v ca (1, 1, 0), (1, 0, 0), (1, 1, 1) hoc (0, 0, 0).
Bi ton 2. Cho a, b, c l di 3 cnh ca mt tam gic. Chng minh rng:
a(b c)
2
+b(c a)
2
+c(a b)
2
+ 4abc > a
3
+b
3
+ c
3
(1)
(1)
Li gii. - Khng mt tnh tng qut, gi s 0 < a b c < a + b
Ta c: (1) c
3
(a +b)c
2
(a
2
+b
2
2ab)c + (a
3
+b
3
a
2
b ab
2
) < 0
Xt hm s f(c) = c
3
(a +b)c
2
(a
2
+b
2
2ab)c + (a
3
+b
3
a
2
b ab
2
), c [b, a + b)
f

(c) = 3c
2
2(a +b)c (a b)
2
f

(c) = 0 3c
2
2(a + b)c (a b)
2
= 0
_
_
c
1
=
(a+b)

(a+b)
2
+3(ab)
2
3
c
2
=
(a+b)+

(a+b)
2
+3(ab)
2
3
Nhn xt c
1
0, b < c
2
< a +b
Bng bin thin
V 0 < a b nn f(b) = a
2
(a 2b) < 0. Suy ra f(c) < 0, c [b; a +b).
2 Loi 2: Bin i quy v mt trong cc tham s bin
thin cho trc.
Bi ton 3. : [IM025] Cho 3 s thc dng x, y, z thay i v tha mn h thc x +y +z = 1.
Tm gi tr ln nht ca biu thc P = xy +yz +zx 2xyz.
Li gii. Khng mt tnh tng qut, ta gi s x y z. T gi thit , suy ra 0 < z
1
3
Ta c:
P = xy(12z) + z(1z)
_
x +y
2
_
2
(1 2z) + z(1 z)
=
_
1 z
2
_
2
(1 2z) + z(1 z) =
1
2
z
3
+
1
4
z
2
+
1
4
196
- Xt hm s f(z) =
1
2
z
3
+
1
4
z
2
+
1
4
, z 0;
1
3

(z) =
3
2
z
2
+
1
2
z =
z
2
(3z 1), f

(z) = 0 z =
1
3
(v 0 < z
1
3
)
Lp bng bin thin, ta nhn c P f(z) f(
1
3
) =
7
27
Kt lun: max P =
7
27
t c khi x = y = z =
1
3
.
Bi ton 4. a, b, c l di 3 cnh ca tam gic c chu vi bng 3. Tm gi tr nh nht ca
biu thc:
T = 3(a
2
+b
2
+ c
2
) + 4abc
Li gii. Khng mt tnh tng qut, ta gi s: 0 < a b c
Mt khc a + b + c = 3 v c < a +b nn 1 c <
3
2
Ta bin i:
T = 3(a +b)
2
6ab + 3c
2
+ 4abc = 3(3 c)
2
+ 3c
2
+ 2ab(2c 3)
= 3(3 c)
2
+ 3c
2
2(3 2c)ab 3(3 c)
2
+ 3c
2
2(3 2c)
_
3 c
2
_
2
T c
3

3
2
c
2
+
27
2
Xt hm s: f(c) = c
3

3
2
c
2
+
27
2
vi 1 c <
3
2
, f

(c) = 3c
2
3c
f

(c) = 0 3c(c 1) = 0 c = 1 (V 1 c <


3
2
.)
Lp bng bin thin, ta nhn c: T f(c) f(1) = 13
Kt lun: min T = 13 t ti a = b = c = 1.
3 Loi 3: Hnh thnh bin s mi.
Bi ton 5. Cho cc s thc dng x, y tha mn iu kin x
2
+ y
2
= x
_
1 y
2
+ y

1 x
2
.
Tm gi tr nh nht ca biu thc P = x
2
+
1
x
2
+y
2
+
1
y
2
Li gii. - p dng bt ng thc csi, ta c:
P (x
2
+ y
2
)
_
1 +
4
(x
2
+y
2
)
2
_
= (x
2
+y
2
) +
4
x
2
+y
2
- Theo gi thit, ta c: x
2
+y
2
= x
_
1 y
2
+y

1 x
2
p dng bt ng thc csi, ta c:
x
_
1 y
2
+ y

1 x
2

1
2
(x
2
+ 1 y
2
) +
1
2
(y
2
+ 1 x
2
) = 1
T , ta c 0 < x
2
+ y
2
1
Khi , ta c: P t +
4
t
, t = x
2
+y
2
, 0 < t 1.
Xt hm s: f (t) = t +
4
t
, trn tp (0; 1]
f

(t) = 1
4
t
2
< 0, t (0; 1]
197
Lp bng bin thin ta nhn c P f(1) = 5
Kt lun: min P = 5 t c x = y =

2
2
.
Bi ton 6. Cho cc s thc a, b, c khng m tha mn a +b +c = 3. Tm gi tr ln nht ca
biu thc: P = (a
2
ab +b
2
) (b
2
bc +c
2
) (c
2
ca +a
2
) .
Li gii. Da theo gi thit ta gi s rng: 0 a b c 3.
Suy ra:
_
a
2
ab 0
a
2
ca 0

_
a
2
ab +b
2
b
2
a
2
ca +c
2
c
2
T ta nhn c:
P b
2
c
2
b
2
bc +c
2
_
= b
2
c
2
_
(b +c)
2
3bc

Ta c: b +c 3 2

bc b +c 3 0 bc 9/4
Do : P b
2
c
2
(9 3bc) = 9b
2
c
2
3b
3
c
3
t t = bc, 0 t
9
4
. Khi : P 9t
2
3t
3
Xt hm
s: f(t) = 9t
2
3t
3
, t
_
0,
9
4

(t) = 18t 9t
2
f

(t) = 0 t
1
= 0, t
2
= 2
Lp bng bin thin , ta nhn c P f(t) 12
Kt lun: max P = 12 t ti a = 0, b = 1, c = 2 v cc hon v ca (a, b, c).
4 Loi 4: Chn hm s c trng, trng hp cc bin
phn li
Bi ton 7 (Olimpic Hng Kng). Cho cc s thc dng a, b, c, d tha mn iu kin a +b +
c +d = 1. Chng minh rng:
6 a
3
+ b
3
+c
3
+d
3
_
a
2
+b
2
+ c
2
+d
2
+
1
8
(1)
Li gii. Ta c: (1) 6a
3
a
2
+ 6b
3
b
2
+ 6c
3
c
2
+ 6d
3
d
2

1
8
Xt hm s f(x) = 6x
3
x
2

5
8
x, x (0, 1), f

(x) = 18x
2
2x
5
8
f

(x) = 0 x
1
= 1/4 (0, 1) , x
2
= 5/36 (loi)
Lp bng bin thin , ta nhn c 6x
3
x
2

5
8
x
1
8
() , x (0, 1)
Da theo kt qu (*) ta c: 6a
3
a
2
+6b
3
b
2
+6c
3
c
2
+6d
3
d
2
4
1
8
_
+
5
8
(a +b +c +d)
6 (a
3
+b
3
+ c
3
+d
3
) a
2
+ b
2
+c
2
+d
2
+
1
8
ng thc xy ra khi a = b = c = d =
1
4
.
Bi ton 8. Cho 3 s thc x, y, z thay i v tha mn h thc x +y +z = 1 Tm gi tr nh
nht ca biu thc P =

x
2
+ 1 +
_
y
2
+ 1 +

z
2
+ 1
Li gii. Xt hm s f(t) =
t+3

t
2
+1
. Tp xc nh D = R
Ta c: f

(t) =
13t

(t
2
+1)
3
, f

(t) = 0 1 3t = 0 t = 1/3
Lp bng bin thin , ta nhn c f (t)

10, t R
t+3

t
2
+1

10, t R t + 3
198

10

t
2
+ 1, t R(1)
- S dng kt qu (1), ta c.
_
_
_
x + 3

10

x
2
+ 1
y + 3

10
_
y
2
+ 1
z + 3

10

z
2
+ 1
Do
(x +y + z) + 9

10
_

x
2
+ 1 +
_
y
2
+ 1 +

z
2
+ 1
_
1 + 9

10
_

x
2
+ 1 +
_
y
2
+ 1 +

z
2
+ 1
_

10

x
2
+ 1 +
_
y
2
+ 1 +

z
2
+ 1
ng thc xy ra khi x = y = z =
1
3
.
Kt lun: min P =

10, t c khi x = y = z =
1
3
.
5 Loi 5: Chn hm s c trng, trng hp cc bin
cha phn li
Bi ton 9. (Darij Grinberg Old and new lnequalityl) Cho cc s thc dng a, b, c. Chng
minh :
a
(b +c)
2
+
b
(c +a)
2
+
c
(a +b)
2

9
4 (a + b + c)
Li gii. Do tnh ng cp, nn ta gi s a +b +c = 3 suy ra a, b, c (0; 3).
Khi bt ng thc cn chng minh tr thnh
a
(3a)
2
+
b
(3b)
2
+
c
(3c)
2

3
4
Xt hm s f(x) =
x
(3x)
2

1
2
x, x (0; 3), f

(x) = 0 x = 1 (0; 3) . Lp bng bin thin ,


ta c
f(x)
1
4
, x (0; 3)
x
(3 x)
2

1
2
x
1
4
() , x (0; 3)
Vn dng bt ng thc (*) ta nhn c.
a
(3a)
2
+
b
(3b)
2
+
c
(3c)
2

1
2
(a + b + c)
3
4

a
(3a)
2
+
b
(3b)
2
+
c
(3c)
2

3
4
Bt ng thc c chng minh du ng thc xy ra khi a = b = c = 1.
Bi ton 10 (USAMO). Cho cc s thc dng a, b, c. Chng minh rng:
(2a +b +c)
2
2a
2
+ (b + c)
2
+
(2b +c +a)
2
2b
2
+ (c +a)
2
+
(2c +a +b)
2
2c
2
+ (a +b)
2
8.
Li gii. - Do tnh ng cp nn gi s a + b + c = 3, suy ra a, b, c (0; 3).
Khi bt ng thc cn chng minh c vit thnh:
a
2
+ 6a + 9
3a
2
6a + 9
+
b
2
+ 6b + 9
3b
2
6b + 9
+
c
2
+ 6c + 9
3c
2
6c + 9
8
199
Xt hm s f(x) =
x
2
+6x+9
3x
2
6x+9

4
3
x, x (0; 3), f

(x) = 0 x = 1 (0, 3)
Lp bng bin thin , ta c
x
2
+6x+9
3x
2
6x+9

4
3
x
4
3
, x (0; 3) ()
Vn dng bt ng thc (*) ta nhn c.
a
2
+ 6a + 9
3a
2
6a + 9
+
b
2
+ 6b + 9
3b
2
6b + 9
+
c
2
+ 6c + 9
3c
2
6c + 9

4
3
(a +b +c) + 4 = 8
Bt ng c chng minh, du ng thc xy ra khi a = b = c = 1.
6 Loi 6:Chn hm c trng, s dng h thc ph tr.
Bi ton 11. Cho cc s thc x, y, z thuc on [0; 3]. Tm gi tr nh nht ca biu thc.
P =
1
4 + 2 ln(1 + x) y
+
1
4 + 2 ln(1 + y) z
+
1
4 + 2 ln(1 + z) x
Li gii. - Tht vy vi 2 ln vn dng bt ng thc c si, ta nh gi v phn li c:
P
9
12 + [2 ln(1 + x) x] + [2 ln(1 + y) y] + [2 ln(1 + z) z]
. - Lc ny cc s hng v phi c sp xp theo cng mt quy lut, iu ny lm h l
hm s c trng: f(t) = 2 ln(1 + t)t, t [0; 3].
T gi thit ta c:
_
_
_
4 + 2 ln(1 + x) y > 0
4 + 2 ln(1 + y) z > 0
4 + 2 ln(1 + z) x > 0
S dng bt ng thc c si, ta c:
[4 + 2 ln(1 + x) y] + [4 + 2 ln(1 + y) z] + [4 + 2 ln(1 +z) x]
3
3
_
[4 + 2 ln(1 + x) y] [4 + 2 ln(1 +y) z] [4 + 2 ln(1 +z) x]
- Li p dng bt ng thc c si, ta c.
P 3
3

1
[4 + 2 ln(1 + x) y] [4 + 2 ln(1 +y) z] [4 + 2 ln(1 +z) x]
Nhn v theo v 2 bt ng thc trn, ta c.
[4 + 2 ln(1 + x) y + 4 + 2 ln(1 + y) z + 4 + 2 ln(1 + z) x] .P 9
P
9
12 + [2 ln(1 + x) x] + [2 ln(1 + y) y] + [2 ln(1 + z) z]
Xt hm s:
f(t) = 2 ln(1 + t)t, t [0; 3], f

(t) =
2
1 + t
1 =
1 t
1 + t
f

(t) = 0 t = 1
200
Lp bng bin thin , ta nhn c : 0 f(t) 2 ln 2 1, t [0; 3]
S dng kt qu trn, ta c:
0 [2 ln(1 +x) x] + [2 ln(1 + y) y] + [2 ln(1 + z) z] 6 ln 2 3
Do : P
9
12+(6 ln 23)
=
9
9+6 ln 2
=
3
3+2 ln 2
ng thc xy ra khi: x = y = z = 1.
Kt lun: min P =
3
3+2 ln 2
t c ti x = y = z = 1.
Bi ton 12. Cho A, B, C l cc gc nhn trong tam gic ABC. Tm gi tr nh nht ca biu
thc
T = tan A + tan B + tan C +
1
cot
A
2
. cot
B
2
. cot
C
2
Li gii. Trc ht ta chng minh. tanA + tanB + tanC cot
A
2
+ cot
B
2
+ cot
C
2
Mt khc:
cot
A
2
+ cot
B
2
+ cot
C
2
=
1
tan
A
2
+
1
tan
B
2
+
1
tan
C
2
=
tan
B
2
. tan
C
2
+ tan
C
2
. tan
A
2
+ tan
A
2
. tan
B
2
tan
A
2
. tan
B
2
. tan
C
2
=
1
tan
A
2
. tan
B
2
. tan
C
2
= cot
A
2
.cot
B
2
. cot
C
2
Do : tanA + tanB + tanC cot
A
2
.cot
B
2
.cot
C
2
.
T , ta nhn c.
T cot
A
2
.cot
B
2
.cot
C
2
+
1
cot
A
2
. cot
B
2
. cot
C
2
t t = cot
A
2
.cot
B
2
.cot
C
2
= cot
A
2
+cot
B
2
+ cot
C
2
3
3
_
cot
A
2
. cot
B
2
. cot
C
2
.
Suy ra: cot
A
2
.cot
B
2
. cot
C
2
3

3.
Xt hm s: f(t) = t +
1
t
vi t 3

3
f

(t) = 1
1
t
2
=
t
2
1
t
2
> 0, t 3

3,
Lp bng bin thin , ta nhn c: T f(t)
28
3

3
, t 3

3
Kt lun: min T =
28
3

3
t c khi A = B = C =

3
.
7 Loi 7: Kho st hm s pht sinh.
Bi ton 13. Cho x, y, z l cc s thc thuc on [0; 1]. Chng minh rng: 2 (x
3
+y
3
+ z
3
)
(x
2
y + y
2
z +z
2
x) 3 (1)
201
Li gii. Xt hm s f(x) = 2x
3
yx
2
z
2
x + 2 (y
3
+z
3
) y
2
z trn tp [0; 1].
f

(x) = 6x
2
2yx z
2
f

(x) = 0 x =
1
6
_
y
_
y
2
+ 6z
2
_
Kho st hm s f(x), ta nhn c. Max
x[0,1]
f(x) = Max {f(0), f(1)}
Nhn xt: f(0) = 2 (y
3
+z
3
) y
2
z 2 (y
3
+z
3
) y
2
z + (2 y z
2
) = f(1)
Nh vy ta cn phi chng minh. f(1) 3
Tht vy: t
f(1) = g(y) = 2 y
3
+z
3
_
y
2
z + 2 y z
2
_
= 2y
3
zy
2
y + 2z
3
z
2
+ 2
g

(y) = 6y
2
2zy1, g

(y) = 0 y =
1
6
z

z
2
+ 6
_
Kho st hm s g(y) trn on [0; 1], ta nhn c. Max
y[0,1]
g(y) = Max {g(0), g(1)}
Ta li c
g(0) = 2z
3
z
2
+ 2 2z
3
z
2
+ 2 + (1 z) = g(1)
= z (z 1) (2z + 1) + 3 3, z [0; 1]
Bt ng thc (1) c chng minh.
Bi ton 14. Cho a, b, c l cc s thc thuc on [0; 1]. Tm gi tr ln nht ca biu thc.
P =
a
b
3
+c
3
+6
+
b
c
3
+a
3
+6
+
c
a
3
+b
3
+6
.
Li gii. Xt hm s [f(c) =
a
b
3
+c
3
+6
+
b
c
3
+a
3
+6
+
c
a
3
+b
3
+6
trn on [0; 1].
f

(c) =
3ac
2
(6+b
3
+c
3
)
2

3bc
2
(c
3
+a
3
+6)
2
+
1
a
3
+b
3
+6
f

(c) =
6ca(6+b
3
2c
3
)
(6+b
3
+c
3
)
3

6bc(6+a
3
2c
3
)
(6+c
3
+a
3
)
3
0, c [0; 1]
Suy ra hm s f

(c) nghch bin trn on [0; 1].


Do f

(c) f

(1) =
1
6+a
3
+b
3

3a
(7+b
3
)
2

3b
(7+a
3
)
2

1
8
2.
3
49
> 0
Suy ra hm s f(c) ng bin trn on [0; 1].
Do f(c) f(1) =
a
b
3
+7
+
b
a
3
+7
+
1
a
3
+b
3
+6
Ta li xt hm s. g(a) =
a
b
3
+7
+
b
a
3
+7
+
1
a
3
+b
3
+6
trn on [0; 1].
Kho st hm s g(a) trn on [0; 1], ta nhn c. P g(a) g(1) =
2
b
3
+7
+
b
8
Ta li xt hm s h(b) =
2
b
3
+7
+
b
8
trn on [0; 1].
Kho st hm s h(b) ta nhn c h(b) ng bin trn on [0; 1].
Suy ra P h(b) h(1) =
3
8
Kt lun: max P =
3
8
t c khi a = b = c = 1.
8 Mt s bi tp
Bi ton 15. Cho cc s thc dng a, b, c tha mn iu kin a
2
+ b
2
+ c
2
= 3. Chng minh
1
2a
+
1
2b
+
1
2c
3
202
Bi ton 16. Cho cc s thc khng m a, b, c tha mn iu kin a +b +c 3. Chng minh
rng:
1
a
2
+b+c
+
1
b
2
+c+a
+
1
c
2
+a+b
1
Bi ton 17. Cho cc s thc dng a, b, c tha mn iu kin a + b + c = 3 Chng minh
1
a
2
+
1
b
2
+
1
c
2
a
2
+b
2
+ c
2
.
Bi ton 18. Cho cc s thc khng m a, b, c tha mn iu kin a +b +c = 1. Chng minh
rng: 10 (a
3
+b
3
+ c
3
) 9 (a
5
+b
5
+ c
5
) 1
Bi ton 19. Cho tam gic nhn ABC. Chng minh: tan A+tan B+tan C+6(sin A+sin B+
sin C) 12

3.
Bi ton 20. Cho cc s thc a, b, c tha mn iu kin a + b + c = 1. Chng minh:
a
1+a
2
+
b
1+b
2
+
c
1+c
2

9
10
Bi ton 21. Tm gi tr ln nht v gi tr nh nht ca hm s: y =
3+2 sin x

1+cos x
+

1 cos x
trn on
_

2
;

2

Bi ton 22. Xt cc tam gic ABC vi 3 gc nh u nhn. Tm gi tr nh nht ca biu


thc.
P =
sin A + sin B + sin C
cos A + cos B + cos C
Bi ton 23. Xt tam gic ABC tm gi tr nh nht ca biu thc.
P = cos A + cos B + cos C +
4
sin
A
2
. sin
B
2
. sin
C
2
Bi ton 24. Tm gi tr ln nht v gi tr nh nht ca biu thc: P = x
4
+y
4
x
2
y
2
Trong
: x, y l cc s thc tha mn iu x
2
+y
2
xy = 1.
203
THM MT PHNG PHP MI CHNG
MINH BT NG THC
Nguyn Ti Chung, Trng THPT Chuyn Hng Vng - Gia Lai
1 Thm mt phng php mi chng minh bt ng
thc
Chng ta u bit rng th gii Bt ng thc v cng rng ln v phong ph, l lnh vc pht
trin nht ca Ton s cp. Do vy cc phng php v k thut chng minh bt ng thc
ngy cng nhiu, v c nhiu phng php mi. Theo di cc ti liu trn Internet ta thy trong
thi gian gn y Bt ng thc c quan tm, trao i, tho lun v pht trin rt nhiu,
xut hin nhiu cao th v bt ng thc l cc Hc sinh, Sinh vin ngi Vit Nam, theo
c mt s phng php mi tn cng cc bi ton v Bt ng thc. Trong qu trnh
ging dy Bt ng thc cho cc em hc sinh gii, ti thy c mt phng php mi l v rt
c o l s dng Dy s chng minh Bt ng thc. Vn ny cng kh rng ln,
phi k n nh dng Gii hn dy s ch ra hng s tt nht lm cho Bt ng thc ng,
tm tt c cc gi tr ca tham s Bt ng thc ng, Bt ng thc trong Dy s, dng
Gii hn dy s chng minh Bt ng thc ba bin i xng. Bi vit ny cp n k
thut dng Gii hn dy s chng minh mt s Bt ng thc kh dng ba bin i xng,
y l vn mi m nn cc v d m ti su tm c cn t. Hy vng rng sau khi c bi
vit, bn c s pht trin thm k thut ny v su tm, sng to thm nhiu bi ton gii
c bng k thut ny.
1. Mt s lu v phng php. Tt c cc bt ng thc i xng ba bin s u c th
quy v cc hm i xng c bn ca
p = x +y +z, q = xy +yz + zx, r = xyz.
Sau khi vit bt ng thc cn chng minh theo p, q, r ta ch cn kho st bt ng thc
ny theo ba bin mi p, q, r.
C th thy ngay li ch ca phng php ny l mi rng buc gia cc bin p, q, r m cc
bin dng x, y, z ban u khng c nh
p
2
3r, p
3
27r, q
2
3pr, pq 9r...
204
Mt s biu din theo p, q, r :
x
2
+y
2
+z
2
= p
2
2q
x
3
+y
3
+z
3
= p
3
3pq + 3r
x
4
+y
4
+z
4
= p
2
2q
_
2
2 q
2
2pr
_
(x +y) (y +z) (z + x) = pq r
xy (x +y) + yz (y +z) + zx (z +x) = pq 3r
x
2
y
2
+y
2
z
2
+z
2
x
2
= q
2
2pr
x
3
y
3
+y
3
z
3
+z
3
x
3
= q
3
3pqr + 3r
2
x
4
y
4
+y
4
z
4
+z
4
x
4
= q
4
4pq
2
r + 2p
2
r
2
+ 4qr
2
.
Vic chng minh cc cng thc trn l n gin. Sau y l mt vi chng minh .
Ta c
x
4
+y
4
+z
4
= x
2
+y
2
+z
2
_
2
2 x
2
y
2
+y
2
z
2
+z
2
x
2
_
= p
2
2q
_
2
2
_
(xy +yz +zx)
2
2xyz (x +y +z)

= p
2
2q
_
2
2 q
2
2pr
_
.
Ta c
(x +y) (y +z) (z + x) = (p x) (p y) (p z)
=p
3
(x +y +z) p
2
+ (xy +yz + zx) p xyz
=p
3
p
3
+pq r = pq r.
Ta c
xy (x +y) + yz (y +z) + zx(z +x)
=xy (p z) + yz (p x) + zx (p y)
=p (xy +yz + zx) 3xyz = pq 3r.
Do hng ng thc
A
3
+ B
3
+C
3
3ABC = (A +B +C) A
2
+ B
2
+C
2
AB BC CA
_
nn
x
3
y
3
+y
3
z
3
+z
3
x
3
=3x
2
y
2
z
2
+ (xy +yz + zx)
_
x
2
y
2
+ y
2
z
2
+ z
2
x
2
xyz (x +y +z)

=3r
2
+ q q
2
2pr rp
_
= q
3
3pqr + 3r
2
.
Bt ng thc Schur : Nu x, y, z l cc s thc dng v t l mt s thc dng, th
x
t
(x y) (x z) + y
t
(y z) (y x) + z
t
(z y) (z x) 0. (1)
205
Chng minh. Khng mt tnh tng qut, gi s x y z > 0. Gi P l v tri ca (1). Khi

P x
t
(x y) (x z) + y
t
(y z) (y x)
= (x y)
_
x
t
(x z) y
t
(y z)

(x y)
_
y
t
(x z) y
t
(y z)

= (x y)
2
y
t
0.
Du bng xy ra khi v ch khi x = y = z hoc x = y v z = 0 cng cc hon v ca n.
Lu . Nu t l s nguyn dng, chng hn t = 1, t = 2 th ch cn iu kin ca x, y, z khng
m l bt ng (1) ng. Bt ng thc (1) cng ng khi t < 0, tht vy, nu t < 0 th vi
gi s x y z > 0, ta c z
t
y
t
, suy ra
P y
t
(y z) (y x) + z
t
(z y) (z x)
= (y z)
_
y
t
(y x) z
t
(z x)

(y z)
_
z
t
(y x) z
t
(z x)

= (y z)
2
z
t
0.
Hai trng hp quen thuc c s dng nhiu l t = 1 v t = 2. Vi x 0, y 0, z 0 ta
c
x (x y) (x z) + y (y z) (y x) + z (z y) (z x) 0. (2)
x
2
(x y) (x z) + y
2
(y z) (y x) + z
2
(z y) (z x) 0. (3)
Cc bt ng thc (2) v (3) cn c vit li l
x
3
+y
3
+z
3
+ 3xyz xy (x +y) + yz (y +z) + zx (z +x) . (4)
x
4
+y
4
+z
4
+xyz (x +y +z) x
3
(y +z) + y
3
(z +x) + z
3
(x +y) . (5)
Cn nu vit theo p, q, r th t (4) v (5) ta c
p
3
4pq + 9r 0, p
4
5p
2
q + 4q
2
+ 6pr 0.
Tht vy, ta c
(4) p
3
3pq + 3r + 3r xy (p z) + yz (p x) + zx(p y)
p
3
3pq + 6r p (xy +yz + zx) 3xyz p
3
4pq + 9r 0
v
(5) p
4
4p
2
q + 2q
2
+ 4pr
_
+pr x
3
(p x) + y
3
(p y) + z
3
(p z)
p
4
4p
2
q + 2q
2
+ 5pr x
3
+y
3
+z
3
_
p x
4
+y
4
+z
4
_
p
4
4p
2
q + 2q
2
+ 5pr p
3
3pq + 3r
_
p p
4
4p
2
q + 2q
2
+ 4pr
_
p
4
5p
2
q + 4q
2
+ 6pr 0.
206
2 Mt s bi ton.
Bi ton 1. Cho ba s dng a, b, c tho mn iu kin a +b +c = 1. Chng minh rng
(1 ab bc ca) (1 27abc)
6 [1 + 3 (ab + bc + ca)]
_
(ab + bc + ca)
2
3abc

.
Gii. t p = a +b +c, q = ab +bc +ca, r = abc. Khi p = 1. Ta cn chng minh
(1 q) (1 27r) 6 (1 + 3q) q
2
3r
_
. (1)
Ta c 0 q = ab +bc +ca
1
3
(a + b +c)
2
=
1
3
. Khi 0 < q
1
4
, ta c
(1 q) (1 27r)
3 (1 27r)
4
. (2)
6 (1 + 3q) q
2
3r
_
6
_
1 +
3
4
__
1
16
3r
_
=
42
4
_
1
16
3r
_
. (3)
V
3 (1 27r)
4

42
4
_
1
16
3r
_
3 81r
42
16
126r 45r
3
8
(ng)
nn t (2) v (3) suy ra (1) ng. Tip theo ta xt
1
4
q
1
3
. Xt dy s (a
n
) nh sau
a
1
=
1
4
; a
n+1
=
1 + a
n
5 3a
n
, n = 1, 2, . . . (4)
Ta c a
1

1
3
. Gi s a
k

1
3
. Xt hm s f(x) =
x + 1
3x + 5
, khi hm f ng bin v
f

(x) =
8
(5 3x)
2
> 0.
Vy a
k+1
= f (a
k
) f
_
1
3
_
=
1
3
. Theo nguyn l quy np, suy ra
0 a
n

1
3
, n = 1, 2, . . .
Do
a
n+1
a
n
=
1 + a
n
5 3a
n
a
n
=
3a
2
n
4a
n
+ 1
5 3a
n
> 0.
Vy dy s (a
n
) tng v b chn trn nn c gii hn hu hn. t lim
n+
a
n
= L. T (4) cho
n +, ta c
L =
1 + L
5 3L
3L
2
4L + 1 = 0
do L
1
3
L =
1
3
.
207
V dy s (a
n
) tng v
1
3
v q
_
1
4
;
1
3
_
, suy ra tn ti k N

sao cho
a
k
q a
k+1
q
1 + a
k
5 3a
k
5q 3qa
k
1 + a
k
3q 3qa
k
+ 1 a
k
2 2q
1
2
(1 a
k
) (3q + 1) 1 q.
T a
k
q
1
3
, ta c
(q a
k
)
_
q
1
3
_
0 q
2

q
3
+qa
k

a
k
3
q
2
q
_
1
3
+ a
k
_

a
k
3
.
Theo bt ng thc Schur, ta c p
3
4pq + 9r 0, m p = 1 nn 4q 1 9r. Do
q
2

1 + 9r
4
_
1
3
+a
k
_

a
k
3
q
2

(9r + 1) (3a
k
+ 1) 4a
k
12
q
2

27ra
k
+ 9r + 3a
k
+ 1 4a
k
12
q
2
3r
27ra
k
27r + 1 a
k
12
q
2
3r
(1 a
k
) (1 27r)
12
.
Vy ta chng minh c
1
2
(1 a
k
) (3q + 1) 1 q. (5)
q
2
3r
(1 a
k
) (1 27r)
12
. (6)
rng 1 a
k
> 0, 3q 1 0, 1 q > 0, q
2
3r 0, 1 27r 0. T (6) suy ra
1 a
k

12 (q
2
3r)
1 27r
Thay vo (5) ta c
1 q
1
2
.
12 (q
2
3r)
1 27r
(3q + 1) 6 q
2
3r
_
(3q + 1) (1 q) (1 27r) .
Php chng minh hon thnh.
Lu . Dy s (a
n
) c tm ra nh sau : Ta cn xy dng dy s (a
n
) tho mn iu kin
a
1
=
1
4
v dy (a
n
) tng v
1
3
. V
1
4
q
1
3
nn tn ti k N

sao cho a
k
q a
k+1
. Ch
rng iu kin a
k
q tng ng vi
(q a
k
)
_
q
1
3
_
0 q
2

q
3
+qa
k

a
k
3
q
2
q
_
1
3
+ a
k
_

a
k
3
.
208
V 4q 9r + 1 nn
q
2

1 + 9r
4
_
1
3
+a
k
_

a
k
3
q
2

(9r + 1) (3a
k
+ 1) 4a
k
12
q
2

27ra
k
+ 9r + 3a
k
+ 1 4a
k
12
q
2
3r
27ra
k
27r + 1 a
k
12
q
2
3r
(1 a
k
) (1 27r)
12
.
Vy chng minh 6 (q
2
3r) (3q + 1) (1 q) (1 27r), ta cn chng minh
1
2
(1 a
k
) (3q + 1) 1 q 5q 3qa
k
1 + a
k
q
1 + a
k
5 3a
k
.
V vy, ta dn n tng l chn lun a
k+1
=
1 + a
k
5 3a
k
v xt dy s (a
n
) nh trn.
Bi ton 2. Xt ba s thc dng a, b, c tho mn a +b +c = 1, chng minh rng
1 + 162a
2
b
2
c
2
27abc + 54 a
3
b
3
+b
3
c
3
+c
3
a
3
_
.
Gii. t p = a +b +c, q = ab +bc +ca, r = abc. Khi p = 1 v
a
3
b
3
+b
3
c
3
+c
3
a
3
= q
3
3pqr + 3r
2
= q
3
3qr + 3r
2
.
Ta cn chng minh
1 + 162r
2
27r + 54 q
3
3qr + 3r
2
_
54q q
2
3r
_
1 27r. (1)
Ta c 0 q = ab +bc +ca
1
3
(a +b +c)
2
=
1
3
. Khi 0 < q
1
4
, ta c
54q q
2
3r
_

27
2
_
1
16
3r
_
(2)
27
2
_
1
16
3r
_
1 27r 27 (1 48r) 32 864r 432r 5 ng
_
. (3)
T (2) v (3) suy ra (1) ng. Tip theo ta xt
1
4
q
1
3
. Xt dy s (a
n
) nh sau
a
1
=
1
4
; a
n+1
=
2
9(1 a
n
)
, n = 1, 2, . . . (4)
Ta c a
1

1
3
. Gi s a
k

1
3
. Xt hm s f(x) =
2
9(1x)
, khi hm f ng bin v f

(x) > 0.
Vy a
k+1
= f (a
k
) f
1
3
_
=
1
3
. Theo nguyn l quy np, suy ra
0 a
n

1
3
, n = 1, 2, . . .
209
Do
a
n+1
a
n
=
2
9 (1 a
n
)
a
n
=
9a
2
n
9a
n
+ 2
9 (1 a
n
)
0.
Vy dy s (a
n
) tng v b chn trn nn c gii hn hu hn. t lim
n+
a
n
= L. T (4) cho
n +, ta c
L =
2
9(1 L)
9L
2
9L + 2 = 0
do L
1
3
L =
1
3
.
V dy s (a
n
) tng v
1
3
v q
_
1
4
;
1
3

, suy ra tn ti k N

sao cho
a
k
q a
k+1
q
2
9 (1 a
k
)
0 1 a
k

2
9q
.
T a
k
q
1
3
, ta c
(q a
k
)
_
q
1
3
_
0 q
2

q
3
+qa
k

a
k
3
q
2
q
_
1
3
+ a
k
_

a
k
3
.
Theo bt ng thc Schur, ta c p
3
4pq + 9r 0, m p = 1 nn 4q 1 9r. Do
q
2

1 + 9r
4
_
1
3
+a
k
_

a
k
3
q
2

(9r + 1) (3a
k
+ 1) 4a
k
12
q
2

27ra
k
+ 9r + 3a
k
+ 1 4a
k
12
q
2
3r
27ra
k
27r + 1 a
k
12
q
2
3r
(1 a
k
) (1 27r)
12
.
Vy ta chng minh c
1 a
k

2
9q
. (5)
q
2
3r
(1 a
k
) (1 27r)
12
. (6)
rng 1 a
k
> 0, 1 27r 0, nn t (5) v (6) suy ra
q
2
3r
2
9q
.
1 27r
12
54q q
2
3r
_
1 27r.
Php chng minh hon thnh.
Bi ton 3. Xt ba s thc dng a, b, c tho mn a + b +c = 1, chng minh rng
7 (ab + bc + ca)
2
18abc + 27 a
3
b
3
+b
3
c
3
+ c
3
a
3
_
.
210
Gii. t p = a +b +c, q = ab +bc +ca, r = abc. Khi p = 1 v
a
3
b
3
+b
3
c
3
+c
3
a
3
= q
3
3pqr + 3r
2
= q
3
3qr + 3r
2
.
Ta cn chng minh
7q
2
18r + 27 q
3
3qr + 3r
2
_
27q q
2
3r
_
7q
2
18r 81r
2
27q q
2
3r
_
7 q
2
3r
_
+ 3r (1 27r)
(27q 7) q
2
3r
_
3r (1 27r) . (1)
Ta c 0 q = ab + bc + ca
1
3
(a + b + c)
2
=
1
3
. Khi 0 < q
1
4
, th (1) ng, do v tri m,
cn v phi khng m. Tip theo ta xt
1
4
q
1
3
. Xt dy s (a
n
) nh sau
a
1
=
1
4
; a
n+1
=
7a
n
3
27a
n
11
, n = 1, 2, . . . (4)
Ta c a
1

1
3
. Gi s a
k

1
3
. Xt hm s f(x) =
7x3
27x11
, khi hm f ng bin v f

(x) > 0.
Vy a
k+1
= f (a
k
) f
1
3
_
=
1
3
. Theo nguyn l quy np, suy ra
0 a
n

1
3
, n = 1, 2, . . .
Do
a
n+1
a
n
=
7a
n
3
27a
n
11
a
n
=
(27a
2
n
18a
n
+ 3)
27a
n
11
=
3 (3a
n
1)
2
27a
n
11
0.
Vy dy s (a
n
) tng v b chn trn nn c gii hn hu hn. t lim
n+
a
n
= L. T (4) cho
n +, ta c
L =
7L 3
27L 11
3(3L 1)
2
= 0 L =
1
3
.
V dy s (a
n
) tng v
1
3
v q
_
1
4
;
1
3
_
, suy ra tn ti k N

sao cho
a
k
q a
k+1
q
3 7a
k
11 27a
k
11q 27qa
k
3 7a
k
a
k
(27q 7) 11q 3. (5)
Nu
1
4
q
7
27
th (1) ng. Tip theo xt
7
27
< q
1
3
. T (5) ta c
a
k

11q 3
27q 7
1 a
k
1
11q 3
27q 7
1 a
k

16q 4
27q 7
.
T a
k
q
1
3
, ta c
(q a
k
)
_
q
1
3
_
0 q
2

q
3
+qa
k

a
k
3
q
2
q
_
1
3
+ a
k
_

a
k
3
.
211
Theo bt ng thc Schur, ta c p
3
4pq + 9r 0, m p = 1 nn 4q 1 9r. Do
q
2

1 + 9r
4
_
1
3
+a
k
_

a
k
3
q
2

(9r + 1) (3a
k
+ 1) 4a
k
12
q
2

27ra
k
+ 9r + 3a
k
+ 1 4a
k
12
q
2
3r
27ra
k
27r + 1 a
k
12
q
2
3r
(1 a
k
) (1 27r)
12
.
Vy ta chng minh c
1 a
k

16q 4
27q 7
. (6)
q
2
3r
(1 a
k
) (1 27r)
12
. (7)
T (6) v (7) ta c
(27q 7) q
2
3r
_
(27q 7) .
(1 a
k
) (1 27r)
12

(16q 4) (1 27r)
12
=
(4q 1) (1 27r)
3
3r (1 27r) (do 4q 1 9r) .
Php chng minh hon thnh.
Lu . Cc bi ton trn cn c gii bng cch vn dng "Phng php ABC" (mt phng
php mi, c a ra bi Nguyn Anh Cng), tuy nhin khi i thi Hc sinh gii Quc gia,
nu lm theo cch ny th phi chng minh nhiu nh l, H qu phc tp, iu khng
thch hp do thi gian hn hp trong phng thi. Cn nu gii theo phng php dng gii hn
dy s nh trn th ta ch cn chng minh kt qu p
3
4pq + 9r 0, iu ny rt ngn gn,
n gin, ch vi dng l xong.
212
MT S VN V PHP NGHCH O TRONG
MT PHNG V NG DNG
T Nguyn, Trng THPT Chuyn L Qu n - Khnh Ha
Php nghch o l mt mng hnh hc phng hay v kh. Bi vit ny cho d ti c gng
n my vn cha th lt t ht c v p ca php bin hnh ny. Trong bi ny, ti s trnh
by mt vn v php bin hnh nghch o cng nh ng dng ca php ny trong mt s
bi tp v hnh hc. phng, khun kh ca chng trnh ging dy cc trng chuyn, lp
chn bi dng cho hc sinh gii cc cp. Phn ln th cc li gii trong bi ny theo hng
suy lun ch quan nn khng th trnh c nhiu thiu st. Do cc em hc sinh sau khi
nghin cu bi ny nn tp gii theo hng khc nhng vn da trn php bin hnh ny
ci thin kh nng t duy sng to, v bit mnh dn t ph, tm ti ci mi hn. Nhm bo
m v tnh h thng, mt s kin thc, mc d c trnh by trong mt s ti liu vn
phi nu li trong bi vit ny.
1 nh ngha
nh ngha 1. Cho im I v s thc R
2
= 0. Mt php bin hnh i t mt phng P vo mt
phng P sao cho mi im M thuc P ng vi im M cng thuc P tha: IM.IM

= R
2
(*)
gi l php bin hnh nghch o cc I, phng tch R
2
v k hiu: N(I, R
2
)
ng thc (*) cn ni ln mt iu: nh, to nh v cc lun thng hng. Hn na cc lun
nm ngoi on thng ni nh v to nh. Thng th ta hay k hiu: nh ca M l M hoc
M*
Ch :
ng trn (O, R ) l ng trn nghch o qua N(O, R
2
) cng l tp hp cc im kp trong
php nghch o ny.
OM.OM

= OH.OH

= OT
2
1
2 nh ca mt hnh qua php nghch o N(I,k)
Qua mt php nghch o th:
1/ ng thng qua cc bin thnh chnh n .
2/ ng thng khng qua cc bin thnh ng trn qua cc v ngc li, hn na tm ng
trn nm trn ng thng qua cc vung gc vi ng thng cho
Ch :
Nu N(O, R
2
) : A B th A v B l tng ng mt i mt nn tip im gia ng thng
vi ng trn, ng trn vi ng trn bin thnh tip im gia 2 nh ca n.
213
Hnh 2:
V d 1. Cho im I c nh nm trong ng trn (O,R). Mt dy AB di ng quanh I. Tm
tp hp giao im M ca 2 tip tuyn ti A,B vi ng trn.
Hnh 3:
Li gii. t k = R
2
. Dng php nghch o N(O, k):
ng trn (O,R) bin thnh chnh n. 2 tip tuyn ti A v B bin thnh 2 ng trn ng
knh OA,OB.
Vy M l giao ca 2 tip tuyn bin thnh K l giao ca 2 ng trn , nn

OKB = 90
0
OKA =

K AB. V : O,I c nh, suy ra K l ng trn ng knh knh OI. M l ng thng
vung gc ng knh OI ti H c nh l nh ca I qua php nghch o N(O,k) . (pcm)
Bi tp tng t vi bi trn. Chng minh rng trong mt lc gic khng c cp cnh i no
214
Hnh 4:
song song vi nhau m ngoi tip mt ng trn th 3 dy cung ni bi 2 tip im ca 2 cnh
i ng quy khi v ch khi giao cc cp cnh i thng hng.
V d 2. ng trn ni tip (I,r) tip xc cc cnh BC,CA,AB ca tam gic ABC ti A, B,
C.Gi M l giao ca BC v BC. CM: AA vung gc vi MI.
Li gii. Xt php nghch o N(I, r
2
). Khi : BC bin thnh (ACIB) BC bin thnh
Hnh 5:
ng trn ng knh IA . M bin thnh K l giao ca: (ACIB) v ng trn ng knh
IA M,K I thng hng.
IKA =

Mt khc do:

IKA

= 90
0
A,K, A thng hng.
Vy MIAA

. (pcm)
V d 3. Cho ng trn (O,r) v mt ng thng d c nh nm ngoi (O).Hy ch ra cch
dng ng trn () tip xc ng thng d ti A c nh cho trc ng thi tip xc (O,r)
215
Hnh 6:
Li gii. Xt php nghch o N(A,k) vi k = P (A/(O).Khi : d d, (O) (O), () d
1
hoc d
2
//d v tip xc vi (O) ti A
1
v A
2
. T suy ra cch dng.
V d 4. Cho 2 ng trn (O
1
) v (O
2
) tip xc ngoi vi nhau ti H. Trn tip tuyn vi
chung vi nha ti H ly im P c nh. Hy dng ng trn qua P tip xc vi 2 ng trn
cho.
Li gii. V P thuc trc ng phng ca 2 vng , ta t k l phng tch ca P i vi 2
vng . S dng php nghch o cc P, phng tch k th 2 vng trn bin thnh chnh n.
2 vng trn qua P tip xc vi 2 vng trn cho bin thnh 2 tip tuyn chung ngoi ca
chng. Tip im ca ng trn qua P vi (O
1
) l T
2
, E
2
tr thnh tip im ca tip tuyn
vi (O
1
) l T
1
, E
1
. Vic dng tip tuyn chung ngoi vi 2 ng trn l bi ton quen thuc
v c bn. T d dng dng c 2 vng trn qua P tha yu cu bi ton.
2.1 ng trn khng qua cc bin thnh ng trn khng qua
cc, nhng tm khng bin thnh tm.
Li gii. Gi s php nghch o N(I,k) bin (C) thnh (C). A thnh A: IA.IA

= k (1)
Gi p l phng tch ca I i vi (O) khi ta c: IA.IB = p (2)
T (1) v (2) IA

=
k
p
IB A

= V
k
p
I
(B) Khi B vch nn (C) th A cng vch nn (C).
Vy qua php N(I,k) th (C) bin thnh (C) l nh ca (C) qua php v t tm I t s
p
k
, cng
l nh ca (C) qua php nghch o N(I,k) . (pcm)
216
Hnh 7:
Hnh 8:
2.2 di on thng nh
a. Nu AB l nh ca AB qua N(O, R
2
) th: AB = R
2
.
AB
OA.OB
b. Nu ng trn bn knh
r l nh ca ng trn bn knh r tm I th:
217
Chng minh. a/ p dng tam gic ng dng bn c d dng chng minh c iu ny.
b/ Gi s AB l ng knh ca vng (I,r) nm trn ng thng OI. D l khng cch OI. Theo
bi trn th:
AB = R
2
.
AB
OA.OB
2r = R
2
2r
|d r| .(d +r)
r = R
2
r
|d
2
r
2
|

V d 5. Cho t gic ABCD. Chng minh rng: AB.CD + BC.DA AC.BD. Du bng xy
ra khi no?
Li gii. Xt php nghch o N(A,k) , k > 0 th 3 im B,C,D bin thnh 3 im B, C, D
Hnh 9:
p dng bt ng thc gia 3 cnh ca tam gic BCD:
B

= k.
BC
AB.AC
, B

= k.
BD
AB.AD
, C

= k.
CD
AC.AD
218
V B

+C

AB.CD +BC.DA AC.BD. Du bng xy ra khi B,C, D thng


hng.
Do qua php nghc o, ng thng khng qua cc bin thnh ng trn qua cc nn A,B,C,D
cng thuc mt ng trn hay t gic ABCD ni tip ng trn. (pcm)
V d 6. Gi d l khang cch 2 tm ca vng trn ni tip v ngoi tip trong mt tam gic
. r,R tng ng l bn knh ca chng th : d
2
= R
2
2Rr (Euler).
Li gii. Gi I v O l tm vng trn ni tip v ngoi tip tam gic ABC. D,E,F l tip im
Hnh 10:
ca vng trn ni tip vi BC,CA,AB. Thc hin php nghch o qua ng trn ni tip th
A, B, C bin thnh A, B, C l trung im cc dy cung EF, FD,DE. ng trn qua A,B,C
c bn knh r/2
Mt khc theo tnh cht 4 th nh ca vng trn ngoi tip c bn knh r = r
2
.
R
|d
2
R
2
|
T ta c:r
2
.
R
|d
2
R
2
|
=
r
2
d
2
= R
2
2Rr
2.3 Php nghch o bo ton gc
Cho 2 ng trn (C
1
) v (C
2
) ct nhau ti A v ti y chng c 2 tip tuyn. Ta gi gc
gia 2 tip tuyn l gc gia 2 ng trn cho. Khi 2 tip tuyn vung gc th ta ni 2 ng
trn l trc giao.
219
B 2. Cho php nghch o N(O, k) bin ng cong (C) thnh ng cong (C). Nu
A (C), A

(C

) l nh ca A qua php N(O, k) v ti y chng c cc tip tuyn th cc


tip tuyn ny i xng nhau qua trung trc AA.
Tht vy: Ta c : M

. |k|
MA
IA.IM
.
Khi M A th M

ct tuyn tr thnh tip tuyn, ng trn qua 4 im AMMA tr


thnh ng trn () tip xc vi 2 ng cong trn. 2 tip tuyn ny l 2 tip tuyn ca ng
trn (). V vy giao 2 tip tuyn ny nm trn trung trc AA. T ta suy ra nh l sau
nh l 1. Php nghch o l php bo ton gc.
V d 7. Cho t gic ABCD c 2 gc A v C ph nhau. Chng minh rng: AB
2
.CD
2
+
BC
2
.DA
2
= AC
2
.BD
2
Li gii. Xt php nghch o N(B,k) , k > 0 th 3 im A,C,D bin thnh 3 im A, C, D
Hnh 11:
p dng bt ng thc gia 3 cnh ca tam gic BCD:
A

= k.
AC
BA.BC
, A

= k.
AD
BA.BD
, C

= k.
CD
BC.BD
V 2 gc A v C ph nhau nn tm ca 2 vng trn ngoi tip 2 tam gic ABD v CBD cng
vi B v D to thnh t gic ni tip.
220
T suy ra:

O
1
BO
2
=

O
1
DO
2
= 90
0
. Vy cho nn 2 vng trn (ABD) v (CBD) trc giao
vi nhau suy ra nh ca chng cng vung gc nhau.
Do : A

D
2
+C

D
2
= A

C
2
AD
2
BC
2
+ CD
2
BA
2
= AC
2
BD
2
(pcm)
V d 8. Cho 2 ng trn khng bng nhau v khng ng tm. Chng minh lun tn ti mt
php nghch o bin chng thnh 2 vng trn ng tm.
Li gii. Gi s l 2 vng trn (C
1
) v (C
2
).
Hnh 12:
Trc ht gi d

l trc ng phng ca 2 vng (C


1
) v (C
2
). D

vung gc vi O
1
O
2
ti I.
Dng tip tuyn IT
1
vi (C
1
). ng trn (C) tm I bn knh IT
1
ct O
1
O
2
ti 2 im O v J.
Gi P l phng tch ca I i vi 2 vng trn , khi : (C) trc giao vi (C
1
) v (C
2
).
Qua php nghch o cc O, phng tch P th (C) bin thnh ng thng d

vung gc vi
C

1
v C

2
.
V (C) vung gc vi (C
1
) v (C
2
). nn d

vung gc vi (C

1
) v (C

2
) v vy d

phi qua tm
ca (C

1
) v (C

2
). Do 2 tm ca (C

1
) v (C

2
) nm trn O
1
O
2
nn 2 tm ny phi trng nhau.
Vy lun tn ti mt php nghch o bin chng thnh 2 vng trn ng tm. (pcm) Chng
hn qua bi trn ta cht nhn ra tng cch dng cc ng trn ni ui tip xc nhau v
cng tip xc vi 2 ng trn cho trc nh hnh v trn.
221
Hnh 13:
3 Php nghch o i vi 2 ng trn
3.1 Trng hp tng qut
Nu 2 ng trn (C) v (C) khng bng nhau khng tip xc nhau v khng ng tm.
Khi c 2 php v t : V
R

R
o
v V

R
o
bin ng trn (C) thnh ng trn (C). Cc tm
v t khng nm trn 2 ng trn nn cng c 2 php nghch o bin ng trn (C) thnh
ng trn (C).
a/ Php nghch o cc O phng tch k =
R

R
.p vi p l phng tch ca O i vi (C).
b/ Php nghch o cc O phng tch k =
R

R
.p

vi p l phng tch ca O i vi (C)


3.2 Trng hp c bit
a/ Nu 2 ng trn (C) v (C) bng nhau m khng tip xc nhau th c mt php v t
tm O bin ng trn ny thnh ng trn kia.Nn ta ch c mt php nghch o cc (O)
vi phng tch k

= p

trong p l phng tch ca O i vi (C) bng phng tch ca O


i vi (C).
b/ Nu 2 ng trn (C) v (C) khng bng nhau m tip xc nhau th tip im l tm v t
222
nhng khng phi l cc nghch v ch c tm v t cn li l cc O ca mt php nghch o.
c/ Nu 2 ng trn (C) v (C) bng nhau m tip xc nhau th khng c php nghch o
no bin ng trn ny thnh ng trn kia.
V d 9. Chng minh rng: ng trn i qua cc trung im cc cnh tam gic tip xc vi
ng trn ni tip v 3 ng trn bng tip.
Li gii. Gi s ng trn tm I ni tip tam gic ABC, tip xc cc cnh BC,CA ti F,
Hnh 14:
D. ng trn bng tip gc A tip xc BC, AB ti E v H. M, N, P l trung im 3 cnh
BC, CA, AB. Gi BC = a, AC = b, AB = c . t p l na chu vi tam gicABC. Ta u bit
rng:AH = p, BF = CE = pb.
Suy ra: ME = MF. Do vy m M thuc trc ng phng ca 2 vng ni tip v bng tip
223
gc A. Gi k l phng tch ca M i vi 2 vng ny th :
Php nghch o cc M phng tch k bin 2 vng ny thnh chnh n.Cn ng trn qua
M,N,P l ng trn qua cc th bin thnh ng thng khng qua cc M . Nu ta chng
minh c nh ca ng trn qua M,N,P l tip tuyn vi 2 vng ni tip v bng tip gc A
l xong. Do vy ta dng tip tuyn chung trong vi 2 vng ny l KL (K AC, L AB) ri
chng minh n l nh ca vng (MNP) qua php nghch o N(M,k).
Tht vy gi N,P l giao ca KL vi MN v MP. Ta c:
KL v BC u l 2 tip tuyn chung trong vi 2 vng trn. AI l trc i xng ca 2 vng ny nn:
KL = BC, AK = AB = c, AL = AC = b, PL = b
c
2
. Ta li c:
PP

AK
=
LP
LA
PP

= c.
(2bc)
2b
=
c(2bc)
2b
T m: MP = |MP PP

| =

b
2

c(2bc)
2b

=
(bc)
2
2b
Suy ra: MP

.MP =
(bc)
2
4
(1)
Biu thc
(bc)
2
4
c b v c bnh ng vi nhau nn hon ton tng t ta cng vn c:MN

.MN =
(bc)
2
4
(2)
T (1) v (2) ta c: MP

.MP = MN

.MN =
(bc)
2
4
Hay vng trn (MNP) bin thnh tip tuyn LK qua php nghch o N(M,k). (pcm)
V d 10. A l im ty thuc na ng trn (C) ng knh BC. H ng cao AH ca
tam gic ABC. Gi (C
1
) v (C
2
) l 2 na ng trn ng knh HB v HC cng pha vi na
ng trn (C).
a/ Hy nu cch dng 2 ng trn (
1
) v (
2
) ln lt tip xc vi (C), ng cao AH, (C
1
)
v (C
2
).
b/ Chng t rng bn knh ca 2 vng (
1
) v (
2
) bng nhau.
Li gii. a/ Gi O, O
1
, O
2
, K l tm cc vng trn ng knh BC, BH, CH v (
1
), T
1
, I v M
tng ng l tip im ca vng (
1
) vi (C), (C
1
) v AH. Khi : T,K,O thng hng , O
1
, I, K
thng hng t d dng C, M, T
1
thng hng v B,I,M thng hng.
Li c: CM.CK = CH.CB = CA
2
C thuc trc ng phng ca (C
1
) v (
1
). Vy CI l
tip tuyn chung ca 2 vng (C
1
) v (
1
). Hn na ta cng c : CI = CA. T cc d liu v
s thng hng ca 3 im v di bng nhau ta bt u hnh thnh cch dng (
1
) nh sau:
I l giao ca vng trn tm C,bn knh CA vi vng (C
1
).BI ct AH ti M.CM gp (C) ti T
1
.
D nhin tm K l giao ca OT
1
vi O
1
I. Do tnh bnh ng gia 2 vng (
1
) v (
2
) trong bi
ton m (
2
) c dng hon ton tng t.
b/ Gi R, r
1
, r
2
, r ln lt l cc bn knh ca 4 vng trn (C), (C
1
), (C
2
)v
1
. D thy rng:
r
1
+r
2
= R. Do tnh bnh ng vi nhau gia 2 vng (
1
) v (
2
) trong bi ton so vi cc vng
(C), (C
1
), (C
2
)
Theo iu kin tip xc ca 2 vng (
1
) v (C
1
) v h thc lng trong tam gic vung BMH
m ta c:
r
r
1
=
IM
IB
=
IM.IB
IB
2
v IM.IB = IH
2

r
r
1
=
IM
IB
=
IH
2
IB
2
(1)
T 2 tam gic ng dng l: CIB v CHI ta li c:
IH
IB
=
IC
BC

IH
IB
_
2
=
IC
BC
_
2
=
AC
BC
_
2
(2)
M theo h thc lng trong tam gic vung ABC th: CA
2
= CH.CB = 4R.r
2
. (3)
T (1) , (2) v (3) ta c: r =
4r
1
.r
2
R
BC
2
=
r
1
.r
2
R
224
Hnh 15:
Vy 2 vng trn (
1
) v (
2
) bng nhau. (pcm)
V d 11. Gi s cc ng trn C
1
, C
2
, C
3
cng tip xc trong vi ng trn C(O, R) ln
lt ti A
1
, A
2
, A
3
v i mt tip xc ngoi vi nhau. B
1
, B
2
, B
3
l tip im ca C
2
v C
3
, C
1
v C
3
, C
2
v C
1
. Chng minh: A
1
B
1
, A
2
B
2
, A
3
B
3
ng quy.
Li gii. Dng php nghch o tm A
1
, phng tch k = 4R
2
. Th vng (C
1
) v (C) thnh 2
ng song song. 2 vng (C
2
) v (C
3
) thnh 2 vng tip xc nhau v cng tip xc vi 2 ng
thng song song . Khi A
2
, B
1
, B
2
c nh l P, B

1
, B

2
(nh hnh v di) thng hng nn
suy ra t gic A
1
A
2
B
1
B
2
ni tip ng trn. A
1
B
1
l trc ng phng ca ca 2 vng
(A
3
A
1
B
3
B
1
) v (A
1
A
2
B
1
B
2
).
Do trc ng phng ca 2 vng i mt trong 3 vng trn khng ng tm th ng quy
ti tm ng phng. T 3 ng thng A
1
B
1
, A
2
B
2
, A
3
B
3
ng quy ti mt im.
(pcm) Ch rng: Gi s () l vng trn tip xc ngoi vi cc vng C
1
, C
2
v C
3
. Ta gi
C
1
, C
2
, C
3
ln lt l tip im ca () vi C
1
, C
2
, C
3
v C
3
, C
2
v C
1
. Khi ta cng vn c:
B
1
C
1
, B
2
C
2
, B
3
C
3
ng quy.
M rng bi ton Edose
Bn trong tam gic ABC ly im O ty . Gi cc khong cch t O n cc cnh BC,
225
Hnh 16:
Hnh 17:
226
CA, AB l p, q, r. v n cc nh A, B, C l x, y, z. Chng minh:
a/ax bq + cr , ax br + cq (I)
b/ax +by + cz 4S (II)
c/(x +y + z) 2(p +q +r) (III)
d/xyz (p +q)(q +r)(r + p) (IV)
Ta c: 2S = ap +bq +cr
Hnh 18:
Mt khc:
x +p h
a
ax + ap 2S ax + 2S
1
2S ax qb +rc
Gi O l im i xng ca O qua phn gic trong gc A ax rb + qc x
b
a
r +
c
a
q ,
tng t nh th ta c: (*)
_
_
_
x
b
a
r +
c
a
q
y
c
b
p +
a
b
r
z
a
c
q +
b
c
p
x +y +z (
a
b
+
b
a
)r +(
a
c
+
c
a
)q +(
c
b
+
b
c
)p 2(p +q +r) Trong tam gic DEF, h OH vung
gc EF th ng cao ny c vai tr tng t nh OP trong tam gic ABC. Ta c :
OH.EF = OE.OF sin A = qr sin A = qr = 2S(OEF).
227
Hnh 19:
Mt khc: EF = AOsin A = x sin A =
ax
2R
, t OH =
qr
x
, V vy tng t vi bt (III)
ta c:
p +q +r 2
_
qr
x
+
rp
y
+
pq
z
_
Hay:
1
pq
+
1
qr
+
1
rp
2(
1
px
+
1
qy
+
1
rz
) (VI)
T bt: ax bq +cr , ax br +cq 2ax (b +c)(q + r)
8abcxyz (a + b)(b + c)(c +a)(p +q)(q +r)(r +p) xyz (p + q)(q + r)(r +p)
V ngoi ra ta cn c: xyz 8pqr
Tng t vi bt (IV) ta c: pqr (
qr
x
+
rp
y
)(
rp
y
+
pq
z
)(
pq
z
+
qr
x
)
Hay: 1 (
1
px
+
1
qy
)(
1
qy
+
1
rz
)(
1
rz
+
1
px
)(VII)
By gi thc hin php nghch o cc O phng tch k, khi ng trn (OEAF) bin thnh
ng thng da vung gc OA, ng trn (OFBD) bin thnh ng thng db vung gc OB,
ng trn (ODCE) bin thnh ng thng dc vung gc OC. 3 ng thng ny lp thnh
tam gic ABC.
Theo php nghch o:
B

=
EF.k
OE.OF
, OA

=
k
p
. OA ct BC ti K th OK =
k
p
T bt (III) ta c:
1
p
+
1
q
+
1
r
2(
1
x
+
1
y
+
1
z
)
(VIII)
228
Hnh 20:
T bt (IV) ta c:
1
pqr
2(
1
x
+
1
y
)(
1
y
+
1
z
)(
1
z
+
1
x
) (IX)
C th thy s tng ng cc yu t ca 3 tam gic trong bng sau y:
Hnh 21:
229
V d 12. Cho ng trn (S
2
) nm trong ng trn (S
1
). Gi d l khong cch 2 tm, R
1
v R
2
l 2 bn knh ca vng (S
1
) v (S
2
). Chng minh rng: a/ Nu tn ti t gic ni tip
trong (S
1
) v ngoi tip quanh (S
2
) th cc on thng ni cc tip im ca cc cnh i nhau
vi (S
2
) lun vung gc vi nhau.
b/ C v s t gic cng tnh cht nh vy.
c/
1
(R
1
+d)
2
+
1
(R
1
d)
2
=
1
R
2
2
Li gii. a/ Gi s tn ti t gic ABCD tha iu kin bi ton nh hnh v. Gi M,N,P,Q
Hnh 22:
l tip im ca AB, BC, CD, DA vi vng (S
2
).Thc hin php nghch o theo ng trn
(S
2
) khi A, B, C, D bin thnh trung im A,B,C,D ca cc dy MQ, MN, NP, PQ. V
chnh cc im ny ang nm trn vng trn (S

1
) l nh ca (S
1
) qua php nghch o trn.
Ta bit rng trung im cc cnh ca t gic l nh ca hnh bnh hnh c cc cnh song song
vi 2 ng cho, nn ABCD l hnh bnh hnh ni tip ng trn. Suy ra ABCD l hnh
ch nht .
Vy MP vung gc NQ. (pcm)
230
Hnh 23:
b/ Mi im N
1
ty trn (S
2
) ta dng dy B
1
C
1
ca (S
1
) tip xc vi (S
2
) ti N
1
. Sau ta
dng dy B
1
A
1
v C
1
D
1
tip xc (S
2
) ti M
1
v P
1
. Qua N
1
dng dy N
1
Q
1
vung gc P
1
P
1
.
Th th do A
1
, Q
1
, D
1
u c nh l cc im ca vng trn ng knh O
2
Q
1
nn chng phi
thuc ng thng vung gc vi O
2
Q
1
ti Q
1
. Tc l A
1
D
1
tip xc (S
2
) ti Q
1
. (pcm).
c/ chng t:
1
(R
1
+d)
2
+
1
(R
1
d)
2
=
1
R
2
2
ta nhn xt rng khi lm thay i hnh dng t gic ni
tip trong (S
1
) v ngai tip (S
2
) th ng thc cn chng minh khng h thay i. Vy ta nn
chn t gic c bit nh hnh thang cn ABCD
(AB //CD) chng hn gii quyt bi ton n gin hn.
i vi hnh thang cn ABCD th ng thng O
1
O
2
l trung trc 2 y AB v CD.
t AB = 2a, CD = 2b v O
1
O
2
= d. M, N l trung im AB v CD. H O
2
H vung gc BC.
Do nh l: 2 tip tuyn xut pht t mt im ngoi ng trn ta d dng suy ra tam gic
BO
2
C vung ti O
2
. V vy bn knh O
2
H = R
2
.
Nn ta c: BH.CH = O
2
H
2
= R
2
2
a.b = R
2
2
.
Mt khc: T 2 tam gic vung AO
1
B v DO
1
N, ta c: R
2
1
= a
2
+(R
2
+d)
2
, R
2
1
= b
2
+(R
2
d)
2
T :
a
2
b
2
= [R
2
1
(R
2
+ d)
2
].[R
2
1
(R
2
d)
2
] R
4
2
= [R
2
1
(R
2
+d)
2
].[R
2
1
(R
2
d)
2
]
231
R
4
2
= [R
2
1
(R
2
+d)
2
].[R
2
1
(R
2
d)
2
]
1
(R
1
+d)
2
+
1
(R
1
d)
2
=
1
R
2
2
Cc bi tp ngh
1. Cho 4 ng trn S
1
, S
2
, S
3
, S
4
. Gi s S
1
ct S
2
ti A
1
v A
2
, S
2
ct S
3
ti B
1
v B
2
, S
3
ct S
4
ti C
1
v C
2
, S
4
ct S
1
ti D
1
v D
2
, CM: nu cc im A
1
, B
1
, C
1
, D
1
nm trn mt
ng trn S(hoc ng thng) th cc im A
2
, B
2
, C
2
, D
2
cng nm trn mt ng trn
S(hoc ng thng).
2. Cho 2 ng trn (O
1
) v (O
2
) trc giao ti A v B. C (O
1
), D (O
2
). CM:(ACD) trc
giao vi (BCD).
3. Trong mt phng cho ng trn (O,R) v ng thng () khng ct n cch O mt khang
bng d. 2 im M,N di ng trn () sao cho ng trn ng knh MN tip xc (ngoi hoc
trong) vi ng trn cho. CM: trn mt phng c 1 im P c nh lun nhn MN di gc
khng i.
4. T mt im M nm trong tam gic ABC dng cc ng trn i xng ca cc ng trn
ngoi tip cc tam gic : MAB,MBC,MCA ln lt qua AB, BC, CA. CM: cc ng trn ny
giao nhau ti mt im.
5. Dng ng trn tip xc vi ng trn (O) cho trc v tip xc ng thng () cho
trc ti A cho trc. CM: ng ni cc tip im ca cc ng trn vi ng trn (O)
lun qua im c nh khi () quay quanh A.
6. Trn tip tuyn ti T ca mt ng trn ly 2 on TM v TN bin thin sao cho TM.TN =
const . Gi T l im xuyn tm i ca T trn ng trn .
a/ CM: on ni giao im ca cc ng thng TM v TN vi ng trn cho i qua
im c nh.
b/ Gii bi ny cho ng thng ni cc tip im ca cc tip tuyn vi ng trn cho v
qua MN.
c/ Tm tp hp giao im ca 2 tip tuyn ny.
7. Cho 2 ng trn (O,R) v (O,4R) tip xc trong nhau ti A.
a/ Hy nu cch dng ng trn (I,r) tip xc ngoi vi (O) v tip xc trong vi (O).
b/ Tm tp hp tm cc ng trn nh vy.
8. Cho 2 ng trn ng knh AB v AC tip xc ngoi nhau ti A. k 2 tip tuyn ti B v
C vi 2 ng trn. Hy nu cch dng cc ng trn tip xc vi 2 ng trn v tip
xc vi 2 tip tuyn trn.
9. Cho 3 im A, B, C thng hng. P nm ngoi ng thng AC. CM: cc tm ca cc ng
trn (ABP), (BCP), (CAP) v P cng thuc mt ng trn.
10. Dng ng trn qua 2 im cho tip xc vi mt ng trn (ng thng) cho.
11. Qua mt im cho trc dng ng trn tip xc vi 2 ng trn (hoc ng thng vi
ng trn) cho trc.
12. Dng ng trn tip xc vi 3 ng trn cho trc.(Apoloniut)
13. ng trn S tip xc ngoi vi ng trn (S
1
) v (S
2
) ti cc im A v B. CM: ng
thng AB i qua tm ca php v t bin (S
1
) thnh (S
2
).
14. Cho 3 im A, C, B thng hng theo th t. ly 3 on ny dng 3 na ng trn cng
232
pha so vi AB.
a/ ng trn S tip xc vi 3 na ng trn . CM: ng knh ca S bng khong cch t
tm ca S n ng thng AB.
b/ Qua C dng na ng thng vung gc AB. Trong cc tam gic cong ACD v BCD ni
tip cc ng trn (S
1
) v (S
2
). CM: (S
1
) v (S
2
) bng nhau.
c/ CM: tip tuyn chung ca na ng trn ng knh AC v (S
1
) lun qua im c nh
khi C di ng trn AB.
15. Cho 2 ng trn tip xc trong nhau (O
1
) v(O
2
). Mt chui cc ng trn S
0
, S
1
, . . . S
n
ni ui nhau lin tip v tip xc nhau v cng tip xc vi 2 ng trn trn m tm ca
S
0
, O
1
, O
2
thng hng. Cc ng trn ny cng nm mt pha so vi OO. Gi dn l khong
cch t tm Sn n OO. CM: d
n
= 2nr
n
.
16. Chng minh: t s kp ca 4 im A, B, C, D khng thay i qua php nghch o.
17. Qua 2 im A v B ca ng trn S k 2 tip tuyn ct nhau ti C. D l trung im AB.
CM: C v D l nh ca nhau qua php nghch o qua ng trn S.
18. Ly 4 im A, B, C, D cng thuc mt ng trn. CM: nu cc tip tuyn ca ng trn
ti A v C ct nhau trn BD th cc tip tuyn ca ng trn ti B v D ct nhau trn AC.
19. Hai ng trn di ng (S
1
) v (S
2
) tip xc ngoi nhau v cng tip xc vi dy v cung
ca hnh vin phn. Chng minh rng:
a/ Tip im gia (S
1
) v (S
2
) chy trn ng c nh v ng ni tm tip xc ng cong
c nh.
b/ Tip tuyn chung trong gia (S
1
) v (S
2
) i qua im c nh.
20. Hy ch ra cch dng 2 ng trn (S
1
) v (S
2
) nh trn.
21. Cho a gic u A
1
A
2
. . . A
n
. M l im ty trn cung A
1
A
n
, d
i
= MA
i
. CM rng:
a/
n1

i=1
1
d
i
d
i+1
=
1
d
1
dn
b/ Nu n l th: d
1
+d
3
+d
5
. . . + d
n
= d
2
+d
4
+d
6
+. . . d
n1
.
2. Cho 3 im A, B, C thng hng vi d l trung trc AB. Mt ng trn (O) thay i i qua
A, B ct d D v E. Cc ng CD, CE ct (O) D v E. Tm tp hp cc im D v E
23. Hai ng trn tip xc nhau ti A. Mt tip tuyn ti M ca ng trn th nht ct
ng trn th hai B v C. CM: ng thng AM l phn gic ca gc to bi 2 ng AB
v AC.
24. a gic 2n cnh A
1
A
2
. . . A
2n
ngoi tip ng trn (O). Gi s p
1
, p
2
, . . . p
2n
l khong cch
t M trn ng trn n cc cnh A
1
A
2
, A
2
A
3
, . . . A
2n
A
1
. CM : p
1
p
3
p
5
. . . p
2n1
= p
2
p
4
p
6
...p
2n
.
25. A l im ty thuc na ng trn (C) ng knh BC (khng trng trung im cung
(BC)). Gi O l tm ca (C).H AH vung gc BC. Tip tuyn vi na ng trn ng knh
BC ti A ct BC ti H. (C
1
), (C
2
) v (C
3
) l 3 na ng trn ng knh HB, HC, HO cng
pha vi na ng trn (C).
a/ Nu cch dng ng trn (
1
) tip xc (C), (C
1
), (C
3
) v (
2
) tip xc vi (C), (C
2
) v
(C
3
).
b/ Chng t rng 2 vng (
1
) v (
2
) bng nhau.
233
Ti liu tham kho
[1] Cc bi ton v hnh hc phng ca V.V Praxolov.
[2] Tuyn tp 30 nm Tp ch Ton hc tui tr.
234
S DNG MT S TNH CHT CA NH X GII
BI TON PHNG TRNH HMS
Trn Vn Trung, Trng THPT chuyn L Qu n Ninh Thun
K thut gii mt bi ton v phng trnh hm s c trnh by trong nhiu ti liu
(xem [1]). Trong bi vit ny ti mun gii thiu mt cch c h thng v k thut ng dng
tnh n nh, ton nh, song nh v im bt ng ca nh x gii mt bi ton phng
trnh hm.
A. K thut s dng tnh n nh-ton nh-song nh.
* Mt s lu :
a/ Nu f : X Y l n nh th t f(x) = f(y) x = y.
b/ Nu f : X Y l ton nh th vi mi y Y tn ti x X f(x) = y.
c/ Nu f : X Y l song nh th f va l n nh va l ton nh v c bit l c nh x
ngc.
- Nu hm s f l n nh th ta thng dng k thut tc ng f vo 2 v.
- Nu hm s f l ton nh th s dng tn ti x
0
sao cho f(x
0
) = 0 v sau tm .
c bit hm s f l song nh th ta c thm mt k thut na l tm hm s ngc.
Bi ton 1. Tm tt c cc hm s f : R R v tha mn iu kin
f (x + y + f(y)) = f (f(x)) + 2y; x, y R (1)
Bi gii. Trc ht ta chng minh f n nh. T iu kin ca bi ton ta hon v vai
tr x v y cho nhau ta c :
f (y +x +f(x)) = f (f(y)) + 2x; x, y R (2)
* Gi s f(x) = f(y) t (1) v (2) ta c :
f (f(x)) + 2y = f (f(y)) + 2x 2x = 2y x = y
f l n nh.
Thay y = 0 vo (1) ta c :
f (x +f(0)) = f (f(x)) ; x R
Do f l n nh nn : f(x) = x +f(0), tc l f(x) = x + a, a R.
Th li ta thy hm s ny tha mn.
Vy f(x) = x +a, a R
Bi ton 2. Tm tt c cc hm s f : Z
+
Z
+
tha mn iu kin
f (f(n) + m) = n + f (m+ 2012) ; m, n Z
+
Bi gii.
Trc ht ta chng minh f n nh. Tht vy nu f(n
1
) = f(n
2
) th
f (f(n
1
) + m) = f (f(n
2
) + m) (1)
235
n
1
+f(m+ 2012) = n
2
+f(m+ 2012) n
1
= n
2
Trong (1) thay m = f(1) ta c :
f (f(n) + f(1)) = n +f ((f(1) + 2012)
f (f(n) + f(1)) = n + 1 + f (2012 + 2012)
f (f(n) + f(1)) = f ((f(n + 1) + 2012)
V f n nh nn : f(n) + f(1) = f(n + 1) + 2012,
hay f(n+1) = f(n) +f(1) 2012 f(n+1) f(n) = f(1) 2012. suy ra f(n) l cp s cng
c cng sai d = f(1) 2012 v s hng u n
1
= f(1), vy nn
f(n) = f(1) + (n 1)[f(1) 2012]
f(n) = f(1) + nf(1) f(1) 2012n + 2012
f(n) = [f(1) 2012]n + 2012
tc l f(n) = an +b.
Thay vo quan h hm ta c : f(n) = n + 2012; n Z
+

Bi ton 3. (IMO 2002) Tm tt c cc hm s f : R R tha mn iu kin


f (f(x) + y) = 2x + f (f(y) x) ; x, y R (1)
Bi gii.
Ta chng minh: f l ton nh. Tht vy thay y = f(x) vo (1) ta c :
f (f(f(x)) x) = f(0) 2x; x R
Do v phi l hm bc nht vi x nn tp gi tr ca f l R f ton nh.
V f l ton nh nn a R sao cho : f(a) = 0
Thay x = a vo (1) ta c
f (f(a) + y) = 2a +f (f(y) a)
f(y) = 2a + f (f(y) a)
f(y) a = f (f(y a) + a
V f ton nh nn : f(x) = x a
Th li, ta thy f(x) = x a tha mn.
Bi ton 4. Tm tt c cc hm s lin tc f : R R tha mn iu kin :
f (x +f(y)) = 2y +f(x); x, y R (1)
Bi gii.
a/ Chng minhf l n nh tht vy nu f(x
1
) = f(x
2
) th :
_
f (x +f(x
1
)) = 2x
1
+f(x)
f (x +f(x
2
)) = 2x
2
+f(x)
2x
1
+f(x) = 2x
2
+f(x) x
1
= x
2
b/ T k (1) f l ton nh (v tp gi tr ca f l R)
Do f l ton nh nn a R f(a) = 0. Ta chng minh a = 0
Th y = a vo (1)
f (x + f(a)) = 2a +f(x) f(x) = 2a + f(x)
a = 0 f(0) = 0
T (1) thay x = 0 ta c : f (f(y)) = 2y +f(0) = 2y
236
Trong (1) thay x bi f(x) ta c :
f (f(x) + f(y)) = 2y + f (f(x))
f (f(x) + f(y)) = 2y + 2x
f (f(x) + f(y)) = 2(x +y)
f (f(x) + f(y)) = f (f(x +y))
(f(x) + f(y)) = f(x +y) (do f n nh)
Do f lin tc f(x) = kx.
Thay vo (1) k =

2
Th li ta c hai hm s f(x) =

2x, tha iu kin bi ton.


Bi ton 5. Tm tt c cc hm s lin tc f : R R tha :
f (xf(y) + x) = xy + f(x); x, y R (1)
Bi gii.
Thay x = 1 vo (1) ta c :
f (f(y) + 1) = y +f(1)
D dng ta chng minh c f song nh.
Thay x = 1, y = 0 vo (1) ta c
f (f(0) + 1) = f(1) f(0) + 1 = 1 (Do f l n nh)
f(0) = 0
Vi x = 0 t y =
f(x)
x
khi (1) tr thnh
f (xf(y) + x) = 0 = f(0)
xf(y) + x = 0 (v f n nh)
f(y) = 1 (V x =0)
V f l ton nh nn b R f(b) = 1
Vy f
_

f(x)
x
_
= f(b)
f(x)
x
= b f(x) = bx x =0
Kt hp vi f(0) = 0 ta c f(x) = bx x R Thay f(x) = bx vo (1) b = 1
Vy f(x) = x
Bi ton 6. Tm tt c cc hm s f xc nh trn tp s thc R, ly gi tr trong R v
tha mn ng thi cc iu kin sau :
1/ f l ton nh t R R.
2/ f l hm s tng trn R.
3/ f (f(x)) = f(x) + 12x; x R
Bi gii.
Nu f(x) = f(y) th f(f(x)) = f(f(y))
f(x) + 12x = f(y) + 12y 12x = 12y x = y
f l n nh. Theo bi f l ton nh t R R suy ra f l song nh
hm s ngc f
1
v f
1
l hm s tng trn R f
1
cng l hm tng trn R.
Thay x = 0 vo phng trnh hm ta c f(f(0)) = f(0).
Do f song nh nn f(0) = 0 f
1
(0) = 0
237
t f
n
1
(x) = f
1
f
1
...f
1
(x)
__
. .
n ln
;
Ta cng c f
n
1
l tng nn f
n
1
(0) = 0
Xt dy s (a
n
) vi
_

_
a
0
= f(x)
a
1
= x
a
n
= f
1
(a
n1
); n 2
Thay x bi f
1
(a
n1
) vo f (f(x)) = f(x) + 12x ta c a
n2
= a
n1
+ 12a
n
Gii phng trnh ta tm c
f
n
1
(x) = a
n+1
=
4xf(x)
7
(3)
n
+
3x+f(x)
7
4
n
Xt x > 0 c nh, khi f
n
1
(x) > 0; n (do f
n
1
tng) 3x +f(x) > 0. Cho n = 2k, 2k + 1
ta c :
4
3
_
2k1
>
4xf(x)
3x+f(x)
;
4
3
_
2k
>
fx4x
3x+f(x)
Cho k + ta c 4x f(x) 4x f(x) = 4x
Khi x < 0 f
n
1
(x) < 0; n, 3x +f(x) < 0.
Hon ton tng t ta cng c f(x) = 4x vi x < 0.
Vy f(x) = 4x; x
B. ng dng im bt ng
Cho nh x f : X Y a Xm f(a) = a ta gi a l im bt ng ca f. Vic nghin
cu im bt ng cho ta mt s k thut tm hm s .
Bi ton 7. Tm tt c cc hm s f : R R sao cho :
f (f(x)) = x
2
2; x R
Bi gii.
t: g(x) = f (f(x)) = x
2
2; g(x) c 2 im bt ng l - 1, 2
g
o
g(x) = g (g(x)) = (x
2
2)
2
2 c 4 im c bt ng 1, 2,
1+

5
2
,
1

5
2
Ta c: g
_
1+

5
2
_
=
1

5
2
v g
_
1

5
2
_
=
1+

5
2
Gi s tn ti f : R R sao cho f
o
f = g th f
o
g = f
o
f
o
f = g
o
f
Khi f(1) = f (g(1)) = f
o
g(1) = g
o
f(1) = g [f(1)] f(1) l im bt ng ca g.
Tng t f(2) l im bt ng ca g.
Vy {f(1), f(2)} = {1; 2}
v ta c:
_
f(1), f(2), f
_
1+

5
2
_
, f
_
1

5
2
__
=
_
1, 2,
1+

5
2
,
1

5
2
_
* Xt f
_
1+

5
2
_
Nu f
_
1+

5
2
_
= 1 f(1) = f
_
f
_
1+

5
2
__
= g
_
1+

5
2
_
=
1

5
2
iu ny mu thun v f(1) {1, 2}.
Tng t :
f
_
1+

5
2
_
= 2, mt khc f
_
1+

5
2
_
=
1+

5
2
238
v
1+

5
2
khng phi l im bt ng ca g.
Do vy ch c kh nng f
_
1+

5
2
_
=
1

5
2
.
Nhng khi th
f
_
1

5
2
_
= f
_
f
_
1+

5
2
__
= g
_
1

5
2
_
=
1

5
2
iu ny mu thun vi g
_
1

5
2
_
=
1+

5
2
{a, b}
Vy khng th tn ti mt hm f tha mn cc iu kin ni trn.
* Ta c th chng minh mt kt qu tng qut hn: Cho S l mt tp hp v
g : S S l mt hm s c chnh xc 2 im c nh {a, b} v g
o
g c chnh xc 4 im c nh
{a, b, c, d} th khng tn ti hm s g : S S g = f
o
f
* Chng minh: Gi s g(c) = y. Th c = g(g
o
g) = g(y) , nn y = g(c) = g(g(y)). Do vy
y l mt im c nh ca g
o
g. Nu y = c th c = g(y) = g(c), tc c l im c nh ca g, mu
thun.
T suy ra y = d, tc g(c) = d, v tng t th g(d) = c.
Gi s tn ti f : S S sao cho f
o
f = g. Th f
o
g = f
o
f
o
f.
Khi f(a) = f(g(a)), nn f(a) l mt im c nh ca g.
Bng vic kim tra tng trng hp ta kt lun f {a, b} = {a, b} , f {a, b, c, d} = {a, b, c, d}
Xt f(c). Nu f(c) = a, th f(a) = f(f(c)) = g(c) = d, mu thun do f(a) nm trong {a, b}.
Tng t cng khng th xy ra f(c) = b. Ngoi ra ta cng khng th c f(c) = c v c khng
l im c nh ca g.
Do vy ch c kh nng l g(c) = d. Nhng khi th f(d) = f(f(c)) = g(c) = d.
Mu thun, v iu ny khng th xy ra do d khng phi l im c nh ca g. Do vy khng
th tn ti hm f tha mn yu cu ca bi ton.
* Bi ton trn l trng hp c bit ca bi ton ny.
Bi ton 8. .(IMO 1994) Gi s S l tp hp cc s thc ln hn 1. Tm tt c cc hm
s f : S S sao cho cc iu kin sau y c tha mn
a) f [x +f(y) + xf(y)] = y +f(x) + yf(x) x, y S
b)
f(x)
x
l hm thc s tng vi 1 < x < 0 v vi x > 0.
Bi gii.
a) Tm im bt ng.
T iu kin (b) ta nhn thy phng trnh im bt ng f(x) = x c nhiu nht l 3 nghim
(nu c) : mt nghim nm trong khong (1; 0), mt nghim bng 0, mt nghim nm trong
khong (0; +).
b) Nghin cu im bt ng ca hm s Gi s u (1; 0) l mt im bt ng ca f.
Trong iu kin (a) cho x = y = u ta c
f(2u + u
2
) = 2u +u
2
Hn na 2u + u
2
(1; 0) v 2u + u
2
l mt im bt ng na ca hm s trong khong
(1; 0)
Theo nhn xt trn th phi c
2u + u
2
= u u = u
2
(1; 0)
239
Hon ton tng t, khng c im bt ng no nm trong khong (0; +). Nh th 0 l
im bt ng duy nht ca hm s (nu c).
c) Kt lun hm Cho x = y vo (a) ta c
f [x +f(x) + xf(x)] = x + f(x) + xf(x) x S
Nh vy vi mi x S th x + (x + 1)f(x) l im bt ng ca hm s. Theo nhn xt trn
th
x + (x + 1)f(x) = 0, x S f(x) =
x
1+x
, x S
Th li thy hm ny tha mn yu cu bi ton.
Bi ton 9. .(IMO 1983)- Tm cc hm s f : R R tha mn hai iu kin:
lim
x
f(x) = 0 v f (xf(y)) = yf(x), x, y R
+
Bi gii.
a) Tnh f(1).
Cho x = y = 1, ta c f (f(1)) = f(1). Li cho y = f(1) ta c
f (xf (f(1))) = f(1)f(x) f (xf(1)) = f(1)f(x).
Mt khc f (xf(1)) = f(x) nn ta c
f(x) = f(x)f(1) f(1) = 1 (do f(x) > 0)
b) im c nh ca hm s
Cho x = y vo quan h hm ta c
f(xf(x)) = xf(x), x R
+
.
Suy ra xf(x) l im bt ng ca hm s f.
c) Mt s c im ca tp im c nh.
Nu x v y l hai im c nh ca hm s, th
f(xy) = f(xf(y)) = yf(x) = xy
Chng t xy cng l im bt ng ca hm s. Nh vy tp cc im bt ng ng i vi
php nhn. Hn na nu x l im bt ng th
I = f(1) = f
1
x
f(x)
_
= xf(
1
x
) f
1
x
_
=
1
x
Ngha l
1
x
cng l im bt ng ca hm s. Nh vy tp cc im bt ng ng vi php
nghch o.
Nh vy ngoi 1 l im bt ng ra, nu c im bt ng no khc th hoc im bt ng
ny ln hn 1, hoc nghch o ca n ln hn 1.
Do ly tha nhiu ln ca im ny ln hn 1 cng s l im bt ng. iu ny tri vi
iu kin th 2 trong quan h hm.
d) Vy 1 l im bt ng duy nht ca hm s, do xf(x) l im bt ng ca hm s vi
mi x > 0 nn t tnh duy nht ta suy ra f(x) =
1
x
.
D thy hm s ny tha mn yu cu bi ton.
BI TP
Bi 1 Tm tt c cc hm s f : R R tha mn iu kin
f (xf(x) + f(y)) = (f(x))
2
+y; x, y R
240
Bi 2 Tm tt c cc hm s f : R R tha mn iu kin
f (x
2
+ f(y)) = (f(x))
2
+y; x, y R
Bi 3 ( ngh IMO 2002) Tm tt c cc hm s f : R R tha
f (f(x) + y) = 2x +f (f(y) x) ; x, y R
Bi 4 (THPT T8/360) Tm tt c cc hm s f : R
+
R
+
tha
f(x).f(y) = f (x +y(f(x)) ; x, y R
Bi 5 Tm tt c cc hm s f : Z
+
Z
+
tha mn
f (f(n) + m) = m+f(m+ 2012); m, n Z
+
Bi 6 (APMO 1989) Xc nh tt c cc hm s f : R R tha mn 3 iu kin sau y :
1/ f c tp gi tr R
2/ f tng nghim ngt
3/ f(x) + f

(x) = 2x; x R
Bi 7 (IMO 1996) Tm tt c cc hm s f : N N sao cho
f (m+f(n)) = f (f(m) + f(n)) ; m, n N
Ti liu tham kho
- Bi ton Hm s qua cc k thi Olympic ca Nguyn Trng Tun
- Cc ti liu bi dng h cho gio vin chuyn ca Trng i hc Khoa hc t nhin i
hc Quc gia H Ni t chc.
- Chuyn bi dng v Phng trnh hm ca Trng chuyn Quang Trung Bnh Phc.
- Bo Ton hc Tui tr.
- Tuyn tp Cc thi Olympic 30/4.
- Cc thi HSG Quc gia v Quc t.
- Ngun Internet.
241
T GIC LNG TIP
Nguyn Hu Tm, Hong T Quyn, Trng THPT chuyn L Qu n Bnh nh
1 M u
Nicolas Fuss (1755-1826), nh ton hc ngi c, ng l ngi dnh trn cuc i
thnh ph P-tc-bua, nc Nga, trong cng vic th k cho nh khoa hc Leonhard Euler.
Trong thi gian lm vic ti Vin hn lm Petersburg, ng vit nhiu bi bo v cc ch
khc nhau nh (spherical) hnh hc, (differential) hnh hc, phng trnh v nhiu lnh vc
khc. Tuy nhin, c lp vi nhng ch trn, N. Fuss kho cu v a gic lng tm,
c nh ngha l mt a gic c c ng trn ni tip v ngoi tip, hay ni cch khc l n
va l a gic ngoi tip, va l a gic ni tip. ng tm ra mi lin h gia 2 bn knh v
ng ni tm ca 2 ng trn trong trng hp t gic, ng gic, lc gic, tht gic v bt
gic. Bi vit ny, chng ti cp n t gic lng tip v mt s tnh cht ca n.
2 T gic lng tip v nh l Fuss
T gic lng tip l t gic c c ng trn ngoi tip (i qua tt c cc nh ca t gic)
v ng trn ni tip (tip xc vi tt c cc cnh t gic). Tt c cc tam gic u lng
Hnh 24:
tip vi mt ng trn ni tip v mt ng trn ngoi tip. Euler a ra biu thc th
hin mi quan h gia bn knh ni tip r, bn knh ngoi tip R v di ng ni tm d
nh sau :
d
2
= R
2
2Rr
M rng ra vi t gic lng tip, ta c nh l sau:
242
nh l 2. (nh l Fuss) Gi s c mt t gic ni tip ng trn (C) bn knh R v ngoi
tip ng trn (C

) bn knh r. Khong cch gia hai tm l d, khi ta c


(R
2
d
2
)
2
= 2r
2
(R
2
+d
2
) (1)
Vic tm ra mi quan h ny c nh gi l mt trong 100 vn ton hc s cp v
i nht trong lch s. Bn cnh , Fuss cng tm ra mi quan h tng t cho mt s trng
hp khc, chng hn nh: Vi trng hp lc gic ta c cng thc:
3p
4
q
4
2p
2
q
2
r
2
(p
2
+q
2
) = r
4
(p
2
q
2
)
2
(2)
Vi bt gic, ta c
r
2
(p
2
+q
2
) p
2
q
2
_
4
= 16p
4
q
4
r
4
(p
2
r
2
)(q
2
r
2
) (3)
Trong , p = R +d, q = R d.
Vic xy dng cc cng thc (2), (3) kh phc tp, y ta ch xy dng cng thc (1) cho
trng hp t gic.
Trc ht ta chng minh b sau :
B 3. Trong t gic lng tip, hai ng
thng ni cc cp tip im i nhau th vung
gc vi nhau.
Tht vy, gi s c t gic ABCD lng tip, v l gc ti nh Av C; K, L, M, N l
cc gc c k hiu nh hnh v. Gi gc to bi hai dy ni tip im l . Ta c
Hnh 25
+K + N + = 360
0
+ M +L + = 360
0
.
Suy ra + + K +N +M +L + 2 = 720
0
T suy ra = 90
0
.
Nhn xt: T tnh cht trn ta c cch dng t
gic lng tip nh sau: Trong mt ng trn v hai
dy cung vung gc vi nhau. T cc u mt ca hai
dy cung v cc tip tuyn vi ng trn. Phn gii hn
bi bn tip tuyn l mt t gic lng tip.
Quan st hnh v, ta c cc nhn xt: ng thng KM v LN chia t gic lng tip
ABCD thnh bn t gic AKON, BLOK, CMOL v DNOM c tnh cht ging nhau. Ta s
nghin cu dng t gic ny bng cch kho st qu tch ca mt im P c xy dng nh
sau.
Cho ng trn (C), v im O nm trong ng trn; X, Y l cc im trn ng trn
sao cho

XOY = 90
0
vung, P l giao im ca hai tip tuyn ca (C) ti X v Y . Cu hi t
ra l, qu tch ca P khi X, Y chuyn ng trn ng trn l g?
Cu tr li l, qu tch im P l mt ng trn. Vic chng minh kt qu ny s dn n
vic xy dng cng thc (1) Chng minh.
243
V MP, gi N l giao im ca MP vi XY . Ta c MP XY , v OF XY, F XY .
t

OMP = , OM = e, MP = p v r l bn knh ng trn (C).
Xt tam gic vung OXY ta c OF
2
= FX.FY (). V MP l trung trc ca XY , nn NX =
NY v

MNY = 90
0
. T ta c NF = e sin .
Hnh 26
Do OF = MN e cos , FX = NX e sin ,
FY = NX +e sin . Th cc ng thc trn vo (*) ta c
(MN e cos )
2
= (NX e sin )(NX +e sin )
MN
2
2.MN.e cos +e
2
= NX
2
M MX = r, MX
2
= MX
2
MN
2
, MX
2
= MP.MN = p.MN, nn t y ta c
2r
4
r
2
e
2
=
2r
2
e
r
2
e
2
.p cos +p
2
(4)
Ta thy, p v l nhng gi tr ph thuc vo v tr ca im P; r v e l hng s khng i.
Khi gi s P nm trn mt ng trn bn knh R = SP v tm S nm trn MO. t
d = SM l khong cch gia hai tm ng trn. Xt tam gic SMP ta c
R
2
= d
2
+r
2
+ 2dp cos (5)
So snh (4) v (5) ta thy R l hng s khi d =
r
2
e
r
2
e
2
.
Do vy qu tch imP l ng trn tmS, bn knh R. Tr li vi t gic lng tip
ABCD th ng trn ngoi tip t gic chnh l qu tch imP ni trn. Khi gc

XOY quay
n v tr m OX hoc OY trng vi ng knh, ta thu c mt t gic lng tip trong
trng hp c bit, c mt ng cho l ng knh chung ca hai ng trn, khi ta tnh
c e theo d, R, r v t thu c cng thc (1).
Cng thc (1) cng c th c chng minh cch khc nh sau: Gi I, O ln lt l tm ng
trn ni tip v ngoi tip t gic ABCD v gi E, F ln lt l giao im th hai ca cc tia
AI v CI vi ng trn tmO. Ta thy E v F l cc im chnh gia cua hai cung BD nn
EF i qua tm O.
Hnh 27
Ta c IO
2
=
IE
2
+ IF
2
2

EF
2
4
. Suy ra 2(d
2
+R
2
) = IE
2
+IF
2
.
Mt khc, IA.IC = IE.IF = R
2
d
2
. Do vy ta c
2(d
2
+ R
2
)
(R
2
d
2
)
2
=
IE
2
+IF
2
(R
2
d
2
)
2
=
1
IA
2
+
1
IC
2
=
sin
2
A
2
+ sin
2
C
2
r
2
=
1
r
2

244
3 nh l Poncelet
Vic chng minh cng thc lin h hon tt. Song hn bn c s quan tm n tnh
ng n ca mnh o: Ta c nh l sau y:
nh l 3. Cho hai ng trn (C) v (C

) c bn knh ln lt l R v r, (C

) nm trong
(C), d l khong cch gia hai tm ng trn sao cho tha mn biu thc
(R
2
d
2
)
2
= 2r
2
(R
2
+d
2
)
Khi tn ti mt t gic va ni tip (C) va ngoi tip (C

). C v s t gic nh vy.
Chng minh. Gi O v I ln lt l tm ca (C) v (C

). Ly mt im A bt k thuc ng
trn tm O, cc dy cung AB, BC, CD cng tip xc vi ng trn tm I. Ta cn chng minh
AD cng tip xc vi (I).
Ko di CI ct (O) ti F, v ng knh FE, ko di EI ct (O) ti A

. T ng thc
trn ta c
2(d
2
+R
2
)
(R
2
d
2
)
2
=
1
r
2

IE
2
+ IF
2
IC
2
.IF
2
=
sin
2
A
2
+ sin
2
C
2
r
2
Do
IE
2
IC
2
.IF
2
+
1
IC
2
=
sin
2
A
2
r
2
+
1
IC
2

IE
IC.IF
=
sin
A
2
r
M IC.IF = IE.IA

nn suy ra
r = IA

. sin
A
2
= r. sin

BAD
2
= r. sin

BA

I = d(I, A

B)
Nh vy A

B tip xc vi ng trn (I), do A A

. Khi A, I, E thng hng. M E l


im chnh gia cung BD cha C, nn d(I, AB) = d(I, AD) = r hay AD tip xc vi ng
trn tm I. iu ny cho thy s tn ti (v tn ti v s) t gic va ni tip ng trn (C),
va ngoi tip ng trn (C

).
Vic m rng cho n gic qu tht khng n gin. Poncelet a ra nh l kn Poncelet
c lin quan mt thit n a gic lng tip v gii quyt c mt phn quan trng ca bi
ton trn.
Xt hai ng trn (C) v (C

) trong mt phng, (C

) nm trong (C). T mt im P bt
k trn (C) ta v tip tuyn n (C

), ko di tip tuyn trn ct (C), t giao im va ct, tip


tc v tip tuyn n (C

), v thc hin thao tc tng t cho n khi to thnh ng cong


kn (c mt giao im trng vi P). Khi ta nhn c mt n gic, c gi l ng gp
khc Poncelet.
nh l Poncelet c th c pht biu nh sau:
nh l 4. Nu mt ng gp khc Poncelet n cnh c xy dng trn hai conic cho trc
l kn vi mt im bt u bt k th cng kn vi mi im bt u.
245
Do , nu c mt a gic lng tip c (C) l ng trn ngoi tip v (C

) l ng trn
ni tip, khi c v s a gic lng tip khng xc nh nhn (C) v (C

) l ng trn
ngoi tip v ni tip.
Vic chng minh nh l Poncelet cho n gic (i vi Hnh hc Euclide v tng qut trong
Hnh hc x nh) khng tin trnh by y, hy vng s c trnh by trong mt bi vit
khc.
Vn m hin nay vn cha c gii quyt trn vn l tm cng thc lin h gia d, R, r
tn ti mt n lng tip hai ng trn cho trc.
4 Mt s bi ton lin quan n t gic lng tip
Bi ton 1. Cho t gic ABCD ni tip ng trn (O; R) v ngoi tip ng trn (I; r) c
khong cch gia hai tm l d. Gi s M, N, P, Q theo th t l tip im ca ng trn ni
tip vi cc cnh AB, BC, CD, DA. Chng minh rng
1) MP vung gc vi NQ;
2) I v O thng hng vi trng tm G ca t gic MNPQ, hn na

IG =
r
2
d
2
R
2
.

IO;
IZ = 3) I v O thng hng vi giao im Z ca cc ng cho AC v BD, hn na

2r
2
d
2
R
2
.

IO;
Hnh 28:
Li gii. 1) Gi s A

, B

, C

, D

ln lt l trung im ca QM, MN, NP, PQ. Khi cc


ng thng AA

, BB

, CC

, DD

cng i qua I. V IA.IA

= IB.IB

= IC.IC

= r
2
nn php
nghch o cc I phng tch r
2
bin A thnh A

, B thnh B

, C thnh C

, D thnh D

; do
php nghch o ny bin ng trn ngoi tip t gic ABCD thnh ng trn ngoi tip
246
t gic A

. Nh vy, hnh bnh hnh A

c ng trn ngoi tip nn n l hnh


ch nht. Do A

suy ra NQ MP.
2) Gi O

l tm ng trn ngoi tip t gic A

, t php nghch o nn trn ta


c I, O, O

thng hng v ta cng nhn c h thc sau

IO

=
r
2
P
I/(O)
.

IO =
r
2
d
2
R
2
.

IO
Do A

l hnh ch nht nn O

trng vi trng tm ca hnh ch nht ny v cng trng


vi trng tm G ca t gic MNPQ. Nh vy, I, O, G thng hng v ta c

IG =
r
2
d
2
R
2
.

IO
3) Theo mt kt qu quen thuc v t gic ngoi tip th bn ng thng AC, BD, MP, NQ
ng quy; do vy Z cng l giao im ca MP v NQ. rng t gic ni tip MNPQ c
hai ng cho vung gc vi nhau v ct nhau ti Z nn tm ng trn ngoi tip, trng
tm G, v Z thng hng, hn na

IZ = 2

IG.
Nh vyI, O, Z thng hng v ta c

IZ =
2r
2
d
2
R
2
.

IO.
Bi ton 2. Cho t gic ABCD ni tip ng trn (O; R) v ngoi tip ng trn (I; r) c
khong cch gia hai tm l d. Chng minh cc ng thc sau:
1)
AB
IA.IB
=
CD
IC.ID
,
AD
IA.ID
=
BC
IB.IC
.
2)
AC
IA.IC
=
BD
IB.ID
=
2R
R
2
d
2
.
3)
1
IA
2
+
1
IC
2
=
1
IB
2
+
1
ID
2
.
4)
AC
2
IA
2
+ IC
2
=
BD
2
IB
2
+ID
2
.
5) 8Rr
1
AC
+
1
BD
_
= AB +BC +CD +DA.
Li gii. Ly A

theo th t l giao im th hai ca cc ng thng AI, BI, CI, DI


vi ng trn tm O. Ta thy A

v C

l im chnh gia ca hai cung trng bi dy BD,


B

v D

l im chnh gia ca hai cung trng bi dy AC; do vy A

l hnh ch nht
v A

= B

= 2R.
t IA.IA

= IB.IB

= IC.IC

= ID.ID

= k. Ta c
A

AB
=
IA

IB
=
k
IA.IB
. Suy ra
AB
IA.IB
=
A

k
. Tng t,
CD
IC.ID
=
C

k
. M A

l hnh ch nht nn A

= C

.
T
AB
IA.IB
=
CD
IC.ID
.
247
Hnh 29:
Chng minh tng t ta cng c
AD
IA.ID
=
BC
IB.IC
.
Ta c
AC
IA.IC
=
A

k
=
2R
R
2
d
2
v
BD
IB.ID
=
B

k
=
2R
R
2
d
2
. T y ta nhn c ng
thc (2).
By gi, v A

l hnh ch nht nn IA
2
+ IC
2
= IB
2
+ ID
2

k
2
IA
2
+
k
2
IC
2
=
k
2
IB
2
+
k
2
ID
2
. T y ta nhn c ng thc (3).
ng thc (4) c suy ra t (2) v (3). Ta chng minh ng thc cui cng nh sau:
Qua I v ng thng song song vi AB, ct AD v BC ti M v N. Gi P v Q theo th
t l hnh chiu vung gc ca I trn AD v BC. Ta c,
MN = IM + IN =
IP
sin

IMP
+
IQ
sin

INQ
=
r
sin A
+
r
sin B
M BD = 2Rsin A, AC = 2r sin B nn MN = 2Rr
1
AC
+
1
BD
_
.
K mt tip tuyn song song vi AB ca ng trn tm I, ct AD, BC, CD ln lc ti
E, F, G. Ta c 2MN = AB +EF().
D thy tam gic GCF v tam gic GED ng dng, hn na chng u nhn (I) l ng
trn bng tip ca hai gc tng ng ti nh E v nhC, do vy hai tam gic ny bng nhau.
Vy ta c CG = EG, DG = FG, suy ra CD = EF().
T (*) v (**) ta nhn c
MN =
1
2
(AB + CD) =
1
4
(AB +BC +CD +DA)
Do
2Rr
1
AC
+
1
BD
_
=
1
4
(AB +BC +CD +DA) = MN
248
y l pcm. T chng minh trn ta thy MN l hng s vi mi trng hp song song vi
cc cnh ca t gic.
Bi ton 3. Gi s cc ng thng AI, BI, CI, DI ct ng trn tm (O) ti im th
hai l A

; gi O
1
, O
2
, O
3
, O
4
theo th t l tm ng trn ngoi tip cc tam gic
IA

, IB

, IC

, ID

; gi E, F, G, H theo th t l hnh chiu vung gc ca I trn AB,


BC, CD, DA. Chng minh rng
1) T gic A

l mt hnh ch nht, hai ng cho ca n l ng knh ca ng


trn tm O v chng ln lt vung gc vi hai ng cho ca t gic ABCD.
2) Bn im O
1
, O
2
, O
3
, O
4
cng nm trn ng trn tm I bn knh
R
2
d
2
2r
.
3) T gic O
1
O
2
O
3
O
4
l nh ca t gic EFGH qua php v t tm I t s
R
2
d
2
2r
2
.
Hnh 30:
Li gii. 1) V I l tm ng trn ni tip t gic ABCD nn A

l im chnh gia cung


BCD, C

l im chnh gia cung BAD; do vy A

l ng knh ca ng trn (O) v


vung gc vi AC. Nh vy A

l mt hnh ch nht.
t k = IA.IA

= IB.IB

= IC.IC

= ID.ID

= R
2
d
2
, ta c
IO
1
=
A

2 sin

IB

=
A

2 sin

AIB
249
M
A

AB
=
IA

IB
=
k
IA.IB
nn
IO
1
=
kAB
2IA.IB. sin

AIB
=
k
2IE
=
R
2
d
2
2r
Nh vy O
1
nm trn ng trn tm I bn knh
R
2
d
2
2r
. tng t th O
2
, O
3
, O
4
cng nm
trn ng trn ny.
2) Ta c

O
1
IB

= 90
0


IA

= 90
0

IAB =

EIB. Suy ra E, I, O
1
thng hng. Tng t
ta cng c F, I, O
2
thng hng; G, I, O
3
thng hng. T y suy ra t gic O
1
O
2
O
3
O
4
l nh
ca t gic EFGH qua php v t tm I t s
R
2
d
2
2r
2
.
Trc khi n vi bi ton tip theo, ta c mt s nhn xt sau y.
Nhn xt. Gi s A
1
A
2
A
3
A
4
l t gic ni tip ng trn (O, R) v ngoi tip ng
trn (I, R). Cc tip im trn A
1
A
2
, A
2
A
3
, A
3
A
4
, A
4
A
1
ln lt l M, N, P, Q. t MA
1
=
t
1
, MA
2
= t
2
, PA
3
= t
3
, PA
4
= t
4
, khi
1) t
i
+ t
i+1
= A
i
A
i+1
, i = 1, 2, 3, 4, quy c t
5
= t
1
, A
5
A
1
.
2) t
1
t3 = t
2
t
4
= r
2
,
3) t
1
t
2
+t
2
t
3
+t
3
t
4
+t
4
t
1
= 2(R
2
d
2
).
4) Gi t
M
, t
m
ln lt l di on tip tuyn ln nht v b nht t (O, R) n (I, r).
Khi ta c
t
m
=
_
(R d)
2
r
2
, t
M
=
_
(R + d)
2
r
2
.
Hnh 31:
Bi ton 4. Cho A
1
A
2
A
3
A
4
l t gic ni tip ng trn (O, R) v ngoi tip ng trn (I, R).
Cc tip im trn A
1
A
2
, A
2
A
3
, A
3
A
4
, A
4
A
1
ln lt l M, N, P, Q. t MA
1
= t
1
, MA
2
=
250
t
2
, PA
3
= t
3
, PA
4
= t
4
, t
M
, t
m
ln lt l di on tip tuyn ln nht v b nht t (O, R)
n (I, r). Chng minh rng
1) 2r t
1
+ t
3
t
m
+t
M
,
2) 2r t
2
+ t
4
t
m
+t
M
,
3) 4r t
1
+ t
2
+t
3
+t
4
4r.
R
2
+d
2
R
2
d
2
,
4) 4r
2
t
2
1
+t
2
2
+t
2
3
+t
2
4
4(R
2
+ d
2
r
2
).
Hnh 32:
Li gii. Xt ng trn (C) c ng knh AB = t
m
+ t
M
. Gi P thuc AB sao cho
AP = t
M
, PB = t
m
. Khi , vi mi t
i
, i = 1, 2, 3, 4; v t
m
t
i
t
M
, nn lun c hai im Q v
R thuc (C) sao cho: t
i
= PQ, t
i+2
= PR, vi PQ+PR = QR. Ta c PQ.PR = t
m
.t
M
= P
P/(C)
.
Hin nhin t
i
+ t
i+2
t
m
+ t
M
v t
m
+ t
M
l ng knh. Mt khc, r rng t
i
+ t
i+2
2r
v r
2
= t
m
t
M
. Vy, hai bt ng thc u c chng minh.
chng minh bt ng thc tip theo, ta rng
t
m
=
_
(R d)
2
r
2
= r.
R d
R +d
, t
M
=
_
(R + d)
2
r
2
= r.
R + d
R d
.
T ta c
4r t
2
1
+t
2
2
+t
2
3
+t
4
2(t
m
+ t
M
) = 4r.
R
2
+d
2
R
2
d
2
Vi bt ng thc cui, rng t
1
t
3
= t
2
t
4
= r
2
, ta c
4r
2
t
2
1
+t
2
3
+ 2t
1
t
3
, 4r
2
t
2
2
+t
2
4
+ 2t
2
t
4
Suy ra
4r
2
t
2
1
+t
2
3
+ t
2
2
+t
2
4
251
Ta li c
t
1
+ t
3
t
m
+t
M
, t
2
+ t
4
t
m
+t
M
, t
1
t
3
= t
2
t
4
= t
m
t
M
Do
t
2
1
+t
2
2
+t
2
3
+ t
2
4
2(t
2
m
+t
2
M
) = 4(R
2
+d
2
r
2
)
Bi ton 5. Cho t gic lng tip A
1
A
2
A
3
A
4
v cc di t
i
, i = 1, 2, 3, 4 nh gi thit bi
ton trn. Chng minh:
4
r

1
t
1
+
1
t
2
+
1
t
3
+
1
t
4

4(R
2
+d
2
)
r(R
2
d
2
)
Li gii. Ta c t
1
+t
3
2r v t
1
t
2
= r
2
nn
1
t
1
+
1
t
3
=
t
1
+t
3
r
2

2
r
Tng t,
1
t
2
+
1
t
4
=
t
2
+t
4
r
2

2
r
T
1
t
1
+
1
t
2
+
1
t
3
+
1
t
4

4
r
Mt khc do t
1
+ t
3
t
m
+t
M
, t
1
t
3
= t
m
t
M
nn ta c
1
t
1
+
1
t
3

1
t
m
+
1
t
M
. T
1
t
1
+
1
t
2
+
1
t
3
+
1
t
4

1
t
m
+
1
t
M
_
=
4(R
2
+d
2
)
r(R
2
d
2
)
.
Bt ng thc c chng minh.
5 Li kt
T gic v rng hn l a gic lng tip v ang l mt vn hp dn khng nhng
vi cc nh nghin cu m cn i vi cc bn tr yu ton. Bi vit trn y phn no a
ra nhng nh l v nhng bi ton c bn v vn ny. Song v d lng c hn v kh nng
hn ch nn bi vit khng cp n nhng vn chuyn su hn trong cc dng a gic v
cc bi ton phc tp. V lc gic v bt gic lng tip s c nghin cu su hn trong cc
chuyn tip theo.
kt thc bi vit, xin a ra mt s bi tp lin quan n t gic lng tip bn c
tham kho.
Bi ton 6. Cho t gic ABCD lng tip; M, N, P, Q ln lt l tip im ca cc on thng
AB, BC, CD, DA vi ng trn ni tip. Z l giao im hai ng cho ca t gic. Chng
minh ZM, ZN, ZP, ZQ ln lt l phn gic cc gc AZB, BZC, CZD, DZA.
252
Bi ton 7. T gic li ABCD c ng trn ni tip (I; r) v ng trn ngoi tip (O; R).
t OI = d. Chng minh rng:
R
2
d
2
=
8R
2
r
2
AC.BD
.
Bi ton 8. Xt lp cc t gic li ABCD va ni tip (O; R), va ngoi tip (I; r) c nh.
Chng minh cc tnh cht sau:
1) AB.CD = const ;
2) IA.IB.IC.ID = const ;
3) sin A. sin B. sin C. sin D = const ;
4)
AB + BC +CD + DA
AC +BD
= const ;
5) Giao im ca hai ng cho AC v BD l mt im c nh nm trn ng thng
OI;
6) Tn ti mt t gic A
0
B
0
C
0
D
0
c din tch v chu vi ln nht, cng tn ti mt t gic
A
1
B
1
C
1
D
1
c chu vi v din tch nh nht.
Bi ton 9. Cho ABCD l mt t gic lng tip, (O; R), (I; r) ln lt l ng trn ngoi
tip v ni tip t gic. Chng minh rng:
1) 8r AB + BC + CD + DA 8r.
R
2
d
2
R
2
+ d
2
;
2) 4(R
2
d
2
+ 2r
2
) AB
2
+ BC
2
+ CD
2
+DA
2
4(3R
2
2r
2
).
TI LIU THAM KHO.
[1] Trn Vn Tn, Bi tp nng cao v mt s chuyn Hnh hc 11, NXB Gio dc 2008.
[2] Fuss problem of the Chord-Tangent quadrilateral, Mgr. Barbora Stastna, Masaryk Uni-
versity, Faculty of Science.
[3] Some relations concerning triangles and bicentric quadrilaterals in connection with Pon-
celets closure theorem, Mirko Radic, Math. Maced Vol1 (2003), 35-58.
[4] http://mathworld.wolfram.com; www.mathlinks.ro.
253

You might also like